1

Ace your homework & exams now with Quizwiz!

TO be able to use Azathioprine and/or Mercaptopurine the patient must have normal function of TPMT. explain why?

- For loss of function of TPMT allele, find alternative treatment or start a very low dose - Patients with genetic deficiency of TPMT may have high risk of myelosuppressive effects > These patients with low or absent TPMT activity are at risk for sever myelotoxicity; occurs in around 10% of patients

Patient called pharmacy saying their child exposed to topical cream and don't know what to do?

1. call 800-222-1222 2. Remove any contaminated clothing and run water over the skin for 10 minutes then soap and rinse

What is the normal level of Calcium and some important information about Calcium?

1. correct calcium if low albumin 2. It increase due to calcium supplementation, vitamin D, thiazide diuretics 3. Decrease due to corticosteroids, long term heparin, loop diuretics, bisphosphonates, calcitonin, foscarnet, Topiramate 4. supplement in pregnancy and other conditions

What are the 3 lifestyle modifications to reduce ED?

1. lose weight 2. limit alcohol 3. quiet tobacco

What is the treatment goals for chronic Angina?

1. reduce risk of an acute coronary syndrome (UA/Myocardial infraction) 2. Provide relief from the angina pain

What is the life time dose of Mitoxantrone?

140 mg/m2

Ketorolac (Sprix NS)

1st IV > Always start IV, IM, or nasal spray and continue with oral > 5 days total maximum treatment > Can cause severe adverse effects including GI bleeding and perforation, post op bleeding, acute renal failure, liver failure and anaphylactic shock

What are some possible treatment of HE?

- Identify and treating precipitating factors - Reducing blood ammonia levels through diet > limiting the amount of animal protein - Drug therapy

Prasugrel (Effient)

- Indicated for the reduction of thrombotic evens in patients with ACS who are to be managed with PCI - Keep in original container - Do not start in patient likely to undergo GABG d/c 7 days prior to any major surgery CI - *Active pathological bleed; patients with a history of TIA or stroke*

Isosorbide dinitrate / hydrALAZINE (BiDil)

- Indicated in self-identified black patients with NYHA FC III or IV who are symptomatic despite optimal therapy with ACE inhibitors and beta blockers. - CI > CI with PDE-5 inhibitors - WARNING > Drug-induced lupus erythematosus (DILE - dose and duration related - report fever, joint/muscle aches, fatigue) - SIDE EFFECTS > Ha, Dizziness, hypotension, tachycardia, weakness - MONITORING > HR, BP, signs and symptoms of HF

List some induction immunosuppressant medications ?

- Induction immunosuppressant therapy is given before or at the time of transplant to prevent acute rejection during the early post-transplant period by providing high degree of immune suppression. > Basiliximab, an interleukin-2 (IL-2) receptor antagonisti > Thymoglobulin (Rabbit Antithymocyte Immunoglobulin)

Important about the use of prednisone in RA

- Initial dose 5-60 mg daily, some use alternate day therapy (ADT) dosing in which twice the usual daily dose is given every other day to decrease adrenal suppression/reduced toxic effects - Indicated for acute inflammation/pain and as bridge therapy while waiting for DMARDs to take effect. Steroids should not be used long term. However, some patients may use chronically <10mg daily due to disease severity.

Antiandrogen-Antiestrogen: LHRH

- Initially increase production of androgens and estrogens, which can cause an initial tumor flare; followed by down regulation through a negative feedback loop which decrease gonadotropin release, decrease LH and FSH, resulting in a chemical castration/oophorectomy. - Goserelin (Zoladex) - Leuprolide (Leupron) - Decrease bone density and increase risk for osteoporosis; consider calcium and vitamin D supplementaiton, weight bearing exercise and DEXA sreening. CI - Pregnancy - Breastfeeding (leuprolide) - Vaginal bleeding (Leuprolide) SE - Hot flashes, bone pain, impotence, injection site pain/swelling, dyslipidemia, QT prolongation, gynecomastia (men), peripheral edema

Notes on Vitamin D Selection

- NOF recommend 800 - 1000 IU of vitamin D for >=50 - NIH recommend Vitamin D for people up to age 70 is 600 IU daily and 71+ years is 800 IU daily. - The 50,000 units vitamin D2 supplement (ergocalciferoL; the green capsule) which is used in renal disease or short term to replenish stores - Cholecalciferol or vitamin D3 is the preferred source although vitamin D2 is often the type in supplements and will provide benefit.

What are the reasons to use Nasal Sprays (NSL)

- Nasal sprays such as Afrin and Flonase are used primarily to treat localized nasal symptoms. The nasal route has a faster onset than the GI route, and is useful for acute conditions that should be treated quickly, including pain. Nasal sprays bypass gut absorption; proteins that would get destroyed in the gut (calcitonin) can be given nasally. A compound that require a gut factor for absorption can also be given nasally (vitamin B12) Example > Lazanda - Fentanyl NS > Flumist - Influenza live vaccine NS

What are the side effects of methotrexate?

- Nausea - Vomiting - Diarrhea - Increase in LFT - Stomatitis - Alopecia - Photosensitivity

What are the major side effects of Acetylchoinestrase inhibitor?

- Nausea , Diarrhea, Dizziness

Pulmonary Toxicity

- Associated with require pulmonary function tests if a high risk agent, including alkylators (busulfan, carmustine, lomustine) bleomycin and methotrexate. Monitor lung function tests

Counseling with Nasal Spray

- Do not sniff, swallow, or inhale through the nose while administrating - Head tilted slightly back

List the Monoamines that would have reduced metabolism with monoamine oxidase inhibitors?

- Dopamine - Epinephrine - Norepinephrine - Serotonin - Tyramine (which is a problem with foods rich in tyramine)

Where is Daytrana applied? A. Hip B. Lower abdomen c. Upper outer arm D. Upper thigh

A. hip Different areas of the body absorb varying amounts of the medicine. The pharmaceutical company that developed Daytrana determined that the hip was the most effective place to apply it.

What are the goals PER ADA?

A1C < 7% Preprandial capillary plasma glucose 80-130 mg/dL Postprandial plasma (1-2 hours after start of meal) <180mg/dL

Important information about HPV vaccine

ACIP recommends the 2 dose series between the age 11-12 years, with catch-up vaccination at age 13-26 years. Vaccination can begin at age 9 years and ideally prior to sexual activity - Males 9-26 years to reduce the likelihood of genital warts or anal cancers (HPV4) - Required 3 doses. The 2nd dose is 1-2 months after the 1st and the 3rd 6 months after the 1st.

What is the MOA of this natural product (Huperzine A) and what is it used for?

AChE inhibitor and NMDA receptor antagonist Used for Dementia/Memory

Bupropion

- Start 1 week before quit date - D/C if no progress by week 7 - BBW > Serious neuropsychiatric events including depression, suicidal thoughts and suicide have been reported in patients taking bupropion; not approved for usre in children; not approved for bipolar - CI > Seizure disorder > Patient receiving other doses of bupropion Warning - Use with caution in patient underlying psychiatric disorders - Avoid use in pilots, air traffic controllers, commercial truckers, bus drivers SE - Dry mouth, insomnia, tremors/seizures (dose-related) - No effects on 5HT and therefore no sexual dysfunction NOTE - can be used in combination with NRT - Delays weight gain, can be used with CVD risk - Do not exceed 450mg/day d/t seizure risk - MedGuide Requires

What to do if you want to switch from anticoagulants other than warfarin to rivaroxaban?

- Start rivaroxaban <= 2 hours before the next scheduled evening dose and discontinue other anticoagulant. For UFH given via continuous infusion, stop UFC and give rivaroxaban at the same time

What to do if you want to switch from Rivaroxaban to warfarin?

- Stop Rivaroxaban and begin a parenteral anticoagulant and warfarin when the next schedule dose of rivaroxban would have been taken, discontinue parenteral anticoagulant when INR reaches therapeutic range

Testesterone

- Testesterone replacement is restricted in men with severe BPH - Even with mild or moderate BPH, if dispensing a 5-a-reductase inhibitor for BPH that blocks the conversion of testosterone active from, it would not make sense to dispense another drug that provide testosterone directly - Common side effects of testosterone include increased male pattern baldness, acne, and gynecomastia.

Define Minimum Inhibitory Concentration (MIC)?

- The lowest drug concentration that prevents visible microbial growth after a 24 hours incubation

What is the nasal ointment of Mupirocin used for?

- The nasal ointment is used to prevent the spread of a bacteria known as MRSA and should be administered into each nostril twice daily for 5 days or UTD.

What is Hydroxyzine used for?

- Vistrail - an FDA approved or anxiety and is considered second-line. - It is occasionally used for short-term anxiety in lieu of BZDs that have the potential for abuse. - This is a sedating antihistamine and works by sedating the patient, rather than treating any underlying cause. IT should not be used long-term. - It is also used for pruritus

What is used in pregnancy?

- all D > Possible risk of cleft hip and palate or floppy baby syndrome - if benzo is required use Alprazolam or lorazepam

IMPORTANT!!!!! In chronic myelogenous leukemia (CML), the philadelphia chromosome is a specific abnormality which leads to a fusion protein of abl (Abelson) with bcr (breakpoint cluster region), termed bcr-abl. These drugs target and inhibit abnormal fusion protein. list drugs that require + BCR-ABL

1. Imatinib (Gleevec) I Dont Need BP 2. Dasatinib (Sprycel) 3. Nilotinib (Tasigna) 4. Bosutinib (Bosulif) 5. Ponatinib (Iclusig)

Important about Calcitonin Nasal Spray and Injection

Calcitonin (Miacalcin, Fortical) > Inhale 1 spray (200 IU) in one nostril daily (alternate nostril daily) CI > Allergy to calcitonin salmon SE > Rhinitis, back pain, muscle aches, HA etc

List some natural products used in ADHD

Fish oil - omega-3 fatty acids - primrose oil (omega 6 fatty acids) - SAMe - St john wort (Strong inducer) - Ginkgo

What are the side effects associated with NIACIN?

Flushing, pruritus (itching), N/V, diarrhea, GI istress, hyperglycemia, hyperuricemia (or gout), cough, hepatotoxicity, orthostatic hypotension, hypophosphatemia. Monitor > check LFTs at the start , every 6-12 weeks for the first year then around 6 months

When is a fibrinolytic indicated?

For patients presenting with STEMI, a fibrinolytic may be administered if the patient is not at a PCI-capable hospital or is not able to receive PCI within 2 hours (120 minutes) from presentation to the hospita.

Where does the RX approval process for RX starts?

For prescriptions drugs, the drug approval process begins with pre-clinical (animal) research, which is followed by an Investigational New Drug (IND) application to conduct human clinical trials. Of, if the company is requesting approval of a genetic drug, they file an Abbreviated New Drug Application (ANDA) The steps are as follow: - Pre-clinical - IND - clinical phases 1,2,3 - NDA - FDA after acceptance can also request the MFG conduct additional post-marketing studies (AKA phase IV) If want generic approved then file ANDA

BIG INHIBITORS

Grapefruit PIs Protease inhibitor (don't miss ritonavir) but check all PIs since many are potent inhibitors Azole antifungals, the agents that are used oral and IV: fluconazole, itraconazole, ketoconazole, posaconazole, and voriconazole Cyclosporine Cimetidine: H2RA that is the most difficult to use due to *Drug Interactions and Androgen-blocking effects (that can cause gynecomastia-swollen, painful breast tissue or impotence* Macrolides (calrithromycin and erythromycin), *not Azithromycin*, but do include telithromycin Amiodarone (and dronedarone) Non-DHP CCBs dilitazem and verapamil

What pregnancy category is methotrexate?

Pregnancy category X

What may be used in wound prophylaxis and for routine boosting?

TD > If patient has received 3 or more doses of tetanus, and the wound is NOT clean or minor, revaccination with TD if more than 5 years since the last dose. They may also require tetanus immunoglobulin (TIG). If less than 3 doses or unclear history with serious wound, they may require both. > A person who gets a cut or burn might need a dose of Td or Tdap to prevent tetanus infection. Tdap should be used for anyone who has never had a dose previously. Td should be used if Tdap is not available, or for anybody who has already had a dose of Tdap, and for adults 65 and older.

What is the purpose of using Placebo in a study?

This practice is called "blinding" or "masking" the treatment allocation in a clinical study

Enoxaparin (Lovenox) [Dose]

Treatment of VTE and UA/NSTEMI - 1mg/kg SC Q 12 h (or 1.5mg/kg SC daily for VTE in patient treatment only) - CrCl < 30 ml/min 1mg/kg SC daily Treatment of STEMI - In patients < 75 years: 30mg IV bolus plus 1mg/kg SC dose followed by 1mg/kg SC Q 12 h (max 100mg for the first two doses only) - CrCl <30ml/min > 30 mg IV bolus plus 1mg/kg SC dose, followed by 1mg/kg SC daily - In patient >=75 > 0.75mg/kg SC Q 12 h (no-bolus-max 75mg for the first two doses only) CrCl < 30ml/min > 1mg/kg SC daily (no bolus) * In patient managed with PCI: if the last SC dose was given 8-12 hours before ballon inflation, give 0.3mg/kg IV bolus *

What is the drug of choice if HIT occurred in the hospital setting? A. Argatroban B. Rivaroxaban C. Dabigartan D. NONE

A. Argatroban - Intravenous *direct thrombin inhibitor * which have very important clinically since they do not cross react with heparin induced thrombocytopenia (HIT) antibodies.

Which of the following have the greatest risk of bleeding? A. homozygous for the cyp2c9*3 B. homozygous for the cyp2c9*2 C. homozygous for the cyp2c9*1

A. leads to a large reduction in metabolism of warfarin

Which of the following statements concerning case-control studies is correct? A. They are retrospective B. The patient serves as their own control C. The researcher analyzes individual patient cases D. They include cases without the intervention E. They provide conclusive evidence of cause and effect

A. they are retrospective C. The researcher analyzes individual patient cases D. They include cases without the intervention

What are the goals per AACE?

A1C <= 6.5% (ADA support this for young adults and patient not experiencing hypoglycemia) Preprandial <110mg/dL Postprandial <140mg/dL

Meddwatch is used to report problems with the following? A. Drugs B. Biologics C. Medical devices D. Nutritional products E. Cosmetics

ALL OF THEM

List Tyramine rich food?

Aged cheese Air-dried meats Certain wines and beers Other foods which have been aged, fermented, pickled , or smoked

Which drugs cause the most Acneiform rash?

Agents that inhibit epidermal growth factor receptor (EGFR) - Cetuximab - Erlotinib - Panitumumab - Sorafenib - Sunitinib

What is the difference between errors of Omission and errors of Commission?

An error of omission means leaving something out that is needed for safety, such as missed instructions or failure to provide a dose at the required dose An error of commission mean that something was done incorrectly such as prescribing bupropion to a patient with history of Epilepsy or dispensing sulfamethoxazole to a patient with sulfa allergy.

List other medical problems associated with SCD

Anemia > Require foid acid Kidney > chronically infected, urination problems are common including uncontrolled urination during sleep > Treatment with ACE inhibitor should be started for proteinuria or microalbuniria in aduls. Priapism > occur in 30% of male patients Liver enlargement Gallstones Spleen > From trapped blood > Spleen injury will put patient at increases risk for serous infections Pregnancy/Contraception > Due to the increase risk of stroke in SCD, *progestin-only contraceptives, levonorgestrel IUDs, and barrier methods of contraception are recommended.*

Ganciclovir (Cytovene IV, Zirgan) ValGanciclovir (Valcyte)

BBW 2 1. Myelosuppression 2. Carcinogenic and inhibition of spermatogeneis in animal SE > Fever, nausea, vomiting, diarrhea, anorexia, thrombocytopenia, neutropenia, leukopenia, anemia

What is the most important medication error reduction tool in our arsenal now?

Barcoding > The barcode follows the drug through the medication use process to make sure it is being properly stocked

Must kno wabout Interleukin 2 (IL2) receptor antagonist

Basiliximab (Simulect( BBW > Should only be used by physicians experienced in immunosuppressive therapy SIDE EFFECTS * Hypertension, fever, weakness, stomach upset/nausea/vomiting/cramping, peripheral edema, dyspnea/upper respiratory irritation/infection, cough, tremor, painful urination *

Why Carvediolol IR converson is not 1:1 to carvediolol CR?

Because CR has less bioavailability than carvediolol IR, therefore, dose conversions are not 1:1

Why Anemia is present in CKD?

Because Erythropoietin is produced by the kidney and stimulate production of reticulocyte (the immature red blood cells) in the blood marrow

Antiandrogens: Androgen Receptor Antagonists used in Prostate Cancer

Bicalutamide (Casodex) Flutamide (Eulexin) Nilutamide (Nilandron) - These agents are used 1-4 weeks prior to starting LHRH agonists to mitigate tumor flare SE - Hot flashes, edema, pain, asthenia, heart failure , gynecomastia, Visual distrubances (nilutamide only)

List 1 drug that have low oral bioavailability.

Bisphosphonates - Annual IV dose will be less than the weekly dose of the oral formulation

important contraindication to the use of Estrogen

Boxed Warning - Do not use if patient is > 35 years and smoke due to cardiovascular risk (contraindicated)

ACE inhibitors

Boxed warning - Can cause injury and death to developing fetus; discontinue as soon as pregnancy is detected Contraindication - Angioedema - Do not use in bilateral renal artery stenosis - Do not use concurrently with aliskiren in patient with diabetes SE - Cough - Hyperkalemia - Hypotension , dizziness, ha Monitoring - BP, K, SCr, BUN Notes - Pregnancy Category D

Which of the following is the best option to correct acidosis? A. Dextrose 5% B. NaCl 0.9% C. Lactated Ringer's (LR) D. Multiple electrolyte Injection (Plasma-Lyte A)

C and D

Which of the following is classified as molds fungi? A. Candida B. Cryptococcus C. Aspgergillus D. Mucor E. Histoplasma F. Blastomyces

C. Aspgergillus D. Mucor

Pyrazinamide

CI - Acute gout, severe hepatic damage SE - GI upset , malaise, hepatotoxicity, hyperuricemia, gout

hydralazine

CI > Mitral valve rheumatic heart disease WARNING > Drug-induced lupus erythematosus (DILE-dose and duration related) Side Effects > Headache, reflex tachycardia, palpitations Monitoring > HR, BP, signs and symptoms of HF, ANA titer

What would you expect if Lithium levels are > 3 mEg/L?

CNS depression, arrhythmia, seizures, irreversible brain damage, coma

If there is a K-Ras mutation (positive), then we can't use 2 medication used in cancer treatment of Colorectal. These agents will not be effective in patients with colorectal cancer who have K-ras mutation?

Cetuximab (Erbitux) Panitumumab (Vectibix)

How often to check Blood Pressure?

Check BP 2-4 weeks after starting therapy and until goal BP is reached.

These lower Testosterone levels:

Chemotherapy drugs used for prostate cancer Methadone Cimetidine Spironolactone

Drug interactions with B12 - there is one that cause decrease in B12 absorption

Chloramphenicol, colchicine, ethanol, and long treatment with metformin may decrease B12 absorption

Which drugs should not be used if patient has G6PD deficiency?

Chloroquine ,Dapsone Methylene blue ,Nitrofurantoin Quinine , Quinidine Sulfonamides ,Primaquine Probenecid

What cause increase in Monocytes?

Chronic infection inflammation and stress

How to determine clearance? CL?

Cl = Dose / AUC FOR IV Cl/F = Dose / AUC for PO > No F in equation #1 because F = 1 100% bioavailable

List 2 drugs that must be avoided with true egg allergy?

Clevidipine (Cleviprex) Propofol (Diprivan)

List 3 medications that must be avoided with peanut allergy?

Clevidipine (Cleviprex) Propofol (Diprivan) Progesterone in Prometrium capsules

Leflunomide (Arava)

BBW (2) 1. Women of childbearing potential should not receive leflunomide until pregnancy has been excluded 2. Hepatotoxicity Contraindication > Pregnancy SE > Hepatotoxicity, etc Notes > Pregnancy category X

What is ACLS TIN?

Drugs that can't be placed in PCV due to sorption Amiodarone infusion greater than 2 hours Carmustine Lorazepam Sufentanil Thiopental Regular human insulin Nitroglyceirn

Cyclobenzaparine (Flexeril)

Dry mouth Serotonergic May exacerbate cardiac arrhythmia

what are the signs and symptoms of lower urinary tract infections (cystitis) ?

Dysuria: Painful or difficult urination Urgency Frequency Burning nocturia suprapubic heaviness and/or hematura

What is synergy?

Effect of two or more agents combined to produce a greater effect than the sum of their individual effects.

What is the BBW associated with Plavix?

Effectiveness depends on the activation to an active thiol metabolite mainly by CYP 2C19. Poor metabolizers exhibit higher cardiovascular events than patients with normal CYP 2C19 function. Tests to check CYP 2C19 genotype can be used as an aid in determining a therapeutic strategy.

Pregnancy Category B

Either animal studies have not demonstrated a fetal risk but there are no controlled studies in pregnant women, or animal studies have shown an adverse effect that was not confirmed in controlled studies in women in the 1st trimester

List one treat that was approved for treatment of severe aplastic anemia?

Eltrombopag (Promacta) > Will increase platelet counts > Bone Marrow Stimulator It can treat thrombocytopenia and aplastic anemia.

Which patches are applied weekly?

Clonidine (Catapres-TTS) Estradiol (Climara, Menostar) Estradiol/Levonorgestrel (ClimaraPro) *Ortho Evra* Weekly for 3 weeks, then off for the 4th week

What is code blue?

Code Blue refers to a patient requiring emergency medical care, typically for cardiac or respiratory arrest. The overhead announcement will provide the patient's location. The code team will rush to the room and begin immediate resuscitative efforts.

Morphine

Constipation > Tolerance usually develops to opioid side effects except constipation > When opioid are ATC, constipation will likely require stimulant laxatives (senna, bisacodyl) or osmotic laxative (MOM). > Rarely docusate alone is enough.

What are the classic symptoms of ANEMIA ?

Fatigue, Malaise, Weakness, SOB, exercise intolerance, headache, dizziness, anorexia, and/or pallor

List some patient risk for developing N/V ?

Female Patient < 50 y.o Dehydration History of motion sickness History of N/V with prior regimens

What is the ANTIDOTE for Benzodiazepines?

Flumazenil > Can precipitate seizures when used in benzodiazepine-dependent patients > Also used for non-benzo hypnotic such as Zolpidem

Which Triptan drugs used for migraine has the longest half life but short onset of action?

Frova (Frovatriptan) Naratriptan (Amerge) > Can be used w/ short acting and fast onset triptan > Must monitor serotonin carefully

What is the first anti-diarrheal drug for HIV/AIDS patients approved by FDA?

Fulyzaq Brand name Crofelemer Generic name > Due to cost, first consider trial of loperamide (Imodium) or Lomotil

Topoisomerase II inhibitor

G2- phase: block the coiling and uncoiling of DNA helix by facilitating single strand breaks followed by religation - Etoposide > VePeside capsules require frig - IV administration can cause hypotension due to rapid infusion rate, infuse slowly over at least 30-60 minutes. - Use non-PVC IV bag and tubing due to leaching of DEHP BW > Bone marrow suppression SE - Hypotension, etc.

Patient has hypovolemic shock, was given crystalloids and did not respond to therapy, what is the next step?

Give Vasopressor therapy > * Vasopressor will not be effective without adequate fluid (at least 30 mL/kg) *

Enterococcus , Staphylococcus aureus, Staphylococcus epidermidis , Streptococcus pneumoaniae ??

Gram + cocci

what about group 2,3,4,5?

Group 2 > Pulmonary Venous Hypertension from left-sided heart disease Group 3 > PH from hypoxia or chronic lung disease such as COPD or interstitial lung disease Group 4 > Chronic thrromboembolic PH (CTEPH) >> Warfarin to an INR goal of 2-3 is recommended given he history of clot >> If warfarin is CI then give Riocioguat (Adempas) Group 5 > whoever does not fit in the previous groups

What are the contraindications associated with Warfarin?

Hemorrhagic tendencies (cerebrovasclar hemorrhage, bacterial endocarditis, pericarditis, pericardial effusions), blood dyscrasias, pregnancy, uncontrolled hypertension, non-complicance, recent or potential surgery of the eye or CNS, major regional lumbar block anesthesia or traumatic surgery resulting in large, open surfaces, pericarditis or pericardial effusion, bacterial endocarditits (pre-eclampsia), threatened abosrtion, and pregnancy (except with mechanical heart valves at high risk of thromboembolism)

Piroxicam (Feldene)

High risk for GI toxicity and sever skin reactions, including SJS/TEN

What is ABVD used for?

Hodgkin's lymphoma A= Adriamycin (Doxorubicin) B= Bleomycin V= Vinblastine D= Decarbazinel

What is the safest DMRD drug to give a pregnancy woman who has RA?

Hydroxychloroquinoe

What are the SE of ESA

Hypertension, thrombosis, etc. NOTE IV route is recommended for patients on hemodialysis.

Hypernatremia: Hypovolemic Hypervolemic Isovolemic

Hypovolemic: is caused by dehydration, vomiting, diarrhea, and it is treated with fluids Hypervolemic is caused by hypertonic and is treated with diuresis Isovolemic is frequently caused by diabetes insipidus (DI) which can decrease ADH. it is treated with desmopressin

If patient can't take capsule due to Gelatin, what is the other type of capsule that could be dispensed to these customers and it is not made from animal?

Hypromellose

Everyone should be encouraged to maintain a healthy lifestyle to reduce cancer risk which include the following EXCEPT? A. Avoid tobacco B. Maintain a healthy weight C. Exercise regularly D. Eat healthy with plenty of fruits and veggies E. Limit alcohol intake F. Protect skin from harmful UV rays G. Assess cancer risk, family history and individual history H. Have regular check up and cancer screening tests. I. All of the above are correct

I

Which testing is required prior to use Maraviroc (Selzentry) for HIV?

If CCR5-positive, we can use the drug.

When to reach steady state?

If drug follows first order kinetics in a one-compartment distribution model (the drug is rapidly and evenly distributed throughout the body) and if a loading dose has not been given, it takes * 5 half-lives to reach steady state

What are the recommended order if patient taking several medications?

If patient is taking several medications, the recommended order is as follows: bronchodilator, hypertonic saline, dornase alfa, chest physiotherapy, then inhaled antibiotics.

What is the relationship between HTN and HF?

If you have hight blood pressure, this means that your heart has to work harder to push blood around the body. To cope with this extra effort, your heart becomes thicker and stiffer which makes it less able to do the job.

What is the time frame for imaging to be interpreted?

Imaging should be interpreted within 45 minutes of the patient's arrival in the emergency department by physician with expertise in reading the studies.

Reticulocyte count

Immature red blood cells: test can determine if the bone marrow production of RBCs is acceptable

What is the difference between Single-blind trial and Double-blind trial?

In a SBT, generally the subject (the patient) is unaware of the treatment allocation, whereas the investigator is aware of the treatment the patient is receiving. In a DBT, nether the subjects nor the researchers know who is receiving active drug or placebo.

which beta2 agonist LABA used daily?

Indacaterol (Arcapta Neohaler) > DPI: 75 mcg capsule via Neohaler device daily Vilanterol/fluticasone (Breo Ellipta) > DPI: 1 inhalation daily Olodaterol (Striverdi Respimat) > MDI: 2 inhalations daily

What is the MOA of MAO-I?

Inhibit the enzyme monoamine oxidase, which breaks down catecholamines, including 5-HT, NE, EPI, DA. If these NTs increase dramatically, hypertensive crisis and death can result.

List only 1 important counseling about Granules, powders or capsules that can be opened and sprinkled into soft food or water?

Instruct patient not to chew any long-acting pellets or beads that are emptied out from a capsule, not to let the mixture sit too long (take within the time directed) and no to add anything warm or hot (the contents will dissolve too quickly) * To swallow with out chewing requires that the drug be placed in a small amount of soft food *

List the high alert medications?

Insulin Oral Hypoglycemia Opioids Anticoagulants Antiarrhythmics Anesthetics Chemotherapeutics Injectable KCL Phophate Magnesium Hypertonic saline *SEDATIVE*

Which drugs cause hypoglycemia?

Insulin is the #1 drug that can cause hypoglycemia. Drugs that make the body secrete more insulin such as sulfonylureas and meglitinides (insulin secretagogues) are also high risk for causing hypoglycemia. Pramlinitide is high risk since it is used concurrently (but injected separately) with insulin at mealtime.

What are the symptoms of hyperkalemia?

Insulin regulate potassium by causing potassium to shift into the cells Symptoms: Muscle weakness Bradycardia Fatal arrhythmias may develop

What is the difference between Intravascular administration and Extravascular administration?

Intravascular administration is where the drug is placed directly into the blood either intravenously or intraarterially. Extravascular administration include Oral (PO), sublingual, buccal, intramuscular, subcutaneous, dermal, pulmonary, topical ocular, intraocular and rectal

List few tools used to estimate kidney function

Inulin or radioactive substance (limited use due to $$) BUN: If high then kidney damage but can be high due to dehydration Creatinine

What is Cost-Effectiveness Analysis?

Is defined as a series of analytical and mathematical procedures that aid in the selection of a course of action from various alternative approaches. * The main advantage of this method is that the outcomes are easier to quantify when compared to other analyses, and clinicians and decision makers are familiar with these types of outcomes since they are similar to outcomes seen in clinical trials and practice.*

Define Revascularization?

Is the restoration of perfusion to a body part or organ that has suffered ischemia. It is typically accomplished by surgical means.

What is Pharmacogenomics?

Is the science which examines inherited variations in genes that dictate drug response and explores way that the variations can be used to predict wether the patient will have a good, bad, or no response to a drug.

What is Ampyra used for?

It is a potassium channel blocker, may increase nerve signal conduction. - The generic name for Ampyra is Dalfamperidine. - Contraindication > History of seizures, CrCl < 50ml/min

What are the formulation that Olanzapine comes in?

It is an antipsychotic with various formulation available: immediate-release (IR) tablet, oral disintegrating tablet (ODT), short-acting injection, and long-acting injection

What is statistical power?

It is used to avoid type II error SP = 1 - B A higher statistical power means that we can be more certain that the null hypothesis was correctly rejected

Important about dosing methadone

It should be started at very low doses of * no more than 2.5 mg PO BID or TID *, and escalated slowly

What was one significant thing about ACCORD lipid study?

It showed no significant difference in experiencing a major cardiac event between patient treated with fenofibrate plus simvastatin compared with simvastatin alone

What is indicated for urgent reversal of warfarin?

Kcentra is a newer product available as a single-use vial containing coagulation factors II, VII, IX and X and antithrombotic proteins C and S as a lyophilized concentrate.

If patients received a LD of theophylline within past 24 hours. What will be the LD?

LD = Cp - Co (Vd) where Vd = 0.5 L/kg Cp = desired theophylline concentration Co = initial theophylline concentration

You mentioned 2 drugs (lactulose and Rifaximin) which one is the first line treatment for HE?

Lactulose is first line therapy for both acute and chronic (prevention) therapy, followed by rifaximin

Sulfonylureas (SUs) What is glimepridine (Amaryl) dose?

MOA: Sulfonylureas work by stimulating insulin secretion from the pancreatic beta cells. Do not use with meglitinides due to similar MOA 1st generation (do no use) > ChlorproPAMIDE (Diabinese) > TOLAZamide > TOLBUTamide 2nd generation > Glipizide (Glucotrol, Glucotrol XL, Glipizide xl) > Glimepiride (Amaryl) > Glyburide (Diabeta) [+metformin: Glucovance] CI > Type 1 diabetes, diabetic ketoacidosis, concurrent use with glyburide WARNING > Sulfa allergy SE > Hypoglycemia, weight gain DOSE > 1-2 mg daily (maximum is 8mg daily)

Which drugs used for high cholesterol are potential for hepatotoxic?

Many of the drug classes used for cholesterol management are potentially hepatotoxic > Statins > Niacin > Potentially fibrates > Zetia

What is the initial treatment of tremor in younger patients?

May be treated with anticholinergic > They can tolerate the side effects unlike elderly who can't tolerate. Amantadine MOA-I also an option to be used initially for mild benefit - Benztropine (Cogentin) - Trihexyphenyidyl

What is Mean?

Mean is the average value of a data set. It is calculated by adding up the value in a list, and dividing by the numbers of value present.

What is anti-switching rule for CMS?

Medicare has an anti-switching rule that prohibits suppliers from encouraging patients to switch glucose meters or test strips. If the pharmacy does not have the test strip that the meter requires then the patient can ask about alternative brands but the pharmacy (or any other supplier) cannot initiate the conversation

Simeprevir (Olysio) for hepatitis C. Which testing required/recommended?

NS3 Q80K = if negative use it.

What is the treatment for Dematophytes?

Nail bed infections: Itraconazole, terbinafine, or fluconazole (confirm fungal infection prior to treatment)

What is the ANTIDOTE for opioid? legal and illicit (heroin)?

Naloxone (Narcan, Evzio)

Which one is the active component of Nebulized racemic epinephrine?

Nebulized epinephrine is a 1:1 mixture of dextro (D) isomers and levo (L) isomers (the L-isomer is the active component)

What is the difference between No and Ha

No = we want to reject Ha = we want to proof

What is the role of Beta Blocking Agents in HTN?

No longer recommended unless patient has (S/P MI, HF, others)

Do you need to make adjustments using formulas for protein-bound drugs if *Free* levels of these highly protein bound drugs are given (Ca , Valproate, Phenytoin)?

No.

My skin is irritating because of the patch.. can i apply hydrocortisone before the patch?

No. applying hydrocoristone before the patch wil prevent the patch from sticking well. You can apply OTC hydrocortisone after the patch.

Potassium

Normal Range 3.5 - 5 mEg/L Increase due to ACE inhibitors, ARBs, aldosterone receptor antagonist (ARAs), aliskiren, NSAIDs, cyclosporine, tacrolimus, everolimus, mycophenolate, potassium supplements, drospirenone-containing oral contraceptives, sulfamethoxazole/trimethoprim, chronic heparin use, canagliflozin, and pentamidine Decrease due to corticosteroids, beta2 agonist, Diuretics, insulin

Can you recommend NSAID after heart attack?

Not a contraindication, but NSAIDs (except aspirin) are unsafe after a heart attack; try to avoid use. Avoid with heart failure.

Can you cut Fentanyl?

Not advisable since prescribed doses can change and medication errors can happen.

What is the drug of choice for Intestinal gas for Children <12 months old

Simethicone drops

What is the difference between Spironolactone and Eplerenone?

Spironolactone is a non-selective aldosterone receptor blocker (also blocks androgen). Eplerenone is a selective aldosterone receptor blocker and does not exhibit the endocrine side effects. > These agents get ride of Na and H2O and conserve K and H ions.

CF provide best environment for bacteria which cause lung infections. What are the common organisms?

Staphylococcus aureus H.influenzae P.aeruginosa

Why we taper patient on steroids who use it for longer than 14 days?

Steroid taper, which is used to wean patients off therapy by a steady reduction in dose for patients using steroids for longer than 14 days. Some package inserts state to taper if used longer than 10-14 days. * This taper is needed due to suppression of the hypothalamic-pituitary-adrenal (HPA) axis and is designed to give the patient's body time to increase its own endogenous cortisol production which would have decreased during the extended period of systemic steroid use.

List some drugs that are associated with photosensitivity? Photosensitivity can cause rashes, fever, fatigue, joint pain, and other symptoms in people with both cutaneous (skin) and systemic lupus.

Sulfa-antibiotics NSAID Tetracycline Oral and Topical Retinoids Tacrolimus Cyclosporine Voriconazole Griseofluvin Diuretics, thiazides and loops Amiodarone Chloroquine Coal Tar QUinine Fluoroquinolones Topical fluorouracil St John's wort Tigecycline

What is the ANTIDOTE for Anticholinergic overdose (Atropine, Diphenhydramine, Dimenhydrinate, Atropa belladonna (deadly nightshade), Jimson weed, scopolamine ?

Supportive care , or Physostigmine

What is the difference between Syrups and Elixirs?

Syrups (sugar base) Elixirs (Alcohol base)

T or F. Pharmacoeconomic studies serve to guide optimal healthcare resource allocation, in a standardized and Evidence based manner.

T

What is the typical onset of HIT?

The typical onset of HIT occurs 5-14 days after the start of heparin or within hours if a patient ha been exposed to heparin within the past 3 months.

List the 5 classes or types of ointment bases?

Oleaginous bases such as white petrolatum Absorption bases (such as lanolin, Aquaphor) W/O emulsion bases O/W emulsion bases (hydrophilic ointment, Dermabase) Water soluble or water miscible bases (PEG ointment)

Which fibrate has the indication to be used with statin?

Only Trilipix has the indication for use with a statin

In some patients chronic opioid use can worsen pain sensitivity, a phenomenon known as ....

Opioid hyperalgesia What is Tolerance? is a person's diminished response to a drug, which occurs when the drug is used repeatedly and the body adapts to the continued presence of the drug.

* Important about Hydromorphone *

Opioid-naive patients should start with no more than 2 to 4 mg orally or 1 to 1 mg IV by injection every four to sexy hours

Define Ostomy?

Ostomy refers to surgery that diverts part of the bowel through the abdominal wall to the skin to collect urine or stool in a pouch (plastic bag)

Where does Thiazide type diuretic works?

Thiazides and Thiazide-type diuretics inhibit Na+ reabsorption in the *distal convoluted tubules* of the nephron causing increase in excretion of Na and water as well as K and H ions.

Cohort Study

This study type follows the cohort (who share a common characteristics) over time (longitudinal) and the outcomes are compared to a subset of the group who were not exposed to the intervention such as a drug (e.g. the Framingham studies) Cohort studies may be prospective in design (carried out into the future) but can be done retrospectively as well (by reviewing patient medical charts)

What cause stress ulcer in ICU?

Patient with critical illness have reduced blood flow to the gut as blood flow is diverted to the major organs of the body. This results in breakdown of gastric mucosal defense mechanisms including prostaglandin synthesis, bicarbonate production and cell turnover.

What are the common side effects associated with Dihydropyridine CCBS?

Peripheral edema, fatigue, dizziness, headache, palpitations, flushing,tachycardia/reflex tachycardia, hypotension, *gingival hyperplasia* NOTES > Covera HS, Adalat CC, Sular have capsular shells that can be seen in feces (ghost shells) > Cardene IV require light protection during administration

What does Pharmacoeconomic research identifies?

Pharmacoeconomic research identifies, measures, and compares the cost (direct, indirect and intangible) and consequences (clinical, economic and humanistic) of pharmaceutical products and services.

What is the difference between Pharmacokinetics and Pharmacodynamics?

Pharmacokinetics is what the human body does to the drug and pharmacodynamics is what the drug does to the human body

Monophasic Action Potential (Cardiac Muscle Cell)

Phase 0 - Class 1 agent works Phase 1 - Na+ channels close - Phase 2 - Ca+2 channel open (entering cell) - K+ channels open (exiting cell) > class 2 and 4 works - Phase 3 > Ca+2 close , K+ continue exiting > class 3 works - Phase 4 > Slow increase in potential (Automaticity)

List the oral non-selective MAO-I?

Phenelzine Tranylcypromine Isocarboxazid > Due to a safety concerns (risk of D-D interactions and D-F interactions) their use is restricted to patients unresponsive to other treatment.

What is the drug used if vesicant develops while patient received Dopamine, Norepinephrine, and epinephrine?

Phentolamine

BIG INDUCERS

Phenytoin Smoking Phenobarbital Primidone Oxcarbazepine Rifampin (and rifabutin, rifapentine) Carbamazepine (and it is auto-inducer) St. John's wort

Important about phosphorous in Chronic Kidney Disease

Phosphorous is eliminated by kidney, if the kidney is not working then you have high levels of phosphours and the more phosphours you have the more calcium you need so this means calcium will be absorbed by the bone and this will cause bone metabolism abnormalities. So to solve this issue which is due to high phosphorous in blood, we need to get ride of phosphorous.

What is the ANTIDOTE for Warfarin?

Phytonadione (vitamin K) (Mephyton)

Treatment for PF? Pirfenidone (Esbriet) and Nintedanib (Ofev)

Pirfenidone (Esbrit) > Photosensitivy warning plus increse in LFT Nintedanib (Ofev) > Increase in LFT and may increase bleeding and cause thromboembolic event (MI) > Do not cause or chew > Hazard

Study TIP for the doses

Please Let Green Brown Bring Peaches TO Your weeding Pink 1mg Lavender 2mg Green 2.5mg Tan/Brown 3mg Blue 4mg Peach 5mg Teal 6mg Yellow 7.5mg white 10mg

Which immunizations are recommended for Chronic Stable Angina?

Pneumovax + Annual influenza vaccine

What is the difference between SNP and SV?

Polymorphism most commonly involve variations at a single base pair within the DNA. This is called a single nucleotide polymorphism (SNP). Polymorphism also involve larger stretches of DNA called structural variations (SV).

Which poor metabolizers will have higher carisoprodol concentrations?

Poor 2C19 metabolizers will have higher carisoprodol concentrations (up to 4 fold)

ACT to monitor what?

To monitor anticoagulation in the cardiac catheterization lab during percutaneous coronary intervention (PCI) and in surgery

Anti-Xa To monitor what?

To monitor low molecular weight heparin (LMWH) or Heparin (prefer aPTT) or Fondaparinux - Monitoring is recommended in pregnancy, mechanical heart valves, and possibly in obesity, low body weight, pediatrics, elderly, or renal insufficiency

Platelet to monitor what?

To screen to abnormal levels; platelets are required for clot formation - Decrease due to heparin, LMWHs, fondaparinux, glycoprotein IIB/IIIA receptor antagonist, linezolid, valproic acid, chemotherapy

Lopinavir + Ritonavir (Kaletra)

Treatment Naive > 800mg lopinavir/200mg ritonavir daily or 400/100mg BID > Take oral solution with food. Take tablets without regard to meals > Solution: Refrigerate. Good for 2 months if left at room temperature. Contain 42% alcohol. SE > PR and QT prolongation, Increase in LFT, rash, URTI, ec Notes * Avoid once daily dosing with carbamazepine, phenytoin, phenobarbital and in pregnant women*

Signs and symptoms of Serotonin Syndrome

Tremor , Agitation , confusion, Hallucination Tachycardia, sweating *Diarrhea* Muscle rigidity , shivering

T or F. All patients presented with ACS symptoms should receive a beta blocker within 24 hours of presentations.

True

if Mg2+ is < 1 mEq/L with life-threatening symptoms (seizure, or arrhythmias) IV replacement is recommended. if Mg2+ is < 1 mEq/L with-out life threatening IM or IV is ok if Mg2+ is > 1 and less than 1.3 PO is ok and magnesium replacement regimens should continue for 5 days to fully replace body stores

True

T or F. All antiepileptic drugs require MedGuide.

True - " Like other anti-epileptic drugs, this medication may cause suicidal thoughts or actions in a very small number of people about 1 in 500 "

T or F. If the patient has a P.aeruginosa infection, 2 drugs given IV are recommended to provide potential synergy and prevent resistance.

True - Aminoglycoside - Beta-lactams - Quinolones

T or F. Glucagon (GlucaGen) is only used if the patient is unconscious or not conscious enogh to self-treat the hypoglycemia.

True.

T or F. If SD is small, we can conclude that the drug being studied had a similar effect on most subjects.

True.

T or F. pharmacogenomic testing can identify wether an individual is homozygous or heterozygous for a specific SNP and is used to predict what type of response a patient may have to a drug.

True.

T or F. There is no drug absorption when drug is administered via *intravascular * route.

True. The drug is placed directly in the systemic circulation.

T or F. Warfarin is recommended in PAH. If yes, what is the target INR?

True. The target INR is 1.5 - 2.5

T or F. Mean, Mode, Median usually have same # or close to that #.

True. usually close to that number

T or F. It is safe not to use ointment paper to prepare creams and lotions because these two formulation have the highest content of water. Water content will cause the paper to moisten, and possibly tear.

True. we can use Glassine weighing paper

What are the 4 types of hypersensitivity reaction?

Type 1 reaction are immediate (15-30 minutes of exposure). The severity range from minor inconvenience to death Type 2 reaction occur minutes to hours after exposure. Example hemolytic and thrombocytopenia Type 3 reaction is immune-complex reaction occur at 3-10 hours after exposure example: lupus and serum sickness Type 4 reactions are delayed hypersensitivity reactions. they can occur anywhere from 48 hours to several weeks after exposure. example PPD skin test for tuberculosis

Is it UA , NSTEMI or STEMI

UA: + Symptoms - Cardiac enzymes - ECG changes NSTEMI: + Symptoms + Cardiac enzyme - ECG changes STEMI: + symptoms + Cardiac enzyme + ECG changes > ST segment elevations in at least 2 contiguous leads of > 2mm in men and > 1.5 mm in women in leads V2-V3

Drug interactions that can decrease efficacy of Hormonal Contraceptives

Use back up while taking the antibiotics listed > Rifampin: Use other form of birth control since the induction will last - if switching back, a back-up method needs to be used for 11/2 months after rifampin is stopped. > Rifapentine and rifabutin are also strong inducers > Depo-Provera does not have drug interactions (although it does lower bone density and should be avoided in women at risk of osteoporosis)

When to use Median or Mean?

Use mean if you have a lot of #s or data is not skewed. When the data is skewed median value should be used. Small #s use the median.

What are the Warning and SE and Notes associated with Ranolzine?

Warning > Can cause QT prolongaton > Acute renal failure has been observed in some patients with CrCl < 30 mL/min SE > Dizziness, constipation, headache, nausea NOTES > Has little to no clinical effects on HR or BP

Anticholinergic (WARNING - SIDE EFFECTS - MONITORING - NOTES

Warning > Use caution in patients with MG, narrow-angle glaucoma, urinary retention, benigin prostatic hyperplasia, bladder neck obstruction SE - Dry mouth (much more common with tio) , upper respiratory tract infections, nasopharyngitis, sinusitis, cough and bitter taste - Notes > Avoid spraying in the eyes > Do NOT swallow capsules of tio > Combivent >> discard 3 months from when cartrigde is inserted into device > Tudorza > Discard product 45 days after opening pouch, when device locks out, or when dose indicator displays "0", whichever comes first.

*Systematic Reviews and Meta-Analysis*

When there are many studies available in the literature, a systemic review and possible meta-analysis is useful to summarize the main findings in order to guide evidence based medical decisions.

What is the treatment option for acute SLE?

PredniSONE (or methylPREDNISolone) > Used acutely to control flares at higher doses; taper to lower doses for chronic, suppressive therapy.

Where is the preferred area to apply vivelle-Dot?

Preferred site is Lower abdomen but can apply also to Buttock

What is the DOC for Toxoplasma gondii?

Preferred: TMP/SMX 1 DS tab PO daily Alternative: TMP/SMX 1 DS PO TIW or 1 SS PO daily or Dapsone 50mg po daily + Pyrimethamine 50mg po weekly + leucovorin 25mg po weeky

which of the following are contraindicated in pregnancy? A. nicotine gum B. nicotine patch C. nicotine nasal D. all of them FYI: nicotine gum and lozenges are category C

Pregnancy is contraindicated in all nicotine replacement therapy

Which population is the only one that is recommended to get primary prevention of stroke?

Primary prevention of ischemic stroke is only recommended for patients with A-FIB.

Important screen for patients with non-life threatening arhythmias

Prior to starting any medication for non-life threatening arrhythmias, be sure to always check the patient's electrolytes and run toxicology screen.

What kind of drugs dose big inducer provide highly active drug ?

Prodrugs Inducer can increase an enzyme that is responsible for converting the substrate into a *more* active form (instead of a *less* active or inactive form).

List some drugs that are micronized to increase dissolution rate?

Progesterone Fenofibrate (Antara) (micronized to treat hypertriglyceridemia)

What is the prophylaxis treatment for cystitis in females of child bearing age ?

Prophylaxis: >= 3 episodes in 1 yr; use 1 SMX/TMP SS daily, nitrofurantoin 50mg PO daily or 1 SMX/TMP DS post coitus.

What is the ANTIDOTE for Heparin, LMWH?

Protamine > 1mg will reserve around 100 units of heparin and is used as a reversal agent for LMWH

What is home testing kits?

Provides convenience and privacy. There are many including kits that test for HIV infection, herpes, and urine drug analysis kits that identify the presence of illicit substances and opioid.

What is Point-Of-Care testing?

Provides results obtained right at the site of the patient care and include many tests: A1C, cardiac enzyme, INR, various infection diagnosis kits and others.

What is the ANTIDOTE for Isoniazid?

Pyridoxine (vitamin B6) > Oral Pyridoxine 10-15 mg is used daily with isoniazid to prevent neuropathy and in high IV dose to treat toxicity

What is the first line recommendation by ACOG for OTC product that treat N/V?

Pyridoxine which is vitamin B6 with or withou doxylamine - Natural product maybe helpful is ginger, in tea form or cooked but do not recommend supplement

Which antipsychotic medication cause the least QT prolongation?

QT prolongation (lower risk) (maybe seroquel) - double chekc

What is the relative risk for this example: Intervention group (4 patients on drug x with 1 patient experiencing pain) and Control group (6 patients with 5 patients experiencing pain) ?

RR = 0.25 / 0.83 = 0.3 if asked % = 30%

What makes you have a false negative in diagnosis of H.pylori?? Fecal antigen test

Recent use of H2RAs, PPIs, bismuth, or antibiotics > discontinue these drugs at least 2-4 weeks prior to test

How to calculate the Relative Risk Reduction (RRR)?

Relative Risk reduction measures how much the risk is reduced in the treatment group compared to control group. It is calculated by using the absolute risk reduction by the control group risk rate or by *subtracting the relative risk as decimal from 1 (1-RR)* RRR = (% risk in control group - % risk in treatment group) divided by % risk in control grup Ex. 28% - 16% / 28% = 43% . Patients taking metoprolol are 43% *less likely* to experience heart failure progression.

Patient called pharmacy saying their child got an eye drops in their eye and don't know what to do?

Remove contact lenses Rinse eye or eyes with water from tap or hose with a gentle stream for at least 15 minutes

What is NNH

Same calculation as above BUT *ROUND DOWN*

What is status epilepticus?

Seizures that last longer than 5 minutes or 2 or more seizures between which the patient does not regain consciousness are called status epilepticus, which is a medical emergency.

Which nutrients are depleted when patient taking Lamotrigine?

Selenium Zinc due to alopecia

Which nutrients are depleted when patient is on long therapy with Valproic aicd /Divalproex?

Selenium Zinc Calcium/Vitamin D

What is the difference between Sensitivity and Specificity?

Sensitivity and Specificity are concepts often applied to diagnostic testing for diseases. Sensitivity is the proportion of time a test is positive in patients who have the disease. A test with 100% sensitivity will recognize all patients with the disease by testing positive. Sensitivity is the percentage of *true-positive* results and is equal 1-type II error. Specificity is the proportion of time a test is negative in patients who do not have the disease. A test with 100% specificity will read negative and accurately exclude disease from all healthy patients. Specificity is the percentage of *true-negative* results and is equal to 1-type I error. Ex. If patient wants to buy HIV kit from walgreens. Is it important to buy test A. more sensitive B. more specific (answer is more sensitive)

When does Serotonin syndrome occur?

With administration of one or more serotonergic medications (and higher doses increase risk) but is most severe when an MAO I is administered with another serotonergic medication)

If given a figure on the exam and it was bell-shaped then is this normal distribution?

Yes Bell-shaped curve means they have normal distribution. Bell-shaped curve means 95% of the population are +/- 2SD from the mean

IF the 95% confidence intervals for each of the risk ratio exclude 1, can we reject null hypothesis?

Yes.

Is it safe for pregnant women to receive an influenza vaccine that contains thimerosal?

Yes. A study of influenza vaccination examining over 2,000 pregnant women demonstrated no adverse fetal effects associated with influenza vaccine. Case reports and limited studies indicate that pregnancy can increase the risk for serious medical complications of influenza. One study found that out of every 10,000 women in their third trimester of pregnancy during an average flu season, 25 will be hospitalized for flu related complications.

* Important PI side effects *

- Hyperglycemia, insulin resistance, new-onset diabetes (highest risk with indinavir, lopinavir/ritonavir) - Fat maldistribution: lipohypertorphy - Hyperlipidemia (lowest risk with atazanavir and darunavir: increased TG highest with lopinavir/ritonavir and fosamprenavir - Hepatitis and hepatic decompensation (highest risk with tipranavir) - Bleeding events (in patients with hemophilia) - Increased CVD risk (lowest with atazanavir and darunavir) - EKG changes (especially saquinavir, lopinavir/ritonavir, and atazanavir)

Anthyrcyclines

- Serial moniotring of cardiac output is necessary with anthracycline doses exceed 250mg/m3 doxorubicin or 320 mg/m3 daunorubicin - Do not exceed max lifetime doxorubicin 450-550 (450 with mediastinal radiation due to cardiotoxicity) - The cardioprotective agent dexrazoxane (Zinecard) should be considered when cumulative doxorubicin doses > 300mg/m2 - MitoXantrone (Novantrone) > An anthracenedione similar to anthracyclines in toxicity and turns body fluid blue rather than red as with other anthracyclines

Risk factors for COPD

- Smoking or smoke exposure - Alpha-1 antitrypsin deficiency - Occupational dusts and chemicals - Indoor an Outdoor air pollution

List some natural products used for depression

- St. John's wort or SAMe (S-adenoysl-L-methionine) > St. John's wort is a broad spectrum CYP 450 enzyme inducer and has many significant drug interactions > It is also a photosensitizer and is a sertotonergic. > > Cause with other 5HT - L-methylflolate (Deplin) is a medical food product being used for depression

Who should receive myeloid growth factor?

- The NCCN recommends all patients with > 20% chance of developing chemotherapy induced ferbrile neutropenia to receive myeloid growth factors

What is Suvorexant (Belsomra) used for?

A new agent with a unique mechanism: It blocks the orexin neuropeptide signaling system, which is invovled with promoting wakefulness.

When do you want to use Benzodiazepines?

- They provide fast relief 30 mins to 2 hours - Appropriate in short term situations where anxiety is acute and can cause extreme stress, prevent proper sleep, and disrupt life. > Ex. Death of loved one, earth quake, motor vehicle accident, etc. - Should use it for less than 1-2 weeks and then discontinued. - Should not be used in long term - Do not use in elderly > confusion, dizziness, and fall

Notes about Sulfanomides

- To avoid confusion mg/kg doses should alway reference to trimethoprim component - Bactrim IV - Store at room temp, dilute with D5W. Require light protection during administration - Bactrim suspension should be stored at room temp and protected from light - IV to PO ratio is 1:1 - Caution with concurrent use of warfarin - Sulfamethoxazole: Trimethoprim dose is always a 5:1 ratio

OTC for GERD/Heartburn/Gas Pains for pregnancy

1st recommend to eat smaller, more frequent meals, avoiding foods that worsen GERD, and if symptoms occur while sleeping, recommend elevating the head of the bed and not eating three hours prior to sleep - If this does not work, calcium antacids are first line, such as calcium carbonate in Tums or store brands. This is a good antacid choice since calcium intake is often deficient in pregnancy - If gas is a concern, simethicone is considered safe (Gas X, Mylicon) and the Mylicon infant drops are safe for infants - H2 antagnist are all pregnancy category B and many doctors recommend OTC or Rx doses - PPI are B's or C's

What is the drug of choice for ICU patient who experiencing Delirium?

1st. we need to control patient's environment (light, noise, stimuli) to decrease incidence of delirium, but no medication is recommended to prevent. > Quietiapine which is mildly sedating and has little risk for movement disorder can be useful. > * providing sedation with dexmedetomidine as opposed to benzodiazepines may decrease incidence of delirium and shorten the duration in patient who already have it *

Fill in the blank. For traditional multiple daily dosing of gentamicin and tobramycin the peak and trough goals are int he ranges of ........ and ...... respectively.

5-10 mg/L and < 2 mg/L respectively

When to administer Platelet transfusions?

< 10,000/mm3 < 20,000/mm3 if active bleed is present

Vaccinations

< 2 years - Prevnar 13 - H.influenza vaccine >= 2years - Pneumovax x 2 > at least 2 months apart from prevnar 13 and at 5 years - Meningococcal conjugate vaccine x 2 > Given between 2-6 years and 5-9

What is the goal for patients with history of severe hypoglycemia, limited life expectancy, extensive comorbid conditions, advanced complications, or longstanding diabetes where the goal is difficult to attain despite optimal efforts.

< 8%

List some commonly used bases in suppositories preparations?

cocoa butter (theobroma oil) Glycerin Hydrogenated vegetable oils Polyethylene glycol (Carbowax)

Dabigatran BBW CI SE NOTE

- BBW (2) > Discontinuing dabigatran puts patients at increase risk for thrombotic events. If dabigatran must be discontinued for reasons other than pathological bleeding, consider the use of another anticoauglant during the time of interruption > Patients receiving neuraxial anesthesia or underoign spinal puncture are at risk of hematomas and subsequent paralysis CI - Active bleed and patient with mechanical prosthetic heart valve SE - Dyspepsia, gastritis-like symptoms, bleeding (including more GI bleed) Note - Preg. C - Cause increase in aPTT, PT/INR -D/C 1-2 days before invasive surgery if normal renal - D/C 3-5 days if CrCl < 50ml/min

Pregabalin (Lyrica)

- C-V - SE > Dizziness, somnolence, peripheral edema, weight gain, ataxia, diplopia, blurred vision, dry mouth, mild euphoria - Preg C

T or F. Immunosuppressive agents can be used if patient fails above therapy or they can be used in combination with above therapies; often called immunomodulators or steorid sparing therapies

k

Why preservative-free formulations if there is no harm from multi-dose vials?

no scientifically conclusive evidence exists of harm from exposure to thimerosal preservative-containing vaccine, whereas evidence is accumulating of lack of any harm resulting from exposure to such vaccines. Therefore, the benefits of influenza vaccination outweigh the theoretical risk, if any, for thimerosal exposure through vaccination" (MMWR 54 [RR08]: 1-40, 2005). Nonetheless, FDA is in discussions with manufacturers of influenza vaccine encouraging them to further increase the supply of preservative-free formulations.

What is the cornerstone of heart failure therapy?

- Diuretics to control fluid volume - ACE-I / ARB and beta blockers to delay or half the progression of cardiac dysfunction and improve survival - The above medications should be utilized in all heart failure patients who do not have a contraindication or intolerance to their use - Digoxin might be provided later to help with symptoms - Survival advantages [aldosterone receptor antagonist and/or hydralazine/nitratel]

Signs and Symptoms (S/Sx)

A symptom is subjective information, described by the patient, such as "my lower back hurts" A sign is generally objective (described by the clinician), such as recording the patient's vital signs * Occasionally objective evidence (such as a skin rash), can be seen by either the patient or family members or the clinician *

What is the difference between Systematic Review and Meta-Analysis?

A systematic review is a structured literature review that uses a step by step protocol with preset criteria for selecting and rigorously evaluating studies Meta-Analysis is a statistical technique that can be used to combine results from multiple studies to develop a single conclusion that has greater statistical power than is possible in the individual smaller studies

How can you educate the patient who CC: my patch is bothering my skin. What can i do ?

A. check if the patient is alternating the application site B. The skin should not be shaved shortly before applying; shaving is irritating to the skin C. In some cases a topical steroid, such as hydrocortisone (OTC) can be applied *after* the patch is removed. * If hydrocortisone is applied before application it will prevent the patch from sticking well*

The non-pharmacologic and pharmacologic treatment of Chronic Angina

A: Anti-platelet and Anti-angina drugs B: Blood pressure and beta blockers C: Cholesterol (statin) and cigarettes (cessation) D: Diet and Diabetes E. Exercise and Education

When to administer anti-emetics for chemotherapy patient?

Administer anti-emetics at least 30 minutes prior to chemotherapy and provide take-home anti-emetics medications

List a recombinant humanized monoclonal antibody used in MS?

Alemtuzumab (Lemtrada) > 2 course treatment 12 months apart BBW (4) 1. Serious (sometimes fatal) autoimmune conditions (ITP), infusion reactions, and malignancies. Available only through the Lemtrada REMS CI HIV (cause prolonged decrease in CD4 count) SE Infections (herpes viral infections, etc) Thyroid gland disorders , etc Notes > Indicated for those with inadequate response to >= 2 MS drugs > Complete all vaccinations 6 weeks before therapy > Pre-medicate with 1 gram methlyprendisolone (or equivalent) immediately prior to infusion and for the first 3 days > Start antiviral prophylaxis on first day of each course and continue 2 months or until CD4 count > 200 (whichever is later)

What is used to treat Bacterial Meningitis in Neonates and up 2+ plus?

Always think about gram - and gram + For gram + we use Vancomycin For gram - we use 3rd generation cephalosporin such as Ceftriaxone or Cefotaxime -> We don't use Ceftriaxone in neonate because of it displace bilirubin in albumin and cause bilirubin induced brain damage and also risk of embous.

List Aromatase inhibitors

Anastrozole (Arimidex) Letrozole (Femara) Exemestane (Aromasin) Mitotane (Lysodren) -> Decrease bone density and increase risk for osteoporosis; consider calcium and vitamin D supplementation, weight bearing exercise, DEXA screening -> Increase in cardiovascular disease compared to SERMs. -> Mitotane require glucocorticoid and mineralcorticoid supplementation. CI -> Pregnancy SE - Arthralgia, edema, lethargy/fatigue, rash, menopausal, hot flashes, hepatotoxicity, nausea, vomiting, weakness, joint pain, bone pain, HTN, depression.

Beta blockers - Bisoprolol and Metoprolol

BBW - Beta blockers should not be withdrawn abruptly (particularly in patients with CAD), gradually taper over 1-2 weeks to avoid acute tachycardia, HTN, and/or ischemia. CI Sinus bradycardia 2nd or 3rd degree block, sick sinus syndrome, cardiogenic shock; do not inititate in patient with active asthma excacerbation SE - Decrease in HR, hypotension, fatigue, dizziness, depression, etc. Monitoring - HR, BP (titrate over 2 weeks as tolerated); decrease dose if HR < 55 BPM; signs and symptoms of HF, renal function, liver function Notes - Metoprolol tartrate IV does is not equivalent to oral IV:PO 1:2.5

What are the important points about cyclosporine?

BBW *7* - Renal impairment (with high doses), increased risk of lymphoma and other malignancies, increased risk of skin cancer, increased risk of infection, may cause hypertension, dose adjustment should only be made under the direct supervision of an experienced physician. Cyclosporine (modified- Gengraf/Neoral) has increased bioavailability compared to cyclosporine (non-modified-Sandimmune) and cannot be used interchangeably. SE > Hypertension, nephropathy, hyperkalemia, hirsutism, gingival hyperplasia, edema, hyperglycemia, QT prolongaton. etc. Monitoring > Trough levels, etc

What is the difference between Crystalloids and Colloids?

Crystalloids pass freely between semipermeable membranes. Most of the administered volume does not remain in the intravascular space (inside the blood vessels). It goes into the extravascular space or interstitial space. They cost less and have fewer adverse reactions than colloids. * sepsis require balanced sodium restriction * Colloids AKA plasma expanders are large molecules (typically protein or starch) dispersed in solutions that primarily remain in the intravascular space. Colloids provide greater intravascular volume expansion than equal volumes of crystalloids, but are more expensive and provide questionable clinical benefit over crystalloids.

Important interaction between Cyclosporine and Tacrolimus

Cyclosporine will decrease mycophenolate and increase sirolimus and everlimus (and will incresase some of the statins, which transplant patients are usually taking) and the mTOR inhibitors (sirolimus and everolimus) are enzyme inhibitors and increase cyclosporine levels. Both cyclosporine and tacrolimus are CYP 450 3A4 and P-gp substrates. Inducers of either enzyme will decrease the CNI concentration and inhibitors will increase the CNI concentration. Both will interact with the majority of drugs.

What are the SE associated with Non-CCB?

Edema, HA, Dizziness, AV block, bradycardia, hypotension, arrhythmias, HF, constipation (with verapamil), gingival hyperplasia Note > Verapamil: Require light protection during administration

Fill in the blank ...................... a single agent that is used to soften and smooth the skin. A moisturizer is sometimes referred to as ................, but the term ............ is used for single agent, and moisturizers often have coloring, scents, and other ingredients added

Emollients

What is the drug of choice for Pre-Exposure prophylaxis (PrEP)/

Emtricitabine/Tenofovir (Truvada) 1 tab po daily in combination with safer sex/behavior risk reduction practices, to reduce their risk of becoming infected.

Fill in the blank ............ Is used to stabilize the emulsion. ........... are usually surfactants (wetting agents) that reduce the surface tension between the two liquids so that the two different substances can move close to each other but they will not be mixed together

Emulsifiers

What is the difference between Phase I and Phase II reaction?

Enzyme metabolism invovles phase I reactions (oxidation, reduction, and hydrolysis), followed by phase II which normally terminates the activity of the drug. Phase I provides a reactive functional group on the compound that permits the drug to be attacked by phase II enzymes)

What is the mainstay in advanced cardiac life support and anaphylactic shock?

Epinephrine

Can you cut some XL tablets?

Yes. example Toprol XL and Sinemet CR. but you can't crushed. * Note the danger if long-acting formulations are crushed: a fatal dose could be released. This include ER opioids *

What is the difference between PHASE 1 2 3 4

phase 1: Focus on *safety* and pharmacology. Low doses of the compound are given to a small group of healthy volunteers Phase 2: Examine *effectiveness* of a compound. Patient without complications and co-morbidities are often selected for a trial to reduce the number of confounding variables that may influence the trial. Phase 3: Researcher try to confirm *efficacy* of the new drug for the target indications in a *large populations* These studies usually last from 2 to 10 years and typically involve hundreds to thousands of patients across multiple sites. Phase 4: post-marketing studies.

What you know about Hepatitis B? How is it transmitted ? Is there a vaccine? Is there a treatment? Who is at high risk? Which population is at risk for Hepatitis B reactivation?

Hepatitis B virus is a vaccine preventable disease that cause acute illness and may lead to chronic infections, cirrhosis of the liver, cancer, and death - It is transmitted via contact with infectious blood, semen, or other body fluid - Many antiretroviral (NRTIs) and interferons are used for chronic therapy of HBV. - Treatment duration is not well defined - HepB reactivation is a concern in patients with prior HBV exposure undergoing certain immunosuppressive therapies and require screening, monitoring, and managing patients to minimize the risk of HBV reactivation.

Is there a vaccine for Hepatitis C? How is it transmitted? What is the treatment?

Hepatitis C virus (HCV) is a non-vaccine preventable disease that can cause acute disease, but more commonly is associated with chronic disease with consequences similar to hepatitis B - Transmission is through blood and is most commonly transmitted in the United State via IV drug use - There are 6 different genotypes (1-6) and various subtypes (for example 1a or 1B) > Genotype 1 is the most difficult to treat - Treatment include peginterferson, ribavirin, sofosbuvir, simeprevir, and the combination of ledipasvir/sofobuvir.

Which test must to be positive before we can initiate Trastuzumab (Herceptin), Ado-trastuzumab emtansine (kadcyla)?

Her2/neu oncogene -> If positive, can use drug

List some Risk factors for stroke

Hypertension A-Fib Gender (male more) Ethnicity (highest in AA) Age >= 55 years Atheroscleorsis Diabetes Transient Ischemic Attack (TIA) Prior history of stroke Smoking Dyslipidemia Patent Foramen Ovale (PFO) Sickle Cell disease

how many half-lives it take for drug to reach steady state?

It takes about 4-5 half lives

What are the monitoring parameters for Statins?

LFT as baseline and as clinically indicated thereafter; obtain a lipid panel 4-12 weeks after initiation or up titration of therapy to assess for medication adherence; then every 3-12 months thereafter

Be carfull if you want to remove the fluid from Ascites. Why?

Large volume paracentesis (removal of > 5 L) has been associated with significant fluid shifts and the addition of albumin is recommended to prevent progression to *hepatorenal syndrome*

What is the normal value of MCV and why it is important?

MCV = 80 - 100 MCV < 80 is called Microcytic anemia MCV > 100 is called Macrocytic anemia < 80 is due to Iron deficiency > 100 due to B12 or Folate deficiency

ON EXAM if patient has allergies to NSAID non-selective. can we recommend COX-2 selective.

NO!! Cox-2 selective NSAID are used clinically, but on licensing exams it may be prudent to avoid all NSAIDs

List the Beta-1 and Beta-2 blockers (Non-selective)

Nadolol (Corgard), Penbutolol (Levatol), Pindolol, Propranolol (Inderal La, InnoPran XL), Timolol NOTES > Propranolol has the highest lipid solubility (lipophilic); therefore it is associated with more CNS side effects since it crosses the blood brain barrier. CNS side effects include sedation, depression, cognitive effects and others.

List one Monoclonal Antibody that is used in MS and list the BBW associated with this drug.

Natalizumab (Tysabri) - given every 4 weeks - BBW > Risk of progressive multifocal leukoencephalopathy (PML) - monitor mental status changes. Risk factors for PML include: Anti-JC virus antibodies, increase treatment duration and prior to immunosuppressant use. CI > History of PML SE > Infusion reactions, headache, fatigue, etc NOTE Rems (TOUCH) Med-Guide

What is the best method to increase dissolution rate for drugs that has poor absoprtion?

One of the methods used to increase the dissolution rate is to reduce particle diameter, which increase the surface area. - These drugs with very small diameters are called micronized. Why called Micronized? Because the diameter was measured in micrometers. Now they have some tablets in nanometers.

Why zileuton is a less desirable alternative?

Only used in step 3 and 4 along - Less desirable due to limited studies as adjunctive therapy and the need to monitor liver function. - Theophylline require moniotring of serum concentrations levels

Important about Ospemifene?

Oral SERM indicated for dyspareunia (painful intercourse). One of the boxed warning is for risk verus benefit; this drug has VTE risk and is not indicated for mild symptoms. Only used for a short treatment period for moderate to severe symptoms.

Pregnancy Category D

Positive evidence of fetal risk is available, but the benefit may outweighs the risk if life-threatening or serious disease

Which supplements are required if patient is diabetic?

Potassium Magnesium

What is the usual cause of fatality with an opioid overdose?

Respiratory depression and sedation is the most important predictor of respiratory depression so it must be monitored especially patients on IV opioids.

Aztreonam Lysine inhalation solution (Cayston)

SE - Allergic reactions (may be severe), bronchospasm, fever, wheezing, cough, chest discomfort NOTE - doses should be taken at least 4 hours apart - Use with Altera nebulizer system - Need to reconstitute with 1 mL of sterile diluent (provided); give immediately - Recommend to refrigerate; can be kept at room temperature up to 28 days - Protect from light

Nitroclygerin

SE - HA, Dizziness, ligtheadedness, flushing, hypotension, tachyphylaxis (decrease effectiveness/tolerance), syncope Notes - Counsel patients to do the medication so they have a 10-12 hours nitrate-free period to decrease tolerance

Tobramycin Inhalation Powder (TOBI podhaler)

SE - Ototoxicity , tinnitus, voice alteration, dizziness, bronchospasm NOTE - Little systemic absorption - User with podhaler - Doses should be taken at least 6 hours apart - Store capsules at room temperature in a dry place

RisperiDONE (RisperDAL) - Also approved for irritability associated with autism

SE - Sedating - * Incresae in prolactin - sexual dysfunction, galactorrhea, irregular/missed perioids * - Orthostasis - Weight gain, increase in lipids , increase in glucose - QT prolongation NOTE > 6mg increse prolactin and EPS

GuanFACINE ER (Intuniv) Tenex - For HTN

SE - Somnolence, dizziness, headache, fatigue, hypotension, nausea, etc.

What is the ANTIDOTE for Salicylates?

Sodium bicarbonate > Salicylates are acidic; sodium bicarbonate is an alkalinizing agent

What are the target organs for cystic fibrosis?

The thick mucus mostly affects the lungs, pancreas, liver and intestine, primarily causing difficulty breathing and lung infections as well as digestive complications.

T or F. Adasuve (Loxapine) require REMS

True

T or F. Chemotherapy should be avoided during pregnancy and breastfeeding.

True

T or F. If compounding IV medications in a sterile hood and glass ampules are used, a filter needles are used to prevent particles from getting into the IV bag and a filter may be required in the line too.

True

What is the difference between stable and unstable angina?

UA is a medical emergency where the pain is increasing in frequency, intensity or duration or occurring at rest. Stable chronic angina is also knowns as stable ischemic heart disease (SIHD) is associated with predictable chest pain often brought by exertion or emotional stress and relieved within minutes by rest or with nitroglycerin.

Which benzodiazepin comes as solution?

- Alprazolam - Lorazepam - Diazepam

List some medications that can cause Diarrhea?

- Antacids containing magnesium - Antibiotics, especially broad-spectrum antibiotics and clindamycin, erythormycin (due to prokinetic activity) > rule out C. difficile infection - Colchicine - Laxatives - Metoclopramide - Misoprostol - Quinidine - Many drugs include diarrhea as possible side effect

Food Care

- At least one per year

The following hiV drugs should be taken w/o food?

- Atripla - Didanosine - Efavirenz (small aount of food ok) - Fosamprenavir (oral suspension) - Indinavir (unboosted)

List the Interferon beta-1a used for treatment for patient with relapsing forms of ms?

- Avonex - Rebif

Hemolytic anemia

- A type of anemia where RBCs are destroyed and removed from the bloodstream before their normal lifespan of 120 days. - The direct coombs test is used to detect antibodies that are stuck on the surface of red blood cells.

ARIPiprazole

- Ability discmelt ODT - Also approved for irritability associated with autism - SE - Akathisisa, anxiety, inosmina - constipation - less weight gain, some degree of QT prolongation * take in the AM *

Pain OTC fore pregnant woman

- Acetaminophen is pregnancy category C and is the analgesic and antipyretic drug of choice during pregnancy and considered safe if breast feeding

Azcyclovir (Zovirax) Valacyclovir (Valtrex) Famciclovir (Famvir)

- Dose for Acyclovir is based on IBW in obese patients - Famciclovir is prodrug of penciclovir - Warning > Thrombocytopenic purpura/hemolytic uremic syndrome (TTP/HUS) has been reported in immunocompromised patients SE > Malaise, HA, nausea/vomiting, diarrhea, incresae in SCR/BUN

Alpha/Beta Agonist: used for neurogenic orthstatic hypotension

- Droxipoda (Northera) BBW - Supine hypertension: monitor supine BP prior to and during treamtent SE - Syncope, falls, headache, UTI

What cause increase in Eosinophils?

- Drug allergies - Asthma - Inflammation - Parasite infection

What are the important information to know about Propofol (diprivan)?

- CI: Hypersensitivity to egg, egg product, soy and soy product. - Side Effects > Hypotension, apnea, *hypertriglycerdemia* , green urine/hair/nail beds, propofol-related infusion syndrome (PRIS - rare but can be fatal) - In oil-in-water emulsion (provides 1.1 kcal/mL) - Monitor: Triglycerides among other stuff - use strict aseptic technique due to potential for bacterial growth - Do not use filter of < 5 micron for administration *Should only be administered through a filter w/ a pore size of ≥5 micron* - *Do not require refrigeration*

* Normocystic anemia *

- CKD causes anemia due to deficiency in erythropoietin (EPO), a hormone that is produced by the kidneys. - Erythropoitein stimulates the bone marrow to produce RBC in response to falling levels of O2 in the tissues - Iron levels are important to asses because if they are low, ESA will be ineffective. - The majority of patients who receive iron by injection are on hemodialysis since hemodialysis results 6-7 mg of iron loss per day of dialysis and is further compounded by physiologic and venipuncture losses. - ESAs should be used at the lowest effective dose to reduce the need for blood transfusions. Start when Hgb is < 10 g/dL and reduce or stop therapy when Hgb is near 11g/dl. - Transferring saturation should be at leasts 20% and ferritin levels should be at least 100 ng/mL prior to starting ESA levels.

How Cannabinoids may work to prevent N/V?

- Cannabinoids may work by activating cannabinoid receptors within the central nervous system and/or by inhibiting the vomiting control mechanism in the medullar oblongata - Dronabinol (Marinol) > Refrigerate capsules > C-II Nabione (Cesamet) No frig CII SE - Somnolence , euphoria, Increase appetite, orthostatic hypotension

What is expected to be performed upon arrival of patient to the ER?

- Cardiovascular support and breathing support, including assisted ventilation may be required - Cardiac monitoring with an ECG will be needed with the drugs that affect heart rate or rhythm or with any cardiac abnormality present - With some drugs , specific antidotes or dialysis may be used.

Important information about Typhoid Fever

- Caused by Salmonella Typhi - Human primary source which is spread through consumption of water or food that has been contaminated by the feces of someone with acute infection or from a chronic asymptomatic carrier - Sexual contact is rate but can occur - Vaccine provide about 50-80% effectiveness, safe food and water precautions and wash hands should be followed even by person who is vaccinated - Vivotif Berna is oral, live attenuated vaccine consists of 4 capsules, 1 taken every other day. The capsules are refrigerated (not frozen)- each capsule taken with cool liquid 1 hour before a meal - Typhim VI, the injectable vaccine is one 0.5 mL dose given IM 2 or more weeks before expected exposure. - PO not used in Children < 6 years and IV not recommended for children < 2 years

Type 1 diabetes

- Caused by a cellular mediated autoimmune destruction of the beta cells in the pancreas - Family history is less of a risk factor than in type 2 - Patients may present with DKA which is life-threatening condition. - Present in younger, thinner patients - Therapy must include insulin

List soem Stimulants and Irritants

- Caution when recommending agents by brand name as many brands can refer to multiple products > Senna (ex-lax) > Bisacodyl (Dulcolax) > Cascara

What is the treatment for acute meningitis 2 years - 50 years (primarily S.pneumonaie and N.meningitdis?

- Cefotaxime or Ceftriaxone or Meropenem plus - Vancomycin plus/minus - Dexamethasone * do not use ceftriaxone in neonate * instead add ampicillin + Cefotaxime * Ampicillin is added for Listeria coverage which is common if patient age < 1 month or >50years *

Important contraindication to Cephalosporin

- Ceftriaoxone: Hyperbilirubinemic neonates (cause biliary sludging), concurrent use with calcium containing IV products in neonate <28 days years old

Folate Antimetabolites

- Cell cycle specific - S-phase: Prevent DNA synthesis Methotrexate > Trexall, Rheumatrex > High doses require leucovorin or levoleucovorin rescue to decrease toxicity > Avoid renal transport inhibitors that can decrease elimination >> Aspirin, beta lactams, probenecid and NSAIDs >> Maintain proper hydration >> Alkalinize urine with IV sodium bicarbonate to decrease toxicity BBW - Pregnancy, etc - Preserved formulation not for intrathecal administration or high dose therapy SE - Myelosuppression, mucositis, hepatic and renal toxicity (renal is dose-related), pulmonary toxicity (rare), hand-food syndrome, tumor lysis syndrome

Why bone marrow transplant can be dangerous for adults?

- Children with severe sickle cell disease have been cured with bone marrow transplant after undergoing a regiment in which their own marrow as completely destroyed with chemotherapy. - Too toxic for adults because they have accumulated organ damage

Chantix Counseling

- Choose a quit date to stop smoking - Start taking the medication 1 week before the quit date - Take the medication after eating and with a full glass of water - Before taking this medication, tell your healthcare provider if you have ever had depression or other mental health problems - If you, your family or caregiver notice agitation, hostility, depression, or changes in behavior or thinking, stop taking the medication and call your doctor. - * serious skin reactions occured * - Use caution driving or operating machinery

NOTES about fluoroquinolones

- Cipro Oral Suspention should not be given through the NG or other feeding tube (oil-based suspension adheres to tubing) shake for 15 seconds each time before use - Do not refrigerate the oral suspension - Cirpo IR: can crush immediate release tablet , mix with water and give via feeding tube - Take Levaquin on empty stomach; store at room temperature -

Class II Antiarrhythmic Agents

- Class II antiarrhythmics drugs block beta receptors and indirectly block calcium channels in the SA and AV nodes, resulting in decreased automaticity and conduction velocity in the nodes. - They slow ventricular rate in the supraventricular tachyarrhythmias - Esmolol (Brevibloc) > Injection only - Propranolol (Inderal LA, InnoPran XL) > injection and oral

Diagnosis of Diabetes

- Classic symptoms of hyperglycemia (polyuria, polydipsia and unexplained weight loss) or hyperglycemic crisis AND a random plasma glucose >= 200mg/dL OR - FG >= 126mg/dL - fasting is defined as no caloric intake for at least 8 hours OR 2-hr plasma glucose >= 200mg/dL during a 75 gram oral glucose tolerance test (OGTT) OR A1C >= 6.5%

What is the dose for Colcrys used initially with prophylaxis treatment?

- Colchicine at a dose of 0.6 mg once or twice daily or NSAIDs are used concurrently for the first 3 - 6 months.

Important about diazepines

- Diazepam > Lipophilic, fast onset, long half-life, high abuse potential - Alprazolam > fast onset, often abused due to quick action - To avoid withdrawal symptoms > taper off slowly - Accohol withdrawal > chlordiazepoxide, diazepam (fastest onset, comes as injection), lorazepam (injection) or oxazepam (preferred BZD if liver disease)

What is the ANTIDOTE for Heavy metals such as arsenic, copper, gold, lead, mercury?

- Dimercaprol: given first since it cross BBB - Succimer: Only given in asymptomatic children with serum lead > 45mcg/dL - Penicillamine or Calcium disodium edetate (EDTA) > EDTA does not cross BBB > Give CaNa2 EDTA

Oral Antihistamines (1st generation)

- DiphenhydrAMINE HCL (benadryl) > 25-50 mg po Q 4 to 6 H > Age < 6 yrs; do not use for self care - SE > Somnolence, etc Notes > 1st generation should not be taken in lactating women > 2nd generation agents are preferred

Antihistamines

- Diphenhydramine (Benadryl, Sominex, Unisom) - Doxylamine (Aldex, Unisom Nighttime) - Side Effects > Due to the side-effect profile considered * Do not use drugs in elderly * > Possible anticholinergic side effects: Sedation, tolerance to sedative effects can develop after 10 days use. > Confusion > Peripheral anticholinergic side effects: Dry mouth, Urinary retention, Dry/blurry vision, risk increased IOP, constipation > Best to avoid in BPH (worsen symptoms) and glaucoma (may elevate IOP)

Direct Renin Inhibitor (DRI)

- Directly inhibit the renin which is responsible for the conversion of angiotensinogen to angiotensin I (AngI). A decrease in the formation of Ang I results in a decrease in the formation of Ang II, a potent vasoconstrictor. - Aliskiren (Tekturna) - Avoid high fat food (reduce absorption) BBW, CI and WARNING , SE, Monitoring, NOTE > Same as other ACE-I * Do not use concurrently with ACE inhibitors or ARBs in patients with diabetes * - New 2012 updated.

How to manage someone on statin and having muscle pain?

- Discontinue the statin and evaluate symptoms - Evaluate patient for other conditions that may increase the risk of muscle damage - If muscle symptoms resolve and if no contraindication * restart same statin at the same or lower dose * - If myalgias return, discontinue original statin. once muscle symptoms resolve, use a *low dose of a different statin* - If low dose of a different statin is tolerated, gradually increase the dose as tolerated.

NOTES about Macrolides

- Do not refrigerate azithromycin oral suspension (Zmax) - Take Biaxin XL with food. Do not refrigerate Biaxin oral suspension (can gel) - Must refrigerate erythromycin ethylsuccinate (E.E.S) oral granule suspension and use within 10 days - Erythromycin powder suspension stable at room temperature x 35 days - Azithromycin is a substrate of 3A4 (minor) and inhibitor of 1A2 (weak) and P-pg; * it does not have as many clinically significant drug interactions * - All macrolides: do not use concurrently with agents that can prolong the QT interval *

Acetylcholinesterase Inhibitors

- Donepezil (Aricept, Aricept ODT) > Aricept 23 mg used for advanced disease >> minimal additional benefit > Do not crush or chew - Rivastigmine (Exelon, Exelon Patch) __ < 6mg PO daily = 4.6 mg patch __ 6-12 mg PO daily = 9.5 mg patch - Galantamine (Razadyne, Razadyne ER) > Only one that comes in solution

What is the MOA of Bupropion and Mirtazapine ?

- Dopamine (DA) and Norepinephrine (NE) Reuptake Inhibitor *Bupropion* _ Mirtazepine: Tetracyclic antidepressant that works by its central presynaptic alpha2 adrenergic antagonist effect, which result in increased relase of NE and serotonin

What cause increase in RBC?

- Erythropoiesis stimulating agents (ESA) - Smoking - Polycythemia

What is used in Restless legs syndrome?

- Dopamine agonist > Pramipexole (Mirapex) and ropinirole (Requip) > Rotigotine (Neupro): Patch - They are used in long-acting formulatin or are taken TID, but for RLS these are taken in the *immediate release formulation * 1-3 hours before bedtime. - Parkinson patients start with the 2mg patch, but for RLS they start with 1mg patch > Do not apply a heat source over the patch > Remove the patch if receiving an MRI procedure > The same site cannot be used again for 14 days - Dopamine agonist cause orthostasis, somnolence and nausea which is dose-related.

Important about storage of vaccinations

- Dormitory style refrigerators should not be used - Store vaccines on the shelves away from the walls. Vaccines should never be stored in the door of the freezer or temperature because temperature there is unstable - Read and document refrigerator and freezer temperatures at least twice each workday AM and PM - Keep temperature logs for at least 3 years - Store refrigerated vaccines between 35F and 46F (2C and 8C). Store frozen vaccines between -58F and +5F (-50C to -15C)

[Cyclosporine (Neoral, Gengraf, SandIMMUNE)]

- Dose depends on transplant type and formulation - Cyclosporine (modified): > Renal 9 +/- 3 mg/kg/day, divided twice daily > Liver 8 +/- 4 mg/kg/day, divided twice daily > Heart 7 +/- 3 mg/kg/day, divided twice daily - Cyclosporine (non-modified) > 3-10mg/kg/day for maintenance * Conversion to cyclosporine (modified) from cyclosporine (non-modified): start with daily dose previously used and adjust to obtain pre-converson cyclosporine trough concentration; monitor every 4-7 days and dose adjust as necessary* - Trough 100-400 ng/mL (nephrotoxicity can occur at any level)

What you know about G6PD?

- Drugs that can cause hemolysis with G6PD deficiency should not be used in these patients. If hemolysis with these drugs test for G6DP deficiency - If patient is having RBC destruction and he has low G6PD then most likely is due to these drugs * chloroquine, dapsone, methylene blue, nitrofurantoin, primaquine, probenecid, quinidine, quinine and sulfonamides * and * FAVA BEANS * - This enzyme helps protect red blood cells from damage and premature destruction.

Asthma

- Drugs used to treat asthma are classified as controllers (maintenance) or relievers (rescue). - All patients with Asthma (step 1-6) need a "rescue inhaler for acute asthma symptoms. - Controllers are taken on a chronic daily basis to keep asthma under control, primarily by reducing inflammation. - Relievers (Rescue) are used to quickly reverse broncho-constriction, or preventively for exercise-induced bronchospasm (EIB). - Inhaled forms deliver drugs directly into the lungs, have reduced toxicity and are the preferred delivery vehicle

What are the side effects associated with Alpha-2 agonist?

- Dry mouth, somnolence, headache, fatigue, dizziness, constipation -skin rash, pruritus, erythema, contact dermatitis - with patch - etc. - For For methyldopa: same side effects as above plus hypersensitivity reactions, hepatitis, myocarditis, positive coombs test (risk for hemolytic anemia), drug-induced fever, drug-induced lupus erythematosus (DILE) and can increase prolactin levels.

Important information must know about Conviptan (Vaprisol)?

- Dual AVP antagonist (V1A and V2) - Do not use if CrCl < 30 mL/min - CI > Corn/corn products > Hypovolemic hyponatremia > concurrent use with strong 3A4 inhibitors > Anuria - WARNING > Overly rapid correction of hyponatremia > 12 mEq/L/24 hours) associated with osmotic demylination syndrome (life-threatening) - Side Effects > Orthostatic hypotension > Fever > Hypokalemia > Infusion site reactions (> 60%)

Why schizophrenia treatment is often difficult to obtain?

- Due to the patient's inability to recognize their illness.

Nitrates use in Stable Angina

- Forms free radical nitric oxide which increase cGMP, producing vasodilation of veins more than arteries, decrease myocardial oxygen demand by decreasing preload; improves collateral blood flow - Recommend SL tablets or Spray for fast relief of angina. Call 911 if chest pain does not go away after the first SL tab or first spray > Continue to take 2 additional doses (up to 3 doses total) at 5 minute interval while waiting for he ambulance to arrive - Long acting nitrates > Long acting nitrates are used as *add-on therapy* with beta-blockers and/or CBs. Long-acting nitrates require a nitrate-free interval to prevent tolerance.

Phenytoin

- Fosphenytoin is dosed in phenytoin equivalents (PE): 1mg PE = 1 mg phenytoin - Therapeutic range > Total PHT: 10-20 mcg/mL > free PHT: 1-2.5 mcg/mL - Exhibits saturable, or michaelis-menten, kinetics; a small change in dose can cause a large change in serum level. - If the albumin (alb) is low (< 3.5 g/dL), the true phenytoin level will be higher than it appears - adjust with formula below if CrCl >= 10 ml/min or measure a free phenytoin level. - PTH correction = PHT measured / 0.2 x alb + 1 - With CrCl < 10ml/min and low albumin, use corretion formula PHT correction = PHT measured / 0.1 x alb + 0.1

Important facts about Yellow Fever

- Found in tropical and subtropical areas in south america and Africa - Reducing mosquito exposure is essential - Aspirin and NSAIDS cannot be used due to the increased risk for bleeding - Vaccine should be provided only to those at high risk of exposure or to those who require proof of vaccination to enter the country due to * high risk of serious adverse effects after yellow fever vaccination* - Neurologic disease - After vaccine; patient is provided an "International Certificate of Vaccination or Prophylaxis" which is called the Yellow Fever Card. The card is valid only if the vaccination is completed 10 days before arrival - CI in infants < 6 months due to ADRS - CI in patients with hypersensitivity to eggs, egg product, chicken proteins or gelatin - Additional CI include a thymus disorder or MG - This is Live Vaccine and cannot be used with immunosuppression including HIV patient with CD4+ coutn < 200mm3 - * Woman should wait 4 weeks after receiving the vaccine before conceiving Yellow fever vaccine *

Counseling with inhaler

- Frequent, continuous puffing for 20 minutes is advised with each cartridge - It is only good for one day - Clean mouthpiece with soap and water regularly - In cold weather, the inhaler and cartridge should be kept in an inside pocket or other warm area - Acidic beverages interfere; avoid for 15 minutes before or during the use of nicotine inhaler

Amphotericin B Deoxycholate and Lipid Formulations

- Fungicidal - Amphotericin B binds to ergosterol, altering cell membrane permeability in suscepible fungi and causing cell death - BBW (2) 1. Medications errors from confusion between lipid-based forms of amphotericin (Abelcet, Amphotec, AmBisome) and conventional amphotericin B for injection have resulted in death 2. Amphotericin B for injection doses should not 1.5mg/kg/day due to overdose which may cause cardiopulmonary arrest SE > Fever, Chills, Headache, Malaise, Rigors, Hypokalemia, Hypomagnesemia, Nephrotoxicity, anemia, hypotension/hypertension, thrombophlebitis, nausea, vomiting NOte > Compatibile with D5W only - Lipid formulation has decreased infusion reactions and decreased nephrotoxicity compared to conventional formulation. - Amphotericin B conventional require light protection during administration and pre-medicaiton to reduce infusion related reaction

What is the indication of Naloxone given by itself?

- Given by itself, naloxone is used for opioid overdose - As an opioid antagonist, it replaces the opioid on the mu receptor

Another mainstay of treatment for patient with relapsing forms of MS is Copaxone? List some important information about Copaxone?

- Glatiramer acetate (Copaxone) - 20mg SC *daily* or 40mg SC *3 times per week* (at least 48 hours apart) Side Effects > Infection side reactions (inflammation, erythema, pain, pruritus, residual mass), infection, pain, flushing, diaphoresis, chest pain, weakness, anxiety, rash, nausea, vasodilation, lipoatrophy and skin necrosis at injection site.

Ado-Trastuzumab Emtansine (Kadycyla)

- HER2/neu over-expression required for use - use 0.22 micron filter BBW - HF, Hepatotoxicity, ado-trastuzumab emtansine and conventional trastuzumab are not interchangeable , etc. SE - cardiac dysfunction

Herpex Simplex Virus (HSV)

- HSV-1 most commonly associated with oropharyngeal disease - HSV-2 associated closely with genital disease - First episode of genital herpes usually begins within 2-14 days post exposure but up to 50% are *asymptomatic* - Symptoms appear first as flu-like symptoms then lesions on external genitalia - Treatment must be initiated during prodrome or within 1 day of lesion onset for the patient to experience the full benefit - If there is a resistant to acyclovir then there will be resistant to Valacyclovir and to famiciclovir.

What are the clinical diagnosis of COPD?

- Dyspnea - Chronic cough or sputum production and a history of exposure to risk factors for the disease especially cigarette smoke. - Spirometry (testing to measure lung function) is required to make the diagnosis. > The presence of a post-bronchodilators FEV1/FVC < 0.70 confirms the presence of airflow limitation and thus of COPD

Trastuzumab (Herceptin)

- Her2/neu over-expression required for use - Avoid anthracyclines due to increased cardiotoxicity - Pharmacogenomics > Trastuzumab binds to and reverses effects of overactive HER2 receptors > Must be >= 2+ by immunohistochemical (IHC) testing to respond/ use this drug - Obtain MUGA or ECHO at baseline and during treatment. - Not interchangeable with ado-trastuzumab ematansine. SE - Cardiomyopathy (HF, decrease in LVEF) etc.

What is the treatment for acute meningitis > 50 years?

- Here we are concern about a new bacteria (Listeria) so we need to add something to cover it - Vancomycin + ceftriaxone/cefotaxime + ampicillin Allergy to PCN - Vancomycin + chloramphenicol +/- SMX/TMP for suspected listeria

Valproate warnings..

- Hyperammonemia. > Give Carnitine - Side effects > Nausea, vomiting, anorexia, abdominal pain, dizziness, somnolence, headache, tremor, alopecia (treat with multivitamins containing selenium and zinc), * weight gain*, edema , polycystic ovary syndrome (PCOS), Vitamin D and calcium deficiency (bone loss) > Dose related: thrombocytopenia, dipolpia, blurred vision - Monitoring > LFTs (at baseline and frequently during first 6 months) - Note > Pregnancy category D/X (for migraine prophylaxis) > Supplementation with calcium and vitamin D recommended.

Docetaxel (Taxotere)

- Hyperesenisitivty - Cario-pulmonary side effects include fluid retention > Pre-treat with dexamethasone to decrease fluid retention - For all taxanes, use non-PVC IV bag and tubing due to leaching of DEHP BBW - Neutropenia, Edema, Hepatic disease, Hypersensitivity increase mortality SE - Myelosuppression, fluid retention, peripheral neuropathy etc.

What are the SE associated with Tacrolimus?

- Hypertension, nephrotoxicity, hyperglycemia, tremor, hyperkalemia, hypomagnesemia, edema etc. QT prolongation ( check out what antibiotics he is on that might affected the QT Interval)

List important electrolyte that can put patient at risk of digoxin toxicity

- Hypokalemia - Hypomagnesemia - Hypercalcemia

What are the side effects associated with Amiodarone?

- Hypotension, bradycardia, corneal microdeposits, dizziness, ataxia, GI upset, constipation, peripheral neuropathy, tremor, hypothyroidism/hyperthyrodisim (more hypo), optic neuritis, pulmonary fibrosis, photosensitivity, increase in LFT, slate blue (blue-grayish) skin discoloration.

What is the dose of the new hydrocodone ER? and what is the brand name?

- Hysingla ER - 20mg Q 24 H (this is the difference once daily dose)

Integrase Strand Transfer Inhibitor Drug interactons

- INSTI > Should be taken 2 hours before or 6 hours after taking cation-containing antacids or laxatives, sucralfate, iron supplements , etc < H2RAs and PPIs do not pose an interaction with INSTIs - Raltegravir > Metabolized by the UGT1A1-mediated glucuronidation pathway. Rifampin , a strong inducer of UGT1A1 will decrease level of raltegravir. > Stribild: Cobicistat, a component of Stribilid, is strong inhibitor of 3A4, 2D6 and P-pg. > Stribild should not be co-administered with any other ART

Integrase strand Transfer Inhibitors (INSTis)

- INSTIs block the integrase enzyme needed for viral DNA to integrate wit the host cell DNA/human genome - Class Side Effects > HA > Insomnia > Increase in CPK (mainly with Raltegravir) - Notes about the whole class > No renal except with Stribid > With or without food except Stribild

When is prophylaxis treatment indicated for migraine?

- If a patient uses acute treatments > 2 times per week or if the migraine decrease their quality of life, or if the patient desires to try an agent to decrease migraine frequency (with any incidence) a prophylactic can be used to decrease migraine occurrence.

Antidepressant use in pregnancy, postpartum depression

- If a women is on antidepressant and wish to become pregnant, it may be possible to taper the drug if the depression is mild and she has been symptoms free for the previous six months. - Mild depession > ACOG recommend psychotherapy first, followed by drug if needed (SSRI are often used initially and are pregnancy category C), except Paroxetine is D (due to cardiac effects) - New formulation of Paroxetine is Brisdelle category X - SSIR potential for risk of persistent pulmonary hypertension of the new born PPHN > Tricyclic preg C is the second group most commonly used. >>> Except doxepin

Notes about SSRI

- If cardiac risk is present, best to avoid citalopram. Sertraline is often the top choice for an SSRI in cardiac patients. - 2011, the FDA issued a warning regarding SSRI use during pregnancy and potential risk of persistent pulmonary hypertension of the new born (PPHN) - To switch to flu 90mg/weekly from flu daily, start 7 days after last daily dose

Ulcer in duodenal and gastric

- If the ulcer is duodenal (usually caused by H.pylori), eating generally lessens the pain. With gastric ulcers (primarily from NSAID), eating generally worsens the pain. Other symptoms include heart burn, bleching, bloating, nausea and anorexia

Omalizumab (Xolair)

- IgG monoclonal antibody that inhibit IgE binding to the IgE receptor on mast cells and basophils. Omalizumab is indicated for moderate to severe persistent, allergic asthma in patients with a positive skin test to perennial aeroallergens and inadequate controlled symptoms on inhaled steroids (step 5 or 6 per guidelines)

TKIs targeting BCR-ABL

- Imatinib (Gleevec) > Philadelphia-chromosome positive (Ph+) chronic myelogenous leukemia (CML) or cKIT (CD117)-positive gastrointestinal stromal tumors (GIST) - SE > Fluid retention, skin rash, diarrhea, etc. - Dasatinib (Sprycel) -SE > Pleural effusion, fluid retention, edema, etc. -Nilotinib (Tasigna) > BBW: Qt prolongation -Bosutinib (Bosulif) > SE: Diarrhea

What is the patophysiology of PAH?

- Imbalance of vasoconstrictor and vasodilator substance and an imbalance of proliferation and apoptosis in the pulmonary arteries. - PAH cause walls thicken and scare and the lung arteries become increasingly narrow > As a result; these changes make it hard for the right ventricle to pump blood through the pulmonary arteries and into the lungs due to the increased pressure. This cause heart failure which is the most common cause of death in people who have PAH

Celiac Disease

- Immune response to eating gluten, a protein found in wheat, barley and rye - The primary and effective treatment is to avoid gluten entirely - It is found in food and many drug excipients - Even a small exposure will trigger a reaction - The common symptoms of celiac disease are diarrhea, abdominal pain , bloating and weight loss.

How to treat hypoperfusion in ADHF?

- In hypoperfusion or cardiogenic shock, IV vasodilators are contraindicated - Initiate IV inotropic drug (dobutamine, dopamine , milirinone) > Dobutamine has some vasodilatory effect (beta2_ and is used if *SBP > 90 mmHg * > Milrinone is a phosphodiesterase-3 inhibitor that can increase CO ina dose-dependend manner. > Milirinone can only be used if blood pressure is adequate > Dopamine is dose dependent effect (dopaminergic receptor agonist at low doses, beta1 at moderate doses, alpha1 at higher doses)

* Fluoroquinolones *

- Inhibit bacterial DNA topoisomerase IV and inhibit DNA gyrase (topoisomerase II). This prevents supercoiling of DNA and promote breakage of double-stranded DNA. - Exhibit Concentration dependent killing - Bactericidal activity - Gemifloxacin, Levofloxacin, and moxifloxacin are often referred to as respiratory fluoroquinolones due to enhanced coverage of streptococcus pneumoniae and atypical coverage. - BBW (2) > Tendon inflammation and/or rupture (most often in Achilles tendon) - Increase risk with concurrent corticosteroid use, organ transplant patients > 60 years of age > May exacerbate muscle weakness related to MG

Rifaximin (Xifaxan)

- Inhibit bacterial RNA synthesis by binding to bacterial DNA- dependent RNA polymerase. - Traveler's diarrhea > 200mg PO TID X 3 days - Reduction of Hepatic Encephalopathy Recurrence > 550 mg PO BID * Not effective for systemic infections because < 1% oral absorption *

* Vancomycin *

- Inhibit bacterial cell wall synthesis by blocking peptidoglycan polymerization by binding to the D-alanyl-D-alanine portion of cell wall precursors. - Exhibit time-dependend killing - Bactericidal - Coverge for MRSA,Enterococci (NOT VRE) and C.difficile - MRSA infections: 15-20mg/kg IV Q8-12 H - C.DIFF: 125-500mg PO QID X 10-14 days (higher if recurrent or sever) - Infusion reaction/red man syndrome > Reason not to infuse at a concentration > 5mcg/mL - Target troughs at 15-20 mcg/mL - pneumonia, endocarditis, osteomyelitis, meningitis, bacteremia -Target troughs at 10-15 mcg/mL for other infections - * consider aleternative agent when MIC of organism is => 2mcg/mL *

* Tetracyclines *

- Inhibit bacterial protein synthesis by reversibly binding to the 30S ribosomal subunit with bacteriostatic activity to the total exposure of the drug - Option for the treatment of MRSA in mild skin infections and VRE in urinary tract infections - WARNING > Children < 8 years of age, pregnancy and breastfeeding - Photosensitivity - Increase BUN - DRESS SE > N/V/D/Rash NOTES > Take with 8oz water to minimize GI irritation > Doxycycline IV to PO ratio is 1:1 > Doxycycline oral suspension should not be refrigerated > Doxycycline IV: requires light protection during administration

Important about Granules, powders or capsules that can be sprinkled into soft food or water

- Instruct patient not to chew any long acting pellets or beads that are emptied out from a capsule, not to let the mixture sit too long (take within the time directed) and not to add anything warm or hot (the content will dissolve too quickly)

Is A-FIB a supraventricular or ventricular ?

- Supraventricular

Echinocandins

- Echinocandins inhibit synthesis of B(1,3)-D-glucan, an essential component of the fungal cell wall and are consiered to be fungicidal - Activity against Aspergillus spp but should be used part of combination regimen - Caspofungin (Cancidas) - Micafungin (Mycamine) - ndiulafungin (Eraxis) Side Effects > Increase in LFT > Hypotension > Fever, Diarrhea > Hypokalemia > Hypomagnesima NOTES Micafungin requires light protection during administration

What is the MOA of Valproic Acid/Valproate?

- T-type calcium channel blocker and fast sodium channel blocker that increase in gamma aminobutyric acitvity (GABA), an inhibitory neurotransmitter. Valproic acid and divalproex dissociate to valproate in the GI tract.

What is interferon Alfa used for?

- Interferon alfa are indicated for treatment of HBV , HCV - Pegylated forms (Pegasys or Peg-Intron) have polyethylene glycol added to the interferon via pegylation which prolongs the half-life and reduce the dosing frequency to once weekly - Dose reduction is required in thrombocytopenia and neutropenia (ANC<500/mm3 and plt < 25,000/mm3 - BBW > May cause or exacerbate autoimmune disorder; may cause or aggravate ischemic and hemorrhagic cerebrovascular events; combination treatment with ribavirin may cause birth defects and/or fatal mortality and/or hemolytic anemia - SE > Flu-like symptoms 1-2 hours after administration (fever, chills, headache, malaise, arthralgia, mylagia, diaphoresis- can last 24 hours) > can treat the flu-like symptoms with acetaminophen and antihistamine > CNS effects (fatigue, anxiety, depression, weakness) > GI upset (N/V anorexia, weight loss) > Increase in LFT (5-10x ULN during treatment > Myelosuppression, mild alopecia * MedGuide Required*

PRAMLINTIDE

- Is a synthetic analog of the human neuroendocrine hormone, amylin. Amylin is produced by pancreatic beta cells to assist in postprandial glucose control. Amylin helps slow gastric emptying, prevent increase in serum glucagon following a meal and increase satiety. This is an amylinomimetic agent. - Pramlintide (Symlin Pen 60, SymlinPen 120) - Can be used in both type 1 and 2 DM > Decrease rapid-acting, short-acting and fixed mix insulins by 50% when starting this drug - Administered SC in abdomen or thigh prior to each meal (>= 250 kcal or >= 30 grams of carbohydrates, if consuming less than the above quantity, skip dose) BBW > Co-administration with insulin may induce severe hypogycemia (usually within 3 hours after administration) CI > Gastroparesis, hypoglycemia unawareness SE > Nausea (30%), anorexia (15%), weight loss, etc.

List the pegylated interferon beta (Plegridy) that was approved and what is the advantage of using Plegridy?

- It allows for more convenient SC dosing every 14 days. - Peginterferon beta-1a (plegridy, Plegridy starter pack)

What is the disadvantage of using POPs?

- It is easier to get pregnant accidentally on a POP if the woman is not lactating.

What is the classic symptoms of Rheumatoid Arthritis (RA)?

- Joint swelling - Stiffness - Pain - Bone deformity (Eventually)

Important about Didanosine (Videx, Videx EC)

- Take on empty stomach at least 30 minutes before meal or 2 hours after - Oral solutions: Stable for 30 days if refrigerated - BBW > Pancreatitis (sometimes fatal) SE > Peripheral neuropathy, Insulin resistance/diabetes, optic neuritits, etc Monitoring > LFT, eye exam Notes > Do not use with tenofovir due to resistance and virologic failure as well as increased didanosine concentration

What is the correct way for drug disposal in a trash?

- Keep medication in its original resistant container. Scratch or mark out the patient information on the label - Place some H2O into solid medication such as pills or capsules then add something like coffee grounds - Close and seal the container lids tightly with packing or duct tape - if discarding blister packs then wrap in multiple layers of duct tape - Place in durable packaging that does not show what's inside - place in the trash close to garbage pickup time

Levetiracetam

- Keppra , Keppra XR - MOA is unknown; may inhibit voltage-dependent N-type calcium channels facilitate GABA-ergic inhibitory transmission through displacement of negative modulators, reduce delayed rectifier potssium current and/or bind to synpatic proteins which modulate neuroranmitter release. - SE > Somnolence, dizziness, weakness, infection, behavior changes, anorexia, etc.

Venous Thromboembolism/Mechanical valves recommendation in pregnancy

- LMWH or UFH > Converte to UFH during last month of pregnancy since is has short half-life > * warfarin is category x *

LamoTRigine

- Lamictal (tabs/chewable/ODT) - Must titrate up to prevent rash - Doses are different wether patient is on inducer/inhibitor or nothing BBW - Serious skin reactions - Titration schedule is based on wether patient is on valproate, inducer anticonvulsant, or no concomitant anticonvulsant. Warning - Aseptic meningitis, blood dyscrasias SE - Nausea, vomiting, insomnia, somnolence, rash, headache, etc. Note - Preg C - Starter kit (orange) - no inducer no inhibitor - Starter kit (blue) - if inhibitor - Starter kit (green) - if inducer

Tyrosine Kinase Inhibitor (TKI) Targeting HER-2 neu

- Lapatinib (Tykerb) > Inhibit HER-2/neu (ErbB2) and EGFR (Erb1) tyrosine kinases. Used in metastatic breast cancer. > Pharmacogenomics: HER2 gene over-expression BBW - Hepatotoxicity SE - Increase in LFT, Decrease in LVEF Qt-prolongation, etc.

Immnodoulators used in Multiple Myeloma

- Lenalidomide (Revlimid) - Pomalidomide (Pomalyst) - Thalidomide (Thalomid) Unique Concerns > Pregnancy category X > Patient, prescriber and pharmacist must be registered with these drugs - Consider prophylactic anticoagulation due to increase VTE risk, seek medical care if signs and symptoms of DVT/PE develop: SOB, chest pain, or arm or leg swelling BBW - Fatal risk/pregnancy, thrombosis , hematologic toxicity SE - Neutropenia, thrombocytopenia , etc.

Why Levothyroxine is the drug of choice?

- Levothyroxine is the drug of choice due to chemical stability, once-daily dosing, inexpensive , free of antigenicity and has more uniform potency

What is the drug of choice of Status Epilepticus?

- Lorazepam (ativan) is the benzodiazepin of choice for treating status epilepticus due to the longer duration of action in the CNS which provides longer protection. - Diazepam is highly lipophilic (quick onset) but rapidly redistributes into fat causing the CNS half-life to be shorter with a shorter duration of effect. > Diazepapam IV > Diazepam rectal gel (DIASTAT AcuDIal) >> 2.5 - 20 mg PR. may repeat once if necessary - Following the benzodiazepin, patient should be treated with phenytoin (max rate 50 mg/minute ) or fosphenytoin (max rate 150 mg PE/min)

What is atonic seizure?

- Loss of muscle tone and falls to the ground; known as "drop attacks"

List some causes of ED?

- Low T - Psychological issues (including depression and stress) - Neurological illness can also cause ED

What is the MOA of Taxanes and list the drugs?

- M-phase specific: Inhibit microtubule function and angiogensis (dysfunctional microtubule bundling) - Administration of taxens is reduced when given immediately after administration of cisplatin or carboplatin - Paclitaxel (Taxol) - DOCEtaxel (Taxotere) - Cabazitaxel (Jevtana) * if neurotpenia; do not give *

What are the 2 most common cause of HF in US?

- MI - HTN

Pathophysiology of Thyroid Disorders

- Elevation in T4 levels will inhibit the secretion of TSH, and create a negative feedback loops - T3 is more potent than T4 but shorter half-life - Less than 20% of T3 is produced by the thyroid gland; T3 is primarily formed from the breakdown of T4 by peripheral tissues - T3 and T4 bound to protein so it is important to measure the FT4 levels as this is the active form. - In hypothyroidism, there is a deficiency in T4, and consequently an elevation in TSH. In hyperthyroidism, there is over-secretion of T4 and consequently a low level of TSH

Causes of Hyperthyroidism

- Most common cause is Grave's disease - Drugs that can cause hyperthyroidism include iodine, amiodarone, and interferons. - RAI-131 is the treatment of choice in Grave's disease - Beta blockers can be used first for symptom control (to reduce palpitations, tremors and tachycardia) - PTU or Methimazole can be used as temporary measure until surgery is complete - Initially, when treating with rugs, it takes 1-3 months at higher doses to control symptoms at which point the dose is reduced to prevent Hypothyroidism from occurring.

What is Bipolar I ?

- Most severe version of the disorder. The classic diagnostic symptoms is mania, but patients may also experience bouts of depression - Symptoms in BP-1 can be severe that can require psychiatric hospitalization, impaire social or occupational functioning or have features of pscyhosis such as hearing voice or delusions.

Boceprevir (Victrelis) for treatment of hepatitis C

- Must be given with preginterferon alfa and ribavirin - CI > All Ci to Preginterferon and Ribavirin - SE > Fatigue , Anemia (requiring ESA use), neutropenia, dysgeusia - Never reduce the dose or interrupt therapy as treatment failure may result. Never use as monotherapy; must always be combined with peg and ribavirin. - Strong CYP3A4 inhibitor

Alteplase (Activase)

- Must confirm clot on brain imaging (head CT scan) before use. - Infuse 0.9 mg/kg (maximum dose 90mg) IV over 60 minutes with *10%* of the dose given as bolus over 1 minute - Once reconstituted use within 8 hours CI - Bleeding , BP > 185/110, etc. SE - Major bleeding , etc. Notes - Treatment must be initiated within 3 hours of symptom onset (guidelines state benefit in select patients up to 4.5 hours but this is not FDA approved) - If patient are receiving fibrinolytic therapy, the drug should be given <= 60 minutes of the patient's arrival in the ED (door to needle time)

Indomethacin

- NSAID approved for gout - High risk for CNS SEs and GI Toxicity

Side effects of Stimulants for ADHD

- Nausea, loss of appetite, insomnia, etc. - Increase in BP and HR - Exacerbation of mixed/mania episodes if bipolar disorder - Peripheral vasculopathy may worsen, including in Raynaud's - Risk of seizures (caution with seizure history), priapism

Which drugs cause Nephrotoxicity/Bladder toxicity?

- Nephrotoxicity is caused by bevacizumab, methotrexate, and platinum agents (Cisplatin and Carboplatin) - Bladder toxicity caused by Cyclophosphamide and Ifosfamide

What is the pathophysiology of

- Neurtitic plagues and tangle in brain tissue; neuron singnaling is interrupted - Alteration of neurotransmitters (decreased acetylcholine)

Normal uric level is 2 - 7.2 mg/dL. Patient has uric acid of 10 but does not have any symptoms. Do we treat this patient?

- No - Asymptomatic hyperuricemia is not treated. The treatment goal is a urate level < 6mg/dL

Intranasal Corticosteroids (1st line for moderate-severe rhinitis)

- Fluticasone (Veramyst) , Mometasone (nasonex), and Traicminolone (Nasacort) can be given to 2 years and up - Prolonged use can increase risk of secondary infection or limit response to vaccine. Avoid exposure to chickenpox. caution in patients with untreated fungal, viral, or bacterial infections. - can take one week for full relief - Budesonide Preg B the preferred inhaled steroid in pregnancy

Alkylators

- Non-cell cycle specific: Cross-Links DNA - Cyclosphoshamide (Cytoxan) - Ifosfamide (Ifex) - WARNING > Bladder toxicity with high dose cyclophosamide and all doses of ifosfamide - give MESNA (Mesnex) to protect against hemorrhagic cystitis and ensure adequate hydration - Temozolomide (Temodar) > Cross BBB; taken on an empty stomach or QHS to decrease nausea - Lomustine PO is dosed QHS with an antiemetic - Procarbazine is MAO inhibitor, avoid interacting food/drugs - carmustine: use non-PVD bag and tubing due to sorption of DEHP

List the conditions of Steroid induced immunosuppression?

- 20mg or more per day of prednisone for 14 days or longer - 2mg/kg or more per day of prednisone for 14 days or longer - NOT intra-articular injections, metered-dose inhalers, topical, alternate day or short course for less than 14 days.

Notes about Stimulatns

- Focalin XR, Rialin LA, Metadate CD and Adderall XR can be taken whole or the capsules sprinkled on applesauce (if not warm and used right away, do not chew) - Concerta OROS delivery > The outer coat dissolved fast to give immediate action, and the rest is released slowly - Daytrana patch is QAM, applied to alternate hip 2 hours before desired effect (or as soon as the child awakens so it starts to deliver prior to school). Remove after 9 hours (at night, so the patient can sleep)

List one drug that is Selective T cell costimulator; inhibit T cell activation by binding to CD80 and CD86 on cells that present these antigens (activated cells are detected in the synovium of RA joints)

- Abatacept (Orencia) WARNING > Screen for latent TB and HBV prior to initiating therapy. Do not give with other biologics or live vaccines Notes > Stable in NS only; require filter and light protection during administartion do not shake * Caution in those with COPD - may worsen symptoms *

These are the *common* drugs that require filters during administration

- Abatacept (Orenicia): Used to treat moderate to severe RA - Abciximab (ReoPro): Blood thinnger - Albumin - Amiodarone (continuous infusion): Treat heart rhythm problems - Amphotericin B Liposomal (AmBisome): Treat fungal infections - Diazepam (valium) - Digoxin Immune Fab (DigiFab) - Infliximab (Remicade) - Golimumab (Simponi) - Lipids - Lorazepam (Ativan) - Mannitol (Osmitrol) - Phenytoin (Dilantin) - Parenteral Nutrition (PN)

Alpha-Glucosidase Inhibitors

- Acarbose (Precose) - Miglitol (Glyset) MOA > These agents reversily inhibit membrane-bound intestinal alpha-glucosidasaes which hydrolyze oligosacchardies and disacchardies to glucose and other monosaccharides in the bursh border of the small intestine - Both acrabose and miglitol are started at 25mg with the first bite of each main meal. CI > IBD > Colonic ulcerations > Partial or complete intestinal obstruciton > Cirrhosis (acarbose) Side Effects > GI effects (flatulence) > Diarrhea > Abdominal pain > Increase LFT with acrabose only Important notes > Take with a full glass of water with the first bite of food (the medicine needs to be in the stomach with your food) if you plan to skip a meal, skip the dose for that meal too. > If you get low blood sugar, it wont be from this medication if taking alone, so you can't treat it with sucrose or tablesugar or candy.

Posaconazole (Noxafil)

- Activity against Mucomycosis and Zygomycosis - Must be taken with a full meal (during or within 20 minutes following a meal) WARNING - QT prolongation - correct K, Ca, and Mg prior to initiating therapy - Side Effects > Increase in LFT > Hypokalemia > Hypomagnesemia > Hyperglycemia

Fluorouracil 5-FU

- Adrucil - Pharmacogenomic testing for dihydropyrimidne dehydrogenase (DPD) - deficiency increase risk of severe toxicity - Given with leucovorin to increase efficac of 5-FU SE > Mucositis > hand-food syndrome (with continuous infusions) > etc.

Asthma and Pregnancy

- Albuterol is the preferred short acting beta 2 agonist - Budesonide is the preferred inhaled corticosteroid due to more studies in pregnancy - It is safter to be treated with asthma medications than to have poorly controlled asthma to ensure oxygen supply to the fetus

When is the time that activated charcoal should be administered?

- For some orally ingested compounds taken in potentially toxic amounts, activated charcoal can be administered and is most effective within * one hour * of ingestion. - It can be aqueous, or combined with sorbitol as a cathartic and sweetener. - * Sorbitol should be avoided because it can induce vomiting an cause electrolyte disturbances* - Dose is 1g/kg

Notes about Metformin

- Alcohol can increase risk of lactic acidosis especially when renal impairment and advanced heart disease - Iodinated contrast dye increase risk of lactic acidosis > Hold prior to procedure and wait 48 hours after and restart only once renal function have been confirmed as normal - Metformin can decrease B-12 absorption (and posisbly folic acid), leading to megaloblastic anemia. Consider vitamin supplementations. Seperate ER from Colesevelam. - If using Glumetza, Fortamet, Glucophage XR, you may see a shell of medicine in the stool. this is normal.

Where is the place of Hydralazine/Nitrates in patients with HF?

- Hydralazine is a direct arterial vasodilator which decrease afterload. Nitrates are venous vasodilators and decrease preload. - The combination improves the survival of heart failure patients [not as much as ACE-inhibitor] - * Used at alternative therapy for patients who cannot tolerate ACE inhibitors or ARBs due to poor renal function, angioedema, hyperkalemia. It can also added to standard therapy in black patients based on a study demonstrating improved survival *

GERD

- Normally , gastric contents are prevented from backflow into the esophagus by a ring of muscle fibers called the lower esophageal sphincter (LES). In GERD, the LES pressure (muscle tone) is reduced (or transiently relaxes) and allows for back flow of the stomach contents.

What is the purpose of using ODT triptan or nasal Sprays?

- OTD triptan are given if Nausea is present [No water is required] - Nasal sprays and injections are usefull if nausea and they are faster. > Intranasal onset of action: around 15 to 30 minutes > Sub-Q onset of action: around 10 minutes

Ortho Evra COC contraceptive patch

- Once a week for 21 out of 28 days - Do not apply to breast - Start on either day 1 (no back up needed) or sunday (back up 7 days if not on day 1) - If patch become loose or falls off > 24 hours during first 3 weeks of use or if > 7 days have passed during the 4th week where no patch is required there is a risk of pregnancy, thus a back up method should be used for 1 week while a new patch is put in place - Higher systemic estrogen exposure; avoid carefully in anyone with clotting risk factors - Less effective in women > 198 pounds

List some medications that are constipating

- Opioids - Anti-cholinergic - Anti-histamine (due to anti-cholinergic effects) - *verapamil* and in general the other Non-DHP calcium - Clonidine - Bismuth - Iron (use docusate to avoid hard, compact stools) - Aluminium antacids > Mg often in combination with aluminum to counteract effect - Tramadol/Tapentadol - Colesevelam - Vareniciline - 5-HT3 receptor antagonists (Zofran) - Phentermine/topiramate - Abilify

Why immunosuppresive agents used in organ transplant require careful monitoring?

- All immunosuppressive agents require careful monitoring (including drug trough levels) to minimize toxicities and decrease the incidence of rejection. Keep in mind which maintenance agents have the highest nephrotoxicity (Tacrolimus, cyclosporine), the most likely to worsen diabetes or cause new onset diabetes (tacrolimus, steroids, cyclosporine, and the mTOR inhibitors everlius and sirolimus) which are the most likely to worsen lipid parameters (mTOR, steroids, cyclosproine) and which are the highest risk for blood pressure elevation (steroids, cyclosporine and tacrolimus)

Stavudine d4T (Zerit)

- Oral solution is stable for 30 days in refrigerator - BBW > Pancreatitis (sometimes fatal) has occurred during combinations with didanosine - Side effects > Peripheral neuropathy > Increase in LFT > Lipoatrophy > Insulin resisance/diabetes

What are the most common complains of arrhythmia?

- Palpitations (feeling like there is a fluttering or racing) - dizziness, light-headedness, SOB, chest pain, fatigue - Severe cases > syncope, heart failure, and death

PPIs Notes

- Pantoprazole and esomeprazole are the only PPIs available IV - PPIs inhibit CYP450 2C19 > PPIs reduce the effectiveness of clopidogrel via 2C19 inhibition. if using these agents together * AVOID omeprazole and esomeprazole*

Paliperidone (Invega) - Invega susenna is long acting injfeciton

- Parent compound is riseperidone - SE > Same as Riseperidone

List the first non-hormonal treatment indicated to reduce moderate severe hot flashes?

- Paroxetine (Brisdelle)

Important about Dextroamphetamine and amphetamine?

- Patient should avoid use of acidic foods, juice or vitamin C as amphetamine levels may decrease when taken together.

What to do when you have patient with "Break-Through-Pain BTP" ?

- Also called end of dose pain, is acute pain that occurs despite use of an ER or scheduled opioid. It is treated with an immediate release (IR) opioid. If repeat doses of BTP medications are required then we need to increase the baseline opioid or substitute with different opioid

What is the recommended first-line treatment for Acute Otitis Media (AOM)?

- Amoxicillin 80-90 mg/kg/day in 2 divided doses - Amox/clav [90mg/kg/day of amoxicillin with 6.4 mg/kg/day of clavulanate (amoxicillin to clauvalanted ration is 14:1) in 2 divided doses.

What would be a good option for HIV-associated cyrptoccosu/meningitis?

- Ampho B can increase effect of flucytosine; may be used for synergy with ampho B for crain fungal infections (Cryptococcus spp)

What is Cystic Fibrosis (CF)?

- An autosomal recessive genetic disorder that leads to abnormal transport of chloride, bicarbonate, and sodium ions across the epithelium, leading to thick, viscous secretions.

Paciltaxel (taxol)

- Anaphylaxis reactions > Pre-treat with dexamethasone, diphenhydramine and H2RA (not needed with Abraxane) - For all taxanes, use non-PVC IV bag and tubring due to leaching of DEHP (except Abraxane) - Will increase INR if on warfarin, monitor closely SE - Myelosuppression, peripheral neuropathy - Etc.

Nephropathy Screening

- Annual urine test to quantitate urine albumin excretion is recommended. - Either an ACE inhibitor or ARB should be started in diabetes patients with urinary albumin excretion of >=30 mg/24 hours. - Type 1 diabetes > Annual testing starting 5 years after diagnosis in patients >= 10 years old -Type 2 diabetes > Annual testing starting at the time of diagnosis

* IMPORTANT FOR EXAM *

- Anyone who is allergic to penicillins should be presumed to be allergic to all penicillins and should avoid the entire group, unless they have specifically evaluated for this problem - Cephalosporins are closely related to penicillin. People with a history of penicillin allergy have a small risk of having an allergic reaction to cephalosporin or carbapenem. On exam avoid any beta-lactam. - If you want to use it and no other alternative then * monobactam is the best to choose *

DA- agonit -- INJECTION for "OFF" periods

- Apomorphine (Apokyn) - Last 90 minutes - can be injected up to 5 x day - dosed in mL CI - with 5HT2 antagonist such as zofran > Due to hypotension SE - Severe nausea and vomiting, hypotensioin monitor BP > Due to severe nausea it is recommended to give Tigan or similar drug 3 days prior to the initial dose of apomorphine and continued at least during the first two months of therapy. - Yawning, dyskinesias, somnolence, dizziness, QT-prolongation

Name the drug used for severe, refractory gout disease?

- Pegloticase

Namenda

- Approved for use alone or in combination with donepezil (Aricept) for moderate to severe AD - XR caps can be opened and sprinkled on applesause - SE > Dizziness, constipation, headache > Rare SEs: flu-like symptoms, arthralgia, UTIs, urinary retention , small risk seizures, hypertension NOTE - Do not exceed 5mg BID or 15 mg XR daily if CrCl < 30 ml/min

Digoxin

- Rate control - Therapeutic range for AFib = 0.8 - 2 ng/ml (lower therapeutic range in heart failure) - When CrCL < 50 ml/min, decrease dose or frequency - Antidote: DigiFAB Contraindication > Ventricular fibrillation - Not given alone > used in combination with beta blocker or CCB

What is the advantage of using POPs (Progestin-only pill)?

- Recommended for lactating (breastfeeding) women, because estrogen reduces the milk production. - Good choice for patients who have contraindication to estrogen.

List some natural products used to lower LDL and TG

- Red yeast rice - Garlic - Over-the counter fish oil [can be used to decrease TG in patient with TG >= 500 mg/dL]

Vigabatrin (Sabril)

- Refractory complex parital seizures and infantile spasms - MOA: Irreversibly inhibits GABA transminase, increase level of GABA - BBW: Permanent vision loss - SERM

Telithromycin (Ketex)

- Related to marcolides - FDA approved for community acquired pneumonia only - BBW > Myasthenia gravis due to respiratory failure - Contraindication > MG , jaundice Warning > Acute hepatic failure > Qt prolongation > Visual problems Monitoring LFT * CYP 3A4 inhibitor *

H2RAs

- Reversibly inhibit the H2 receptors on the gastric parietal cells which decrease gastric acid secretion - Used PRN for heartburn and scheduled for GERD - The brand name for Cimetidine is Tagamet - The brand name for Nizatidine is Axid WARNING - Confusion, usually reversible. Risk factor > 50, renal or hepatic impairment. ECG changes with renal dysfunction (famotidine) - Potential for vitamin B12 deficiency with prolonged use (>= 2 years) SE - HA, agitation/vomiting in children < 1 year - Cimetidine: *gynecomastia, impotence *

What is the difference between Argatroban and Bivalirudin?

- Argatroban is indicated for HIT and patients undergoing PCI who are at risk of HIT - Bivalirudin (Angiomax): Indicated for patients with ACS underoing PCI and are at risk for HIT Notes - Pregnancy category B - No corss-reaction with HIT - No antidote

NRTI important interactions

- Ribavirin increase level of all NRTI (combo increase risk of lactic acidosis and LFT). avoid concurrent use of ribavirin with didanosine (Increase liver failure, pancreatitis) and ribavirin with zidovudine (significanlty increase risk and severity of anemia) - Avoid didanosine (ddI) and Stavudine (d4T) due to increase risk of peripheral neuropathy, hyperlactatemia and pancreatitis. - Avoid didanosine and tenofovir combination due to resistance and virologic failure as well as increased didanosine concentrations - Allopurinol can increase didanosine levels. Avoid combo. - Methadone can increase zidovudine levels. Monitor for zidovudine toxicity.

How often to clean Atrovent HFA?

- Rinse mouthpiece under warm running water (but not the metal canister) for 30 seconds to prevent medication buildup and blockage. Shake to remove excess water and * let air dry*. *clean at least weekly*

Primary Prevention of Cardiovascular Disease

- Aspirin (babe aspirin) should be considered for primary prevention in patients with type 1 and type 2 diabetes at increased cardiovascular risk (10-year risk >10%). This include men and women >50 years of age who have at least one additional major risk factor (family history of CVD, HTN, smoking, dyslipidemia, albuminuria).

Oxcarbazepine SE

- SE > Somnolence, dizziness, headache, GI effects (N/V/abdominal pain), * dipolpia *, nystagmus, abnormal visual disturbances, ataxia, tremor, vitamin D and Ca deficiency (bone loss) NOTE - Supplementation with Ca and D recommended

Additional therapy in the treatment of Acute Ischemic Stroke

- Aspirin therapy > 325 mg PO within 24-48 hours after onset of stroke to prevent recurrent stroke. Aspirin should not be giving within 24 hours of fibrinolytic therapy - Hypertension management > Medications should be used to lower BP < 185/110 mmHg prior to giving alteplase. - Diabetes > Must be maintained betwen 140-180

Rituximab (Rituxan)

- Targets CD20 antigen on B lymphocytes, killing the cancer and releasing cytokines - Must be administered in hospital or clinic to monitor for infusion reactions which may occur within 30-120 minutes - Administer diphenhydramine and acetaminophen prior to infusion BBW - Severe infusion reactions, hepatitis B reactivation, etc. SE -* infusion reaction * - Tumor lysis syndrome, etc. - Reactivation of hepatitis B

What is the first drug approved for Non-24 which is a condition where patients have difficulty with obtaining restful sleep because their circadian rhythm is not snchronized with the 24-hour day night cycle.

- Tasimelteon (Hetlioz) > It is a melatonin receptor agonist (Similar to ramelteon)

What are the side effects associated with Propafenone (Rythmol, Rythmol SR)

- Taste disturance (metallic), dizziness, visual distrubances

Example of drugs used for systemic absorption is Forteo (teriparatide). What is this drug used for and what class of drugs belong to?

- The drug is injectable used for osteoporosis - Class: It is a parathyroid Hormone Analog

Sublingual immunotherapy

- The first dose must be given at Dr's office - Patient must be monitored for at least 30 minutes for allergic reactions (boxed warning) - Patient should be prescribed auto injectable epinephrine

What is the most common type of arrhythmia?

- Atrial fibrillation (AFib) is the most common type of arrhythmia. A-fib results from multiple waves of electrical impulses in the atria, resulting in an irregular and usually rapid ventricular response. - This rapid ventricular rate can result in > Hypotension > Worsen underlying ischemia > Worsen underlying HF > Risk of thromboembolism and stroke - Due to risk of stroke, management of A-FIB usually invovles anticoagulation and anti-arrhythmics to either slow the V-rate (rate-control) and/or terminate the a-FIB to restore normal sinus rhythm (rhythm-control).

What you give if patient have a history of tardive dyskinesia (TD) or any type of movement disorder ?

- Avoid riseperidone and paliperidone and lurasidone > * Quetiapine has low risk of movement disorder and is the recommended agent for psychosis in a ptient with parkisnon disease * > * clozapine has very low risk too but is not used highly *

Atomoxetine (Strattera)

- BBW > Risk of suicidal ideation; monitor for suicidal thinking or behavior, worsening, or unusual behavior - Contraindications > Glaucoma, pheochromocytoma, MAO I use within past 14 days - Warning > Rare, but severe hepatotoxicity (most within 120 days of start of treatment) SE > Headache, insomnia, *somnolence*, dry mouth, nausea, abdominal pain, decrease in appetite, nausea, etc. > Priapism NOTE > * do not open capsule * - irritant > CYPD 2D6 substrate > watch out for *LIVER PROBLEMS *

What to do if Traveler's diarrhea develops?

- Hydration is essential - In serious cases, oral rehydration solution can be used for fluid replacement - Over-The-Counter loperamide (imodium) can be used to decrease the frequency and urgency of bowel movement - It should not be used in children < 2 y.o without physician authorization due to risk of toxic megacolon. - another option is bismuth subsalicylate (pepto bismol) which should not be used in anticoagulatns or with salicylate allergy, renal insufficent or children who are < 12 year old or who may have a viral infection (due to risk of Reye's syndrome)

Important facts about Meningitis

- Hyperendemic regions include the meningitis belt of Africa during dry season (DEC - JUN) - Meningococcal vaccine is required by the government of Saudi Arabia - Patient with symptoms of fever, severe, unrelenting headache, nausea, stiff neck - 9-23 months (Menactra-2 doses) - 2-55 years (Menveo) - 56+ (Menomune) - 7-10 days are required after vaccination for protective antibody levels

* Carbapenems *

- Beta-lactams that inhibit bacterial cell wall synthesis - Exhibit time-dependent killing with bactericidal activity - Gram + , - , and Anaerobic pathogens - * Ertapenem does not have activity against Pseudomonas or Acinetobacter - Carbapenems have been assoicated with CNS ADS including confusional states and seizures - Doripenem: Do not used for the treatment of pneumonia including healthcare-associated pneumonia and ventilaor associated pneumonia - * Carbapenems can decrease serum concentration of valproic acid leading to a loss of seizure control *

What are the side effects associated with loop diuretics?

- Hypokalemia, orthostatic hypotension, decrease in Na, Mg, Cl, Ca (different than thiazide which increase Ca), *increase in HCO3/metabolic alkalosis, hyperuricemia (UA), hyperglycemia (BG), high in TG, High in total cholesterol *, photosensitivity, ototoxicity (more with ethacrynic acid) including hearing loss, tinnitus and vertigo

* Tedizolid (Sivextro) *

- Same as Linezolid - WARNING > Consider alternative therapy if neutropenia - SE > HA/Diarrhea

Why the previous 3 drugs not recommended in C.difficile infections?

- The patients body must be able to rid itself of the toxin

TYLENOL

- IV acetaminophen (Ofirmev) > Used inpatient to enable lower opioid doses, and can be used when oral routes are not available. - Keep below 4000 mg/day > Maximum of 325mg per prescription dosing unit per the FDA - Giving Q 4-6 H - Pediatrics (<12 yrs) > 10-15mg/kg Q4-6H [maximum 5 doses/d or use weight and age based dosing table on container] - Infant drops are now the same concentration as the children's suspension (160mg/5mL) to decrease dose confusing and thus decrease toxicity risk - ALL Iv acetaminophen doses should be prepared in the pharmacy

DSM-5 Diagnostic Criteria for ADHD

- Inattention > 6 or more symptoms of inattention for children up to age 16 > 5 or more for ages 17 to adults > symptoms present for a least 6 months - Hyperactivity and Impulsivity > 6 or more symptoms of hyperactivity and impulsivity for children up to age 16 > 5 or more for ages 17 toa dults > symptoms must be present for at least 6 months - In addition > Several inattentive or hyperactive-impulsive symptoms were present before age 12 years > Symptoms must present in 2 or more settings

What are the BBW associated with Tacrolimus?

- Increase susceptibility to infection; possible development of lymphoma; not recommended in liver transplantations; should be administered under the supervision of a physician experienced in organ transplantation in a facility appropriate for monitoring and managing therapy. *extended release tacrolimus associated with increased mortality in female liver transplant recipients *

1st generation cephalosporin

- Increased activity toward staphylococci , but increased covered for streptococci and gram- activity compared to 2nd/3rd generation; covers PEK (proteus gram-, E.coli , Klebisella species gram-) - Cefadroxil - Cefazolin (Kefzol) (NO ORAL) - Cephalexin (keflex)

What the AGA guidelines recommend?

- Increasing fiber intake (dietary and as supplement) with the possibility of adding on a relatively in-expensive osmotic agent (milk of magnesium) or PEG (polyethylene glycol) - * do not use milk of magnesium with severe renal impairment *

Which drugs cause Mucositis?

- Inflammation of the GI tract > 5-flurouracil > Capecitabine > Irinotecan > Methotrexate

DSM-5 ( In addition to the elevated or irritable mood, the DSM-5 diagnostic criteria for mania include 3 or more of the following symptoms. Note that 4 symptoms are required if the mood is only irritable)

- Inflated self-esteem or grandiosity (having an exaggerated belief in one's importance or talents) - decreased need for sleep - More talkative than usual - Flight of ideas (jumping from one topic to the next) or racing thoughts (the mind switches between thoughts very quickly) - Distractibility - increase in goal-directed activity - Excessive involvement in pleasurable activities that have a high potential for painful consequences (buying sprees, sexual indiscretions (gambling)

What is the MOA of Acetylcholinesterase inhibitors?

- Inhibit centrally active acetylcholinesterase, the enzyme responsible for hydrolysis (breakdown) of acetylcholine which results in high concentration of ACh.

Statins

- Inhibit the enzyme 3-hydroxy-3-methylglutaryl coenzyme A (HMG-CoA) reductase preventing the conversion of HMG-CoA to mevalonate (the rate-limiting step in cholesterol synthesis). - Mevacor is the immediate release of Lovastatin and is taken with evening meal - Altoprev is the ER dose of Lovastatin and is taken at bedtime with or without food - Lescol is brand name for Fluvastatin and is taken with evening meal - Lescol XL is taken daily, anytime - Rosuvastatin (Crestor) is the most potent statin

Neuroleptic malignant syndrome (NMS)

- Signs > Muscle rigidity > Hyperthermia > Mental status changes Lab results > High level of creatine phosphokinase > High level of white blood cells Treatment > Stop antipsychotics > Switch to quetiapine or clozapine

What is the guidelines about Occupational postexposure prophylaxis recommendatioin (PEP)?

- Therapy should be started right away, ideally within 72 hours, when treatment is indicated - Per the 2013 updated guidelines a three drug regimen including raltegravir + tenofovir/emtricitibine (truvada) for a 4 week course is the preferred regimen. - The exposed health care personnel should be tested for HIV ab at baseline, 4-6 weeks, 3 months, and 6 months after the exposure event.

* Sofosbuvir and Sofosbuvir/Ledipasvir * for hepatitis C

- Sofosbuvir is inhibitor of HCV NS5B RNA-Dependent RNA polymerase - Sofosbuvir and Ledipasvir( NS5A inhibitor) - Sofosbuvir must be givein with peg and ribavirin or jus iwth simeprevir - Genotype 2 and 3 can be given with Ribavirin only for 12 weeks and 24 weeks respectively - Harvoni (Ledipasvir/Sofosbuvir) is 90mg/400mg given daily for 12 weeks > for genotype 1 infections ONLY!!!! > require acidic environment (combo-only**) - Sofosbuvir is a substrate of drug transporter P-pg avoid giving with BIG INDUCERS - * Partiaprevir/ritonavir/Ombitasvir + dasabuvir (Viekira Pak): 2 tablets of paritaprevir/ritonavir/ombitasvir once daily in the morning and 1 dasabuvir tablet twice daily with meals * > WARNING: Hepatic decompensation and hepatic failure in patient with cirrhosis; risk of increase in LFTs (>5 x ULN) within 4 weeks of treatment, female patient taking ethinyl estradio products are at increased risk: significant drug interactions potential

List some natural products used for Insomnia?

- St john wort ( if depression too) - Chamomile tea - Melatonin - Valerian > There have been isolated reports of valerian causing liver toxicity, this risk is unclear at present

Liopatrophy can significantly occur in these 2 drugs?

- Stavudine - Zidovudine to a lesser extent

How to monitor Extravasation?

- Stop the infusion, elevate the limb and use cold compresses w/ except to the VINCA ALKALOIDS and ETOPOSIDE, use warm compresses.

When do you stop the primarily prophylaxis of MAC (Mycobacterium Avium Complex)

- CD4+ count >= 100 cells/mm3 for >= 3 months of ART

Dextromethorphan (Delsym)

- CI > All dextromethophan containing products should not be used within 14 days of MAOI use. - SE > Serotonin syndrome if co-administered with other sertonergic drugs

Which drugs cause hand-food syndrome?

- Capecitabine - Pazopanib - sorafenib - sunitinib - Vemurafenib - IV agent > 5-fluoruracil > Liposomal doxorubicin an cytarabine

Seizure and Pregnancy --

- Carbamazepine , clonazepam, phenobarbital/primidone, phenytoin/fosphenytoin, topiramate and valproate are pregnancy cateogry D - The rest are C - Valporic acid is X in migraine prophylaxis

Premenstrual dysphoric disorder PMDD

- Choose YAZ or Sertraline or fluoxetine (Sarafem)

Why some clinicians overlap SNRI or SSRI with beznodiazepines?

- Clinicians may overlap SSRI or SNRI with a benzodiazepin for 2-4 weeks to help alleviate the initial stimulatory effect and regulate sleep.

List the clinical signs and symptoms of Hypothyroidism?

- Cold intolerance/sensitivity - Dry skin - Fatigue - Muscle cramps - Voice changes - Constipation - Weight gain - Coarseness or loss of hair - Heavier than normal menstrual periods - Memory and mental impairment - Goiter - Myalgias - Weakness - Depression - Bradycardia

How to treat Congestion in ADHF?

- Congestion is treated with diuretics and possibly IV vasodilators

ACE inhibitors and Angiotensin Receptor Blockers

- Decrease in preload and after-load - They decrease pathologic cardiac remodeling - Improve left ventricular function - Reduce morbidity and mortality * The use of ACE inhibitor is indicated for all heart failure patients regardless of symptoms (NYHA FC I-IV)

Tretinoin, AKA All-trans Retinoic Acid, ATRA

- Decrease proliferation and increase differentiation of APL cells. - 1st line for APL - Pregnancy Category D - BW - RA-APL differentiation syndrome, leukocytosis, pregnancy

What is so special about Firmagon ?

- Degarelix the generic - True blockdage of GnRH so does not cause increasae in testosterone with tumor flare CI - pregnancy SE - Qt prolongation - etc.

Valproate , Valproic acid (Depakene, Stavzor, Depacon)

- Depacon - IV - Depakote Sprinkle > Capsules can be opened and sprinkled on food > Also used for bipolar and migraine prophylaxis - Delayed-release divalproex decrease GI upse - Therapeutic range > 50-100 mcg/mL - If the albumin is low < 3.5 g/dL then the true valproate level will be higher than it appears > Adjust with the same formula used for phenytoin

What is the treatment of Syphilis caused by Treponema pallidum, a spirochete?

- Depends if primary, secondary, or early latent (<1year duration) > Penicillin G benzathine (Bicillin L-A- do not substitue with Bicilllin C-R) > 2.4 million units IM x 1 Alternative > Doxycycline 100mg PO BID or > Tetracycline 500mg PO QID x 14 days Pregnant > Desensitized to PCN then give PCN - If Latent (> 1 year), tertiary, or latent syphillis of unknown duration > 2.4 million units IM weekly for 3 weeks (7.2 MU total) Alternative > Doxycycline 100mg PO BID or > Tetracycline 500mg PO QID x 28 days Pregnant > Desensitized then treat with PCN

Cytarabine

- DepoCyt > Preservative free for intrathecal administration - To prevent cytarabine syndrome treat w/ corticosteroids BBW - Bone marrow suppression (conventional formulation) CI - Active meningeal infection (DepoCyt only)

L-Asparaginase (Elspar) - Derived from E.coli Asparaginase (Erwinaze) - Derived from Erwinia chrysanthemi Pegaspargase (Oncaspar) - Peg form of L-Asparaginase

- Deprives leukemia cells of *asparagine* which is an essential amino acid in leukemia - * Erwinaze is FDA approved for patients who develop allergic reactions to the E. coli dervied asparaginase * - CI - Hypersensitivity to E.coli or L-asparaginase, etc. SE - Hypersensitivity reactions , pancreatitis, Prolonged prothrombin time (PT/INR) and thrombin time (TT)

Mitomycin (Mutamycin)

- Dervied from streptomyces caespitosus - Vesicant, do not extravasate - Antidote is dimethyl sulfoxide (DMSO) and cool compresses.

List one drug that is used commonly in mild UC and CD as antispasmodic?

- Dicyclomine (Bentyl) SE > Dizziness, dry mouth, nausea, blurred vision

What to do if you want to switch from warfarin to rivaroxaban?

- Discontinue warfarin and start Rivaroxaban when INR < 3

Apixaban - Switching from warfarin to apixaban

- Discontinue warfarin and start apixaban when the INR < 2

How to diagnose ACS

1. A 12 lead ECG should be performed at presentation (within 10 minutes) 2. If not diagnostic, but symptoms remain then ECG should be performed every 5-10 minutes to detect ischemic changes 3. Cardiac troponin I or T levels should be obtained at presentation and 3-6 hours after symptom onset in all patients with ACS symptoms 4. * BNP may be obtained afterwards for prognostic information *

Treatment principle for CDI include

1. Discontinue the offending agent immediately, if possible 2. Avoid anti-motility agent due to the risk of toxic mega-colon 3. Wash hands with soap and water to prevent transmission. (Hand sanitizer containing alcohol do not kill C. difficile spores) 4. Isolate patient to prevent transmission (single patient room, gloves, gowns) 5. Metronidazole should not be used beyond the 1st recurrence (use Vancomycin taper/pulse therapy instead) due to the potential for cumulative neurotoxicity 6. Probiotics (Lactobacillus) are not beneficial for treatment, but may have some benefit for prophylaxis.

How to prevent recurrence of Breast Cancer?

1. ER/PR+ (pre-menopausal and peri-menopausal) female patients > Tamoxifen for 5 years, after which they should receive additional therapy based on menopausal status. If pre-menopausal, continue tamoxifen for a total of 10 years. If post-menopausal, continue tamoxifen for a total duration of 10 years or an aromatase inhibitor for total duration of 10 years. 2. ER/PR+ (Post-menopausal) female > tamoxifen for 10 years; an AI for 5 years; tamoxifen for 5 years then switching to an AI for up to 5 years; or tamoxifen for 2-3 years then switching to an AI for up to 5 years 3. ER/PR-: do not have benefit with hormonal agents 4. ER/PR+ male patients: oral hormonal therapy (SERM agent for 5-10 years; tamoxifen is DOC)

Metronidazole (Flagyl) and Tinidazole (Tindamax)

- These agents cause a loss of helical DNA structure and strand breakage resulting in inhibition of protein synthesis - Bactericidal - Mild-to-Moderate C.diff infections > 500mg IVPO TID for 10-14 days BBW - Possibly carcinogenic based on animal studies CI - Pregnancy (1st trimester) - Use of Disulfiram within past 2 weeks (metronidazole) - Use of alcohol or propylene glycol-containing products during therapy or within 3 days of therapy discontinuation SE - Metallic taste Notes - IV:PO ratio is 1:1 and require light protection during administration - Do not refrigerate (metronidazole) IV (Crystal will form which will dissolve upon warming to room temperature) - Pregnancy Category B metronidazole - Metronidazole and Tinidazole should not be used with alcohol (within and for 3 days after discontinuation of therapy) due to a potential disulfiram-like reaction

What is the rule of PO steroids for management of Asthma?

- Used as pulse therapy for acute exacerbations; used for maintenance therapy in very severe disease (step 6) at doses of 5-60 mg daily or every other day

Cbazitaxel (Jevtana)

- hypersensitivity reactions - For all taxanes, use non-PVC IV bag and tubing due to leaching of DEHP BW - Neutropenia, hypersensitivity CI - Neutropenia, hypersensitivity

What if patient has Acute bacterial exacerbation of chronic bronchitis (ABECB)?

- if mild-to-moderate > No antibiotic treatment or if used give amoxicillin or bactrim ds - Severe disease > Must have two or more of the following: increased dyspnea, increased sputum production, and increased sputum purulence 1. Inhaled anticholinergic bronchodilator plus oral corticosteroid, taper over 2 weeks

Important Digoxin Drug-Drug and Drug-Disease Interactions

- reduce digoxin dose by 50% if patient is on amiodarone or dronedarone - Hypokalemia ( < 3.5 mEg/L), hypomagnesemia, hypercalcemia. Increase risk of digoxin toxicity. - * Hypothyroidism can increase digoxin levels *

What will happen if patient has blood glucose of < 20mg/dL/

- seizures - coma - death can occur

Clonidine ER (Kapvay)

- start 0.1 mg QHS , titrate weekly, using BID dosing SE - Headache, somnolence, etc. NOTE - Rebound hypertension (with sweating/anxiety/tremors), if stopped aburptly > Must tapter. ER fomulation has less SE * Do not crush *

Varenicline (Chantix)

- start 1 week before quit date BW - Serious neuropsychiatric events including depression, etc. Warning - Angioedema, hypersensitivty rxns, seizures, increase intoxicating effect of alcohol, and serious skin reactions have occured. - use caution in patients with underlying psychiatric disorders and while driving or operating machinery, traffic accidents have occured. Avoid use in pilot, etc. SE - Nausea , constipation, etc > Take w/ food to reduce nausea Note MedGuide required

What are the symptoms of aseptic meningitis (due to Lamtorigene)

- stiff neck - headache - fever, abnormal sensitivity to light - muscle pains, chills and/or confusion

What you do if you miss a dose of Apixaban?

- the miss dose should be taken as soon as possible on the same day and twice daily administration should be resumed. The dose should not be doubled to make up for a missed dose.

Antacids- Cautions

- use caution with administration of calcium carbonate, iron, itraconazole, ketoconazole, the Atelvia formulations of risedronate, dasatinib, bosutinib, erlotinib, nilotinib, pazopanib, rilpirvirine, and atazanvir

Why Chest guidelines for stroke prevention in non-valvular AFib recommend Dabigartan instead of warfarin?

-Dabigatran prevents 5 more strokes per 1000 patients/year than warfarin

What should be given with methotrexate on weekly based doses to decrease side effects associated with methotrexate?

-Folic Acid

Potamine 10mg/mL

1 mg protamine will reverse around 100 units of heparin > Reverse the amount of heparin given in the last 2-2.5 hours; max dose: 50mg > Enoxaparin given within last 8 hours >> 1mg protamine will neutralize 1mg of enoxaparin > Enoxaparin given > 8 hours ago: >> give 0.5mg protamine per 1mg of exnoxaparin given * Give 1mg protamine for each 100 anti-Xa units of dalteparin *

* Must know about Grapefruit Juice/Fruit interactions *

1. Big inhibitor 2. Concurrent use with grapefruit and some drugs (including LSA: Lovastatin, Simvastatin and Atrovastatin) will cause increase in the drug concentration which may or many not be clinically relevant 3. Concurrent use with Rivaroxaban or ticagrelor will cause increase in bleeding risk 4. QT prolongation with Lurasidone, quinidine and many others

Rank the loop diuretic from hight potency 1 to lowest

1. Bumetanide 1mg equalt 2. Torsemide 20mg 3. Furosemide 40mg 4. Ethacrynic Acid (Edecrin) 50mg

What dose should you start a healthy outpatient?

10mg daily for the first 2 days, then adjust dose per INR values. Doses of <= 5mg may be an appropriate starting dose for those who are elderly, malnourished, taking drugs which can increase warfarin levels, liver disease, heart failure, or have a high risk of bleeding _ Highly protein bound

OTC for constipation for pregnancy

1st recommend increasing fluid intake, increasing fiber int he diet, increasing physical activity such as walking. If this does not work *fiber is 1st-line* and *psyllium* is pregnancy category B (Metamucil or store brands)

Neuraminidase

- Inhibit the neuraminidase enzyme which affects the release of viral particles, thereby reducing the amount of virus in the body. active against influenza A and B - Tamiflu - Oseltamivir > Treatment > 12 years: 75mg BID for 5 days > Prophylaxis > 12 years: 75mg QD for 10 days Warning > CNS SE > Vomiting Notes > Start within 48 hours hours of symptoms or contact with an infected individual - Zanamivir (Relenza Diskhaler) > Treatment 7 years or old >> 10mg (two 5mg inhalation) BID x 5 days > Prophylaxis 5 years or older >> 10mg qd x 10 days (household) or 28 days (coumunity) Warning > Bronchospasm risk Notes > Start within 48 hours of symptoms or contact with an infected individual

Fentanyl (Duragesic) CII

- Injection, Actiq SL lozenege , Onsolis SL film, abstral and Fentora SL pills. - All fentanyl transmucosal forms are REMS drugs except patch and injection - IV fentanyl include Alfentanil (Alfenta), Remifentanil (Ultiva) and Sufentanil (Sufenta) - Patch > 12 (delivers 12.5mcg/hr) > Change Q 72 hours > Do not increase dose if pain is controlled but does not last long enough - in this case you shorten the interval - Any form of fentanyl is for chronic pain only > Do not stop other analgesic when you put patient on fentanyl instead just decrease dose 50% and the reason is because it takes 8-16 hours for fentanyl patch to show effects. - Do not apply > 1 patch each time and do not heat up patch or skin area before applying - Do not cover with heating pad or any bandage > Have patient call healthcare providers if they have a fever.

Initiating Insulin Therapy for Patients with Type 1 Diabetes

- Insulin analogs are preferred to reduce hypoglycemia risk and mimic the physiologic pattern of the insulin made by our body - Insulin should be started at a total daily dose (TDD) of 0.6 units/kg/day. If using rapid-acting and basal insulins, known as a basal-bolus strategy (preferred as there are insulin analogs), 50% of the TDD is used as the basal insulin dose and 50% of the TDD is used as the rapid-acting (bolus) insulin - If using NPH and regular insulins, take 2/3 of the TDD as the intermediate-acting (NPH) dose and 1/3 as the regular insulin dose. There are generally doses twice daily 30 minutes prior to breakfast and dinner (evening meal)

Type 2 Diabetes

- Insulin resistance and relative insulin deficiency with progressively lower insulin secretion over time - Can be managed with lifestyle modificaitons, oral medicaitons, and/or injections including insulin - Strongly associated with obesity, physical inactivity and family history of diabetes

There are 3 categories of transmission-based precautions defined by CDC. What you must know about the first one (Contact precautions)?

- Intended to prevent transmission of infectious agent which are spread by direct and indirect contact with the patient and patient's environment - Single patient room is preferred. if not then keep >3 feet spatial separation between beds to prevent inadvertent sharing of items between patients - Healthcare personal should wear gown and gloves for all interactions that may involve with patient or contaminated areas in the patient's room - * contact precautions are recommended for patients colonized with MRSA and VRE *

What is the treatment option for UC and CD [MILD]

- Loperamide - Some times the antispasmodic Bentyl can be usefull > Anticholinergic - Short course of oral or IV steroids are used to treat acute exacerbations in both UC and CD - In UC aminosalicylates are used for maintenance therapy to control inflammation and reduce flare-ups > Mesalamine is the primary amino used > Sulfasalazine is used less commonly due to many side effects with the sulfapyridine component > Efficacy of aminosalicylates have been proven in UC but not in CD so not recommended in CD

These agents are used as Antidiarrheals in MILD symptom control

- Loperamide (Imodium) - Bismuth subsalicylate (Pepto-Bismol) - Diphenoxylate 2.5 mg with atropine 0.025 mg (Lomotil)

What is the SE associated with Rx Amitiza an what is the generic name?

- Lubiprostone > Nausea, Diarrhea, Headache, Hypokalemia

Vinca Alkaloids

- M-phase specific: inhibit microtubule function (destabilizers) - Vincristine, Viblastine (Velban), VinoRelbine (Navelibine) - VinCRIStine (Vincasar) > Max 2mg per cycle (due to neurotoxicity) - These agents are Vesicants > Administer with central line - use warm compress and hyaluronidase if extravasation occurs - IV only, accidental intrathecal administration is fatal - * Vincristine is NOT myelosuppresive * BW - Intrathecal administration (fatal) ,etc SE - Cumulative (dose-dependent) peripheral neuropathy, constipaton ,etc

Metformin (Glucophage, Glucophage XR, Fortamet, Glumetza, Riomet)

- MOA > Metformin works primarily by decreasing hepatic glucose production, decreasing insulin absorption of glucose and increase insulin sensitivity - 1st line therapy per ADA unless CI - Start 500mg BID or 850g daily and Titrate to 1.5-2gram daily - ER (Swallow whole; do not crush, break or chew) - BBW > Lactic acidosis, Increase risk in acute HF, dehydration, excessive alcohol intake, hepatic/renal impairment or sepsis. - CI in SCr >= 1.5 mg/dL (males) or >=1.4 mg/dL (females), Metabolic acidosis, Temporary D/C in patients receiving IV iodinated contrast media SE - B12 defieicny - Weight neutral (some ppl might think they lost weight) - D/N/V/Flatulence - Etc

Antacids

- MOA > Neutralize gastric acid (producing salt and water) thus increasing gastric pH. This provide relief within minutes since antacids do not require systemic absorption. > Work quickly but last 30-60 minutes. - Facts > Alka-seltzer contain Sodium bicarbonate + Aspirin + citric acid Warning - Aluminium and magnesium can accumulate with severe renal dysfunction. Use is not recommended in patients with CrCl < 30ml/min Side effects - Calcium can cause constipation and loose stools - Aluminum may cause constipation - Magnesium may cause loose stools (may use together with aluminum to counter constipation) > Maalox, Roalids contain both Mg and AL

Phenobarbital and Primidone CIV

- MOA > These agents enhance gamma aminobutyric acid (GABA)-mediated chloride influx - Primidone (Mysoline) is the prodrug of phenobarbital and phenylethylmalonamide (PEMA) - both are active metabolites - CI > Makred hepatic impairment, dyspnea or airway obstruction, SC administration SE - Somnolence, cognitive impairment, dizziness/ataxia, physiological dependence, tolerance, hangover effect, depression, vitamin D and calcium deficiency (bone loss), respiratory depression, folate deficiency NOTE - Supplementation with Ca and Vitamin D recommended - Strong CYP450 enzyme inducers

Oxcarbazepine (Trileptal, Oxtellar XR)

- MOA > Voltage-sensitive Na channel blocker, inhibit repetitive firing and reducing the propogation of synaptic impulse. Oxcarbazepine is a prodrug that converts to active 10-monohydroxy derivative (MHD) - FDA for Partial seizure - Carbamazepine to oxcarbazepine dose conversion > 1.2-1.5 x carbamazepine dose - ER take on empty stomach - Warning > 25-30% cross-sensitivity to oxcarbazepine > Increae risk for SJS/TEN, consider screening patient with Asian descent for HLA-B1502 prior to initiating therapy > Hyponaremia (more than carbamezpine)

Important information about the treatment of UTI

- Must treat bacteriuria in pregnant women (for 7 days) even if asymptomatic with negative urinalysis - In pregnant women, avoid quinolones (cartilage toxicity and arthropathies) and tetracyclines (teratogenic). SMX/TMP can cause hyperbilirubinemia and kernicterus in 3rd trimester (pregnancy D); otherwise category C. - Generally, beta-lactams are used (amox/clav or oral cephalosporins) and, if penicillin-allergic, nitrofurantoin can be used.

What are the symptoms of Hepatic Encephalopathy?

- Musty odor of the breath - Changes in thinking, confusion, forgetfulness - Mood changes, poor concentration, drowsiness, - Hand tremor - Sluggish movement - many other, including risk of coma

What are the side effects associated with Statins?

- Myalgias - Arthralgias - Myopathy - Diarrhea, Increase in CPK - Rhabdomyolysis (increase risk with higher doses) - Cognitive impairment (memory loss, confusion-reversible) - Increase in blood glucose - Increase in A1C, and LFT

Important abot Estrogen usage in Osteoporosis patients

- No longer used first-line for osteoporosis because of health risk. - Estrogen with or without a progestin can be used for osteoporosis prevention in post-menopausal women, primarily in women with vasomotor symptoms of menopause and should be given at the lowest possible dose and only for up to 2 years - Conjugated estrogens/bzedoxifene (DUAVEE) contains estrogen plus an estrogen agonist/antagonist (Bazedoxifine) which is also used for prevention of osteoporosis in post-menopausal women with a uterues. - Bazedoxifene prevents endometrial hyperplasia in women with a uterus, just like progestin. - Raloxifene can be used for treating or preventing osteoporosis and is used most commonly in postmenopausal women who are at risk or have fear of breast cancer.

Anthracyclines

- Non-cell cycle specific > works by several mechanisms, including intercallation into DNA, inhibiting topoisomerase II, and creating free radical that damage cells - Very effective but limited use due to cardiac toxicity and N/V - Vesicant > High risk of severe tissue damage with extravasation >> antidote is dexrazoxane (Totect) or dimethyl sulfoxide (DO NOT USE BOTH) - Liposomal product have higher incidence of hand-food syndrome and allergic reaction - require light protection during administration

Linezolid (Zyvox)

- Oxazolidinone class > Binds to bacterial 23S ribosomal RNA of the 50S subunit inhibiting bacterial translation and protein synthesis and is mainly bacteriostatic - Bacteriostatic - Cover MRSA, VRE - used for uncomplicated/complicated skin and soft-tissue infections - CI > Concurrent use or within 2 weeks of MAO inhibitors - Caution with diabetes patients on insulin or hypoglycemia agents SE > HA/Diarrhea - NOTES > IV to PO ratio is 1:1 > Store oral suspension at room temp and use within 21 days after reconstitution. Prior to administration mix gently by inverting bottle * do not shake * - IV: require light protection during administration

What is used for RSV prophylaxis?

- Palivizumab (Synagis) is a humanized monoclonal antibody indicated for the prevention of serious lower respiratory tract disease caused by RSV in children at high risk of the disease - AAP recommends considering palivizumab prophylaxis during RSV reason (Late fall, early winter, early spring) for infants born *before 29 weeks* gestation who are younger than 12 months at the start of RSV season - AAP also recommend considering palivizumab for infants with chronic illness [primarily congenital heart disease or chronic lung disease] - Paliviziumab is dosed monthly at 15mg/kg per dose - Infants should not receive more than 5 monthly doses during RSV season - It is also used in certain infants and children < 24 months with select medical conditions that affect respiration

Secondary Prevention of Stroke

- Patient with previous cardioembolic stroke (A-FIB) should be placed on anticoagulant therapy (Warfarin) for secondary prevention of stroke. - patient with out cardioembolic stroke, the use of antiplatelet agent rather than oral anticoagulant is recommended to reduce the risk of recurrent stroke. - Aspirin or Aspirin + ER dipyrimadole , or clopidogrel are all acceptable options for initial therapy.

Insulin to Carbohydrate Ratio (ICR)

- Patients taking meal time may be counting carbohydrates t better adjust the insulin dose for that meal and every person responds differently to insulin (some are more sensitive and some are less) - An insulin to carbohydrate ratio (ICR) is patient specific and can be calculated by the Rule of 500 (for rapid-acting insulins) or Rule of 450 (for regular insulin) 500/TDD = gram of carbs covered by 1 unit of rapid-acting insulin 450/TDD = grams of carbs covered by 1 unit regular insulin.

What is the treatment of Ascites?

- Patients with ascites due to portal hypertension should restrict dietary sodium intake to < 2 grams per day, avoid sodium-retaining medications including NSAIDs and use diuretics to increase fluid loss. Restriction of fluid is recommended only in patients with symptomatic severe hyponatremia ( serum Na <120 mEg/L - Diuretics therapy should be started with either Spironolactone mono-therapy or combination with furosemide. ( do not give furosemide by it self; it is ineffective) > * keep the ratio of furosemide and spironolactone in 40:100 ratio to maintain potassium balance

What is peak level and when to take the trough level of a drug?

- Peak level is the highest concentration in the blood the drug will reach and require time for the drug to distribute in the body's tissues - Trough level is the lowest concentration the drug will reach in the blood and is drawn right before the next dose

What you know about Flucytosin?

- Penetrate fungal cells and is converted to fluorouracil which compete with uracil interfering w/ fungal RNA and protein synthesis - Fungicidal - BBW > Caution w/ renal dysfunction - SE > Bone marrow suppression > Hepatitis > Increase in Scr and Bun

DA-Agonist - Patch

- Rotigotine - approved for RLS and PD - SE > *peripheral edema*, drowsiness, headache, fatigue, orthostasis, sleep disturbance (trouble initiating/maintaining sleep), hallucinations, application site (skin) reactions, hyperhidrosis, nausea, dyskinesias, arthralgias. NOTE - Apply once daily, same time each day - Do not apply o same site for at least 14 days - Do not apply heat source over patch - Remove patch in MRI, avoid if sensitivity/allergy to sulfites

Sickle Cell Disease

- SCD affect the hemoglobin in red blood cells. - Patients with SCD have a single base substitution on chromosome 11 in the DNA sequence of the gene encoding the beta chain. - The irregular shaped RBCs are unable to transport oxygen effectively and get stuck in smaller blood vessels. - When the tissue is deprived of O2 rich blood, it is ischemic, and painful - An acute episode of ischemic pain is called a sickle cell crisis or vaso-occlusive crisis (VOC) and is the hallmark of sickle cell disease. - If the pain is in the chest it is called acute chest syndrome ACS

What is the most important thing to know about Ramelteon?

- SE - Somnolence, dizziness Notes - Do not take with fatty food

Gabapentin (Neurontin)

- SE > Dizziness, somnolence, ataxia, peripheral edema, weight gain, Diplopia, blurred vision, dry mouth - Monitoring > Edema/weight gain, mental status, seizure frequency

What is the MOA of Vortioxetine (Brintellix)

- SE > Increase bleeding risk with concurrent use of anticoagulatns etc. > Same as other, but claim it has less sexual problems

Which drugs cause Clotting risk?

- SERMS - Bevacizumab (anti-VEGF agents) - Immunodoulators > Thalidomide > Lenalidamide > Pomalidomide

COC start day options

- Start on the Sunday following the onset of menses (will menstruate during the week - most common start is a Sunday start) - Start on 1st day of Menses - If COCs are started within 5 days after the start of the period, no back up method of birth control is needed; protection is immediate. If not within 5 days, use back-up for the first week of use.

Apixaban (Eliquis) - Switching from apixaban to warfarin

- Stop apixaban and begin a parenteral anticoagulant and warfarin when the next scheduled dose of apixaban would have been taken, discontinue the parenteral anticoagulant when INR reaches therapeutic range

Important information about Influenza vaccine

- The Live Attenuated Influenza Vaccine, Quadrivalent(LAIV4, nasal spray) is preferred over the inactivated influenza vaccine (IIV, flu shots) when immediately available for children 2-8 years of age. - All patients must be monitored for at least 15 minutes after vaccination. PRECAUCTIONS > Defer if patient has moderate to severe acute illness > Refer to physician if patient had Guillain-Barre within 6 weeks of a prior dose of flu vaccine > Those who get only hives after eating eggs can receive inactivated vaccine and be observed for *30 minutes* > Flumist Quadrivalent is given as 0.2 mL, divided between the two nostrils

Cough/Cold/Allergies for pregnancy OTC

- The first generation antihistamine are the usual first line recommendation. Chlorpheniramine (drug of choice) and diphenhydamine are pregnancy category B - The non-sedating 2nd generation loratadine and certizine are often recommend during 2nd and 3rd trimester > Do not recommend these just ask to see Doctor. - If nasal steroids are needed for chronic allergy symptoms, budesonide (Rhinocort) and beclomethasone (beconase AQ) considered safe but require Rx - Decongestant (pseudopherine, phenylphrine, oxymetazoline), the cough suppressant dextromepthorphan and mucolytic guaifenesin are category C but recommended by physician - * The oral decongestants should not be recommended during first trimester *

What is the treatment option for chronic Angina?

- To reduce risk of acute coronary syndome > Aspirin DOC > If allergy, move to Plavix > Use combination of ASA + Plavix in high risk pateitns - To relief pain > Beta Blockers are DOC > If BB are contraindicated then use CCB or Long acting nitroglycerin > use CCB or long acting when additional therapy is needed to subside the pain > Ranolazine can be used as a sub or in addition to BB therapy

List one drug that is IL-6 receptor antagonist

- Tocilizumab (Actemra) BBW > Risk of serious infections; screen for latent TB prior to initiating therapy WARNING > Do not start Tocilizumab (Actemra) if ALT or AST is > 1.5 times ULN, ANC < 2000 cells/mm3 or plateletes < 100,000 cells/mm3 SE > Increasae in LFT, infections, myelosuppresion, GI perforation, Increase in LDL and total cholesterol

Antacids - notes

- relief within minutes, last 30-60 minutes - * antacids are the drug of choice in pregnancy * - Alka seltzer Original contains > 500 mg Na and 325 mg aspirin per tab - Interactions > 2 hours before or 6 hours after cipro > 2 hours before or 2 hours after levo > 4 hours before or 8 hours after moxi > 1-2 hours before or 4 hours after Tetra > > > less concern with doxy and mino > Thyroid products should be separated from antacids by at least 4 hours

Loop diuretic use in HF

- used for symptoms (reduction in preload ) and restore euvolemia - work in the thick ascending limb of the loop of henle - They increase excretion of Na, Cl, Mg, ca and water - They don't alter the survival of heart failure patients; therefore the lowest effective dose should be used

Mannitol (Osmitrol)

- used to reduce ICP by withdrawing water from the brain - CI > Severe renal disease (anuria) > Severe dehydration, progressive heart failure and * pulmonary congestion * - Preferred to be given as intermittent boluses - Side Effects > Fluid and electrolyte loss, dehydration, hyperosmolar induced hyperkalemia, acidosis, increase Osmolar gap - Notes > Maintain serum osmolality < 300-320 mOsm/kg

SGAs

- weight gain, lipids and glucose abnormalities are common with clozapine, olanzapine and quetiapine. Risperidone and Paliperidone have moderat risk of metabolic side effect - * Abilify, Geodon, and lurasidone and asenapine have lowest risk of metabolic effects * - ADA recommend when initiating antipsychotic state that patient should first be screen for overweight and obesity, dyslipidemia and hyperglycemia, hypotension and personal or family history for risk.

List some drugs that cause Additive QT prolongation

-Class I (class Ia in particular) and class III antiarrhythmics > Amiodarone, disopyramide, dofetilide, dronedarone, ibutilide, procainamide, quinidine, sotalol, flecainide, lidocaine, and mexiletine. - Antibiotics > quinolones > macrolides > etc - Azole antifungals > Posaconazole > Voriconazole > Etc - Antidepressants including tricyclics , SSRI, SNRI > * Sertraline is preferred in cardiac patients* > etc. - Antiemetic agents including the 5-HT3 receptor antagonists > etc. - Antipsychotics - Oncology - Protease inhibitors - others > Fingolimod > methadone > etc.

Beta-1 blocker and Produces Nitric Oxide-Dependent Vasodilation

5-10mg daily; maximum is 40mg daily If CrL < 30 ml/min or moderate liver impairment, start 2.5mg daily SE - HA , Fatigue , hypertriglycermdia and decrease in HDL

What is the normal CD4+ T Lymphocyte count

800 - 1100 cells/mm3

HEPA filters remove .......... of all air particles ...... mm or larger

99.97% 0.3 mm

What are the 2 BBW associated with Teriflunomide (Aubagio)?

1. Severe liver toxicity 2. Teratogenicity

What does ACIP recommend when it comes to giving more than one vaccine together?

1. Simultaneous administration of all vaccines for which a child is eligible is very important in childhood vaccination programs. It increases the probability that a child will be fully immunized at the appropriate age 2. According to the ACIP, there are no contraindications to simultaneous administration of any of the vaccine currently available in the United States and every effort should be made to provide all necessary vaccinations at one visit to improve compliance.

List some of the patched that must be removed prior to MRI?

1. Testosterone (Androderm) 2. Clonidine (Catapres-TTS) 3. Fentanyl (duragesic, generics) 4. Rotigotine (Neupro) 5. Scopolamine (Transderm-scop) 6. Salonpas Power Plus (OTC) 7. Nicotine (NicoDerm CQ)

* Must know about MAO Is *

1. The non-selective MAO Is have drug interactions that can cause serotonin syndrome, hypertensive crisis, and potentially be fatal 2. Do not use MAO Is with ephedrine and analgos (Pseduoephedrine, etc), bupropion, buspirone, linezolid, lithium, merepidine, SSRI, SNRI , TCA, tramadol etc. 3. The selective MAO Is, the selegiline patch (at high dose) and rasagiline should not be used with tyramine rich food

* Important about the use of ARB-I to prevent the progression of nephropathy *

1. There is strong evidence to support the use of ACE inhibitors and ARBs to prevent the progression of nephropathy in diabetic and non-diabetic patients with proteinuria. 2. It is essential to control the blood pressure tightly. The goal blood pressure in kidney disease is < 140/90 mmHg if no proteinuria is present and < 130/80 mmHg if proteinuria is present. Glycemic control, if an issue will also need to tightly controlled to preserve kidney function

* Must know about Lamotrigine and Valproate *

1. This combination cause a severe rash 2. Any inhibitor of Lamotrigine will require a lower dose titration that is included in the packaging.

What is the reason behind separating black patients to different classes of drugs for HTN?

1. This group is at high risk for stroke 2. BP reduction is not as great with ACE-I or ARBs compared with other thiazide-type diuretic *unless* they have CKD and proteinuria.

What is the IV to PO ratio?

1:2

What is the furosemide IV:PO ratio?

1:2 Furosemide 20mg IV = furosemide 40mg PO - store at room temp - Do not use furosemide solutions if they are yellow in color; must be clear - Preg B

How much iron 4-6 months breast-fed babies need?

1mg/kg/day from 4-6 months old and until consuming iron-rich foods. At about 6 months most breast-fed babies get about half their calories from other food

What is the drug of choice for Dry eyes in SJOGREN'S SYNDROME?

1st try OTC artificial treats, if no improvement then move to Rx (cyclosporine emulsion eye drops - Restasis) Dosing > One drop in each eye BID 12 hours SE > Burning, stinging, etc NOTE > Remove contact lenses prior to administration, re-insert 15 minutes afterwards > Separate from artificial tears by 15 minutes

Drug interactions in organ transplant

1. Caution with additive drugs that are nephrotoxic with tacrolimus and cyclosporine 2. Caution with additive drugs that raise blood glucose with tacrolimus, steroids, cyclosporine and the mTOR inhibitors. 3. Caution with additive drugs that worsen lipids with mTOR inhibitors, steroids and cyclosporine. 4. Caution with additive drugs that raise blood pressure with steroids, cyclosporine and tacrolimus. 5. Mycophenolate cause diarrhea - good option is to switch to the Myfortic - Mycophenolate > Do not take with antacids or multivitamins concurrently. Separate the doses by 2 hours. Avoid use with bile acid resins. > Limit the amount of time you spend in sunlight > Mycophenolic acid (myfortic) and mycophenolate mofetil (cellcept) are not interchangeable. Do not switch between products unless directed by your healthcare provider. These medicines are absorbed differentily . This may affect the amount of medicine in the blood. - Cyclosporine may cause swelling and growth of the gums (gingival hyperplasia) brush your teeth and floss daily to minimize this problem.

How to treat SBP Spontaneous Bacterial Peritonitis?

1. Ceftriaxone for 5-7 days 2. Primary prophylaxis with Norfloxacin or bactrim DS to prevent SBP is indicated in some cases 3. If you do not have Norfloxacin then use cipro

What is the most common use of patient profile?

1. Check for allergies 2. Check for drug interactions

* Must know about Amiodarone *

1. Digoxin, Warfarin, Quinidine an Procanamide must be reduced by 30-50% when starting amiodarone 2. If warfarin and digoxin is on board first, the pharmacist must recognize the interaction and decrease the dose when amiodarone is started.

2 important things about Collection bin for ongoing collection at pharmacies for drug disposal?

1. Drugs should not be reviewed by staff at collection site (wether a pharmacy or community event) before being deposited into a secured collection bin 2. Drugs that are collected should be separated from their containers to reduce costs of disposal

* Must know about Digoxin *

1. Has a narrow therapeutic range > Therapeutic range for HF is 0.4 - 0.9 ng/mL (higher range for A-fib) > Therapeutic range for A-fib 0.8 - 2 ng/mL (lower range for HF) 2. Antidote: DigiFab 3. Digoxin levels increase due to renal function or hypokalemia 4. Be careful with other drugs that have additive effect on lowering heart rate (< 60 BPM). These are primarily beta blockers and the non-DHP calcium channel blockers (dilitiazem and verapamil), amiodarone, dexmetetomine (Percedex), clonidine, and opioids.

What is stroke?

A stroke , or cerebrovascular accident (CVA), occurs when blood flow to an area of the brain is interrupted by ischemia due to a clot (thrombus or emboli) or ruptured blood vessel (hemorrhage)

Beta 1 Selective Blockers

AMEBBA Atenolol, Metoprolol, Esmolol, Bisoprolol, Betaxolol, Acebutolol BOXED WARNING - Beta blockers should not be withdrawn abruptly (particularly in patients with CAD), gradually taper over 1-2 weeks to avoid acute tachycardia, HTN, and/or ischemia CI - Sinus bradycardia, 2nd or 3rd degree heart block, sick sinus syndrome (unless patient has a functioning artificial pacemaker) or cardiogenic shock. Do not initiate in patients with active asthma exacerbation SE - Decrease in HR, Hypotension, Fatigue, Dizziness, depression, decrease Libido, impotence, Hypertirglycerdiemia, decrease in HDL, etc. Monitoring - HR, BP ,etc.

Statin - Contraindications

Active liver disease Pregnancy Simvastatin Lovastatin and Pitavastatin are contraindicated with cyclosporine use

KYNAMRO (mipomersen sodium) injection 200 mg/mL can cause elevations in transaminases and increases in liver fat. Because of the risk of liver toxicity, KYNAMRO is available only through a restricted prescribing and distribution program called KYNAMRO REMS.MOA: Inhibitor of APO B-100 synthesis and APOB is the main component of LDL and VLDLD which is prescuros of LDL. * PCSK9 inhibitors Alirocumab (Praluen and Evolocumab (Repatha) expensie drugs *

Adding Juxtapid to your HoFH (homozygous familial hypercholesterolemia) treatment plan may further lower your bad (LDL) cholesterol level. In one study, there was an average additional decrease of 40% at 6 months. Also require REMS. (MOA for Lomitapide (Jux): Binds to and inhibit micromosla triglyceride transfer protein (MTP) in the endoplasmtic rectiuclum. MTP inhibiton prevent the assembly of apoB containing lipoproteins in enterocyte and hepatocyte resulting in reduced production of chylomicorns and VLDL and subsequently reduced plasma LDL concentrations

Why women with uterus require progestin giving with estrogen?

Adding progestin reduces the risk of endometrial cancer in a women with a uterus

Naproxen

All given BID VIMOVO > It has Naproxen and Esomeprazole >> The PPI is used to protect the gut from damage caused by the NSAIDs

Which of the following are common preservatives? A. Benzalkonium chloride (BAK) B. Benzyl alcohol C. Chlorhexidine D. Thimerosal (contain mercury) E. Sodium benzoate F. Benzethonium chloride G. propylparaben

All of them

It is important to distinguish between physiological adaptation and addiction.

All patients, including addicts become physiologically adapted to opioid after repeated exposures. By contrast, addiction involves a strong desire or compulsion to take the drug despite harm and it is manifested by drug-seeking behavior (exaggerated physical problems) or criminal activity (Rx forgery or theft)

What is the difference between Wild-type allele and Variant allele?

Allele is the specific form of a gene. The gene have different forms because each person has two alleles (one from the mother and one from the father) for each gene. Alleles are also described as wild-type or variants. A wild type allele is the usual (or *normal*) sequence. A mutation and/or polymorphism is referred to as a variant allele.

What is the difference between allograft vs Isograft and Autograf?

Allograft: is the transplant of an organ or tissue from one individual to another of the same species with a different genotype. Isograft: A transplanted organ from a genetically identical donor ( such as an identical twin) Autologous transplant: An autograft is when a tissue is transplant from one site to another on the same patient.

Which drug can help with dyskinesias in later stage disease due to it's antagonism of the NMDA receptor?

Amanatadine

What should be given if patient receiving Cisplatin?

Amifostine (Ethyol) may be used to reduce the risk of cisplatin induced renal toxicity * Do not exceed maximum dose of 100mg/m2/cycle for cisplatin *

List the common drugs that are only compatible with DEXTROSE?

Amphotericin B (Amphotec) Carfilzomib (Kyprolis) Lorazepam (Ativan) Mycophenolate (CellCept IV) Pentamidine Quinupristin/Dalfoprisitin (Synercid) Sulfamethoxazole/Trimethoprim(Bactrim)

List the common drugs that are only compatible with Saline?

Ampicillin Caspofungin (Cancidas) NS or LR Daptomycin (Cubicin) NS or LR Phenytoin Ertapenem (Invanz) Infliximab (Remicade)

List one Prospectively analysis to reduce medication errors?

An analysis can also be done prospectively to identify pathways that could lead to errors and to identify ways to reduce the error risk. Failure mode and effects analysis (FMEA) is a proactive method used to reduce the frequency and consequences of errors.

What is antibiogram?

An antibiogram is a chart that contains the susceptibility patterns of local bacterial isolates to antimicrobial agents at a single institution over a specific period of time (generally 1 year)

What is enzyme-substrate system?

An enzyme is a protein that performs some action. The substrate is a chemical that is acted upon. Drug molecules, foods, and toxins are substrates for CYP enzymes.

What is polymorphism?

An inherited variations in the DNA sequence. Polymorphism occurs with fairly high frequency, but most do not result in a change in phenotype.

List a drug that is IL-1 receptor antagonist (NON-TNF biologics)

Anakinra (Kineret) WARNING > Screen for latent TB prior to initiating > others

Which class of HTN cause Angioedema?

Angioedema is more common in black patients. Although angioedema is more likely with ACE inhibitors than ARBs or aliskiren if a patient has had angioedema with these classes of agents, all others are contraindicated since angioedema can become fatal. * Counsel patients to report any swelling of lips, mouth, tongue, face, or neck immediately *

* Must know about Chealtion Risk with Quinolones, and Tetracycline *

Antacids, Didanosine, Sucralfate, Bile acid resins, Magnesium, Aluminum, Calcium, Iron, Zinc, Multivitamins, or any product containing these multivalent cations can chelate and inhibit absorption: The quinolones separation times vary. The tetracycline class (including doxycycline and minocycline) have the chelation interaction and require separation)

Enoxaparin [Monitoring]

Anti-Xa levels can be used to monitor, but not routine in the general patient population. Monitoring is recommended in pregnancy and in patient with mechanical heart valves. Monitoring may be done in obesity, low body weight, pediatrics, elderly, or renal insufficiency. aPTT is not used. Obtain peak anti-Xa level 4 hours post dose.

When is anti-hypertensive treatment is recommended in pregnant patients?

Antihypertensive therapy is recommended in pregnant patient with SBP >= 160 mmHg or DBP >= 105 mmHg. - First line treatment > Labetolol > Methyldopa (safety issues) > Nifedipine (ER) - Goal is 120-160 and 80-105

What is the requirements of Folic Acid for any woman planning to conceive?

Any woman planning to conceive (and all women of child-bearing age) should be taking a folic acid supplement (400-800 mcg/daily, which is 0.4-0.8 mg/daily) to help prevent birth defects of the brain and spinal cord (neural tube defects)

Lidocaine 5% patches (lidoderm)

Approved for postherpetic neuralgia (Shingles) NOTES > Can cut into small pieces before removing backing > Do not apply more than 3 patches at one time

What is the 2nd line for APL if Tretinoin if patient is contraindicated ?

Arsenic trioxide (Trisenox) - QT prolongation > Monitor ECG, avoid concurrent QT prolonging agents, keep Mg2 , Ca2, K within normal range. > APL differentiation syndrome BW - RA-APL differentiation syndrome ,etc. SE - QT prolongaton , etc.

Which of the following can be put down a G-tube? A. Kadian B. Avinza

Avinza (morphine ER)

What is the DOC for Chlamydial infections caused by Chlamydia trachomatis, intracellular obligate parasite?

Azithromycin 1 gram PO x 1 Or Doxycycline 100mg po BID for 7 days PLUS Ceftriaxone if co-infected with gonorrhea

Important

Azithromycin 2 g PO x 1 is effective to treat both gonorrhea and chlamydia but poorly tolerated (GI effects)

Which of the following must be discarded 45 days after opening pouch, when device locks out, or when dose indicator displays "0: whichever comes first? A. combivent respimat B. Tudorza C. DuoNeb D. Spririva

B

What is the function of Inducer? A. Increase the production of the enzyme (by increasing the expression of the gene sequence that codes for the enzyme) B. Increase the activity of the enzyme

A and B are correct

What is sleep apnea?

A potentially serious sleep disorder in which breathing repeatedly stops and starts.

What is absence seizures?

A type of generalized seizures that is characterized by brief and abrupt staring spells lasting 10-30 seconds.

Which of the following alleles are/is wild type and provide normal enzyme activity when it comes to metabolize of Warfarin? A. CYP2C9-1 B. CYP2C9-2 C. CYP2C9-3 D. A and C

A. CYP2C9*1 is the wild-type allele and results in normal enzyme The CYP2C9-2 and the 2C9-3 alleles are decreased function alleles. This results in elevated S-warfarin concentration leading to increased bleeding

What is the best option for dehydration? A. Dextrose 5% B. NaCl 0.9% C. Lactated Ringers D. Multiple Electrolyte Injection

A. Dextrose 5%

Which of the following levigating agents are the best to prepare aqueous systems (O/W dispersions)? A. Glycerin B. Mineral oil C. Castor oil D. Cottonseed oil E. Tween 80

A. Glycerin > Rest are good levigating agents used to prepare oleaginous systems (W/O) > Additional agents used as levigating agents to prepare aqueous systems consist of Propylene glycol, polyethylene glycol 80

When do you consider the drug studies in a new clinical trial is bad? A. RR = 1 B. RR < 1 C. RR > 1

Answer: C. RR > 1 Explanation: RR = 1: No difference in risk between the 2 groups RR < 1: fewer events are occurring in the treatment group compared to the control group RR > 1: More events are occurring in the treatment group compared to the control group

Apixaban (Eliquis)

BBW 2 1. Pateitns receiving neuraxial anesthesia (epideral, spinal) or undergoing spinal puncture are at risk of hematomas and subsequent paralysis 2. Premature discontinuation of apixaban increases the risk of thrombotic events Contraindication - Active pathological bleed Warning - Use is not recommended with prosthetic heart valve or severe hepatic impairment Monitoring Renal function prior to initiation, when clinically indicated and at least annually in all patients; hepatic function and signs of bleeding SE - Bleeding, anemia Notes - Pregnancy B - No antidote - No monitoring of efficacy required - D/C 48 hours prior to elective surgery in high risk bleeding patients and 24 hours prior to elective surgery with low bleeding risk

Cidovovir (Vistide) - Treatment of CMV in HIV only

BBW 3 1. Dose-dependent nephropathy 2. Neutropenia 3. Carinogenic/teratogenic CI - SCr > 1.5 mg/dL , Urine protein >100mg/dl (<2+ proteinuria), sulfa allergy, use with or within 7 days of other nephrotoxic drugs, direct intraocular injection - SIE > Nephrotoxicity

Where to check IV stability/Compatibility?

AHFS drug information King Guide Micromedex Product package insert Trissel's handbook on injectable drugs

All of the following are SE associated with Gabapentin Except? A. Dizzines, Somnolence B. Peripheral edema, weight gain C. Ataxia, Diplopia D. Blurred vision, dry mouth, mild euphoria

ALL ARE TRUE

All of the following are SE associated with Lyrica Except? A. Dizzines, Somnolence B. Peripheral edema, weight gain C. Ataxia, Diplopia D. Blurred vision, dry mouth, mild euphoria

ALL OF THEM ARE TRUE

CCR5 Antagonist

CCR5 inhibitors bind to the CCR5 co-receptor on the CD4+ cell and prevent HIV cell entry. Unlike others, CCR5 inhibitors do not directly target the HIV cell but rather blocks the human host cell receptor Maraviroc (Selzentry) BBW - Hepatotoxicity with allergic type features CI - Crcl < 30 mL/min - Taking potent 3A4 inhibitor/inducer SE - URTIs, fever, rash (including SJS), cough, musculoskeletal symptoms, dizziness, orthostatic hypotension, etc. Monitoring - Prior to starting therapy, patients must undergo a screening test (Trofile), to determine the tropism of the HIV since this agent will only work for patients with CCR5-tropic disease NOTE - Swallow tablet whole, do not chew, break or crush

What is the major laboratory indicator of immune function in patient infected with HIV and serve as a key factor in determining both the urgency of antiretroviral therapy (ART) initiation and the need for prophylaxis against opportunistic infections (OIs).

CD4+ Count

In which gene does mutation occur in cystic fibrosis?

CF is caused by a mutation in the gene for the protein cystic fibrosis transmembrane conductance regulator (CFTR) > This protein required to regular the components of sweat, digestive juices, and mucus.

Isosorbide mononitrate (Monoket)

CI - CI with PDE-5 inhibitors SE - Headache, dizziness, lightheadedness , flushing, hypotension, tachyphylaxis (need 10-12 hour nitrate free interval), syncope Monitoring HR, BP, signs and symptoms of HF

Rifampin (Rifadin)

CI - Concurrent use with PIs (switch to rifabutin) SE - Increase LFT, GI upset, Rash/Pruritus, Orange-Red discoloration of body secretions, flu-like syndrome, positive coombs test.

MAO I B selective inhibitor

CI - D/C at least 10 days prior to elective surgery requiring general anesthesia, do not use with local anesthesia containing sympathomimetics vasoconstrictors, or food high in tyramine.

Dornase alfa (Pulmozyme)

CI - Hypersensitivity to chinese Hamster Ovary SE - Chest pain, fever, rash, rhinitis, laryngitis, voice alteration, throat irritation NOTE - Store the ampules in the refrigerator ( do not exposre to room temperature >= 24 hours) - Do not mix with any other drug in the nebulizer - Protect from light

Bisphosphonates (ORAL)

CI - Inability to stand or sit upright for at least 30 minutes (60 minutes with once-monhtly Boniva), difficulty swallowing, esophageal stricture, or at high risk of aspiration, Hypocalcemia. SE > Hypocalcemia (mild, transient), musculoskeletal pain, abdominal pain, dyspepsia, N/V, dysphagia, heartburn, esophagitis, skin rash, eye inflammation NOTE > Binosto (effervescen alendronate) contains 650 mg Na+. Avoid in sodium restriced patients such as those with CHF and HTN ec.

2nd generation Antihistamine

CI - Levocertirizine (xyzal) > End stage renal disease, hemodialysis, infants and children 6 months to 11 years of age with renal impairment

NRT

CI - Recent MI (within 2 weeks), life-threating arrhythmia, severe or worsening angina, pregnancy < check on this Side Effect Notes - Patient must show identification for proof of age prior to purchase of nicotine products since the FDA prohibits sale of nicotine products to individuals younger than 18 years of age (REMS) - Gum is good option for patients who want to reduce or delay weight gain - Inhaler has a hand to mouth use; mimics smoking action, providing a coping mechanism - Nasal are the fastest; highest dependence potential among NRT. Avoid in severe reactive airway disease - Patch is typically worn for 24 hours; however can remove at bedtime to avoid insomnia

Febuxostat (Uloric)

CI > Concomitant with Azathioprine or mercaptopurine Warning > Hepatotoxicity > Possible increase in thromoembolic events SE > increase in LFT and rash Monitoring > LFTs at 2 months, 4 months, then periodically Good thing about this drug NO dose reduction necessary in renal impairment

Thiazide-type diuretics

CI > Hypersensitivity to sulfanomide-derived drugs > Anuria > Renal decompensaton Warning > Sulfa allergy > Electrolyte disturbance > Gout can precipitated Side Effects > Hypokalemia > Hyperuricemia (UA) > Elevated lipids (LDL and TG) > Hyperglycemia (BG) > Hypercalcemia (Ca) > Hyponatremia > Hypomagnesemia > Dizziness, photosensitivity, rash

Prostacyclin Analogues (or Prostanoids)

CI > Use of oral treprostinil in Child-Pugh class C hepatic impairment Side Effects - During dose titration > Vasodilation (hypotension, headache, flushing, if happens decrease dose) > Nausea, vomiting, diarrhea, anxiety, chest pain/palpitations, tachycardia, edema and jaw claudication - During chronic use > Anxiety, flu-like symptoms , thrombocytopenia - Treprostinil and illoprost (inhaled) > cough - Treprostinil (oral) > GI adverse effects NOTE - Avoid interruptions in therapy - Avoid large, sudden reduction or increase in dose - Remodulin: SC very painfull - The parenteral agents are considered most potent of all PATH!!!!!!!!!!!

Fibrinolytics

CI (Absolute) - Active bleeding or bleeding diathesis - Any prior intracranial hemorrhage - Recent intracranial or intraspina surgery or trauma (last 3 months) - Intracranial neoplasm, arteriovenous malformation, or aneurysm - Arotic dissection - Sever uncontrolled hypertension - Ischemic stroke within past 3 months, except acute ischemic stroke within 4.5 hrs. Relative - Pregnancy - Active peptic ulcer - Current use of anticoagulant SE - Bleeding, hypotension, intracranial hemorrhage, fever NOTE * Door-to-needle time should be < 30 mins (for fibrinolytcis)

List 3 drugs that are highly protein-bound?

Calcium Valproate Phenytoin -> During lab usually protein-bound levels are given and if this is what we have then we need to use formula to adjust the levels for low albumin and determine the true level of these drugs. - Ceftriaxone > These drugs are not removed by dialysis procedures.

Agents used to treat extended spectrum beta-lactamase producing enteric gram-negative rods (ESBL GNR)- E.coli, Klebsiella pneumonie, P.mirabilis?

Carbapenems

Case-Controlled study

Case-Control studies are good for studying rare diseases or outcomes, can be conducted in less time since the condition has already occurred, and are useful to establish an association. They are often used to generate hypotheses that can then be studied via a prospective cohort or other studies.

Bile Acid Sequestrants/Bile Acid Binding Resins

Cholestryamine Colesevelam (Welchol) Colestipol Contraindications > Cholestyramine - complete biliary obstruction > Colesvelam - Bowel obstruction (TG > 500mg/dL; history of hypertriglyceridemia-induced pancreatitis.

What is the DOC for Bacterial vaginosis in pregnancy?

Clindamycin 300mg poBID or metronidazole 500 PO BID. all for 7 days * Topical therapy for bacterial vaginosis is not recommended during pregnancy *

What is the difference between Dobutamine and Milrinone?

Dobutamine is beta agonist; incresae heart rate and force of contraction which increases cardiac output Milrinone is a selective phosphodiesterse-3 inhibitor in cardiac and vascular tissue. it produces inotropic effects with vasodilation and *less chronotropic effect compared to dobutamine*

All of the following put patient at risk of bleeding especially if on Warfarin. Which of them increase INR? A. Ginkgo Biloba (Dementia) B. Bromelains (indigestion) C. Danshen (Prevent stroke) D. Dong Quai E. Vitamin E G. High doses of Fish Oils H. Garlic I. Ginseng J. Glucosamine K. Grapefruit L. Policosanol (treat leg pain) M. Willow bark (treat low back pain)

Dong Quai Increase INR Glucosamine may raise INR Danshen increase INR Grapefruit increase INR Wintergreen oil increase INR

What is the DOC in health failure patient with SBP < 90?

Dopamine > Inotropes are associated with worse outcomes and should be discontinued once the patient is stabilized.

What is the DOC for CMV?

Cytomegalovirus (CMV) - Ganciclovir 5mg/kg IV q 12 or - Valganciclovir 900mg PO Q 12 H for 21 days

Increase in cAMP by PDE-4 inhibitors is very important in COPD because it will: A. cause vasodilation in the lungs B. cause vasoconstriction in the lungs C. cause vasoconstriction in the penis D. reduction lung inflammation (select all that apply)

D

Which of the following is not approved to be used as Augment AD therapy? A. Aripiprazole (Abilify) B. Olanzapine/Fluoextine (Symbyax) C. Quetiapine extended release D. Quetiapine immediate release

D

What is the purpose of Phase IV?

During clinical studies the population is limited to few hundreds and the complete safety profile may be missed. For this reason the FDA may request Phase IV to examine the risk and benefits of the new drug in a different population or to conduct special monitoring in a high risk population.

All of the following are brand name for Aspirin EXCEPT? A. Bayer B. Ascriptin C. Bufferin D. Ecotrin E. All of the above are Aspirin

E

All of the following are true about Aspirin except? A. anti-platelet B. Antipyretic C. analgesic D. anti-inflammatory . E. all are true about Aspirin

E

All of the following are recommended for long distance travelers who are at risk for VTE except ? A. frequent ambulation B. calf muscle exercise C. Sit in an aisle seat D. use graduated compression stocking with 15-30 mmHg of pressure at the ankle during travel E. Aspirin 81mg before flight. F. Antiocoagulant

E F

Morphine

ER brands: MS contin, AVINza, Kadian, Oramorph SR, Roxanol - CII - IR DOSE > 10-30mg Q4H PRN - Do not crush or chew any ER or CR opioids - Avinza > No alcohol, can shorten extended release duration. Can sprinkle on applesauce, soft food - Kadian > Can be opened and can sprinkle on applesauce, soft food - If renally impaired, start at a lower dose, or avoid morphine, oxycodone, tramadol, or tapentadol which are all renally cleared

What is ECHO model?

Economic outcomes: Include direct, indirect and intangible cost of the drug compared to a medical intervention Clinical outcomes; Include medical events that occur as a result of the treatment or intervention Humanistic Outcomes: Include consequences of the disease or treatment as reported by the patient or caregiver (patient satisfaction, quality of life )

What is the IV and IM ratio strength?

Epinephrine used for IV route is 0.1mg/ml or 1:10,000 ratio strength Epinephrine used for the IM route is 1:1000 ratio strength * Protect norepinephrine from light *

ESAs

Epoetin alfa (Epogen, Procrit) Darbepoetin (Aranesp) CKD - Initiate when Hgb < 10g/dl - Reduce or interrupt dose when Hgb approached or exceeds 11g/dl for CKD on HD or > 10g/dl for CKD not on HD Cancer - Initiate when Hgb < 10g/dl and when at least 2 additional months of chemotherapy planned Epoetin alfa (Epogen, Procrit) - If Hgb increases > 1g/dl in any 2 weeks Periods interrupt or decreasae dose by 25% for CKD and cancer patients Darbepoetin (Aranesp) - If Hgb increases > 1g/dl in any 2 week periods, decrease dose by 25% for CKD patient , 40% for cancer patients.

Oral Iron Therapy

Ferrous sulfate > 325 mg PO daily to TID (65 mg elemental iron) > most commonly used and is least expensive Ferrous fumarate (Ferretts, Hemocyte) Ferrous gluconate (Fergon) Ferrous sulfate, dried (exsiccated) Controlled release (Slow Fe, Feosol) CI Hemochromatosis, hemolytic anemia Warning - Accidental overdose of iron-containing products is leading cause of fatal poising in children under 6. keep iron out of reach of children SE - N, stomach upset, constipation (dose related), dark and tarry stool NOTE - Although fiber is first line treatment for constipation, a stool softener such as docusate is often recommended for iron-induced constipation.

What is the ANTIDOTE for Ethylene Glycol, Diethylene glycol, Methanol?

Fomepizole (Antizol) Preferred or Ethanol (2nd line) -> Femepizole and ethanol inhibit alcohol dehydrogenase (ADH) -> Methanol or Ethylene glycol toxicity should be suspected with anion-gap metabolic acidosis along with an osmolar gap

Nimodipine

For oral administration ONLY. > Start therapy within 96 hours of the onset of subarachnoid hemorrhage > Administer on an empty stomach at least 1 hour before or 2 hours after meals BBW - Nimodipine has inadvertently been administered IV when withdrawn from capsules into a syringe for subsequent nasogastric administration. severe cardiovascular adverse events including death have occurred. NOTE PO ONLY

What are some weird stuff that can develop due to iron deficiency anemia?

Glossitis > Inflammation of the tongue Koilonychias > Spoon shaped nails Pic > Craving and eating non-foods such as chalk or clay

Which testing are required/strongly recommended before the initiation of the following drugs: Abacavir (Ziagan) Abacavair + lamivudine (Epzicom) Abacavair + zidovudine + lamivudine (Trizivir)

HLA-B*5701 -> If positive, avoid use of abacavir -> Hypersensitivity to abacavir is multi-organ clinical syndrome -> D/C as soon as a hypersensitivity reaction is suspected

Why is it important for olanzapine to have IR ODT short-acting and long acting formulation?

IR: it was the original tablet ODT: To prevent cheeking, these formulation are useful to block the patient from hiding the medication in the mouth Long acting injection: The majority of these patients discontinue medication so this is helpful to improve adherence Short acting injection: Works quickly and is useful for acute agitation.

Which institutions promote the use of tall man letters as one mean of reducing confusion between similar drug names?

ISMP FDA TJC Other safety-conscious organizations

Where can a pharmacist find errors that occurred in the past to help prevent these errors from occurring at his/her setting?

ISMP The institute for safe medication practices (ISMP) * The ISMP national medication errors reporting program (MERP) is a confidential national voluntary reporting program that provides expert analysis of the system causes of medication errors and disseminates recommendations for prevention *

What is the best certified sterile compounding environment for injectable product? Which ISO?

ISO class 5 (class 100) laminar air flow hood within ISO class 7 (class 10,000) clean room * with positive air pressure * differential relative to adjacent areas. ISO class 5 (class 100) clean room with positive air pressure differential relative to adjacent areas or a barrier isolator that provides a ISO class 5 (class 100) environment for compounding

What is the IV to PO conversion for metoprolol?

IV:PO 1:2.5

Which bisophosphonate has 3 months duration and comes as injection?

Ibandronate

Is it safe to use Ceftriaxone in neonates? If no, why?

It is not safe to use Ceftriaxone in neonate 0-28 days> Ceftriaxone which is used in adults should be *avoided* in neonates: Ceftriaxone displaces bilirubin from albumin, which can cause bilirubin-induced brain damage (kernicterus). There is risk of an embolus from a ceftriaxone calcium precipitate; a critically ill neonate will be receiving various IV fluids

Short-Acting Beta-2-Agonist

MDI: 1-2 inhalations Q4-6H PRN SE - Tremor, Shakiness, Lightheadedness, cough, palpitations, Hypokalemia, Tachycardia, Hyperglycemia Notes - Preg C - Shake before use - prime 3-4 sprays into the air -Prefer a beta-2 selective agent via inhaled route. - PRN use for asthma - If using SABA > 2 days per week then need to increase maintenance therapy - DOC for EIB (Exercise Induced Bronchospasm) - Levalbuterol is the R-isomer of albuterol - 200 inhalations/canister

What is the purpose of HFA used in MDI?

MDIs use a chemical propellant (HFA) to push medication out of the inhaler. MDIs require a slow and deep breathe in from the device. All MDI need to be shaken since they are suspension except QVAR and Alvesco. They can be used with spacer devices.

What is the ANTIDOTE for Ethanol (alcoholic drinks)?

No ANTIDOTE -> Supportive care, correct hypoglycemia Facts about Hypoglycemia: Research has shown that acute consumption increases insulin secretion, causing low blood sugar (hypoglycemia), and can also impair the hormonal response that would normally rectify the low blood sugar. Drinking as little as 2 ounces of alcohol on an empty stomach can lead to very low blood sugar levels.

Can you give Dofetilide with Thiazide Diuretics?

No [X: Avoid combination] Thiazide diuretics may enhance the QTc prolonging effect of Dofetilide. Thiazide Diuretics may increase the serum concentration of Dofetilide.

Pregnancy Category C

No controlled studies in humans have been performed and animal studies have shown adverse events or studies in humans and animals are not available; give only if potential benefit out-weighs the risk

Is median influenced by the outliers?

No it is not affected by the outliers. Another term for the median is the 50th percentile, where 50% of the values are below the median and the other 50% are above the median.

Can you give Brisdelle with Tamoxifene or Warfarin?

No!! As an SSRI, it will increase risk of bleeding and as a CYP450 2D6 inhibitor, it will block the effectiveness of tamoxifen and block the metabolism of warfarin, causing bleeding risk.

Can you combine Emtricitabine (FTC) with LamiVUDine 3tc?

No, Avoid this combination (no benefit) as both are cytosine analogs: FTC and 3TC

Can the patch be exposed to heat from an electric blanket, heating pad, or body temperature > 38 C?

No, Heat will increase release of a drug. This cause the drug to pour out of the patch, resulting in toxicity. with fentanyl and buprenorphine this can be quickly toxic (fatal)

Do you give Physotimine if patient have anticholinergic overdose who having seizure?

No- we give benzodiazpines/anticonvulsants

What does medicare Part A Part B part C part D covers?

Part A: Hospital inpatient care, skilled nursing, hospice and home health Part B: DME , Lab tests, Diagnostic services, dialysis, Preventative services/education, Prescriber bills, Vaccine Part C: Advantage plans only (They are generally low income) and they pick up some of patient costs Part D: Outpatient drugs except for a few that are covered under part A or B, MTM services

Equations to determine what will be C2 after given Ke and Time and another equation to determine Ke when C1 and C2 and time is given

C2 = C1e to the power -keT Ke = -ln (c1/c2) / t

Which Beta Blocking Agents has intrinsic sympathomimetic activity and are CI in S/P MI patients?

CAPP Carteolol Acebutolol Penbutolol Pindolol

Suvorexant (Belsomra)

CI - Narcolepsy SE - Somnolence, headache, dizziness, abnormal dreams, cough, upper respiratory tract infection Note - Belsomra may cause sleep-driving and other complex behaviors while not being fully awake - * avoid wih strong CYP3A4 inhibitor *

Inhaled Steroids

CI - Primary treatment of status asthmaticus or acute episodes of asthma or COPD SE (Inhaled) > Dyphonia > Oral candidiasis (thrush) > Cough > Hyperglycemia > etc Note > To prevent oral candidiasis, rinse mouth and throat with warm water and spit out or use spacer device if using a MDI > Inhaled steroids are first line for long term control for all ages with persistent asthma * QVAR and Alvesco are MDIs that do not need to be shaken before use *

What Class schedule are the non-benzodiazepines?

CV

What is the generic name for Brands: Tagamet

Cimetidine

What is FOLFOX regimen used for?

Colorectal cancer FOL= Folinic acid (Leucovorin: given 24 hours after methotrexate) F= Fluorourical OX = Oxaliplatin

Phentyoin/Fosphenytoin Safety etc.

BBW - Phenytoin IV administration should not exceed 50mg/minute - Fosphenytoin IV administration should not exceed 150mg PE/minute: if faster hypotension and cardiac arrhythmias can occur. Warning - asian patient should be screen for HLA-B*1502

Heparin [BBW, CI, SIDE EFFECTS, Monitoring]

BW - Some product contain benzyl alcohol as a preservative use of these products is contraindicated in neonates and infants CI - Uncontrolled active bleed - Severe thrombocytopenia - ICH - History of HIT - Hypersensitivity to pork products Warning - Do not give IM due to hematoma risk SE - Bleeding, thrombocytopenia, heparin induced thrombocytopenia, Hyperkalemia and osteoporosis (with long-term) Monitoring - aPTT (or anti-Xa level) - aPTT is taken 6 hours after initiation, every 6 hours until therapeutic range of 1.5-2.5 x control (patient baseline) is reached, then every 24 hours. Also check every dosage change - Platelet count, Hgb, Hct at baseline and daily to monitor for thrombocytopenia and bleeding

Why you are not suppose to start anti-depressant (Unless there is a mood stabilizer ) medications on someone who has mania or depression?

Because giving an anti-depressant will put patient at risk for mania

Inhaled Corticosteroids

Beclomethasone HFA (QVAR) Budesonide (Pulmicort Flexhaler, Pulmicort Respules) Ciclesonide (Alvesco) Fluticasone (Flovent HFA) Mometasone (Asmanex Twisthaler, Asmanex HFA)

4th generation cephalosporin

Best gram - activity including HNPEKS and CAPES (Actinobacter, Pseudomonas, Enterobacter and Serratia species and gram + similar to 3rd generation. - Cefepime (Maximpe)

Important about controlling the rate

Beta blockers (preferred) and non-dihydropyridine calcium channel blockers are recommended for treatment in patient with AFib for controlling ventricular rate. * Patients with underlying HF should not recieve non-dihydropyridine calcium channel blocker * - Amiodarone and digoxin are not first line agent for ventricular rate control, but may be considered for refractory patients.

Heparin [Dose and Monitoring] and Anti-dot

Treatment of VTE - 80 units/kg IV bolus followed by 18 units/kg/hr infusion or fixed dose of 5000 units IV bolus followed by 1000 units/hr infusion. Treatment of ACS/STEMI - 60 units/kg IV bolus (max4000 units); 12 units/kg/hr (max 1000) infusion > use actual body weight Monitoring - Monitor for decrease in platelet count of > 50% from baseline. - HIT has cross-sensitivity with LMWHs - Antidoe: Protamine 1mg protamine will reverse approximately around 100units of heparin; max dose 50mg

Hypophosphatemia is considered severe and usually symptomatic when serum Po2 is < 1mg/dL - When serum Po4 is < 1 mg/dL, IV phosphours is used for replacement. - Less severe hypophosphatemia can be treated orally and full replacement often take one week or longer

True

In order to discharge a patient on fentanyl, they should have been using morphine 60mg/daily or higher (morphine equivalent dose) for at least 7 days.

True

Joint commission standards require that pain be treated in the same manner as vital signs, making it compulsory for health care professionals in accredited facilities to inquire about, measure and treat pain as they would blood pressure, pulse. Pain is considered to be the fifth vital sign.

True

Live vaccines, MMR and Varicella, are only contraindicated for HIV patients with CD4 lymphocyte count < 200 cells/mm3

True

Magnesium is necessary for potassium uptake so make sure you have magnesium. IV potassium should be administered no faster than 10-20 mEq/hr with intermittent doses. Concentration above 80 mEq/L should be administered through a central line

True

NCCN recommends all patients with > 20% chance of developing chemotherapy induced febrile neutropenia to receive myeloid growth factors. T ro F

True

Ordinarily, heparin prevents clotting and does not affect the platelets, components of the blood that help form blood clots. Triggered by the immune system in response to heparin, HIT causes a low platelet count (thrombocytopenia).

True

Patient hospitalized for new-onset or worsening heart failure symptoms are referred to as having acute heart failure or acute decompensated heart failure, respectively.

True

Pregnant woman should receive Tdap with each pregnancy. The optimum time for Tdap vaccination is between weeks 27 and 36 of the pregnancy.

True

Sodium intake (hight) will decrease Li levels, and Low sodium intake will increase Lithium levels

True

T or F. According to CHEST and ASCO guidelines. LMWH is recommended for treatment of VTE over warfarin therapy for patients with cancer.

True

T or F. Antipsychotics can be used to treat delusion/anger, but they increase the risk of death in elderly patients.(BBW( and provide little benefit.

True

T or F. Beyond use date must be applied to each compounded product using USP 795 guidelines for Nonaqueous Liquids and Solid formulation and for Water-containing Formulations.

True

T or F. Both sorbitol and lactose are classified as excipients (or binders) and they help hold tables together.

True

T or F. DtaP series given to children younger than 7 years of age?

True

T or F. Due to the positive response from using methylphenidate, the primary focus of research is on the catecholamine system (dopamine is catalyzed to the two other primary catecholamines, epinephrine and norepinephrine)

True

T or F. EPIPEN injections require rubbing the area where the medicine entered your skin

True

T or F. Each time warfarin molecules pass through the liver, some are captured by the CYP2C9 enzyme and converted into inactive metabolites.

True

T or F. Enteric coated aspirin must be chewed if patient is having Acute Coronary Syndrome.

True

T or F. Ethinyl Estradiol is a semisynthetic estrogen.

True

T or F. Gemfibrozil can increase concentration of repaglinide concentrations and decrease BG, and it is recommended to use fenofibrate instead.

True

T or F. Green tea may lower DM risk

True

T or F. HPV vaccine is not recommended during pregnancy.

True

T or F. If HIT occur, Vitamin K should be given.

True

T or F. If patient has CKD with or with out diabetes, we must use ACE-I or ARB.

True

T or F. If switching to morphine and the patient has renal insufficiency, a 50% reduction in the total daily dose or similar would be WISE.

True

T or F. Immunizing pharmacists should always maintain a current basic life support (BLS or CPR) certification.

True

T or F. MMSE score < 24 indicates memory disorder.

True

T or F. New guidelines for treatment of cholesterol. Non-statin therapies are not recommended unless statins are not tolerated and treating to specific LDL-C goal is no longer recommended.

True

T or F. Stimulatns are * c-II * which means the prescriptions can only be prescribed for one month at a time, which makes it convenient to check blood pressure and heart rate (and weight and height, periodically in children)

True

T or F. The inability to pass a bowel movement after straining or pushing for more than 10 minutes require a digital evacuation.

True

True or False. Tamoxifen is widely prescribed to patients with estrogen receptor-positive breast cancer, and it is a prodrug that requires bioactivation by cytochrome P450 enzymes CYP2D6 and 3A4 to generate the active metabolite, endoxifen.

True

Venous thromboembolism (VTE) is a disease that includes deep vein thrombosis (DVT) and pulmonary embolism (PE). DVT and PE are both forms of VTE, but they're not the same thing

True

Ventricular tachycardia with a pulse is treated with certain antiarrhythmics , wehreas a ventricular tachycardia without a pulse i a medical emergency. Untreated ventricular tachycardia can degenerate into ventricular fibrillation (complete disorganized electrical activation of the ventricles) which is always a medical emergency.

True

When giving treatment for constipation. Make sure you use extra caution in patients with cardiovascular disease, renal insufficiency, if taking diuretics (loops, due to additional fluid loss) and NSAIDs.

True

T or F. All patients on enzyme inducing AEDs should supplement with vitamin D and Ca+2

True > Because these inducer will decrease Conc. of VitaminD by increasing its metabolism.

Per TJC, It is required for all accredited facilities to create and follow a protocol to identify and report critical values to the responsible caregiver who has an established time frame to manage the result.

True - Critical values are potentially life-threatening laboratory results that must be conveyed immediately to the physician

List 2 drugs that require HER2+ to be affective?

Variations in over-expression of the HER2/neu oncogene: If the receptor is over-expressed, the response to trastuzumab (Herceptin) and ado-trastuzumab (Kadcyla) is *positive*

Find Vd. If an IV dose of 150mg of a drug produces an initial plasma concentration of 0.1mg/L What is the VD?

Vd = Dose / Co = 150mg/0.1 = 1500 L

Anticoagulant Recommendations for Acutely ILL hospitalized Patients at increased Risk of VTE

UFC - 5000 units Sub-Q Q 8 to 12 hours Enoxaparin - 30 units Sub-Q BID - CrCl < 30: 30 units Sub-Q Daily - 40 units Sub-Q Daily Dalteparin - 5000 units Sub-Q daily Factor Xa inhibitors Fondaparinux > 2.5 mg SC daily ( do not use if CrCl < 30 or KG < 50 Rivaroxaban > 10mg PO daily ( Do not use if CrCl < 30ml/min Apixaban > 2.5mg PO BID

Which anticoagulants work by binding to antithrombin (AT)?

Unfractionated heparin, low molecular weight heparin (LMWH), and fondaparinus works by binding to AT causing a conformational changes which increasae AT activity to 1000 fold. - AT inactivates thrombin and other proteases invovled in blood clotting including Xa. - LMWHs inhibit factor Xa more specifically than UFH - Fondaparinux (Arixtra) is a synthetic pentasaccharide that requires AT binding to selectively inhibit Factor Xa.

What is so special about Teriparatide (Forteo)?

Unlike other medications (Bisphosphonates, Raloxifene, Estrogens, denosumab) which slow down bone break-down or resorption. Teriparatide (Forteo) does both - it helps prevent bone break-down and helps build bone. It is strong agent and is reserved for high risk patients.

What is the difference between HTN urgency and HTN Emergencies?

Urgency - BP (generally >= 180/110-120) with no acute target organ damage - Treat with any oral medication Emergency - BP (generally >=180/110-120) with acute target organ damage - Reduce MAP or BP by no more than 25% (within the first hour), then if stable to 160/100-110 mmHg within the next 2-6 hours. - Use IV (hydralazine, labetalol. sodium nitroprusside, nicardipine, or others)

Is there a vaccine for Prostate cancer?

Yes. Sipuleucel-T (Provenge) > Autologous cellular immunotherapy which stimulates an immune response targeting prostatic acid phosphatase (PAP), an antigen expressed in most prostate cancer cells.

Is allopurinol used for tumor lysis syndrome?

Yes. in higher doses

Can you give Ibuprofen with Aspirin?

Yes. it happens when patient is using Aspirin for cardio-protection. > Take Aspirin one hour before or 8 hours after ibuprofen.

Which triptan drug used for migraine comes in transdermal patch?

Zecuity Transdermal patch

What dose 5HT3 receptor antagonist do?

Zofran (Ondansetron) prevent nausea/vomiting

Which bisphosphonate has one year duration and it is given as annual infusion?

Zoledronic acid preferable

Renal Cell Cancer (RCC) Tyrosine Kinase Inhibitors targeting multiple kinases PDGF, VEGF-R , etc

SUNitinib (Sutent) > Rash, hand-food syndrome, HF, decrease in LVEF, QT prolongation PAZOPanib (Votrient) > Hypertension, QT prolongation SORAfenib (NexAVAR) > Fatigue, Hypertension, hand-food syndrome, diarrhea, N/V

What is the difference between Somatic Nociceptive and Visceral Nociceptive?

Somatic is from the skin, muscle, bones, joints, and ligaments - this is commonly called musculoskeletal pain or visceral (from the internal organs, such as the heart or lungs)

How to convert to fentanyl?

Some clinician use this estimation: morphine 60mg total daily dose = 25 mcg/hr fentanyl patch > If converting fentanyl using the chart, remember that you are finding the total daily dose in mg, and will then need to converte to mcg (multiply by 1000) then divide by 24 hours because fentanyl is dosed in mcg per hour. * A. calculate daily morphine B. Divide total by 100 C. Times 1000 D. Divid by 24*

What are the acute overdose sings and symptoms?

Somnolence Respiratory Depression with shallow breathing, cold and clammy skin and constricted (pinpoint, miosis) pupils. Can lead to coma and death

What are the side effects associated with Triptan drugs used for Migraine?

Somnolence, nausea, paresthesias (tingling/numbness) etc. Triptan Sensation > Pressure on the chest and neck region and usually dissipate after administration

ARAs - Aldosterone Receptor Antagonists

Spironolactone is non-selective aldosterone receptor blocker (also blocks androgen and weakly activates progesterone receptors). Eplerenone is a selective aldosterone blocker and does not exhibit the endocrine side effects. ARAs compete with aldosterone at the receptor site in the distal convoluted tubule and collecting ducts, increasing Na and H2o excretion while conserving K and H ion. - ARAs reduce morbidity and mortality - ARA should be added to standard therapy in patients with NYHA FC II-IV

* Important for the exam * Allergies to sulfonamides, also known as sulfa drug

Sulfa allergies : patient should avoid using sulfapyridine, sulfadiazine, and sulfisoxazole. The non-arylamine sulfonamides (thiazide diuretics, loop diuretics, sulfonylureas, acetazolamide, zonisamide and celecoxib) usually do not cross react but on EXAM you will likely have to recognize the reaction.

What is the first additions to an Electronic Health Record (EHR)?

The first additions to an EHR are the admission sheet, a service agreement form ("this is what I am having done at this facility"), a page describing the patient's right (a Joint Commission requirement), and an advanced directive to document the patient's wishes concerning medical treatment if he or she is unable to make the decisions on their own behalf.

Which agents cross-interact and what to choose if morphoine group allergy?

The following agents cross-react > morphine, oxymorphone, codeine, hydrocodone, hydromorphone, oxycodone. If morphone group allergy > Choose fentanyl!!!!!!!!!!!!!!!!!

Who require that standard orders sets be used for all anti-thrombotics?

The joint commission requires that standard order sets be used for all anti-thrombotics

Define Minimum Bactericidal Concentration (MBC)?

The lowest drug concentration that reduces bacterial density by 99.9% in 24 hours (kill bacteria)

DKA

Diabetic Ketoacidosis (DKA): Is a hyperglycemic crisis most commonly presenting in type 1 and rarely in type 2 diabetes. - DKA occurs due to non-insulin compliance (ran out, lost/homeless, re-fused to take), sub-therapeutic insulin dose (due to a stressor, such as infection, MI or trauma), or as the initial presentation in type 1 diabetes). - Ketones are present because triglycerdides and amino acids are used for energy which produces free fatty acids. - Glucagon converts the FFAs into ketones, normally, insulin prevent this conversion but, in DKA insulin is absent.

What is the ANTIDOTE for DIgoxin, foxglove, Oleander?

Digifab 40mg vial binds to approximately 0.5 mg digoxin > Adult dose is 20 vials if unknown amount

What is the mechanism of action of Interferon B?

Directly increase expression and concentration of anti-inflammatory agents which down-regulate pro-inflammatory cytokines.

Switching from apixaban to anticoagulants other than warfarin

Discontinue apixaban and start other anticoaug at the next scheduled dose.

What is the important contraindication to Kcentra?

Disseminated intravascular coagulation and known heparin-induced thrombocytopenia (contains heparin)

What are the side effects associated with Flecainide ?

Dizziness, visual distrubrances, headache, dyspnea, new or worsening arrhytmias

If given Volume of Distribution and Plasma concentration of drug then you can easily determine the Dose.

Dose = Vd x Plasma Concentration

What are the dose-related side effects of phenytoin/fosphenytoin and Chronic side effects?

Dose-related (toxicity) - Ataxia, dizziness, somnolence, lethargy, nystagmus, slurred speech, confusion, blurred vision and dipolpia Chronic - Skin thickening (children), gingival hyperplasia, hair growth, vitamin D and calcium deficiency (bone loss), connective tissue changes, coarsening of facial features, folate deficiency, hepatotoxicity

List some common drugs that require light protection?

Doxycycline Dopamine Epoprostenol (Flolan) Linezolid (Zyvox) Metronidazole (Flagyl) Micafungin (Mycamine) Nitroprusside (Nitro-press) Norepinephrine (Levo-phed) Phytonadione (vitamin K) Bactrim Verapamil

What is Tic Tac Toe Craving delicious , excellent, irresistible pho (Pho is Vietnamese soup)

Drugs that can't be placed in PCV due to leach DEHP into the solution - Tacrolimus , Temsirolimus, Teniposide, Cabazitaxel, Docetaxel, Etoposide, Ixabepilone, paclitaxel.

What is the maximum daily limit of vitamin E?

Due to CVD risk (hope trial) -> Patient should not be exceeding 150 IU daily

Stroke and SCD

Due to blockage of coronary blood vessels. Multiple aneurysms are common. Patient with ScD should be screen for HTN and treated

Why Dolasetron IV is CI in CINV?

Due to incidence of QT prolongation

Why you can't give zyperxa with benzodiaezpine IM?

Due to orthostasis risk

Why is the duration of bisphosphonate is kept at 3-5 years in patients at low risk for fracture?

Due to the rare risk of atypical (or low trauma) femur fracture and osteonecrosis of the jaw

What are the Side Effects Associated with NSAID?

Dyspepsia Heartburn Take NSAID with food to decrease nausea Blood pressure may increase Gi irritation/bleeding, renal impairment etc. Rare but cases of severe skin rash SJS and TEN > Stop drug , seek immediate medical help

All of the following are SE of Plavix Except? A. Bleeding B. Bruising C. Rash D. Pruritus E. All are SE of Plavix

E

All of the following are SIDE EFFECTS of ASPIRIN EXCEPT? A. Dyspepsia B. Heartburn C. GI upset D. GI bleed/ulceration E. Bleeding F. Renal impairment G. Tinnitus (in toxicity) H. All of the above

E

Which of the following can make a health claims? A. Drug B. Homeopathic C. Natural Products D. Medical foods E. A and B and D

E. A and B and D

What is LH used for?

Rises mid-cycle causing egg release Tested in urine with ovulation predictor kits for women attempting pregnancy

Injections that patient can self-administer (mostly SUB-Q)

The erythropoiesis stimulating agents (ESAs) are given SC by patients with impaired renal function, and if in ESRD, are given IV at the dialysis center.

What is the goal of treatment of cancer?

The plan may attempt to achieve remission (which curative intent) or be palliative (to reduce tumor size and symptoms)

What cause HE?

The symptoms of HE result from an accumulation of gut-derived nitrogenous substances in the blood (such as ammonia, glutamate, others) due to decrease hepatic functioning and shunting through the porto-systemic collaterals, which bypass the liver.

T or F. Two drugs will be needed if the systolic blood pressure (SBP) is > 20 mmHg or the diastolic blood pressure (DBP) is > 10 mmHg above goal.

This is very true.

Where to report medical errors in a hospital?

To the P&T committe, a staff meetings (as defined by facility), to the medication safety committee

Important about Crystalloids and colloids?

To treat EDEMA; we want to use Albumin 25% because we want something to pull water into the intravascular space

There is one drug that cause these: 1. Metabolic acidosis 2. Oligohydrosis 3. Nephrolithiasis 4. Acute myopia 5. Hyperammonemia 6. it has one brand name that is contraindicated if u drink alcohol within 6 hours prior to and 6 hours after 7. It cause dizziness, sleepy, difficulty with memory, difficulty with concentration, it cause bone loss also. 8. The dose for week1: 25 BID , week 2 50 BID etc.

Topiramate

What cause decrease in HCO3

Topiramate Salicylate overdose

Examples of symptoms and signs typical of anaphylactic reactions

Urticaria (hives) Swelling of the mouth and throat Difficulty breathing wheezing abdominal cramping hypotension or shock

What do you have to watch out for when using combination of two nicotine products ?

Use extreme caution if any underlying cardiovascular condition due to additive side effects

What are the common adverse effects of vaccines?

Usually adverse effects are minor and include mild fever, soreness or swelling at the injection site. Some vaccines may cause headache, loss of appetite and dizziness -- but these quickly dissipate.

Entyvio- what is the generic?

Vedolizumab - SE > Ha, nasopharyngitis, arthralgia, antibodies development NOTE > use immediately after reconstitution. Do not freeze. infuse over 30 minutes > All immunizations must be up to date before starting therapy > Cannot be used with other immunosuppressants > Medication guide required

What cause abnormal muscle turnover so elevated creatine levels can be due to these conditions?

Very high or low muscle mass Obese Liver disease pregnant

What cause Wernikce's encephalophay?

Vitamin B1 (thiamine) Deficiency

Which vitamin is depleted when patient on long term Isoniazid?

Vitamin B6

Why Vitamin D is not considered a True Vitamin?

Vitamin D is thus not a true vitamin, because individuals with adequate exposure to sunlight do not require dietary supplementation.

What is the DOC for Aspergillus?

Voriconazole

Warfarin (Coumadin, JanToven)

Warfarin competitively inhibit the C1 subunit of the multi-unit vitamin K exposide reductase (VKORC1) enzyme complex, thereby reducing the regeneration of vitamin K epoxide and causing depletion of active clotting factors II VII IX X and protein C and S - Racemic mixture of R- and S- enantiomers with S-enantiomer being 2.7-3.8 times more potent

Warfarin

Warfarin is a vitamin K antagonists. Vitamin K required for the carboxylation of clotting factors II, VII, IX, X. Without adequate vitamin K, the liver produces the factors, but they have reduced coagulant activity. Warfarin requires careful patient monitoring - with frequent blood tests to measure the INR, the test used to measure warfarin's safety and efficacy. Warfarin has a narrow therapeutic range and the INR is highly variable and is affected greatly by many drugs or changes in dietary vitamin K intake.

Carbamazepine Warning Plus SE

Warning - Risk of developing hypersensitivity reaction may increase in patients with the variant HLA-A*3101 - Hyponatremia (SIADH) > Because it stimulate release of ADH which lead to more water and this will cause less Na - Mild anticholinergic effect - Hypothyroidism - etc. Side Effects - Dizziness, somnolence, headache, nausea, vomiting, ataxia, dry mouth, pruritus, photosensitivity, blurred vision, rash, vitamin D and calcium deficiency (bone loss), increase in LFT etc. Monitoring - Carbamazepine levels should be monitored within 3-5 days of initiation and again after 4 weeks due to autoinduciton - etc. NOTE - Preg D - inducer and autoinducer - Supplement with Ca plus vitamin D

When do you stop treating HIV+ patient for Toxoplasma gondii?

When CD4+ count > 200 cells/mm3 for > 3 months on ART

What makes the Drospirenone formulations so popular COCs?

Yasmin, Yaz, Gianvi, Loryna, Ocella, Zarah, Daylette, Angeliq, Nikki, Safyral, Syeda, Vestura, Beyaz (Safyral and Beyaz contain folate) > Decrease bloating, PMS symptoms and weight gain. This is due to the progestin drospirenone, which is a potassium sparing diuretic. > Avoid using Drospirenone containing formulation if kidney , liver, or adrenal gland disease. On a case, check potassium level. It should be in the safe range of 3.5 - 5 mEq/L. > This type of progestin puts the patient at a slightly higher risk of clotting, and should be avoided in women with clotting risk

Which testing should be considered for patients starting Allopurinol (Zyloprim)?

HLA-B5801 - Consider testing prior to starting therapy in high risk individuals, such as Korean patients with significant renal impairment or those of Han Chinese or Thai ancestry. - Can cause RASH which can lead to SJS

How to treat anaphylaxis?

If a patient has an anaphylactic reaction, they will need to go to the ED right away (or call 911) and receive an epinephrine injection +/- diphenhydramine +/- steroids +/- IV fluid. * If patient experience severe allergies all the time, they should carry epipen with them plus Diphenhydramine tablets 25mg x 2 and emergency contact information *

Why do we need special formulations for Olanzapine?

Majority of patients with schizophrenia d/c antipsychotics. We need some formulation to prevent this and the best is ODT which prevent patient from cheeking the medication and spitting it out later. A long acting injectable would benefit this group and also a fast acting injection are useful for acute agitation.

What is the name of Equation used to describe the rate of dissolution?

Noyes-Whitney

What are the benefits of using ODT formulation and list some common ODT formulations?

ODT formulation is helpful in A. Dysphagia B. Children C. Nausea D. Non-adherence Example: Metozolv ODT - Metoclopramide Maxalt-MLT - Rizatriptan Zelapar - Selegiline Staxyn = Vardenafil Generic for Levitra > ODT

What is the best thing to do to prevent emergence reactions with ketamine (NMDA antagonist)?

Pretreatment with benzodiazepines can decrease the incidence of emergence reactions

List 2 agents that bind to alpha-2-delta subunit of voltage-dependent calcium channels within the CNS, inhibiting the excitatory neurotransmitter release.

Prgabalin and Gabapentin - Both doses should be decreased if CrCl < 60 - Both cause weight gain - Both cause Edema and dry mouth

Why QT prolongation is dangerous?

Prolongation o the QT interval is a risk factor for Torsade de Pointes, a particularly lethal ventricular tachyarrhythmias which is most commonly associated with drugs and can result in sudden cardiac death.

why Zykadia (CERTINIB) u not suppose to use it if patient has diabetes cardiac and bradycardia?

QT Prolongation Hyperglycemia Bradycardia

Which antipsychotic is a DOC for patient with parkinson disease?

Quetiapine is the preferred drug due to a low risk of movement disorders, but it will require monitoring due to metabolic complications including increase blood glucose and cholesterol.

Name one stimulant that comes as oral suspension?

Quillivant XR extended-release oral suspension is used to treat attention deficit hyperactivity disorder (ADHD).

What is the dose of N-acetylcystein?

PO: 140mg/kg for 1 dose, followed by 70mg/kg every 4 hours for 17 additional doses. Repeat dose if emesis occurs within 1hour of administration IV: 150mg/kg over 60 minutes, followed by 50mg/kg IV over 4 hours followed by 100mg/kg IV over 16 hours

Just the term " As likely " Example:

Page. 119 Subjects treated with metoprolol were only 57% as likely as placebo-treated patients to have heart failure progression

Ex of Intangible?

Pain, suffering, weak, anxiety, etc..

Which 5HT3 receptor antagonist is the only one that is proven effeciacy in delayed CINV?

Palonosetron

Which patients are target for MTM?

Patient with multiple chronic conditions who are taking multiple drugs and are likely to incur annual costs for covered drugs that exceed a predetermined level.

Another formula to calculate RR when it comes to disease progression

RR = a/(a+b) divided by c/(c+d) Ex. In a group of 100 smokers, 40 people developed lung cancer (CA) while 60 people did not. In a similar group of 100 non-smoker, lung CA developed in 10 people. Calculate the relative risk of developing lung CA from smoking RR = 40/40+60 divided by 10/(10+90) = 4 This means: The RR of 4 means that smokers are 4 times as likely to develop lung CA than non-smoker This is same as doing it this way: 40/100 = .4 and 10/100 = .1 then .4/.1 = 4

What type of studies are considered the gold standard trial design in evaluating safety and efficacy of an intervention (e.g drug)?

Randomized Controlled Trial (RCT) are always prospective and are considered the gold standard trial design in evaluating safety and efficacy of an intervention (e.g. drug).

What is the DOC for MAC?

Preferred: Azithormycin 1200 mg PO weekly or Clarithromycin 500mg po BID or Azithromycin 600mg po twice weekly

What is the DOC for PCP for HIV+ patient whose CD4+ count is below 200 cells/mm3?

Preferred: TMP/SMX 1 DS tablet PO daily or 1 SS Po daily Alternative (if drug allergies) Dapsone 100mg po daily or 50mg po BID or Dapsone + pyrimethamine + leucovorin or Aerosolized pentamidine Or Atovaquone or TMP/SMX 1 DS PO TIW

What pregnancy test kits test for?

Pregnancy test kits are positive if hCG (human choriionic gonadotropin) is in the urine. ta

What cause an increase in Hemoglobin Hgb, Hb or Hct Hematocrit?

ESA

Why Anemia no longer treated with ESA?

ESA can shorten survival and increase tumor progression in some cancers

Why you must monitor iron level before giving ESA?

ESAs will not work well to correct anemia if iron levels are inadequate - Levels of folate and vitamin b12 may need to be evaluated, especially if there is a poor response to ESA

Erlotinib (Tarceva) Afatinib (Gilotrif)

Erlotinib (Tarceva) - 150 mg daily PO (1 hour before or 2 hours after meal) - Works best in EGFR positive mutation - Mut test for EGFR mutation status. Must be positive SE - Acneiform rash, diarrhea, hepatotoxicity, GI Perfroation, etc. Afatinib (Gilotrif) - EGFR exon 19 deletions or exon 21 substitution - SE > Acneiform rash, diarrhea, stomatitis, dry skin, paronychia, decrease in appetite, pruritus

Which patch applied twice weekly?

Estradiol (Alora, Vivelle-Dot): Twice weekly Oxybutynin (Oxytrol) Twice weekly

Relationship between Estrogen and Calcium

Estrogen supports this activity by aiding in intestinal absorption of calcium. Having low estrogen levels negatively impacts your body's ability to make use of the calcium you consume. This, in part, explains why you are at more at risk for osteoporosis if you're female, according to Dr. Margery Gass of the University of Cincinnati. Gass points out that women with conditions affecting their estrogen levels, such as early menopause, are at risk for bone loss.

What is EBM?

Evidence Based Medicine: Is an approach to medical practice intended to optimize decision-making by emphasizing the use of evidence from well designed and conducted research.

Fill in the blank ........................ refer to a compound prepared without a specific formula (not from an official compendium) and made specifically to fill the needs of an individual patient

Extemporaneous

Important notes about Selective Beta-1 Blocker

- Avoid abrupt discontinuation - must taper - Take metoprolol immediate release tablets with food - ER can be taken w/o regard to meal

What are the DKA symptoms?

- BG > 250mg/dL, ketones (on lab report, or picked up as fruity breath) with anion gap metabolic acidosis (arterial pH< 7.35, anion gap > 12)

Patient has AOM and doctor wants to prescribe antibiotic but patient has allergies to penicillins. what should we give this patient?

- Cefdinir 14mg/kg/day in 1 or 2 doses - Cefuroxime 30mg/kg/day in 2 divided soes

What immunizations are required for Asthmatic?

Adults aged 19-64 years old with asthma should receive the pneumococcal polysaccharid vaccine (PPSV23 or Pneumovax) Annual flu vaccine should be given to those with asthma

What is the DOC for Vaginal Trichomoniasis?

2g PO metronidazole x 1 ( or 500mg PO BID for 7 days) at any stage of pregnancy.

What is the 1st line therapy for schizphorenia?

2nd generation antipsychotic agents (SGAs) are commonly used first line due to lower risk of EPS, however they are not first line in all patients, some respond better to 1st generation

What is considered an adequate treatment trial to see if anti-depressant medication is working?

6-8 weeks

Non-cell cycle specific

Alkylating agents Anthracyclines Platinum agents ALL ABOUT PROBLEMS

Which of the following used for neuropathic pain/migrain prophylaxis 10-50 mg QHS?

Amitriptyline

What is Cost-benefit analysis?

Cost-Benefit analysis (CBA) is a systemic process for calculating and comparing benefits and costs of an intervention in terms of monetary unit ($$$$).

What is Acetazolamide used for?

It can treat glaucoma, epilepsy, mountain sickness, and fluid retention (edema).

What is the purpose of competitive bidding process for CMS?

It is an effort to keep the taxpayers from being overcharged.

What does the drug distribution depends on?

It is based on drug's lipophilicity, molecular weight, solubility, ionization status, and the extent of protein binding

T or F. You can rinse compounding equipment with tap water?

False - you can only rinse Purified water for rinsing equipment

What is the name of the scale that is used to determine if drug is the cause of an ADR?

Naranjo Scale

1 birth control is the only formulation that is four-phasic, with four phases of estradiol valerate and the progestin dienogest.

Natazia

Age classifications

Neonate 0 - 28 days Infant 1 month - 12 months Child 1 - 12 years Adolescent 12 years - 18 years

What is the DOC commonly used to prevent Raynaud's ?

Nifedipine

What is used to provide fast (immediate) relief of angina?

Nitroglycerin as a sublingual tablet or translingual spray

Which medications you can't have a child-resistant (C-R) containers on?

Nitroglycerin sublingual tablets

Can a pregnant woman receive MMR, Varicella , or any other live vaccine such as live influenza nasal?

No > No live vaccines one month before and during pregnancy

Dabigatran (Pradaxa)

Non-Valvular AFIB 150mg BID; 75mg BID if CrCl 15-30 ml/min Treatment of DVT/PE and Reduction in the Risk of Recurrence of DVT and PE 150mg BID when CrCl > 30ml/min, No recommendation if CrCl < 30

Which Non-prescription drugs require C-R container?

Non-prescription drugs that require C-R containers include anything containing Iron, Diphenhydramine, acetaminophen, salicylates, NSAIDs, "imidazoline" vasoconstriction such as naphazoline and oxymetazoline and drug that have been switched from Rx to OTC plus Turpentin which is very toxic if swallowed.

What is the disadvantage of CEA?

Inability to directly compare different types of outcome. For example, one cannot compare the cost effectiveness of implementing a diabetes program with implementing an asthma program where the outcome units are different (e.g., blood glucose values versus asthma exacerbations)

T or F. When triglycerides > 1000 mg/dL, Chylomicrons are almost present. These low density are very large and may obstruct capillary leading to local ischemia and Acidemia.

True

T or F. You may shower, bathe, swim while wearing Nitroglycerin Patch.

True

T or F. patient with valvular AFib (those with mechanical heart valves) should only be treated with warfarin

True

T or F: If a care indicates hCG+, the patient is pregnant and the teratogenic drug should be discontinued, if possible.

True

The calcineurin inhibitors tacrolimus and cyclosporine and the anticoagulants dabigatran and rivaroxaban are substrates of the P-gp efflux pump. T or F

True

True or False. Substances with a molecular weight below 40,000 daltons can pass through the glomerular capillaries into the filtrate. Larger substance are not filtered and stay in the blood.

True

T or F. Morning stiffness is a clue for RA an day last for up to 2 hours. Osteoarthritis does not cause prolonged stiffness.

True - In diagnostic criteria it says morning stiffness around joints lasting more than 1 hor

Volume of Distribution is how large an area in the patient's body the drug has distributed into and is based on the properties of the drug. T or F.

True Vd = Amount of the dug in the body divided by Concentration of drug in plasma

T or F. For Migraine; Chose among various formulations, if with aura choose POP

True because aura put patient at increased risk of stroke!

T or F. In up to 25% of patients with HIT, the development of thrombosis precedes the development of thrombocytopenia.

True thats why platelet are checked at baseline and monitored frequently

For a right skewed data, the main is greater than the median. T or F

True.

Furosemide-related ototoxicity is usually reversible but may be permanent in rare instances (e.g., in patients with renal failur. True or False

True.

T or F. Alcohol-based hand rubs have poor activity against spores.

True.

T or F. Case reports or Case series may generate hypotheses that can be then studied in larger trials with more robust study designs (eg., prospective cohort or randomized controlled trial)

True.

T or F. Inhibitors decrease the levels of prodrug's active forms.

True.

T or F. Sulfites or sulfate allergies do not cross react with sulfonamide.

True.

T or F. Heart failure is one of the most important conditions to counsel on lifestyle modifications and medications adherence.

True. > They must have sodium and fluid restriction.

Important about Amiodarone drug interactions

When starting amiodarone , decrease dose of digoxin by 50% and decrease dose of warfarin by 30-50% > Do not exceed 20mg/day of simvastatin or 40mg/day of lovastatin in patient taking amiodarone.

* Must know about Statin *

When the statin dose is increased, the risk is higher for muscle toxicity including rapid breakdown of muscle tissue (Rhabdomyloysis) which can cause renal failure as the muscle "breakdown" products enter the blood and travel to the kidney causing damage.

What is sigmoidoscopy?

examination of the sigmoid colon by means of a flexible tube inserted through the anus.

When to discard Serevent Diskus/Breo Ellipta?

discard device 6 weeks after removal from the foil or when the dose counter reads "0" (whichever comes first)

What are the drug of choices to prevent (Primary Prevention) of breaking out the veins balloon due to blood flowing up through the veins int he esophagus?

Non-selective beta-blockers (such as nadolol and propranolol) or Endoscopic Variceal LIgation (EVL) > The beta-blocker should be titrated to the maximal tolerated dose (target HR 55-60 BPM) and continuted indefinitely

NON-SMALL CELL LUNG CANCER

Non-small cell lung cancer is sometimes treated with surgery. Small cell lung cancer (SCLC) usually responds better to chemotherapy and radiation.

What is the difference between orange book and purple book?

Orange book for drugs Purple book for biological

How is Osteopenia is defined by T-score?

Osteopenia is defined by a T-score between -1 and -2.5

How is Osteoporosis defined by T-Score?

Osteoporosis (OP) is defined by a T-score equal to or less than -2.5

What Ovulation kits test?

Ovulation kits test for LH and are Positive if LH is present.

Out all of the psychiatric drugs, which one is considered to have the highest risk ?

Paroxetine (Brisdelle catgory X)

INR above therapeutic range but < 4.5

Reduce or skip dose. Monitor INR. Resume warfarin when INR therapeutic. Dose reduction may not be needed if only slightly above therapeutic range

Which triptan drugs used for migraine is contraindicated with strong CYP 3A4 inhibitors?

Relpax (Eletriptan)

List the REMS drugs?

Relprevv injection (olanzapine) suspention last 2-4 weeks

What formlation does Treprostinil comes in?

SC/IV (Remodulin) Inhaled (Tyvaso) Oal (Orenitram

What u know about prolactin?

Secretion is regulated by dopamine; can increase with haloperidol, riseperidone, paliperidone , methyldopa

Brentuximab (Adcetris)

TNFRSF8 (CD-30) -> If positive can use the drug

Corticosteroids may be used in SJS (benefit is controversial) but are contraindicated in TEN. True or False

True

T or F. DM acts as a sertotonin reuptake inhibitor. DM is a drug of abuse as it acts as NMDA-receptor blocker in high doses leading to euphoric and hallucinogenic properties similar to PCP.

True

T or F. DVT prophylaxis should not be giving with 24 hours of atleplase.

True

T or F. Dabigartan (Pradaxa) , Rivaroxaban (Xarelto), Apixaban (Eliquis) should not be used in patient with prosthetic heart valves.

True

T or F. Overdose of phenytoin will produce CNS-depressant effects.

True

T or F. Patient should receive adequate sedation and analgesia prior to starting a NMBA (neuromuscular blocking agents)?

True

T or F. Patient treated for RA require a bridge therapy with NSAIDs or Steroids which is not recommended for long period due to significant health risks.

True

T or F. Patients receiving anesthetics must be continuously monitored (vital signs and respiration)

True

T or F. Peanuts and soy are in the same family and can have cross reactivity.

True

T or F. Three days immediately from the positive result is the highest chance for pregnancy.

True

T or F. Vertical transmission (from mother to child) may also occur, either during pregnancy, at birth, and through breastfeeding.

True

Mirtazepine

- Remeron SolTab BBW - Antidepressant increase risk of suicidal thinking and behavior in children, adolescents and young adults (18-24 years of age) with major depression disorder and other psyo disorder Warning - Anticholinergic effect, QT prolongaton, blood dyscrasis, CNS depression SE - Sedation and increase appeptite, and weight gain , etc.

Clindamycin (Cleocin)

- Reversibly binds to the 50S ribosomal subunit and inhibit bacterial protein synthesis - BBW > Can cause severe and possibly fatal colitis - SE > *GI upset (N/V/D)*, rash, urticaria, increase LFT

Fosamprenavir, FPV (Lexiva)

- Take with/out food (adults) WARNING - Use caution in patient with a sulfa allergy Side Effects > N/V/D , Rash (including SJS/TEN), HA, nephrolithiasis Warning * Caution when dispensing: potential for medication error among Lexiva, Lexapro, and Levitra. *

Important note about sodium Oxybate (Xyrem)

- Taken with 1/4 cup of water, usually in an empty pharmacy pill container. Patient should lie down immediately after taking and stay in bed. The second dose is taken 2.5 - 4 hours after the first. Will typically fall asleep within 5-15 minutes after taking 1st dose. - Contains a high sodium content

Bile Acid Resins

- Work by binding bile, blocking reabsorption. Bile is produced from cholesterol and cholesterol levels decrease. The mechanism is uknown. Contraindicaiton > History of bowel obstruction > TG > 500mg/dl > History of hypertriglyceridemia-induced pancreatitis SE > Constipation (>10%) > Can increaes TGs by 5% > etc Notes > Welchol may decrease absorption of vitamins A,D,E, and K

Drug treatment for SCD

- immunizations and antibiotics - analgesics - folic acid - Hydroxyurea > To reduce frequency of pain episodes and acute chest syndrome - Iron chelation therapy

What are the core symptoms of ADHD?

- inattention, hyperactivity and impulsivity.

What are the long term complications of acid suppressants therapy?

- increase risk of GI infections (most commonly caused by C.difficile) - Increase risk of nosocomial pneumonia in hospitalized patients - PPIs increase risk of osteoporosis and fractures - BEERS Criteria > All H2RAs should be avoided in elderly patients with delirium, dementia or cognitive impairment because of adverse CNS effects that could make these worse.

List common drugs that increase HTN

- ACTH , Alcohol in excess , Amphetamines , Appetite suppressants, Caffeine, Calcineurin antagonists, Corticosteroids, Decongestants, Erythropoiesis Stimulating Agents, Estrogen, Mirabegron (Mybetriq), NSAIDs, Certain oncology medications, SNRI at higher doses, Thyroid hormone (if overdosed)

What is the difference in components between Kcentra and Bebulin/Profilnine?

- Bebulin and Profilnine contains II IX and X but low or notherapeutic levels of factor VI and should not be confused with Prothrombin Complex Concentrate (human) (factors II, VII, IX and X), protein C and Protein S which contains therapeutic level of factor VII

Why medications normally giving to treat acute gout attacks are giving prior to prophylaxis?

- Because acute attacks which can occur when urice acid is lowered rapidly

Why seizure so danger?

- Because it can damage and destroy neurons, which can cause cognitive deficits and be life-threatening.

Tizanidine (Zanaflex)

- Central alpha 2 agonist > Hypotension, Dizziness, xerostomia, weakness, QT prolongation * important * > Contraindicated with ciprofloxacin and fluvoxamine due to elevated tizanidine levels

Which medication can cause positive coombs test?

- Cephalosporin - Isoniazid - Levodopa - Methyldopa - Nitrofuranotin - Penicillins - Quinidine - Quinine - Rifampin - Sulfonamides

What cause a decrease in RBC?

- Chemotherapy that target the bone marrow - Low production - blood loss - Deficiency anemias (B12 , folate) - Hemolytic anemia - Sickle cell anemia

Which drugs cause the most Cardiotoxicity?

- Chest radiation - Anthracyclines - Tyrosine kinase inhibitor - Breast cancer drugs > Trastuzumab and Lapatinib Monitoring > ECG, ECHO, MUGA * Arsenic trioxide requires ECG and electrolyte (Mg, K) monitoring at baseline and weekly while on treatment to assess for QT prolongation *

Anticholinergics

Short-acting - Ipratropium bromide (Atrovent HFA) > + albuterol (Combivent Respimat, DuoNeb) > Usually given QID Long-acting - Aclidinium (Tudorza Pressair): BID - Tiotropium (Spiriva HandiHaler): QD > requires 2 puffs - Umeclidinium (Incruse Ellipta) : QD

T or F. Niacin and Statin cause HYPERGLYCEMIA.

True

T or F. Omalizumab should always be given at Dr's Office.

True. - BBW > Anaphylaxis > > it can occur after the first dose but also has occurred beyond 1 year after beginning treatment. Warning > Increase risk of CVD and Cerebrovasuclar adverse events > Malignancies have been obsereved

T or F. MedGuide are considered part of Labeling.

True. Medication Guides present important adverse events that can occur with over 300 medications. MedGuide are FDA_approved patient handouts and are considered part of the drug's labeling. If a medication has a MedGuide, it should be dispensed with the original prescription and *each refill*

T or F. Rate control favor because Rhythm put patient in new arrhythmias.

True. Pro-arrythmia

What is a big issue with Vasopressors?

Vasopressors are vesicants that cause severe tissue damage/necrosis with extravasation. This is a medical emergency. For all, treat with phentolamine, an alpha adrenergic blocker that antagonizes the effect of vasopressor

Which formulation of Pancrelipase is not enteric coated and needs to be given with PPI?

Viokace

Minocycline (Minocin, Solodyn)

Warning > < 8 years of age, pregnancy (suppress bone growth and skeletal development, permanently discolors teeth) SE > Gi upset (n/v/d), photosensitivity, rash

Tramadol (Ultram, Ultram ER, Conzip IR/ER)

Warning > Increase seizure risk - avoid in patient with seizure history, head trauma Notes > Can cause physiological dependence > Serotonin syndrome risk if used in combination with others such as SSRIs. > Avoid tramadol with 2D6 inhibitors (require conversion)

What is the difference between SE and ADR?

Side effects or adverse events are not generally avoidable and can occur in anyone with normal doses (although higher doses can increase the side effect severity). Side effects are more common and generally less severe (such as orthostatic hypotension from doxazosin) while adverse events are known complications of a drug but are generally rarer and more severe (such as rash from lamotrigene)

What does the Federal prescription drug disposal guidelines recommend when it comes to proper disposal of prescription drugs?

Federal recommend to mix the unwanted drug with unpalatable substances (dirt, kitty litter, or used coffee grounds) and placing them in a non-de-script container before discarding in the trash unless it is one of the drugs that is okay to be flushed down the toilet or sink.

What is the active ingredient in vyvanse?

Lisdexamfetamine (pro-drug of dextroamphetamine) > can mix capsule contents with water, yogurt or orange juice. take right away > QAM, with or without food * Lisdexamfetamine is hydrolyzed in the blood to active d-amphetamine. If injected or snorted the fast effect would be muted. The design is to decrease the abuse potential *

Patient wants to do TB TEST but also require to get a live vaccine MMR. What is the best advice to give?

Live vaccines can be administered on the same day and is the preferred method to avoid a false negative response to the skin test. If live vaccine is given recently then patient must wait at least 4 weeks before placing the PDD in order to avoid false negative. If patient is given PDD first then wait 48-72 hours and determine the PPD result before administering live vaccine.

What are the two primary site for drug metabolism?

Liver (CYP450) Gut

How to calculate Loading Dose?

Loading dose = Desired concentration (Vd)/ F > Some times it is beneficial to administer loading dose to rapidly achieve the targeted levels.

Hyperthyroidism treatment in pregnancy

Mild cases will not require treatment Severe cases require treatment with Category D drugs : Propylthiouracil is used if trying to conceive and in 1st trimester, then it is generally reasonable to switch to methimazoe. > Both are high risk for liver injury and there is risk with either to the neonate

Lipohypertrophy can be significant with these 2 drugs?

PI and Efavirenz (breast enlargement has been reported)

Agents used to treat VRE. faecalis?

Pen G or ampicillin

Antibioites

Penicillin or Benzathine penicillin IM Q 4 weeks If allergy Erythromycin

Which antibiotics are okay to use in pregnancy?

Penicillins Cephalosporins Azithormycin Erythormcyin * not clarithromycin, which is C *

Succinylcholine is the only available depolarizing agent and is typically reserved for intubation. What is the difference in MOA between depolarizing agent and non-depolarizing agent?

Succinylcholine binds to and activate the acetylcholine receptors and desensitizes them. The non-depolarizing NMBAs works by binding to the acetylcholine receptor and blocking the actions of endogenous acetylcholine

Which drug that have significant QT risk?

- Thioridazine

Chlamydia/Chlamydophilia , Mycoplasma pneumonaie

Atypical

Erlotinib

Tarceva

What is the mechanism of Non-Bendodiazepines (zolpidem etc)

- Act selectively at the benzodiazepine receptors to increase GABA - Intermezzo SL 3.5mg for males - Intermezzo SL 1.75mg for females

Which drugs do not bind to charcoal?

- Alcohol - Alkalis, Acids - Heavy metals (Iron, Lead, Lithium, Mercury

Retinopathy Screening

- Annual dilated, comprehensive eye exam is recommended in patients with diabetes - If the patient has one or more normal eye exams and their blood sugar is well controlled, screening can be done every 2 years - Type 1 > Annual testing beginning 5 years of diagnosis in patients >=10 years old - Type 2 > Annual testing beginning soon after diagnosis

* Must know about Perioperative Antimicrobial Prophylaxis *

- Antibiotic should be initiated 60 minutes before the incision (or 120 minutes for vancomycin or fluoroquinolones) > 120 mins for vancomycin or fluoroquinolones to avoid serious adverse effects - In general 1st cephalosporin is used (Cefazolin) for most procedures - Vancomycin is used as an aletrnative in beta-lactam allergic patients o when MRSA is concern - If surgery involve bowel movement then we will add Metronidazole (cover anaerobic infection) + 2nd or 3rd generation cephalosporin (cefotetan, ceftriaxone) or Ertapenem if allergies to cephalosporin

Which type of schizophrenia cause NMS?

- Antipscyhotics used to be called neuroleptics. NMS is rare but is highly lethal. It occurs most commonly with FGAs and is due to D2 blocked. so what happens is as the intense muscle contractions can lead to acute renal injury due to Rhabdomyloysis from the destruction of muscle tissue, suffocation and death.

Boxed Warning associated with Antipsychotics

- Antipsychotics (APs) increase the risk of mortality in elderly patients with dementia-related psychosis, primarily due to an increased risk of stroke and infection.

List some drugs that can lower the seizure threshold

- Antipsychotics (e.g., clozapine, phenothiazines, butyophenones) - Antivirals - Buropion - Carbapenems - Cephalosporins - Fluoroquinolones - Lindane - Lithium and theophylline - Mefloquine - Mepridine - Metoclopramide - Natural products such as dendrobium, evening primrose oil, gingko, melatonin - Penicillins - Sleep deprivation, alcohol intoxication, mensturation, infection and fever - Tramadol

Sickle Cell Pain Management Principles

- Appoint one health care provider to write prescriptions for long-term opioids - Believe in patient's pain - Patient should be seen every 2-3 months - Encourage fluids, fiber, stool softeners, and stimulant laxatives PRN - Use a partnership agreement and individualized treatment plan (patient right and responsibilities including random drug testing)

What are the 3 BBW with Ketoconazole?

- Associated with Hepatotoxicity - Concimtant use with certain drugs lead to life-threatening ventricular arrhythmias - Use oral tablet only when other effective antifungal therapy is unavailable or not tolerated and the benefits outweigh risks

Counseling with the lozenge

- Dissolve in the mouth - Acidic beverages interfere with the buccal absorption of nicotine so eating and drinking anything except water should be avoided for 15 minutes before or during use of the nicotine lozenge. - Patient should use 1 lozenge every 1-2 hours

Butrans Boxed Warning (patch)

- Do not exceed a one 20mcg/hr patch due to risk of Qt prolongation SE > Sedation, etc.

Important interactions with Lovastatin

- Do not exceed lovastatin 20mg/day with danazol, dilitiazem, dronedarone, or verapamil - Do not exceed lovastatin 40mg/day with amiodarone

Important interactions with Simvastatin

- Do not exceed simvastatin 10mg per day with Verapamil, Dilitiazem, Dronedarone - Do not exceed simvastatin 20mg per day with Amiodarone, amlodipine or ranolazine

Enoxaparin [Notes]

- Do not expel air bubble from syringe prior to injection - Do not administer IM - Store at room temperature - Largely neutralized by protamine > Not a true anti-dote

Ranolazine (Ranexa)

- Do not take if you take any of the following > Ketoconazole (Nizoral) > Itraconazole (Sporanox) > Infection: Clarithromycin (Biaxin) > Depression (Nefazodone) > HIV drugs nd TB drugs > Herbal: St John Worts

Important about Intermezzo (zolpidem)

- Do not take unless planning to sleep 4+ more hours - lifestyle changes should be the primary method to improve sleep, not drugs - Preferred over benzodiazepines for 1st line treatment of insomnia due to less abuse , dependence and tolerance. - Do not take with fatty food, a heavy meal or alcohol

List 2 drugs that are emollients, lubricants (stool softeners)

- Docusate sodium > Colace - Codusate Calcium - Mineral oil > take vitamin at a different time due to risk of fat-soluble depletion

Important about Conjugated estrogens/Bazedoxifene (DUAVEE)

- For prevention only > Osteoporosis prevention (in postmenopausal women with a uterus and for vasomotor symptoms BBW 4 1. Endometrial cancer risk (if used without progestin in women with a uterus) 2. Dementia risk in women >= 65 years of age taking estrogen alone 3. Women taking Duavee should not be taking additional estrogens 4. Do not use to prevent cardiovascular disease due to increase VTE risk in postmenopausal women 50-79 years of age.

List some Topical steroids used in UC and CD?

- Hydrocortisone (Cortifoam, Cortenema) Notes > Topical steroids have not been proven effective for maintenance of remission > Advantage of topical therapy include less systemic absorption and less frequent dosing schedule - Budesonide rectal (foam - Uceris) Note > For mild-moderate distal UC > Propellant is flammable > CYP 3A4

Important about H2RAs

- If you are self-treating for more than 14 days or more than 2 times per week and heart burn persists, discuss your symptoms with a healthcare provider

3rd generation cephalosporin

- Increase gram - activity compared to 2nd generation, including HNPEK +/- Serratia (HNPEKS) and some additional enteric gram - rods; decrease activity toward staphylococcal compared to 1st generation but increase streoptococci acitivity. * Note that ceftazidime has decreased gram + activity but has high gram negative activity including Pseudomonas * - Cefdinir, Cefditoren, Cefixime, Ceftoaxime, Cefpodoxime, Ceftazidime (fortaz, Tazicef), Ceftibuten , Ceftriaxone (Rocephin) - above names contain many Is *HINT*

2nd generation cephalosporin

- Increased activity toward gram - compared to 1st generation with similar gram+ activity including PEK, Haemophilus and Neisseria (HNPEK). * Cefotetan and Cefoxitin have anaerobic activity (Bacteroides fragilis), but decrease gram + activity - Cefaclor - Cefotetan - Cefoxitin -Cefprozil - Cefuroxime (Ceftin, Zinacef)

Important points about the mainstay for treatment for patients with relapsing forms of MS (Interferon beta)

- Avonex: weekly - Rebif: three times per week - Betaseron, Extavia: every other day - Peginterferon beta-1a: every 14 days WARNING - Depression/suicide, injection site necrosis, myelosuppression, Increase in LFT, thyroid dysfunction, infections, anaphylaxis, worsening cardiovascular disesae, seizure risk SIDE EFFECTS - Flu-like symptoms - Injection site reactions Note - Refrigerate all the items above EXCEPT Betaseron and Extavia which can be stored at room temperature. If refrigerated, let stand to room temperature prior to injection. Do not expel small air bubble in pre-filled syringes due to loss of dose.

Tigecycline (Tyacil)

- Binds to 30S ribosomal subunit inhibiting protein synthesis - same as Tetracyclines - Bacteriostatic - Cover MRSA VRE - * No activity agains the 3P's: Pseudomonas, proteus, providenica species * - FDA suggest to use this drug only when other alternative not available BBW > Increase risk of death Notes > Avoid use in bloodstream infections since it doesn't stay in blood (lipophilic) > Reconstituted solutions should have yellow-orange; discard if not this color * Tigecycline can increase INR in patient taking warfarin *

Monoclonal antibodies ( Bevacizumab (Avastin) )

- Binds to VEGF - A - Impair wound healing: stop at least 28 days before elective surgery and may restart bevacizumab 28 days after surgery BW - Bleeding, GI perforation, wound dehiscence SE - bleeding, hypertension, HF, thrombosis (including DVT, PE and stroke), GI perforation, wound dehiscence, etc.

Theophylline

- Blocks phosphodiesterase causing increase in cAMP which promotes release of epinephrine from adrenal medulla cells - Active metabolites are caffeine and 3-methylxanthine - Therapeutic range 5-15 mcg/ml > Measure peak after 3 days of oral dosing, at steady state sE > nausea, loose stools, headache, tachycardia, inosmnia, tremor, and nervousness Note > dose based on IBW > If using IV aminophylline, then multiply by 0.8 Theophylline dose is 80% of aminophylline dose

Aromatase inhibitors

- Blocks the conversion to active estrogen/androgen/corticosteroid/mineralcorticoid to reduce cell growth in breast, prostate and/or adrenal cancer. These agents are approved for post-menopausal women and are not-fda approved for men with breast cancer.

Baclofen (Lioresal)

- Boxed Warning > Avoid abrupt withdrawal of the drug; abrupt withdrawal of intrathecal baclofen has resulted in severe sequelae (hyperpyrexia, obtundation, rebound/exaggerated spasticity, muscle rigidity, and rhabdomyolysis) leading to organ failure and some fatalities. SE > For all muscle relaxants >> Excessive sedation, dizziness, confusion

What to know about Hydroxychloroquine?

- Brand name (Plaquenil) - Contraindications > Retinopathy > Hypersensitivity to 4-aminoquinoline compounds - SE > Diarrhea, Vomiting, Nausea, abdominal pain, rash, pruritus, headache, vision changes (dose-related), pigmentation changes of the skin and hair (rare) - Monitoring > CBC at baseline and periodically, LFTs. Eye exam at baseline and every 3 months during prolonged therapy

What are the recommended vaccinations for patient with COPD?

- Influenza - Pneumococcal

What is the DOC for maintenance and Rescue?

- Inhaled corticosteroids: 1st line treatment for maintenance - SABA (albuterol) for rescue

Important about Estrogen and Progestin combination oral contraceptives (COCs)

- Inhibit FSH and LH which prevent ovulation. - May prevent pregnancy by altering the endometrial lining, altering cervical mucus, interfering with fertilization or transport of an egg, or preventing implantation. - Used for various conditions such as Dysmenorrhea, PMS, etc. - Polycystic ovary syndrome (PCOS) is a common condition ( around 15% of women) with a typical presentation of infrequent or prolong menstrual periods, hirsutism acne and excessive weight.

Fosfomycin (Monurol)

- Inhibit bacterial cell wall synthesis by inactivating the enzyme pyruval transferase, which is critical in the synthesis of cell wall - Bactericidal - * used as single dose to treat uncomplicated UTI (systitis only) due to E.coli and E.faecalis - Female uncomplicated UTI > 3gram PO x 1 dose, mixed in 3-4 oz of cold water

Oritavancin (Orbactiv)

- Lipoglycopeptide that inhibit cell wall biosynthesis by inhibiting the polymerization step by binding to stem peptide of peptioglycan presucrosr and disrupting bacterial membrane integrity leading to cell death - Concentration dependent bactericidal - CI > Use of Unfractionated heparin for 48 hours after Oritavancin due to artificial interference with aPTT lab results - SE > Nausea, vomiting - Compatibile in D5W ONLY!!!! *Oritavancin is a one-time dose *

Benzodiazepines (for sleep)

- Lorazepam (Ativan) - Temazepam (Restoril) - Estazolam (Prosom) > cannot be used with potent 3A4 inhibitor - Quazepam (Doral) > Long half-life: risk of falls, fractures - Flurazepam > Long half-life: risk of falls, fractures - Triazolam (Halcion) > Associated with higher rebound insomnia and daytime anxiety; tapering upon discontinuation

Epothilone

- M-phase: microtubule stabilizer enhacing polymerization of tubules halting cell division; mechanism similar to taxens - Ixabepilone (Ixempra) > Use non-PVC IV bag and tubing due to leaching of DEHP BW - Hepatic disease

What are the side effects associated wit Milnacipran (Savella)

- Nausea, headache - Constipation, dizziness - Insomnia , hot flashes * Increased bleeding risk with anticoagulants or antiplatelets *

What is the difference between simple partial seizures and complex partial seizures?

- Simple partial seizures where consciousness not impaired and complex partial seizures where consciousness impaired

What are some recommendations to give for patients who are on steroid inahler to help reduce fracture risk?

- Smoking cessation, exercise, using lowest, effective steroid dose, Ca2+ and vitamin D supplementation with prescription therapies if needed and obtaining regular bone density screening

List some drugs that put patient at Risk for osteoporosis

- Steroids [Equivalent of 5mg/d of prednisone equivalent for >=3 months) - Depot medorxyprogesterone acetate - Anticonvuslant (carbamazepine, fosphenytoin, phenobarbital, phenytoin, primidone, others) - Heparin and Lithium - Excessive thyroid hormone - Loop diuretics - Aromatase inhibitors used for breast CA - Nafarelin - Androgen blockers used for prostate - Proton pump inhibitor because it will lower ACID environment which is required for calcium absorption - SSRI - TZD (pioglitazone and rosigloitazone)

What is the 1st line drug therapy for ADHD?

- Stimulants > stimulants are tried first (2 or 3 agents), if these are not working enough, then prescriber can try atomoxetine (Strattera), a non-stimulant medciation > Some prescribers start atomoxetine right away if they are concern the possibility of an abuse by the patient or family

What are the two type of IBD?

- The major types of IBD are ulcerative colitis and Crohn's disease. - The hallmark symptom is persistent bloody diarrhea

Pancreatic Enzyme Products (PEPs)

- The thick mucus obstructs pancreatic enzyme flow, resulting in a paucity of these enzymes reaching the GI tract. > Results are greasy, foul-smelling stools are manifestations of pancreatic insufficiency. - Pancrelipase is natural product harvested from Porcine. > Contain Lipase, amylase, and protease - Dose is based on lipase component > dose is adjusted after 3-4 days until stools are normalized. - Do not use doses > 6000 units/kg/meal of lipase due to colonic stricture risk

Does collection bins " Take Back" Container require locks? If yes why?

- There should be two separate locks on the secured collection bin: One key should be in the possession of the pharmacy and the other one with the licensed integrated waste hauler who will pick up what is now classified as " hazardous household waste" - The dual lock ensures that the pharmacy cannot open the collection bin without the presence of the integrated waste hauler, and vice versa - Some pharmacies do not participate in Take Back Programs, but offer services for disposal by mail, including common "takeaway" which is processed by companies such as Sharps.

What are the classic signs and symptoms of asthma?

- Wheezing - Coughing - Breathlessness - Chest tightness

* Must know about Calcineurin Inhibitors (Tacrolimus and Cyclosporine) *

- With transplant drugs the serum level needs to remain constant, around-the-clock to reduce the risk of graft rejection - Many drug interactions - Transplant patients are immune-suppressed and this results in illness fungal infections may be treated with systemic azoles (which are inhibitors), bacterial infections may be treated with macrolides (most are inhibitors) - Grapefruit juice is an absolute * do not take * with CNIs

What is the function of G6PD?

- Without sufficient levels of the enzyme to protect the RBCs, the RBCs become damaged or destroyed via hemolysis 24 to 72 hours after exposure to oxidative stress such as infections, certain food and drugs.

Is gabapentine used for RLS?

- Yes. - The dose must be reduced with renal impairment of CrCl < 60 ml/min to avoid increased side effect: > dizziness, somnolence, ataxia, peripheral edema, weight gain, dipolpia, blurred vision, dry mouth - Usually taken at 5 pm

Leukotriene Modifying Agents

- Zafirlukast (Accolate) > Keep in the original container > CI: hepatic impairment > Take on empty stomach (food decrease bioavailability by 40%) - Montelukast (singulair) > age 1-5: take 4 mg daily in the PM > 6-14: 5 mg daily in the PM > 15 and up: 10mg daily in the PM - Zileuton (zyflo, Zyflo CR) > CI (active liver disease or LFT >= 3 xUNL > Need to monitor LFT every month for first 3 months , then every 2-3 months for the rest of the first year of therapy SE FOR ALL OF ABOVE Headache, etc.

Natural Products used for Colds

- Zinc > Do not use longer than 5-7 days because it cause copper deficiency > Do not recommend zinc nasal swab or nasal spray due to risk of loss of smell - Vitamin C - Echinacea - Airborne has all the above items combined.

Metoclopramide

- dopamine antagonist and at higher doses, it blocks serotonin receptors in the chemoreceptor zone of the CNS - It accelerate gastric emptying - BBW > Cause TD - increase risk in elderly SE > Somnolence, dystonic reactions, restlessness, fatigue etc. Notes - CNS side effects - Avoid use in parkinson disease

What are the risk factors for breast cancer?

- family history - Genetics (BRCA1 or BRCA2 mutations) - Early menarche - Late menopause (which increase lifetime exposure to estrogen) - Late pregnancy > 30 years old - Nulliparity (no pregnancy) - Smoking , obesity and lack of exercise.

Capectiabine (Xeloda)

- pro-drug of 5-FU - PO BID - take within 30 minuets after a meal - Pharmacogenomic testing for dihydropyrmidine dehydrogenase (DPD) > Deficiency put patient at increased risk for toxicity - Capecitabine can increase INR up to 91% due to CYP2C9 inhibition BBW > Increase INR during and up to 1 month after discontinuation; reduce wafarin dose, monitor carefully; fatal bleeding can occur CI - Dihydropyrimidine dehydrogenase (DPD) defieincy - CrCL < 30ml/min SE - Hand-food syndrome (more than 5FU), diarrhea, *mucositis* , gastritis, N/V, etc.

What are the reasons to use Lozenges?

- used for oral mucosa drug administration - Used to treat condition in the oral mucosa - the drug is held in the mouth while the troche slowly dissolves - Example: > Mycelex - Clotrimazole Torche

Important about Methotrexate

- Otrexup and Rasuvo are SC auto-injectors used weekly > Less Nausea and Vomiting - Low weekly doses are used for RA (the weekly dose can be split in 2-3 doses given 12 hours apart); never use daily for RA - BBW > Hepatotoxicity > Acute renal failure > Pneumonitis > Bone marrow suppression > Mucositis/stomatitis > Dermatologic reactions > Malignant lymphomas, others > Renal and lung toxicity more likely when using oncology doses.

PPI

- PPIs block gastric acid secretion by irreversibly binding to the gastric H/K-adenosine triphosphatase (ATPase) pump in parietal cells. - Can take Dexilant (Dexlansoprazole) without regard to meals - Can take Zegerid at HS to control nighttime symptoms - May take Aciphex with or without food WARNING - Can increase risk of C-difficile-associated diarrhea (CDAD) - Can increase osteoporosis-related fractures, especially long term - Can increase risk of pneumonia in hospitalized patients - Hypomagnesemia with long term use and vitamin b12 deficiency with prolonged use (>= 2 years)

What is the difference between Partial seizure and Generalized seizure?

- Partial seizure start in one part of the brain. If there is no loss of consciousness, the seizure is called simple partial. If there is loss of consciousness, the seizure is termed complex partial. - Partial seizures can spread to the other hemisphere of the brain, resulting in secondarily generalized tonic clonic seizures. - Generalized seizures, begin in both hemispheres of the brain and consciousness is impaired

What is used for mild SLE disease?

- Patients with mild disease may do well on an NSAID (dosed at anti-inflammatory doses) but use caution the doses are high and these patients are more sensitive to the GI and renal side effects. - Concurrent use with a PPI is generally recommended to reduce GI side effects of NSAIDs.

Important information about Malaria

- Prophylactic medication available, Transmitted by Mosquito - Malaria is transmitted by the bite of an infected *Anopheles mosquito* - P.vivax causes up to 65% of malaria cases and is becoming resistence to drugs. - P.falciparum is the most deadly species - When deciding which drug to give patient, think about cost, allergies, pregnancy, if an infant or child, and concurrent conditions

What is PAH?

- Pulmonary Arterial Hypertension (PAH) is characterized by continuous high blood pressure in the pulmonary arteries. - The average blood pressure in a normal pulmonary artery (called pulmonary artery pressure) is about 14 mmHg when a person is setting. - mPAP > 25 mmHg in the setting of normal fluid status defines PAH

Methadone (Dolophine) Methadose-Liquid

- REMS drug - Methadone 40mg is indicated for detox and maintenance treatment of opioid addicted patients. > Useful for detox because it relieves opioid craving and blocks euphoric effects of abusable opioids BBW (2) 1. QTc interval prolongation and serious arrhythmias (TdP) have occurred during treatment 2. Respiratory depression NOTE > Serotonergic

ADA treatment guidelines for type 2 diabetes

- Rapid-acting secretagogues may be used in place of SUs - Consider beginning at the 2 drug stage in patient with A1C >= 9% - Consider meglitinides in patients with irregular meal schedule who develop late postprandial hypoglycemia on SUs - Consider starting with insulin in patient with severe hyperglycemia (>=300mg/dL or >= 10%) - After initial treatment of metformin (unless CI), wait 3 months then see if there is improvement in A1C - Do not combine Sulfonylurea with Meglitinides since same moA.

What is the drug of choice for children < 12 months for Nasal congestion?

- Recommend cool mist to loose the congestion - Steaming in the bathroom up with the shower while a parent sits with the baby outside the shower - be carfull not to let hot steam get near child's skin - OTC cough and cold medicines are not used in children < 2 (FDA) or < 6 (AAP) - * Gentle suctioning with saline drops or spray to loosen the mucus can provide relief *

Important concepts about Antimicrobial Stewardship Programs (ASP)

- Reduce the emergence of resistance - Limiting drug-related adverse effects - minimizing risk of unintentional consequences associated with antimicrobial use > How they do these? 1. Restriction and/or pre-authorization policy 2. automatic IV-PO medication interchange 3. De-escalation or streamlining of therapy based on patient response and culture/susceptibility results 4. Disease care pathways or protocols

OLANZapine (Zyprexa) Zydis ODT

- Relprevv injection suspention last 2-4 weeks require REMS - QHS (sedating) BBW - Sedation (including coma) - Deleirium (including agitation, anxiety, confusion) has been observed following IM injection SE - Somnolence - Weight gain, increase in lipids, increase in glucose - QT prolongation (lower risk) NOTE _ Smoking reduce drug levels

* Chloramphenicol *

- Reversibly binds to the 50S ribosomal subunit of susceptible organisms inhibiting protein synthesis - Bactericidal - BBW > Serious and fatal blood dyscrasias (aplastic anemia, thrombocytopenia) - Warning > Gray syndrome SE > Myelosuppresion, aplastic anemia, decrease in vitamin b12

What cause high prolactin levels?

- Risperidone and Paliperidone

Pulmonary Hypertension and SCD

- SCD cause increase in pulmonary Hypertension which put pressure on the right side of heart and lead to heart failure

Long-Acting Beta-2 Agonist

- Salmeterol (Serevent Diskus) - Salmeterol + Fluticasone (Advair Diskus) - Formoterol (Foradil Aerolizer) - Formoterol + Budesonide ( Symbicort) - MDI: 2 inhalations BID Boxed Warning - Increase risk of asthma related death - Contraindicated in Asthma - Once asthma control is achieved and maintained, assess the patient at regular intervals and step down therapy (d/c LABA) if possible w/o losing asthma control NOTE Foradil capsule should be placed in the Refrigerator in the pharmacy and they are kept at room temperature for 4 months once picked up by the patient.

Which of the following haplotype produce less VKORC1 and will require a lower dose of warfarin? A. haplotype A B. Haplotype G

A. Haplotype A produce less VKORC1 so require less warfarin other wise put patient at risk for bleeding

Hyperthyroidism Treatment

BBW - Severe liver injury and acute liver failure (PTU) SE - GI upset, headache, rash, pruritus, fever, constipation etc. - Hepatitis, arganulocytosis (rare) NOTE - Pregnancy category D - PTU preferred in 1st trimester- change to methimazole for 2nd and 3rd trimesters due to increased risk of liver toxicity from PTU - PTU is preferred in thyroid storms * PTU is not a first line treatment for hyperthyroidism except in patient who cannot tolerate other options or conditions where other antithyroid therapies are contraindicated.

What is the age that require Tdap?

DTaP series is indicated for children 6 weeks to 6 years of age - single Tdap dose for ages 7-10 years who were not fully vaccinated with DTaP series (missed dose in series) - Tdap is the one time booster for ages 11-64 years with no previous record of Tdap then one dose of Td every 10 years

What is the purpose of Sedation Vacations ?

Daily interruptions of continuous infusions of sedative drugs ('sedation vacations') are used to assess the readiness to wean off the sedative and to stop the drip as soon as medically feasible.

TYLENOL BBW SIDE EFFECTS and NOTES

BBW > Acetaminophen may cause severe hepatotoxicity, potentially requiring liver transplant or resulting in death > Hepatotoxicity is usually associated with excessive acetaminophen intake > 4g/day, high risk of dosing errors with injection Side Effects > Hepatotoxicity > Severe skin rash (SJS, TEN etc) >>> Stop drug, seem immediate medical help > Nephrotoxicity: rare renal damage but generally safer than NSAIDs in renal disease NOTES Antidote for overdose > N-Acetylcysteine (NAC) >> Restore glutathione >> Should be administered immediately, even before the results of APAP level are obtained; within 8 hours of ingestion >> Loading dose of 140mg/kg PO followed by 70mg/kg Q4H for 17 doses, unless the APAP level is not toxic. - IV form is Acetadote

Important drug interactions with Riva and Apixa

Both are a substrate of 3A4 (major) - Avoid given riva w/ strong inducers or inhibitors - If you give a strong inhibitor with Apixaban at doses > 2.5mg BID then decrease the dose by at least 50% - If patient taken Apixaban at 2.5mg BID > Avoid these strong dual inhibitors

What is the difference between Continental or dry gum method and English, or wet gum method?

Both have same ingredients (oil, purified water and gum (such as acacia) in the ratio of 4:2:1 but the order of mixing is different. Dry method: gum is mixed rapidly with oil then water added all at once Wet method: gum is dissolved in water first, then oil is slowly added

K+-Sparing Diuretic Drug interactions

Boxed Warning - Tumor risk with spironolactone; tumorigenic in chronic rate toxicity studies. Avoid unnecessary use Contraindications - Anuria , Significant renal impairment, hyperkalemia (addison's disease or other conditions that increase K), concomitant use with K-sparing diuretics - *for eplerenone: Concomitant use of strong 3A4 inhibitors; type 2 diabetes with microalbuminuria, SCR >2mg/dL in males, SCR > 1.8 mg/dL in females, CrCl < 50ml/min * SE > Hyperkalemia > Increase in serum creatinine > etc Monitoring > check K before starting and frequently thereafter; BP, electrolytes, SCR/BUN, fluid status (input and output, weight)

Allopurinol

Brand names Zyloprim, Aloprim, Lopurin WARNING > Hypersensitivity reactions can occur including severe rash (SJS/TEN). Can test for HLA-5801 prior to starting treatment (consider testing in Koreans with stage 3 or worse CKD, and Han Chinese or Thai irrespective of renal function) > Hepatotoxicity SE > Rash, acute gout attacks, nausea, diarrhea, increase in LFT Notes > avoid use with Azathioprine or mercaptopurine

HHS

HHS is a hyperglycemic crisis that most often occurs in type 2 and is due to some type of severe stress. - Serum Ketones would be negligible or not present because the type 2 patient has enough insulin to suppress ketoagenesis. Symptoms: BG > 600mg/dL, high serum osmolality > 320 mOsm/L, extreme dehyration, altered consciousness (confusion,dizziness) pH > 7.3, elevated bicarbonate > 18mEg/L, risk of seizure.

What should be measured in all HIV-1 infected patients at baseline and on a regular basis thereafter, especially in patients who are on treatment. because it is the most important indicator of response to ART. It quantifies the degree of viremia by measuring it amount in blood and it is used to assess response to drug therapy and disease progression (along with CD4+) count and possible medication adherence problems or drug resistance.

HIV-1 RNA (viral load)

What are the contraindication and side effects associated with Glycoprotein IIB/IIIa Receptor Antagonist? Abciximab (ReoPro) Eptifibatide (Inegrilin) Tirofiban (Aggrastat)

CI - Thrombocytopenia (< 100,000) - History of bleeding diathesis - Recent (within 6 weeks) GI or GU bleeding - Active internal bleeding - Recent (within 4-6 weeks) major surgery or trauma - high Prothrombin time - Severe Uncontrolled HTN - Hypersensitivity to maurine proteins (abciximab) - Dependency on renal dialysis (eptifibatide) SE Bleeding Thrombocytopenia Hypotension Note Must filer abciximab with administration

Injectable Bisphosphonates

CI > CrCL < 35 mg/minutes and if evidence of acute renal impairment (Reclast only) Warning > Boniva IV is not recommended for CrCl < 30 Note Zoledronic Acid (Reclast) only comes as 5mg etc.

* Clevidipine (Cleviprex) * INJECTION

CI > Do not use in soy or egg allergy, acute pancreatitis, severe aortic stenosis, lipoid nephrosis Side effects > HA , N/V Notes > In a lipid emulsion (provides 2kcal/ml) > Risk of infection and Hypertriglyceridemia. Use strict aseptic technique upon administration. Maximum hang-time per vial/bottles is 12 hours

What important things to know about Aluminum free, calcium Free? Lanthanum carbonate (Fosrenol)

CI in bowel obstruction, fecal impaction, ileus SE: N/V abdominal pain, constipation and diarrhea Monitoring: Ca2+, phosphorous and PTH * Must chew *

What is Cost-Minimization Analysis?

CMA is used when two or more interventions have already demonstrated equivalency in outcome and the cost of each intervention are being compared. * However, the use of this method is limited given its ability to compare only alternatives with demonstrated equivalent outcomes*

Which supplement is required if patient is on long-term Heparin or LMWH?

Calcium

Important drug interactions with Mycophenolate?

Can decrease level of hormonal contraception; mycophenolate levels can be decreased by antacids and multivitamins , cyclosporine , metronidazole, PPIs, fluoroquinolones, sevealmer, bile acid resins and rifampin and derivatives. Acyclovir , valganciclovir and ganciclovir will increase mycophenolate. * Avoid grapefruit juice and st john's worts with either CNI *

COX-2 Selective

Celecoxib (Celebrex) > Highest COX-2 selectivity > Contraindicated with sulfanomide allergy > Pre C < 30 weeks > Pre D >= 30 weeks

What is the separation time between live vaccine and antibody-containing products?

If patient receive vaccine (LIVE) first, then must wait at least 2 weeks before giving antibody - If patient receive Antibody first (IVIG) then must wait at least 3 months or longer before giving vaccine.

Why variations in HLA-B*1502 is significant?

If variations are present, the risk of a sever skin reaction to *carbamazepine or phenytoin * is increased from < 1% (wild type) to 5-10%

What increase WBC?

Increase due to corticosteroids Increase due to CSF (colony stimulating factors) Epinephrine

Na

Increase due to lithium [a sudden increase in sodium might prompt your lithium levels to fall.] , hypertonic saline Decrease due to carbamazepine, oxcabrazepine, SSRi, and diuretics

What are inhibitors?

Inhibitors are compounds (many of which are drugs) that inhibit the activity of the enzyme. Enzyme inhibition result in less drug metabolism. The drug serum level (and therapeutic effect) will increase. This can result in drug toxicity.

What is OTC drug monograph?

It is a "recipe book" of approved ingredients, doses, indications, formulations, and labeling requirements. If the drug and indication are already found in the approved OTC monograph, it may be marketed without further FDA review

What is the function of P-gp efflux pumps?

It is a pump that used to eliminate foreign (toxins) via pushing these toxins (efflux means "to flow out") or drugs back into the gut

* LAG EFFECT AND SUICIDE PREVENTION *

It is important to inform the patient that physical symptoms such as low energy improve within a few weeks but psychological symptoms such as low mood, may take a month or longer > This is why suicidal thinking are increased because they have so much energy from the effect on physical

What does Statistically Significant means?

It means it is unlikely to have occurred by chance. * When the P-value is smaller than the predetermined significance level (or alpha level), the difference found between groups is statistically significant and the study failed to accept (or reject) the null hypothesis.

Where to report medication error in any setting?

MEDWATCH ISMP's medication errors reporting program MERP

Niacin

MOA - Decrease the rate of hepatic synthesis of VLDL (Decresae TGs) and LDL; may also increase rate of chylomicron TG removal from plasma - Nicotinic acid or vitamin B3 - ER niacin (Niaspan) RX ONLY! CI > Active liver disease > Active PUD > arterial bleeding > arterial hemorrhage

What is the difference between Statistically significant and Clinically significant?

The measures of statistical significance is not the same as "clinical significance". Statistical significance reflects the influence of chance on the outcome; clinical significance reflects the clinical value of the outcome. Ex. if a blood pressure drug lowers SBP by 3 mmHg, it may be statistically significant (with a p-value < 0.05) versus placebo, but clinically the drug will not be used since other drugs lower BP to a greater degree.

What is the primary bacterial pathogen that cause Traveler's Diarrhea?

Most cases (>80%) are bacterial and E.coli abroad.

What is P-Value?

P = Probability Probability that the result obtained was due to chance. Generally a P-value < 0.05 (and sometimes < 0.01 or other values, depending on the trial design) indicated statistical significance. If P < 0.05 that means there is a < 5% probability that the result occurred by chance.

T or F. Methotrexate can cause alopecia.

True

Panttumumab = u

humna = 99.99% human

Where is Polymyxins used primarily?

It is used primarily in setting of multi-drug resistant gram- pathogens

T or F. Most cancers will not relapse if a patient remains cancer-free for 5 years.

True

T or F. Naproxen has the lowest CV risk.

True

List some drugs that work by inhibiting factor Xa?

Rivaroxaban (Xarleto) Apixaban (Eliquis) - These drugs work by inhibiting factor Xa - PO

Ethambutol (Myambutol)

SE - Optic neuritis, decrease in visual acuity Notes - Routine vision test (monthly), Serum Creatine

What is the MOA of Ergotamine?

Stimulate cerebral vasoconstriction and has some effect on serotonin

Pregnancy Category X

Studies in animals or humans show abnormalities; use in pregnancy is contraindicated

Where is the preferred place for Vivelle-Dot?

* Lower abdomen *

What is the most important thing to know about Tesimeleton (Heltioz)?

- Incresae ALT SE - HA , abnormal dreams , etc Note - Take w/o food

The difference between Reclast and Zometa?

- Reclast 5mg/year for osteoporisis - Zometa 4mg/month for oncology

What is the ANTIDOTE for Nicotine, including E-cigarettes in teenagers?

- Supportive care - Atropine

Zetia - SE

- URIS -Diarrhea - Arthralgias - Myalgias - Pain in exteremities - Sinusitis

What are symptoms of low Na?

- nausea, tiredness or lack of energy - headache, more frequent or more severe seizures and confusion

List 3 risk factors for NSAID-induced ulcers

1. Elderly 2. Previous bleed 3. chronic NSAID use or high dose 4. Concomitant anticoagulant, steroids, SSRI SNRI 5. Smoking 6. poor health

What is used for treatment of Hypercalcemia of malignancy treatment?

1. Hydration with normal saline and loop diuretics 2. Calcitonin 3. IV Bisphosphonates

List three way to estimate central tendency?

1. Mean 2. Median 3. Mode

Neomycin (Neo-Fradin) can be used to treat HE (hepatic Encephalopathy) List 3 BBW with this drug? This drug can also be used Topically for Infection of skin, In minor cuts, scrapes, and burns; Prophylaxis

1. Neurotoxicity 2. Nephrotoxicity 3. Neuromuscular blockade and respiratory paralysis especially if given soon after anesthesia or with muscle relaxant Side effects > GI upset is a big one

What are the difference study designs within the RCT?

1. Parallel study design 2. Crossover study design 3. Factorial Design

What is the 1st line therapy for treatment of acute migraine and what we give NEXT if contraindication or no benefit with 1st line therapy?

1st line therapy is Triptan If No Triptan We give Ergotamine

What is the maximum hang time per vial/bottle of Clevidipine?

12 hours

Al of the following alleles have reduced function when it comes to metabolization of Plavix EXCEPT? A. CYP 2C19*1 B. CYP 2C19*2 C. CYP 2C19*3 D. All have equal metabolization activity

A

Which of the following is considered with Non-Sterile Preparation? A. USP 795 B. USP 797

A

T or F. Insulin will decrease K level and is used acutely to manage Hyperkalemia.

This statement is True.

What is Normal Distribution?

A "normal" distribution is also known as a bell-shaped curve or Gaussian curve. In a Gaussian or normal distribution, the mean , mode and median would all have the same (or similar) value and would look like the figure.

Define Embolus?

A blood clot that has traveled from its point of origin.

What is Eutectic Mixture?

A eutectic mixture is two or more components that melt at a temperature lower than the melting temperature for the individual components.

What is haplotype?

A group of gene or DNA variations that exist on the same chromosome and are likely to be inherited together.

What are the recommended vaccinatons in Immune-compromised patients?

A. Annual influenza vaccine B. TDap substituted for the Td booster followed by the Td booster every 10 years C. Pneumococcal vaccines (both PCV13 and PSSV23) D. HPV vaccine for men and women up to the age of 26 years are recommended E. others depends.

Which of the following drugs are safe to use in pregnancy? A. Mefloquine B. Atovaquone/Proguanil (Malarone) C. Chloroquine (Aralen) D. Primaquine

A. Mefloquine

what weight we use in oncology?

Actual weight

What is the ANTIDOTE for Organophosphates (OPs), include industrial insecticides (malathion, others) and nerve (warfare) and gases (sarin) ?

Atropine and pralidoxime or the combo (DuoDote)

Which of the following is classified as dimorphic fungi? A. Candida B. Cryptococcus C. Aspgergillus D. Mucor E. Histoplasma F. Blastomyces

E. Histoplasma F. Blastomyces

Which type of CCB are moderate 3A4 inhibitors? A. Dihydropyridine CCBs B. Non-dihydropyridine CCBs

B

Which of the following show benefits in mania? A. Lamotrigiene B. Lithium

B - Lam only in bipolar depression - Lithium is used in mania, depression, and maintenance.

List 2 plasticizers used to soften the capsule?

Glycerol and Sorbitol

Which test and/or recommendations for Rituximiab (Rituxan) and Ofatumumab (Arzerra)?

B-cell CD20 expression If positive, can use drug

Patient is on warfarin 2mg taking three times a week. Patient is put on Rifampin. By how much should the dose of warfarin increase? A. Should be increased between 50-100% B. 100-300% C. 400-500%

B. 100-300%

What is the average life span of platelets? A. 2-5 days B. 7-10 days C. 12-14 days D. 120 days

B. 7-10 days

Which of the following MAO-Inhibitor causes stimulation? A. Isocarboxazid (Marplan) B. Phenelzine (Nardil) C. Tranylcypromine (Parnate) D. EMSAM E. Selegiline ?

C

* chikcen pox* varicella

Itchy rash, fever, malaise The rash appears as crops of sores (head, then trunk, then arms and legs) that turn into blisters, burst then form crusts

Ado-trastuzumab ematansine

Kadcyla

What test is used to distinguish type 1 from type 2 diabetes ?

C-peptide (fasting) A C-peptide test can be done when diabetes has just been found and it is not clear whether type 1 diabetes or type 2 diabetes is present. A person whose pancreas does not make any insulin (type 1 diabetes) has a low level of insulin and C-peptide. A person with type 2 diabetes can have a normal or high level of C-peptide.

What is the average life span of red blood cells RBC? A. 105 days B. 110 days C. 120 days D. 150 days

C. 120 days

We can only use (Cetuximab (Erbitux), Erlotinib (Tarceva), Afatinib (Gilotrif) if ............ positive. A. Her2 B. K-ras mutation C. EGFR

C. EGFR -> These medications have enhanced effectiveness in tumors expressing EGFR or who have EGFR-TK exon 19 deletion or exon 21 substitution mutations.

Which of the following anesthetic route can cause malignant hyperthermia (MH)? A. Topical B. INjectable C. Inhaled

C. Inhaled

Which of the following meningococcal vaccines are recommended for patients 56 yeas old or older? A. Menactra B. Menveo C. Menomune

C. Menomune

Which of the following thiazdies type diuretics works in patient with reduced renal function? A. Chlorothalidone B. Indapamide C. Metolazone (Zaroxoyln) D. Methyclothiazide

C. Metolazone

T or F. Hazard ratio = Relative Risk

True

Which of the following has injection formulation? A. Chlorthalidone B. Inadapmide C. Hydrochlorothiazide D. Chlorothiazde

D

T o F. you can use Beta blockers in Prinzemtal's angina.

False. It will cause more vasoconstriction

Which of the following is delivered via MDI (Select all that Apply)? A. Arformoterol B. Indacaterol C. Breo Ellipta D. Striverdi E. Olodaterol

D and E

What is Discrete Date and what are the 2 types of discrete data?

Discrete Data - can have only a limited, or finite set of values (ie., not continuous) and can assume only whoel numbers. The 2 types 1. Nominal: Consist of categories, where the order of the categories is arbitrary (e.g. martial status, gender, ethnicity 2. Ordinal: consist of *ranked* categories, where the order of the ranking is important. However, the difference between categories cannot be considered to be equal. Ex. Nominal: pass/fail Ordinal: A,B,C,D,F

The Cockcroft-Gault formula may not be preferable in: A. very young children B. End-stage renal disease (ESRD) C. Renal function fluctuating rapidly D. All of the above

D. All of the above

Cetuximab

Erbitux

What is the drug of choice for bladder cancer?

Gemcitabine (Gemzar)

What cause increase in HCO3

Loop diuretic corticosteroids

Hypothyroidism treatment in pregnancy

Must test for it and treat it with LEVO which is Category A

What is the drug of choice for sepsis shock?

Norepinephrine is considered the vasopressor of choice in septic shock

What is the NSAID to give if patient on lithium and wants to use NSAID?

Sulindac

aPTT and PTT to monitor what?

UFH Argatroban

MMA

Used for further workup of macrocytic anemia when B12 deficiency is suspected. Schilling test has been used

Oral Iron Drug Interactions

_ Iron must be taken 2 hours before or 4 hours after antacids. - Iron must be taken 2 hours before or 4 hours after Tetracycline - Iron must be taken 2 hours before or 6 hours after ciprofloxacin, 2 hours before or 2 hours after Levo , 4 hours before or 8 hours after moxi (longer half life) - Iron should be taken 60 minutes after oral ibandronate (Boniva) or 30 minutes after alendronate/risedronate.

Light exposure causes photo degradation that can destroy the drug and in some cases will increase the toxicity. Light sensitive drugs either come in light resistant packaging or can be protected with an.....?

amber light sensitive cover that is laced over the IV container

Agents used for skin and skin structure infections caused by community-associated methicillin-resistant staphylococcus aureus (CA-MRSA): Pick 1 drug

bactrim ds - SMX/TMP

What is recommended for maintenance of NSR?

flecainide dofetilide dronedarone propafenone sotalol

What is benzodiazepine antidote?

flumazenil

Fill in the blank. Example of drugs used for local effects include eye drops. Ex. Latanoprost is used topically to treat........

glaucoma: Prostaglandin analogue

Bevacizumab = zu

humanized = 90% human

T or F. RA is chronic, symmetrical, systemic and progressive disease.

true

- LABA (part 2)

- * Combination therapy with inhaled steroids can increase the risk of pneumonia, however, the combination showed a decrease in exacerbations and improvement in lung function when compared to the individual components * - Arformoterl contains R-isomer of formoterol

Oxymorphone (Opana , Opana ER)

- *Take on empty stomach* (most other analgesic are with food to help avoid stomach upset) - No alcohol with ER formulations

Important about Emtricitabine, FTC (Emtriva) + Tenofovir (Truvada) + Efavirenz and Tenofovir (Atripla) + Tenofovir and rilpirivine (Complera) + Tenofovir and elvitegravir and cobicistat (Stribild)

- 1 tablet daily for truvada, Atripla, Complera, and Stribild - Take Atripla on an empty stomach , preferbly at bedtime, take complera witha meal. Take stribild with food - BBW > May exacerbate Hepatitis B once drug is discontinued or when HBV resistance develops Side Effects > Hyperpigmentation, (mainly in children) > Rash

Elvitegravir (EVG) Cobicistat (COB) Emtricitabine, Tenofovir (Stribild)

- 1 tablet daily with food - Do not initiate if CrCl < 70ml/min; discontinue when CrCL < 50ml/min - Keep in original container BBW 2 1. Lactic acidosis with severe hepatomegaly with steatosis 2. Acute exacerbation of HBV in patients who are co-infected (specific for emtricitabine and tenovir) Side Effects Proteinuria, etc

Plan B

- 1.5 mg tab or two separate 0.75 mg tab of levonoregesterol (steroid form of progestin). This formulation of EC reduce the risk of pregnancy by 89% when started within 72 hours after unprotected intercourse. The sooner it is started, the higher the efficacy. - EC can be an important resource after unprotected sex, such as from missed pills, a condom breaking during intercourse, a diaphragm or cap that moved out of place during intercourse, or if a woman may have been sexually assaulted. - If patient vomits within * 2 hours * of taking the pill/s, they should consider repeating the dose.

Important information about Phenazopyridine (Azo, Uristat, Pyridium)

- 100-200 mg TID (OTC and Rx) x 2 days (max) - CI > CrCL < 50ml/min or liver disease -SE > Ha, dizziness, stomach cramps, body secretion discoloration - NOTES > Take with or following food and 8 oz of water to minimize stomach upset > May cause red-orange coloring of the urine and other body fluids. Contact lenses and clothes can be stained > Can cause hemolytic anemia in patients with G6DP

Important about Chronic Urate Lowering Therapy (ULT)

- 1st line > XO inhibitors ( Allopurinol / Febuxostat) > Titrate up (slowly for allopurinol) to lower uric acid level to a target < 6mg/dl. > Lower allopurinol dose with moderate or severe CKD > * People with high risk of severe allopurinol hypersensitivity reaction (including certain asian groups) should be screened for the HLA-B*5801 - 2nd line > Probenecid > Used if XO inhibitors are contraindicated or not tolerated or *can be added * when the uric acid is not at goal despite maximal doses of XO inhibitors. > * used only in patient with adequate renal function * >> Avoid used of CrCL < 30ml/minute

Oral Antihistamine

- 1st line agent for patients with mild-to-moderate disease. - They are effective in reducing symptoms of itching, sneezing, and rhinorrhea, but have little effect on nasal congestion. - Block H1 receptor site - 2nd generation preferred since they cause less sedation and cognitive impairment.

Tobramycin Inhalation Solution (TOBI, Bethkis)

- 300mg via nebulizer Q12H x 28 days, followed by 28 days off cycle - Indicated in CF patients >= 6 years who are colonized with P.aeruginosa to reduce infection/hospitalization SE - Ototoxicity, tinnitus, voice alteration, dizziness, bronchospasm NOTE > little systemic absorption TOBI: use with PARCI LC Plus reusable nebulizer and DeVilbiss Pulmo-Aider air copressor Bethikis: Use with PARI LC plus nebulizer and VIos air compresison - Doses should be taken at least 6 hours apart * - Recommended to store in frig, can be kept at room temperature up to 28 days - In foil to protect from light

Infections and SCD

- 35% of infants with SCD die from infections > Streptococcus pneumoniae and H.influenza > Seek medical attention if temperature > 101.3 F

Treatment for Intestinal and Nutritional Problems

- A high fat and calorically dense diet to help with nutrition - Pancreatic enzyme replacement to optimize growth and nutritional status and promote healthy bowls - Proton Pump inhibitor to prevent degradation of pancreatic enzymes in the stomach and for treatment of GERD - Vitamin supplements, especially the fat-soluble vitamin ADEK - Insulin for treatment of CF_ related diabetes

Fall Risk/ Fall Prevention Measures

- A home safety assessment should ensure that lighting is appropriate, floors are safe (throw rugs/clutter/cords have been removed), storage is at reasonable heights, bathrooms have safety bars and nonskid floors, handrail are present on all stairs and the stairs are well-lit with non-skid treads or carpet

Aztreonam

- A monobactam (same MOA as beta-lacatams) - The monobactam structure makes cross-allergenicity with beta-lactams unlikely - Cover pseudomonas; *no gram+ activity* - Can be used in PCN-allergic patients

What is cyclothymic disorder?

- A mood disorder that cause emotional highs and lows

What makes you have a false negatives in diagnosis of H.pylori? Urea breath test

- A resent use of H2RAs, PPIs , bismuth or antibiotics > D/C H2RAs, PPIs 1 to 2 weeks and bismuth and antibiotics 4 weeks prior to test

Where to find drug informations about pediatrics?

- AHFS drug information - CDC - Harriet lane of pediatrics - Neofax - Lexicomp's pediatric and Neonatal dosage handbook - ASHP: Pediatric injectable drugs

NSAID

- ALL NSAIDS: known risk factors for GI bleeding: Elderly, previous bleed, chronic or high dose use, hypoxic gut. - BBW > GI bleeding > CVD risk (highest risk with COX2) > Preop GABG Contraindication > Avoid with NSAID hypersensitivity > Nasal polyps > Asthma > Avoid Aspirin (not other NSAIDs) in children (<16 y/o) with any viral infection due to potential risk Reye's syndroem

What important information about Sevelamer? Sevelamer carbonate (Renvela) Sevelamer hydrochloride (Renagel)

* A non-calcium and non-aluminum based phosphate binder that is not systemically absorbed. Also has the benefit of lowering total cholesterol and LDL by 15-30%. Sevelamer carbonate may have the advantage over sevelamer hydrochloride of maintaining bicarbonate concentration. * 1. CI in bowel obstruction 2. SE: N/V diarrhea (>20%), constipation, abdominal pain 3. Monitoring: Ca2 , phosphorus , bicarbonate, CL-, PTH

* Drug induced Lupus Erythematosus (DILE) can be caused by many drugs most likely with Hydralazine, methyldopa, minocycline, etc. *

* Drug induced Lupus Erythematosus (DILE) can be caused by many drugs most likely with Hydralazine, methyldopa, minocycline, etc. *

Which infants/children need vitamin D supplement?

* Exclusively breastfed infants or babies drinking less than 1 Liter of baby formula need 400 IU of vitamin D daily (can use Poly-Vi-Sol or generic) *

Importanta bout patients who experience chest pain while on PDE-5 Inhibitors

* If a patient has taken a PDE-5 inhibitors and then develops angina, Nitrolgycerin should not be used until after 24 hours has elapsed for sildenafil and 48 hours has elapsed for Tadafil. (sometimes nitrates are used in an acute emergency, despite this warning with careful monitoring *

Important about Pneumococcal Vaccine

* Immunocompromised adults 19-64 * * Immunocompetent adult > 19 w/ CSF leak or cochlear implant* > Give prenvar 13 first followed by Pneumovax 23 at least 8 weeks later. > If Pneumovax 23 was given first then wait at least 1 year before give Prevnar 13 > Repeat Pnemova 5 years after first PCV23 * Immunocopetent adult 19-64 year with chronic illness, habits or living conditions that put them at risk of pneumococcal disease (smoking, asthma, long-term care resident, alcholism, liver disease, pulmonary disease, diabetes, heart disease * > Pneumovax 23 x 1 dose * Adults 65 or over (ALL) * > Prevnar 13 (if not given previously) followed by PCV23 6 to 12 months later. If pneumovax 23 was given first, wait at least 1 year before giving the 13.

What is the MOA of Lubiprostone?

- Activate chloride channels in the gut, leading to increased fluid in the gut and peristalsis.

List the components present in Excedrin migraine

- Aspirin - Tylenol - Caffeine

Warfarin Interactions

- Avoid with Tamoxifen - Rifampin (Cause large decrease in INR) - Licorice and St. John's Wort (May Decrease INR) - 2C9 inhibitors such as Fluconazole, TMP/SMX , macrolide antibiotics , Metronidazole etc.. Can increase INR MAT > Metronidazole, Macrolid > Amiodarone, Azole > TMP/SMX and Tamoxifen * When starting amiodarone, decrease the dose of warfarin by 30-50% *

What is the treatment option for UC and CD [Moderate-Severe]

- Azathioprine, mercaptopurine, or methotrexate are used. - Steroids are also used in moderate-severe cases - Anti-TNF agents (infliximab) are used in patietn with IBD that is *refractory to steroids and immunosuppressants *

Nefazodone

- BBW - Antidepressant increase risk of suicidal thinking and behavior in children, adolescents and young adults (18-24 years of age) with major depression disorder and other psyo disorder - NOTE > Less sedation than Trazodone > *HEPATOTOXICITY*

Tricyclics Safety/Side effects/monitoring

- BBW > Antidepressant increase risk of suicidal thinking and behavior in children, adolescents and young adults (18-24 years of age) with major depression disorder and other psyo disorder CI - Concurrent use iwth MAO is, linezoid, methylene blue, etc. SIDE Effects QT prolongation Orthostasis, tachycardia Anticholinergic > Dry mouth, blurred vision, urinary retention, constipation etc. > weight gain

Diclofenac (Cataflam, Voltaren_XR, Arthrotec)

- BBW > Arthrotec: Not to be used in women of childbearing potential unless woman is capable of complying with effective contraceptive measures NOTES > Misoprostol is used to replace the gut-protective prostaglandin to reduce the risk of GI damage from NSAID.

Fentanyl

- BBW > Hypoventilation , Respiratory depression and/or death - Use with strong or moderate CYP 3A4 inhibitor may result in increased effects and potentially fatal respiratory depression Notes - SL forms are REMS - Some patches needs to be removed prior to MRI - After applying patch to hairless skin; press in place for 30 seconds - Dispose patch in toliet or cut it up and put in coffe grounds

Dofetilide (Tikosyn) capsules

- BBW > Must be initiated (or reinitiated) in a setting with continuous ECG monitoring for a minimum of 3 days or 12 hours after cardioversion, whichever is greater - CI > patients with congenital or acquired long QT syndromes, concurrent use of dolutegravir, HCT, itraconazole etc. SE > HA, Dizziness, ventricular tachycardias , increase QT interval Monitor ECG, BP, HR, SCR, electorlyte (K an Mg) > d/c if QT > 500 msec. Notes Medguide required REMS program

Tysabri

- BBW > PML > monitor mental status changes. Risk factors for PML include: anti-JC virus antiboties, increase treatment duraiton and prior timmunosuppressant use. SE > Infusion reactions, headache, fatigue NOTE - Only approved for moderate-severe Crohn's patients who have failed anti-TNF therapy - Discontinue if no response by week 12 - REMS: must be enrolled in Manufacture TOUCH prescribing program - Medication Guide required

* Daptomycin (Cubicin) *

- Binds to cell membrane component causing rapid depolarization, inhibiting all intracellular replication processes including protein synthesis. - Concentration-dependent killing - Bactericidal activity - Cover MRSA VRE -> Approved for complicated skin and soft-tissue infections - Increase CPK and myopathy - CPK must be monitored daily and more frequently if on statin - Do not used to treat pneumonia as a drug is inactivated by surfactant - Compatible with NS and LR only - Can cause false elevation in PT/INR (but no increase in bleeding risk)

What is buspar used for?

- Binds to serotonin type 1A receptors (agonist). Serotonergic activity contributes to anxiolytic actons and anti-depressant actions. - Buspar is a second line option for GAD (only FDA approved indication). It is an alternative for patients who do not respond to antidepressants, are at risk for BZD abuse or added as adjunctive therapy. It has delayed onset of 2-4 weeks, shorter than SSRIs but longer than BZDs.

* Macrolides *

- Binds to the 50S ribosomal subunit resulting in inhibition of RNA dependent protein synthesis with bacteriostatic activity related to total exposure of the drug - CI > history of cholestatic jaundice/hepatic dysfunction with prior use > Clarithromycin: concurrent use with colchicine in patients with renal or hepatic impairment, history of QT prolongation or ventricular cardiac arrhythmia - WARNING > QT prolongation > Hepatotoxicity SIDE EFFECTS > GI upset

Griseofluvin (Grifulvin V, Gris-PEG)

- Binds to the keratin precursor cells which prevents fungal invasion - indicated for dermatomycosis and Tinea infections of skin, hair and nails - SE > Photosensitivity Notes - Pregnancy Category X - Cross reaction w/ PCN allergy - Griseofluvin may increase the metabolism of contraceptives which may lead to contraceptive failure.

What are the SE associated with warfarin?

- Bleeding - Skin necrosis - Purple toe syndrome

What cause a decrease in Hemoglobin Hgb, Hb, or Hct Hematocrit?

- Bleeding - Risk with: Anticoagulants, antiplatelets, P2Y12 inhibitors, Fibrinolyctics, or due to aplastic anemia from chloramphenicol

Class IV antiarrhthmic agents

- Block L-type calcium channels, slowing SA and AV nodal conduction velocity - Used to slow the Ventricular rate (rate control) - Nondihydropyridine calcium channel blockers should not be used in patients with LV systolic dysfunction and decompensated HF due to their negative inotropic effects, but they may be used in patient with HF with preserved LV systolic function

What is the MOA of Methylnatlrexone (Relistor) and Alvimopan (Entereg)?

- Block opioid receptors in the GUT to reduce the constipation effect of opioid - Relistor > SC QOD > Notes: Stay close to toilet after injecting, Decrease dose if CrCl < 30 mL/min, * Only for patients on opioid who have failed DSS + laxative (senna, bisacodyl). DO not use routinly; can often increase laxative until the patient can eavate. - Entereg > PO > Must be hospitalized to receive drug > BBW: MI risk, and this is why it has REMS > Contraindication: Patients who have taken therapeutic doses of opioids for more than 7 consecutive days prior to use > Use is limited to post-surgical patients to decrease the risk of post-operative ileus - Naloxegol (Movantik) > C-II > Peripherally acting mu-opioid receptor antagonist (PAMORA) for the treatment of opioid-induced constipation (OIC) in adults patient with chronic, non-cancer pain >CYP3A4

Memantine

- Blocks NMDA (N-methyl-D-aspartate), which inhibits glutamate from binding to NMDA receptors and decrease abnormal activation

What is myoclonic seizure?

- Brief, lightning-like jerk movements of entire body

What are the side effects of Folic Acid?

- Bronchospasm, flushing, rash, pruritus, malaise

List some drugs that likely to contribute to Insomnia?

- Bupropion - Stimulants - OTC appetite suppressants - Decongestants - MAO-B inhibitors, if taken later in the day - Fluoextine, if taken late in the day - Caffeine - Steroids - Alcohol (initially induces sleep, but prevent deeper stages of sleep and cause nocturia) - any drug that causes urinary retention, or nocturia including antihistamine and diuretics taken later in the day

Sodium oxybate (Xyrem)

- C-III (Narcolepsy) - C-I (abuse) > REMS program: This is a 'date rape' drug (Sedative, called GHB that requires strict measures to ensure it is going to narcolepsy with cataplexy patients only BBW - Respiratory depression - Sodium oxybate is a salt form of hydroxybutyrate (GHB), a drug of abuse SE - dizziness, nausea, somnolence, enuresis (dose-related), daytime hangover effect

When do you treat patient for PCP - Pneumocystis Pneumonia who is HIV+?

- CD4+ count < 200 cells/mm3 or - Oropharyngeal candidiasis

When do you start primary prophylaxis of MAC - Mycobacterium Avium Complex?

- CD4+ count < 50 cells/mm3 after ruling out active disseminated MAC disease

Must know about Bismuth Subsalicylate (Pepto-Bismol)

- CI > Children with viral infections (varicella, influenza) due to risk of Reye's syndrome, patients with a salicylate allergy, history of severe GI bleed or coagulopathy - Side Effects > Black tongue/stool , hearing loss/tinnitus (toxicity)

Benzodiazepines - Safety / side effect / Monitoring

- CI > Myasthenia gravis , severe respiratory insufficiency, severe hepatic insufficiency, sleep apnea syndrome, acute narrow-angel glaucoma, not for use in infants < 6 months of age (oral) - Warning - Anterograde amnesia (after drug is taken some events may not be stored as memories) - CNS depression - Extravasation with IV administration - Potential for abuse - risks in elderly - development of tolerance, withdrawal following abrupt discontinuation or large decrease in dose; taper off slowly - potential for abuse - physiological dependence and tolerance develop with chronic use

Hydroxyurea Side effects

- CI Significant myelosuppression: WBC < 2500 or platelet count < 100,000 or severe anemia - SE Leukopenia, anemia, thrombocytopenia , anorexia, nausea, diarrhea, constipation, hyperpigmentation, scaling, headache, dizziness, etc. Note - Wear gloves when handling and wash hands before and after contact

Zonisamide (Zonegran)

- CI: Hypersensitivity to sulfonamide SE - Oligohydrosis and nephrolithiasis

What are the most side effects associated with AEDs?

- CNS-related > somnolence > fatigue > cognitive impairment > coordination abnormalities such as ataxia and dizziness.

NSAID

- COX1 and COX2 enzymes catalyze the conversion of prostaglandins (PG) and thromboxane A2 from Arachidonic acid AA. The non-selective NSAIDs blocks the synthesis of both COX enzymes. The COX-2 selective agent block the synthesis of COX2 only and this will decrease GI risk because COX1 protects the gut layer. * Both groups will decrease inflammation of the PGs that are involved in decreased inflammation , decreased pain and fever. Blocking COX-1 decrease the synthesis of PG-H2 which decrease the formation of TXA2. TXA2 is required for both platelet activation and aggregation and blocking TXA2 decrease clotting and provide the cardiovascular benefit.

Notes on Calcium Selection and Absorption

- Calcium absorption is saturable; doses should be divided (maximum 500-600mg of elemental calcium per dose) - Calcium requires vitamin D for absorption - Calcium citrate (Citracal, others) has better absorption and can be taken with or without food; usual tab has 315mg of elemental calcium (21% elemental calcium). It may be preferable with little or no stomach acid - such as what occurs with elderly patients and/or with the use of PPIs, which have been shown to increase fracture risk due to impaired calcium carbonate absorption (including the dietary calcium) - Calcium carbonate (Oscal, Tums, others) has acid-dependent absorption and should be taken with meals; usual tab is 500-600 mg of elemental calcium (40% elemental calcium)

Terbinafine (Lamisil, Terbinex)

- Can exacerbate sysemic lupus erythematousus SE > HA , incresae LFT > Strong 2D6 inhibitor and weak/moderate 3A4 inducer

What is the MOA of Pyrimidine analgos antimetabolis and list the drugs

- Cell cycle specific, S-phase: Inhibit pyrimidine synthesis - Capecitabine (Xeloda) - Cytarabine > Conventional (AKA ara-C) > Liposomal (DepoCyt) - Fluorouracil , 5-FU - Gemcitabine (Gemzar)

Important information about Varicella-containing Vaccines?

- Children get varicella vaccine at 12 months and again at 4-6 years - All adults without evidence of immunity to varicella should receive 2 doses of varicella vaccine at least 4 weeks apart - Herpez zoster vaccination (potency 14 times greater than varicella in order to elicit needed immune response - Store vaccine in freezer $ protect from light (keep in original container) - Store diluent in refrigerator or room temperature - LIVE vaccine - Do not give if hypersensitivity to gelatin or neomycin - Give SC. Reconstituted: reconstitute immediately upon removal from freezer and inject: short stability

List the drugs used for acute attack of uric acid?

- Colchicine - NSAIDs - Steroids

Protease Inhibitor Drug Interactions

- All PIs are metabolized in the liver by CYP 450 system and have many drug interactions. - All PIs are 3A4 substrates and most are strong inhibitor of 3A4 - Ritonavir is a potent inhibitor of 3A4 used at low doses to increase, or boost, the level of other PIs. - PIs can alter the INR (Mainly decrease ) in patient taking warfarin due to 2C9 ; the INR should be monitored closely - Atazanavir cannot be used with PPIs - PDE-5 inhibitors: PIs can increase levels of PDE-5 inhibitors and increase risk of toxicity. - Hormonal contraceptives (especially those containing ethinyl estradiol and norethindrone) - Methadone: Levels may be decreased by ritonavir (Via CYP induction). Ritonavir could theoretically induce methadone metabolism, particularly via CYP2B6.4 Ritonavir is also expected to strongly inhibit CYP3A4 mediated methadone metabolism

Calcium and Vitamin D - Patients on medications for low bone density

- All Rx medications for low bone density require adequate calcium and vitamin D taken concurrently - Dietary intake of calcium should be assessed first and is preferred with supplements used if insufficient. - Important in children, pregnancy, during the years around menopause - Vitamin D is required for calcium absorption in the GI - * Vitamin D deficiency in children cause rickets * and in adults cause osteomalacia. - Ensure adequate intake of calcium and vitamin D

Do biologics require Medication Guide?

- All TNF inhibitors carry a boxed warning for risk of serious infections, including tuberculosis, invasive fungal and other OI. All patients should be evaluated for TB before starting these drugs. Patients with latent TB should start prophylactic treatment. Retest for TB annually.

What warning does all antibacterial agents carry?

- All antibacterial agents carry a warning of the risk of superinfection with prolonged use including C. dificile-associated diarrhea (CDAD) and pseudomembranous colitis. This is due to the killing of normal healthy flora in the GI tract resulting over-growth of C.difficile.

Azoles antifungals drug interactions

- All azoles are 3A4 inhibitors - Itraconazole and ketoconazole have PH-dependent absorption: Increase in PH will cause decrease in absorption; avoid using with Antacid , H2RAs, PPIs - Voriconazole is metabolized by several cyp 450 enzymes (2C19, 2C9, and 3A4); the concentration of voriconazole can increase dangerously when given with drugs that inhibit voriconazole's metabolism or with small dose increases - it is first order followed by zero order kinetics - Avoid concurrent use of voriconazole and these drugs > Barbiturates (long acting) > Carbamazepine > Efavirenz (>400mg/day) > Ergot alkaloids > Pimozide > Quinidine > Rifabutin, Rifampin, Ritonavir (>800mg/day) > Sirolimus and St. John's wort

Notes about Azoles antifungals?

- All azoles are cleared hepatically except fluconazole - Only fluconazole and voriconazole penetrate the CNS adequately to treat fungal meningitis and are often associated with CNS toxicities (HA/dizziness/halucination) - Fluconazole IV should not be refrigerated. Fluconazole IV to PO ratio is 1:1

Important about NRTI?

- All require renal adjustment except Abacavir (Ziagen) - No CYPE interactions - No food requirement exception if you have them in combo

Which benzodiazepines comes as ODT?

- Alprazolam - Clonazepam

What are the side effects associated with Bile-Acid Resins?

- Constipation - Dyspepsia - Nausea - Abdominal pain, cramping, gas, bloating, hypertriglycerdemia , esophageal obstruction, increase in LFT

What effect fibrate has on LDL HDL and TG?

- Decrease TG - Increase HDL - Decrease LDL (but can increase LDL when TG are high)

What drugs used to treat Extravasation?

- Dimethyl sulfoxide (DMSO) or dexrazoxane (Totect) for the anthracyclines. - Hyaluronidase for the vinca alkaloids - Sodium thiosulfate for mechlorethamine

What is Aggrenox and what is the most common SE from this medication?

- Dipyridamole / Aspirin - Headache > 10%

Plavix use in ACS

- Do not start in patients likely to undergo GABG surgery and d/c 5 days prior to any major surgery - If patient received fibrinolytic therapy for STEMI and is > 75 years of age, omit the loading dose and start 75 mg daily

Non-Drug treatment for Celiac disease

- Drugs themselves are gluten-free, it is the excipients that may be a problem - The first place to look for excipient content is the package insert which may or may not contain the excipient components. - The key word to look for is "starch" which will be either corn, potato, topioca or wheat. If the package insert lists "starch" alone then the mfg must be consulted to find out if the starch is wheat.

Multivitamins needs in CF

- Due to poor fat absorption, patients use 1-2 multivitamins daily with ADEK and some require additional doses.

Important about Emergency Contraception (EC)

- Emergency Contraception (the "morning after pill") is a form of contraception that prevents pregnancy up to 72 hours (3 days) for levonorgestrel EC and up to 120 hours (5 days) after sexual intercourse for ELLA. Another option to prevent pregnancy is the copper IUD insertion (paraGard)

Adverse effects due to Estrogen

- Estrogen can cause nausea , breast tenderness/fullness, bloating, weight gain or elevated blood pressure. - If mid-cycle breakthrough bleeding can occur and may require a higher estrogen dose. - 3 things to keep in mind > Higher estrogen dose means higher clotting risk > FDA safety annnouncement: Dropirenone-containing birth control pills may be associated with a higher risk of blood clots than other progestin-containing pills. > Ortho Eva patch: higher systemic estrogen exposure than most coC pills.

List the TNFa inhibitors biologics?

- Etanercept (Enbrel, Enbrel SureClick) > 50 SC weekly or 25 SC Twice/weekly - Adalimumab (Humira, Humia Pen) > 40 mg SC every other week >> If not taking with methotrexate can be taken 40mg SC weekly - Infliximab (Remicade) > Given only in combination with methotrexate in RA > IV > Requir filter > Stable inNS only > Infusion reactions > Delyaed hypersensitivity reaction - Certolizumab pegol (Cimzia, Cimzia Prefilled) - Golimumab (Simponi, SImponi) > Given in combo with Methotrexate in RA > Require filter if given IV

If patient has Sulfa allegy; which loop diuretic can be giveN?

- Ethacrynic acid > Does not cross interaction

Factor Xa inhibitors

- Fondaparinux (Arixtra) > A synthetic pentasaccharid that selectively inhibit factor Xa via antithrombin (AT) > Indirect inhibitor of Factor Xa -Treatment for VTE - <50 kg: 5mg SC daily - 50-100kg: 7.5 mg SC daily - >100 kg: 10mg SC daily BBW - Patient recieving neuraxial anesthesia (epidural, spinal), or undergoing spinal puncture are at risk of hematomas and subsequent paralysis CI - Severe renal impairment (CrCl < 30ml/min) - etc - body weight < 50kg (for prophylaxis only) SE > Bleeding > *hypokalemia * _ only one Monitoring -Anti-Xa levels - etc Notes Do not administer IM Store at room temp Do not expel air Preg B

What you know about HLB (hydrophilic-Lipophilic balance) number?

#s range from 0-20 with 10 being the mid-point means it is the break point between water and oil solubility. #>10 means hydrophilic (water-soluble) and #<10 means lipophilic (lipid-soluble)

Labels on Pharmacy-compounded products - Expiration Dates (Beyond Use Dates)

* Water containing oral formulations * > The beyond use date is no later than 14 days when stored at controlled cold temperature * Water containing topical/dermal/mucosal liquid and semisolid formulations * > The beyond use date is no later than 30 days * Nonaqueous formulations* > The beyond use date is no later than the time remaining until the earliest expiration date of any API (use no later than 25% of time of this api) or 6 months, whichever is earlier

Fidaxomicin (Dificid)

- $$$$ - Inhibit RNA polymerase resulting in inhibition of protein synthesis and cell death - Bactericidal - Only use if resistance and/or failure to vancomycin or metronidazole - Not effective for systemic infections - absorption is minimal

Dronedarone (Multaq)

- 400mg po BID with meals - BBW > HF (NYHA class IV or any class with recent hosppitalization) and in patients with permanent AFib - Side effects > QT prolongaton, increase in Serum Cr - Notes - Pregnancy category X

ANTIARRHTHMICS

- Heart rate describe the frequency of depolarization of the ventricles - Normally the resting heart rate in adult is 60 to 100 beats per minute - An arrhthmia can be silent (asymptomatic) which may be only detected during a routine physical exam

Major bleeding from warfarin

- Hold warfarin therapy. Give vitamin K 5-10mg slow IV injection and four-factor prothrombin complex concentrate (PCC)

When do you use Clozapine?

- It has superior to others but comes with multiple BBW - Known for arganulocytosis, seizures and myocarditits - * A clozapine trial should be considered for a patient who has had no or poor response to two trials of antipsychotic medications (at least one should be SGA) or for a patient with significant ADRs)

Bromocriptine

- It is dopamine agonist but it improves glycemic control by working in the CNS to decrease insulin resistance - Bromocritpine (Cycloset) - Parlodel (higher dose) is indicated for hyperprolactinemia, Acromegaly, Parkinson Disease - May reduce ergot effectiveness of migraines if co-administered within 6 hours of ergot-related drug.

What is the pathophysiology of Parkinson Disease?

- It occurs when neurons in a part of the brain called substantia nigra die or become impaired. - The cause of neuronal death is not well understood, but it multi-factorial. Normally, these cells produce dopamine. Dopamine allows smooth, coordinated function of the body's muscles and movement. - When 80% of dopamine producing cells are damaged, the motor symptoms of the disease appear.

Bupropion Counseling

- It takes 1 week for the medication to start working. For your best chance of quitting, you should not stop smoking until you have been taking this medicine for 1 week. Set a date to stop smoking during the second week of starting this medication - Do not take if have a seizure disorder, are taking other forms of bupropion, or have taken an MAO inhibitor within the last 14 days or had an eating disorder - Do not exceed 450mg daily - If you, your family or caregiver notice agitation, hostility, depression, or changes in behavior or thinking, stop taking the medication and call your doctor.

What is the DOC for Trichomoniasis caused by Trichomonas Vaginalis a flagellated protozoan?

- Metronidazole 2g PO x 1 or Metronidazole 500mg po bid for 7 days or - Tinidazole 2g PO x 1

Where to find information about Toxicology?

- Micromedex: POISINDEX - Lexicomp's: Poisoning and toxicology: Lexi-Tox - NLM: TOXNET at toxnet.nlm.nih.gov

Nicotine Patch Counseling

- Patches should be applied as soon as the patient wakes on the quit day. With patients who experience sleep disruption, have the patient remove the 24 hour patch prior to bedtime or use the 16 hours patch

What are the contraindications associated with Non-Dihydropyridine CCBs?

- Severe hypotension (SBP < 90mmHG), 2nd or 3rd degree heart block, sick sinus syndrome (unless the patient has a functioning artificial ventricular pacemaker), cardiogenic shock, pulmonary congestion and HF

Importanta bout Aute (STAT) Treatments and Bowel Preps.

- Sodium phosphates (Fleet enema, OsmoPrep) > Warning: Do not use sodium phosphate in CHF, renal disease, it decrease Ca+2 or Increase PO4 > Onset of action 1 to 5 minutes.

Valproate/Valproic acid (Depakene, Stavzor, Depacon)

- Supplementation with Ca and Vitamin D recommended. - Therapeutic range > 50-125 mcg/mL - BBW (4) 1. Hepatic failure 2. Teratogenicity 3. Pancreatitis 4. Mitochondiral disesae increase risk of acute liver failure CI - Hepatic disease, urea cycle disorders, propylaxis of migraine in pregnancy, unknown mitochondrial disorders SE - Nausea, alopecial (treat with multivatmins containing selenium and zinc), weigh gain, tremor - Dose related SE > Thrombocytopenia, dipolopia, blurred vision

Which agents cause the most alopecia?

- Taxens > Do you have to pay tax in Turkey? - Anthracyclines Others (less important to know) > Carboplatin > Cyclophosphamide > Etoposide > Ifosfamide > Vinca Alkaloids

What is the difference between COPD and Asthma?

- The limitation of airflow is not fully reversible and generally worsens over time.

Is there benefit of using Bupropion + nicotine instead of 2 nicotine products?

- There is less evidence of benefit with this combination versus using two nicotine agents.

Stimulant BBW

- drug dependency > use caution if a history of ethanol or drug abuse > Avoid abrupt discontinuation in patient who have been taking drug for prolonged period - Adderall > Misuse can cause sudden death and serious cardiovascular adverse events

PEMEtrexed (Alimta) cause dermatologic toxicity, how can we prevent that?

- folid acid supplements (1mg po daily_ - Vitamin B12 (cyanocobalamin) - Dexamethasone

What are the typical symptoms of GERD?

- heart burn - Hyper-salivation - regurgitation - acid taste in the mouth

Ticagrelor (Brilinta)

- indicated for reduction of thrombotic events in patients with ACS - BBW 1. Can cause significant sometimes fatal bleeding (same with prasurgel) 2. Maintenance dose of aspirin above 100mg reduce the effectiveness of ticagrelo and should be avoided. After any initial dose, maintenance aspirin should not exceed 100mg daily SE Bleeding, dyspena (>10%) Increase in Scr and bradyarrythmias NOTE Do not start in patients like to undergo CABG surgery and d/c 5 days prior to any major surgery * 2015 drug Cangrelor (Kengreal) IV only. Just know the transition to ORAL P2Y12 inhibitors: Ticagrelor 180mg given during or immediately after stopping cangrelor, prasugrel 60mg and clopidogrel 600mg imeediately after stopping cangrelor (not prior) *

* Treatment for lung problems for CF *

- inhaled bronchodilators to open airways - Hypertonic saline (HyperSal) for hydrating the airway mucus secretions and facilitating mucociliary clearance - DNAse enzyme to breakdown extraceullular DNA from accumulated neutrophils. DNAse therapy is designed to thin mucus and facilitate mucociliary clearance. - Inhaled antibiotics for prevention and treatment of lung and sinus infections - PO azithromycin to reduce inflmamation and disrupt P.aeruginosa biofilm formation - Transplantation, in patients with end stage lung disease

Important notes about Centrally-acting Alpha-2 Adrenergi Agonist

- patch applied weekly - Remove patch prior to MRI - Rebound hypertension, if stopped abruptly (must taper)

AzaTHIOprine

-BBW > Chronic immunosuppression can increase risk of neoplasia > Hematologic toxicities (leukopenai, thrombocytopenia) and mutagenic potential Warning > GI (Severe N/V/D) > * Patients with genetic deficiency of thiopurine methyltransferase (TMPT) are at increased risk of myelosuppression and may require lower dose * Side effects > Severe N/V/D, rash, increase in LFT, hematologic toxicities (leukopenia, thrombocytopenia)

* Must know about Hydrocodone and Tramadol *

-Both of these opioids are metabolized by 2D6 -Patient w/o this enzyme or those on 2D6 inhibitors (fluoxetine, paroxetine, others) would be at increased risk of respiratory depression and at the least have increased side effects with hydrocodone - Tramadol requires conversion by 2D6 to the main active (analgesic) metabolite * CYP2D6 Inhibitors (Strong) may decrease serum concentrations of the active metabolite(s) of Hydrocodone. Specifically, concentrations of hydromorphone may be decreased. Severity Moderate Reliability Rating Fair *

Drug Treatment

-Intranasal corticosteroids (Flonase) are first-line for chronic, moderate to severe rhinitis. Milder, intermittent symptoms can be treated with oral antihistamines. - Decongestants are used for congestion (if present)

What is the treatment for PAH if Acute Vasoreactivity testing is - (for low risk and high risk)

-Low risk and high risk the same > Eras or PDE-5 I or sGC stimulator (oral) > Epoprostenol or Treprostinil (IV) > Treprostinol or LLOprost (inhaled) reprostinil (SC)

How to limit amount of ammonia from protein?

-Patient should get a daily protein intake of 1-1.5 g/kg - Veggies and dairy sources of protein is preferred to animal sources due to the lower calories to nitrogen ratio - Branched chain amino acids (BCAA) (leucine, isoleucine, valine) are favored over aromatic amino acids (AAA); they interfere with AAA ability to cross the blood brain barrier and increase hepatocyte growth factor synthesis.

What must the MFG apply for if they want to change one of the following: A. Labeling changes B. New dose C. New strength D. New manufacturing processes

sNDA Supplemental New Drug Application

What does Participating supplier mean?

" Accepting Assignment" means that the pharmacy agrees to collect only the Medicare portion and will not for ask for or accept any additional payment from the patient. If the pharmacy agrees to accept assignment, they are called a *participating supplier* "

Where is the only place Daytrana placed?

*HIP*

What is the difference between Lunesta 1mg 2mg 3mg?

- 1mg > If difficulty falling asleep - 2mg > If difficulty staying asleep - 3mg > for longer duration

What is the complication of OVER-DOSING levothyroxine in elderly patients ?

- A-fib - Fractures

List 2 monoclonal antibodies to TNF approved (FDA) for UC and CD

- Adalimumab (Humira) - InFLIXimab (Remicade)

Which drug cause Hepatotoxicity?

- Anti-androgens - Folate and pyrimidine analog antimetabolite > Methotrexate - Aramatase inhibitors - Busulfan - Ixabepilone - SERM - Taxanes and some tyrosine kinase inhibitor and the vinca alkaloid

Which medications are the LEAST to cause myelosuppression?

- Asparaginase (MSC) - Bleomycin (MSC) - Vincristine (Vinca) - Monoclonal antibodies (mAbs)

Why Aricept (mainstay of therapy ) administered at QHS?

- Because one of the side effects is NAUSEA

What will happen if oversodes on stimulants?

- Benzodiazepines

List the Interferon beta-1b used for treatment for patient with relapsing forms of ms?

- Betaseron - Extavia

Amantadine (Symmetrel)

- Blocks dopamine reuptake into presynaptic neurons, increases dopamine release from presynaptic fibers; used for mild disease, or for dyskinesias in advanced disease - SE - dizziness , etc. - Toxic delirium - Cutaenous reaction called Livedo reticularis - require drug discontinuation

Ziprasidone (GEODON)

- CI > QT prolongation, contraindicated with QT risk SE - Sedating, respiratoyr tract infeciton ,etc. Noe Take with food * No significant metabolic effects *

To convert from one opioid to another * follow these steps *

- Calculate total 24 hr dose requirement of the current drug - Use ratio-conversion to calculate the dose of the new drug - Calculate 24 hour dose of new drug and reduce at least 25% (unless exam does not specify to reduce dose) - Divide to attain appropriate interval and dose for new drug - Always have breakthrough pain (BTP) medication available while making changes. Guideline recommendation for BTP dosing range from 5-17% of the total daily baseline opioid dose.

List 1 drug that is Non-selective alpha and beta blockers?

- Carvediolol (Coreg, Coreg CR)

Can you take Atazanavir with acid suppression agents?

- Caution required because acid-suppressive agents as they can reduce the absorption (and blood levels) of atazanavir.

Important interactions with Atorvastatin

- Do not exceed 20mg/day with strong inhibitors - Do not exceed 40mg/day with nelfinavir and boceprevir

List the oral disease-modifying agents approved for MS ?

- Fingolimod (Gilenya) - Terflunomide (Aubagio) - Dimethyl Fumarate (Tecfidera)

What is the drug of choice for Cytomegalovirus (CMV)?

- Ganciclovir and Valganciclovr are the drugs of choice for CMV

What is the drug of choice OTC for constipation for children?

- Glycerin suppositories - Miralax (Polyethylene glycol)

What is Bipolar II?

- Has same symptoms of bipolar I disorder, however the symptoms are described as "hypomania" because they are less severe than in pure mania.

What are the required immunization for Diabetes?

- Hepatitis B - Influenza - PPSV23 - Tdap

Must know about Tuberculosis (TB)

- Highly contagious - Latent disease is diagnosed by tuberculin skin test (TST), also called a purified protein derivative (PPD) test. The solution is injected intradermally and the area is inspected for induration 48-72 hours after injection

Important about antimalarial agents used for SLE

- Hydroxychloroquine (Plaquenil) > *Takes 6 months to see maximal effect* > Effecetive for cutaneous symptoms and arthralgias, fatigue and fever - used for mild disease; chronic (not acute) therapy

What cause increase in Basophils?

- Inflammation - Hypersensitivity reaction - Leukemia

What is the MOA of Dipyrimadole?

- Inhibit the uptake of adenosine into platelets and increase cAMP leels, which indirectly inhibits platelet aggregation.

What is the rule of Propronalol in anxiety?

- It is used to reduce symptoms of stage fright or performance anxity. - It is dosed at 10-40 mg 1 hour prior to an event such as a public speech.

Lamotrigine and Estrogen

- Lam concentration is decreased by Estrogen

Live vaccines used in Immune-compromised patients

- Live vaccines: If needed these must be given prior to the start of immunosuppressive drugs. - Yellow Fever vaccine is live, and may be requested for travel, but cannot be given to anyone with severe immune suppresion; advise these patients to avoid traveling to endemic regions.

Important about Vivelle-Dot Patch Application

- Mark the two-day schedule you plan to follow on your caton's inner flap - Apply patch to lower abdomen below the waistline. Avoid the waistline, since clothing may cause the patch to rub off - The area must be clean, dry, free or powder , oil or lotion. This applies to all patches. And never apply a patch to cut or irritated skin (unless it is bandage) - Do not apply patch to breasts (never apply any estrogen patch to the breasts)

Important counseling point for Singulair 4mg oral granules

- Never store any oral granules mixed with food, baby formula, or breast milk for use at later time. Throw away any unused portion. Do not mix Singulair oral granules with any liquid drink other than baby formula or breast milk

What is the used to prevent the vasospasm that occur 4-21 days after the bleed?

- Nimodipine is used to prevent the vasospasm associated with delayed ischemia. - can use prophylactis anticonvuslant but do not use long term unless patient has high risk for seizures

Important facts about Dengue

- No vaccine - Sequential infections put people at greater risk for dengue hemorrhagic fever and dengue shock syndrome both of which can be fatal - Protection from mosquito bites with non-drug measures is essential

Nasal Irrigation and Wetting Agents

- Normal saline - Neti pot > Instruct to use boiled, bottled or distilled water (not tap)

Nasal Decongestants

- Oxymetazoline (Afrin, Neo-Synephrine nighttime 12-hour) - Naphazoline (Privine) - Phenylephrine (Neo-Synephrine 4 hours) - Tetrahydrozoline (Tyzine) > Rx CI - Hypertension, V-tachycardia Notes - Limit use to <= 3 days to prevent rebound congesion

Fibrinolytics

- PCI is preferred if it can be performed within 90 minutes (optimal door to ballon time) or within 120 minutes of first medical contact. If PCI is not possible within 120 minutes of first medical contact, fibrinolytic therapy is recommended and should be given within 30 minutes of hospital arrival (door-to-needle time). - In the absence of contraindications and when PCI is not available, fibrinolytic therapy is reasonable in STEMI patients who are still very symptomatic within 12-24 hours of symptom onset.

Roflumilast (Daliresp)

- PDE-4 inhibitor that increase cAMP levels, leading to a reduction in lung inflammation CI - Moderate to severe liver impairment SE - Diarrhea, * weight loss* , nausea, decrease in appeptite, insomnia, depression and psychiatric event including suicidality * NOTES * - CYP 3A4 substrate (MAJOR)

Which TKIs u give if you have T3151 mutation?

- PONATinib (Iclusig) > Life threatening clots BBW > Thrombotic events, stroke, MI, heart failure, hepatotoxicity

Oral Decongestants

- Phenylephrine HCL > Sudafed PE - Pseudoephedrine > Sudafed CI - Do not use within 14 days of MAOs.

Which agents cause the most Neuropathy?

- Platinum agents - Proteosome inhibitors (MIB) > Bortezomib and Cafilzomib - Taxanes - Vinca alkaloids Remember > PPTV

List some important non-cardiac conditions that can trigger or predispose a patient to arrhythmias?

- Potassium, Magnesium, Sodium, and Calcium

CarBAMazepine (Equetro) - for bipolar

- Potent CYP450 inducer and autoinducer > will decrease level of many other drugs and of itself - Supplement with Ca and Vitamin D - Therapeutic range 4-12 mcg/mL BBW (2) 1. Serious skin reactions, including SJS/TEN >> If Asian descent, test for HLA-B*1502 allele prior ot use >> Blood cell abnormalitites, including risk of arganulocytosis SE - Dizziness, somnolence, headache, nausea, ataxia, rash

What is the drug of choice in pregnancy (ART)?

- Preferred two-NRTI backbone to be combined with either PI or NNRTI *> Zidovudine/maivudine* > Tenofovir/emtricitabine (orlamivudine) > Abacavir/lamivudine * Zidovudine/lamivudine is the combination with the most experience for use in pregnancy. Requires twice daily administration and has increased risk for causing anemia *

What is so special about Lam?

- Preg C - No significant drug interactions - Weight natural

Notes about phenytoin and fosphenytoin

- Preg D - Supplements with folic acid, calcium and vitamin D recommended - Strong CYP450 inducer - Phenytoin IV is compatible with NS ONLY, require a filter and is stable for 4 hours - Fosphenytoin is compatible with NS or D5W and is refrigerated (stable for 48 hrs at room temperature; 30 days if refrigerated) - IV:PO ratio is 1-1 - Enteral feedings decrease phenytoin absorption; hold feedings 1-2 hours prior and 1-2 hours after phenytoin administration

General notes about statins?

- Preg. X - Can take crestor, lipitor, Livalo, Lescol XL and pravachol at any time of the day

What are the two conditions that are absolute contraindication with live vaccine?

- Pregnancy - Immunosuppression

General Notes about Warfarin

- Pregnancy category X/D (women with mechanical heart valves) - Antidote: Vitamin K (PO or IV) - Take at the same time each day - Missed dose > Take the dose as soon as possible on the same day; do not double the dose the next day to make up for a missed dose. > Dental cleanings and single tooth extraction do not generally require a change in warfarin dosing if INR is in therapeutic range.

Why is it important to encourage enrollment in the pregnancy registries?

- Pregnancy exposure registries exist for select disease states (cancer, autoimmune condition, HIV, epilepsy) and for many individual drugs. - They are designed to collect health information from women who take various drugs when they are pregnant and breastfeeding - This information are compared with woman who have no taken medicine during pregnancy and the health of their babies

What is the dose for Potassium Chloride?

- Prevention of Hypokalemia > 20-40 mEq/day in 1-2 divided doses - Treatment of mild Hypokalemia > 40-100 mEq/day in 2-5 divided doses - * no more than 20-25 mEq should be given as a single dose to avoid GI discomfort * - * for infusions > 10 mEq K+/hour, central line and continuous ECG monitoring required *

Ritonavir RTV (Norvir)

- Primarily used as a booster agent and not as a sole PI - 100-400 mg/day- booster dose given in 1 or 2 divided doses. - Take with food - Capsules: Keep refrigerated; can be left at room temperature if used within 30 days - Tablets: store at room temperature - Solution: contains 43% alcohol; do not refrigerate - Keep in original container. BBW > Ritonavir may interact with many medications: including antiarrhythmiacs, ergot alkaloids, and sedative/hypnotics, resulting in potentially serious and life-threatning AE Side Effects > N/V/D, paresthesias, Asthenia, altered taste, PR prolongation, etc.

Which group qualify for High-Intensity statin?

- Primary elevation of LDL >= 190mg/dL - Diabetes and 40-75 years with LDL between 70-189 mg/dl with estimated 10-year ASCVD risk >= 7.5% - Clinical atherosclerotic cardiovascular disease (ASCVD) <= 75 years old

Important about Transplant/Immunosuppression

- Prior to any transplant, tissue typing or cross-matching is performed to assess donor-recipient compatibility for human leukocyte antigen (HLA) and ABO blood group - Panel Reactive Antibody (PRA) test that is taken to gauge the degree to which the recipient is sensitized to foreign (or non-self) proteins. A high score correlates with the likelihood of graft rejection and could necessitate some type of desensitization protocol prior to the transplant.

List one drug that deplets CD20 B cells and it is used in RA treatment given with methotrexate ? > Biologic (Non-TNF)

- RITUXimab (Rituxan) > Need pre-medicate with a steroid, acetaminophen, and an antihistamine BBW (4) 1. Severe and fatal infusion-related reactions, usually with the first infusion 2. Fatal progressive multifocal leukoencephalopahty (PML) due to JC virus infection 3. Hepatitis B virus (HBV) reactivation; some cases resulting in fulminant hepatitis, hepatic failure and death 4. Severe and fatal mucocutaneous reaction (SJS, TEN) WARNINGS > Can cause serious infections > Screen for latent TB and HBV prior to initiating therapy > DO not give with other biologics or live vaccines NOTE > do not shake

Important about Raloxifene

- Raloxifene is an estrogen agonist/antagonist, also called a Selective Estrogen Receptor Modulator (SERM). It acts to decrease bone resorption. Conjugated estrogens/bazedoxifene (Duavee) is an estrogen/SERM combination indicated for both osteoporosis prevention (in postmenopausal women with a uterus) and for vasomotor symptoms (Raloxifene cause vasomotor symptoms) - Used often in women at risk (or have a fear of ) breast CA - Favorable lipid affects (Decrease in CH and LDL; no effect on HDL) BBW2 1. Increase risk of thromboembolic events (DVT, PE, MI, stroke) 2. The risk of death due to stroke may be increased on women with coronary heart disease or in women at risk for coronary events SE > Hot flashes, peripheral edema, arthralgia,etc. Notes > Pregnancy category x > Separate raloxifene and levothyroxine by 12 hours. If on warfarin, monitor INR when tarting or stopping raloxifene.

What are the risk of items that contain albumin?

- Risk of Creutzfeldt-Jacob disease transmission (rare); avoid in albumin-sensitive patients.

Topoisomerasae I inhibitor

- S-phase specific: lock the coiling and uncoiling of DNA helix. Topoisomersae I faciliate single strand breaks followed by religation (putting the DNA strands back together) - Irinotecan (Camptosar) > Acute diarrhea is a cholinergic symptoms >> treat with atropine > delayed diarrhea is treated with loperamide > Pharamcogenomic testing >> Those who are homozygous for UGT1A1 28 allele are at increase risk for neutroepnai BW - Bone marrow suppression , diarrhea SE - Diarrhea, myelosuppression, mucositis etc.

Subarachnoid Hemorrhage

- SAH is associated with a high incidence of delayed cerebral ischemia 2 weeks following the stroke - Vasospasm is thought to be the cause of the delayed ischemia and can occur 4-21 days after the bleed.

What is the 1st line agent for anxiety?

- SSRI and SNRI

Selective MAO-B inhibitors

- Selegiline must be givein with levodopa - Rasagiline can be given monotherapy or adjunctive > Azilect the brand name - Selegiline (Eldepryl) > Emsam - patch for depression > Zelapar (ODT)

What if patient has persistent cough that last longer than 14 days (whooping cough)?

- Severe disease - Azithromycing 500mg x 1 then 250mg daily for 2-5 days Or - Erythromycing for 14 days Or - SMX/TMP DS 1 tab BID for 14 days Or - Clarithromycin 500mg BID or 1gram ER for 7 days

What are some SE of benzodiazepines?

- Somnolence - Dizziness - Weakness - Ataxia - lightheadedness W/ chronic use > tolerance, withdrawal symptoms if stopped NOTE - L-O-T (lorazepam, oxazepam, temazepam) > these are considered less potential harmful for elderly or with liver impairment since they are metabolized to inactive compounds (glucuronidase)

Buprenorphine + Naloxone (to block opioid if used) (Bunavail - Buccal tablets, Suboxone - sublingual tablets )

- Suboxone - Used as alternative for methadone (So patients can get from a regular healthcare provider and filled at any pharmacy and can get over opioid since withrawal symptoms are reduced). use daily for addiction - * To prescribe Suboxone: Prescribers need Drug Addiction Treatment Act (DATA) 2000 waiver. If they have it, the DEA number will start with X * Patch application > Change weekly > do not use same site for at least 3 weeks

What is the difference between systolic and diastolic dysfunction?

- Systolic dysfunction; heart failure with a reduce ejection fraction (HFrEF). Left ventricle is not ejecting enough blood > < 40% LVEF - Impaired ability of the left ventricle to fill with blood > diastolic dysfunction; heart failure with a preserved ejection fraction (HFpEF) > > 40-50% LVEF

List some lifestyle measures for constipation management

- These lifestyle modifications may be helpful to reduce constipation > Look for offending drugs > Correct fluid intake (64 oz - caution with CVD) > Limit caffeine and alcohol (to avoid dehydration) > Replace refined foods with whole grain products, bran, fruits and vegetables, beans. > Increase physical activity > do not delay going to the bathroom when the urge to defecate is present: may need to schedule time.

* Cephalosporins *

- They are beta-lactams that inhibit bacterial cell wall synthesis by binding to one or more penicillin-binding proteins (PBPs) - Cephalosporin exhibit time-dependent killing with bactericidal activity

Important about Atelvia

- This is a long-acting form of risedronate. Take the medicine after breakfast. Sit or stand upright for 30 minutes or longer after taking. Do not use acid suppressing "heartburn" therapy with this medicine. Do not take calcium , iron, magnesium, antacids, or multivitamin supplements until later in the day

Where is the use of Fluoextine unique?

- This medication has longer half life, and this can be used to minimize withdrawal symptoms. - Symptoms such as > Anxiety > Agitation > Insomnia > Dizziness > Flu-like symptoms - Paroxetine and some other agents carry a high risk of withdrawal symptoms and must be tapered upon discontinuation.

What is outcome from problem with sleep initiation, duration or quality?

- This will contribute to daytime impairment with fatigue, somnolence, poor memory and concentration and, if long-term, chronic health conditions.

Serious Adverse Effects

- Thrombogenic disorders, including heart attack, stroke/ DVD/PE. - Risk increase with AGE , SMOKE, Diabetes, Hypertension, and if she requires prolonged bed rest and if she is overweight. - The progestin drospirenone as well as the ortho overa patch are linked to a higher risk of blood clots and are best avoided in at-risk women.

Used Needle-Syringe Safety Tips

- Throw the entire needle/syringe assembly (needle attached to the syringe) into the red plastic sharps container - Place in the sharp container without recapping - Never compress or "push down" on the contents of any sharp containers

Why Meperidine is used in Amphotericin?

- To decreased duration of severe rigors

Common drugs used in Psoriasis

- Topical steroids [use high-potency steroids only short-term due to risk of side effects] - Retinoids - Coal tar, in many products including Neutrogene T, Denorex, Psoriasin, MG217 Psoriasis > Coal tar products are messy, time ocnsuming and can stain clothing and bedding. Side Effects > Skin irritation, photosensitivity - Anthralin - Calcipotriene (Dovonex, Calcitrene, Sorilux) > CI (avoid in hypercalcemia or vitamin D toxicity, do not use on face) - Acitretin (Soriatane) > Used only in severe cases due to numerous contraindications and side effects > Preg X - Ustekinumab (Stelara) > Warning: Risk of serious infections (including active TB, fungal, viral, bacterial or OI, lymphomas and other malignancies. -Apremilast (Otezal) > SE: weight loss, diarrhea, depression and suicidal ideation > Major CYP3A4, Avoid w/ strong 3A4 inducers. - Secukinumba (Cosentyx) > WARNING serious infections > Diarrhea

What is the treatment option for UC and CD [Mild-Moderate]

- Topical steroids or oral budesonide are first line

What are the benefits of using Creams, ointments, gels, solutions for topical conditions?

- Topical treatment used for topical conditions have a decreased incidence of systemic side effects and generally provide faster relief.

Colchicine (Colcrys)

- Treatment > 1.2mg orally followed by 0.6mg in 1 hour ( do not exceed 1.8mg) > Dose may be repeated in 3 days - Prophylaxis > 0.6mg once or twice daily - Renal impairment > Same dose for treatment but repeat in 2 weeks > For prophylaxis 0.3mg/day CI > Concomitant use of P-gp or strong CYP 3A4 inhibitor > Diarrhea , Nausea, etc. Notes > recommended only when treatment is started within 36 hours of onset of symptoms Notes > Fatal toxicity can occur if colchicine is combined with strong 3A4 inhibitors such as *clarithromycin * or a strong inhibitor of P-gp such as *cyclosporine*

Darunavir DRV (Prezista)

- Treatment naive: 800mg + 100mg ritonavir daily - Treatment experienced: 600mg + 100mg ritonavir BID - Take with food: swallow whole. must be given with ritonavir WARNING > Use caution in patient with sulfa drugs SE > N/D/Rash (including SJS/TEN), Increase in LFT, HA Monitoring > LFT, Rash * Make sure patient in their profile don't have sulfa allergies *

Important about Acute Gout attack treatment

- Treatment should be started within 24 hours - Usually one medication is enough but we can use the following combination in severe cases > Colchicine + NSAID > Steroid + Colchicine > Intra-articular steroids + NSAID or Colchicine - Ice bag is used - * Do not stop the prophylaxis medications *

Which agents are recommended to treat depression in parkinson patient?

- Tricyclic antidepressants showed a highest efficacy for treatment in these patients. - Also secondary amines such as desipramine and notriptyline

What is the recommendations given by " The American College of Gastroenterology guidelines " for treatment of H.pylori?

- Triple therapy with a PPI + 2 antibiotics (clarithromycin and amoxicillin ) for 14 days

What is the difference between Quadrivalent and Trivalent flu vaccines?

- Trivalent flu vaccine protect against three influenza viruses: Two influenza A's (an H1N1 and H3N2) and one influenza B. The quadrivalent flu vaccine protect against two influenza A's and two influenza B's.

Can you list some common type of nosocomial acquired infections?

- UTI from indwelling catheters > Remove the catheter ASAP - Blood stream infections from IV lines (central lines have the highest risk) - Surgical site infections - Decubitis ulcers - Hepatitis - Clostridium difficile, other GI infections - Pneumonia (mostly due to ventilator use)

Acetylcholinesterase inhibitors - Safety/Side Effects/Monitoring

- Used for mild-moderate disease (& in Combo for moderate-severe AD) - Side Effects > GI side effects (N/V/loos stools) > Donepezil given QHS to help tolerate nausea > Bradycardia, fainting, insomnia, tremors, weight loss NOTE - Other oral formulations are BID or daily if long-acting - Rivastigmine and galantamine IR with food (breakfast and dinner), galantamine ER with breakfast. Galantamine solution can be mixed with liquid and taken with 4 hours - Recommend Exelon patch or Aricept ODT to decrease GI side effects - if the cost difference is acceptable

What is the reason for use of Expectorants?

- Used for productive cough to decrease phlem viscosity in the lower respiratory tract and increase secretion in the upper respiratory tract to help move phlegm upwards and out - Guaifenesin (Mucinex) - SE > Nausea / vomiting / headache / dizziness

Centrally-Acting Alpha-2 Adrenergic Agonists

- Used for resistant HTN > After running out of other drug classes used to treat hypertension > If patient has problem swallowing tablets; Alpha-2 has agent that comes as a patch and is changed weekly (Helps with adherence) - Clonidine (Catapres, Catapres-TTS patch) > Kapvay for ADHD - GuanFACINE (Tenex) > Intuniv- for ADHD - Methyldopa (for pregnancy) > CI with active liver disease or concurrent use with MAO-I

COMT-Inhibitors

- Used only with levodopa to increase levodopa duration of action. Inhibits the enzyme COMT to prevent peripheral and central conversion of levodopa - Stalevo > Levodopa/Carbidopa + entacapone - Notes > Decrease in levodopa dose of 10-30% is usually necessary when adding COMT inhibitor - Tasmar - Tolcapone

Trametinib (Mekinist)

- V600E or V600K mutations SE - Hand food syndrome, Increse in LFTs, diarrhea, anemia, lymphedema, etc.

List some natural products used for anxiety

- Valerian > some products maybe contaminated with liver toxins - Lemon balm - Glutamine - Passion flower - hops - Chamomile tea - Theanine and skullcap - Kava > Hepatotoxic

LamoTRIgine

- Various formulations - Require titration (Increase every other week due to rash) - Usual dose 25mg Q daily - If using valproic acid start 25 max 100 - if using carbamazepine, phenytoin, phenobarbital or primidone , start at 50 mg max 400mg BBW - Serious skin reactions, including SJS/TEN Warning - Risk of aseptic meningitits, blood dyscrasias, et. SE - Somnolence, dizzines, rash, vision changes/diplopia, nausea, insomnia

SNRI

- Venalfaxine and Duloextine are approved for GAD, Depression - Duloextine is approved for Peripheral neurotpahty and Fibromyalgia , etc - Venalfaxeine approved for Social Anxiety Disorder - Cymbalta good option if patient has pain and depression.

What to do incase of Hyperkalemia?

- We need to stabilize the myocardial cells and rapidly shift potassium intracellular - D/C all potassium sources - Give IV calcium > To stabilize the heart, it will not decrease K - If the potassium is high or the heart rate and rhythm is abnormal, we need to hook up the patient to an ECG - Remove sources of potassium intake - Enhance potassium uptake by the cells via > Glucose to stimulate insulin > Insulin given w/ glucose to prevent hypoglycemia - If metabolic acidosis present, give sodium bicarb - Consider Beta-Agonist such as nebulized albuterol. monitor for tachycardia, chest pain - Increase renal excretion with loop diuretic such as furosemide. - Fludrocorstione (florinef) is optional - Consider the use of cation exchange resin, sodium polystyrene sulfonate (Kayexalate) and this can decrease K by 2 mEg/L with a single enema. > Given PO or rectally > Rectally for Emergency - Emergency dialysis can be used if the hyperkalemia could be fatal or patient with renal failure.

What is the dose for hydrocodone ER ?

- Zohydro - 10 mg Q 12 (opioid-naive)

What are the important information must know about Dexmedetomidine (Precedex)?

- alpha 2 agonist - Used for sedation in intubated and non-intubated patients - Duration of infusion should not exceed 24 hours - SE > Transient hypertension during loading dose (may need to decrease infusion rate), hypotension, bradycardia, dry mouth, nausea

How to minimize risk of hyperkalemia?

- avoid in impaired renal function (CrCl < 30 ml/min) - Do not start if K > 5 - Start with a low dose. Higher risk when concurrent ACE inhibitors or ARBS are used at higher doses - Do not use NSAID concurrently (should be avoided anyway in HF) - Monitor frequently - Counsel patient about increase risk if dehydration occurs

List some natural products used for Vasomotor Symptoms

- black cohosh - Red clover - Soy - Flaxseed - Evening Primrose

What is the function of Anticholinergic?

- block the action of acetylcholine and increase cyclic guanosine monophosphate (cGMP) at parasympathetic sites in bronchial smooth muscle causing bronchodilation.

List some drugs that can become supratherapeutic if smoker quiet long term smoking?

- caffeine - theophylline - estrogen (oral) - fluvoxamine - olanzapine - clozapine > risk for arganuloyctosis

Macrocystic Anemia

- caused by either a vitamin b12 or folate deficiency or both. - If continued long term, the patient is at risk for serious neurological consequences including cognitive dysfunction (dementia) and peripheral nerve damage. - It is diagnosed by low hemoglobin and high mean corpuscular volume (MCV>100 mm3)

How to classify Arrhythmias?

- classified based on their location of origin into two broad categories > Supraventricular (orginitating above the AV node) > Ventricular (orginiating below the AV node)

Rule of Vitamin K

- comes as 2.5-5mg - Oral formulations of vitamin K are preferred in patients without significant or major bleeding - Vitamin K given SC produces a slow onset and a variable response (AVOID) - Vitamin K given IM cause hematoma (AVOID) - IV administration should be given *only* when patient is experiencing serious bleeding. - Infuse slowly because.. > IV injection is reported to cause anaphylaxis in 3 out of 100,000

Which chemotherapy agents put patient at moderate to high risk for Emetic PO

- cyclophasmide >= 100mg/m2/day - Crizotinib - Etoposide - *Lomustine* - single use - Procarbazine - Temozolomide > 75 mg/m2/day - Vismodegib

Hyperglycemia symptoms

- dizziness - Headache - Anxiety - Shakiness - Diaphoresis - Excessive hunger - Confusion - clumsy or jerky movement - Tremors - Palpitations or fast heart rate - blurred vision

What are the classic symptoms of heart failure?

- dyspnea (SOB) - Fatigue - Peripheral edema

NEW DRUGS

- lloperiodone (Fanapt) > Dizziness, somnolence, orthostasis, tachycarida > QT prolongation; avoid use with QT risk - Asenapine (Saphris) > No food/drink for 10 mins after dose SE > sedating, tongue/mouth numbess > EPS (5% more than placebo), QT prolongation; avoid use with QT risk - Lurasidone (Latuda) SE > Sedating, EPS, dystonias, Nausea, agitation, akathisia > Nearly weight, lipid and blood glucose netural NOTE - Take with food >= 350 kcal

Majority of drugs in which filters are necessary use a ...... filters and for lipids they use.....

0.22 micron and 1.2 micron for lipids

CHA2DS2-VASc

0= no ancticouagulaton 1= No anticoguatlion or oral anticoagulation or ASA 2 = Oral anticoagulation is recommended. Options include warfarin dabi, riva and apixaban

What is the dose of activated charcoal?

1 gram per kg

Who is candidate for Tdap?

1. Administer one time dose of Tdap to adults who have not received Tdap previously, and as soon as feasible to all 1. pregnant or postpartum women 2. close contacts of infants younger than age 12 months (e.g., grandparents and child-care providers 3. health care presonnel with direct patient contact - Pregnant women week 27-36 should receive Tdap with each pregnancy.

What you must know about Thioridazine?

1. Belong to 1st generation (low potency) 2. Boxed warning for QT prolongation

How to reduce infections with ventilator use?

1. Elevate head-of-bed 30-45 degrees 2. Assess readiness o wean off ventilator at least daily 3. Use breaks or reductions in sedation use if possible 4. Consider DVT prophylaxis 5. Use stress-ulcer prophylaxis judiciously and only in patients who meet requirements for use

What is the recommended treatment for Asthma in pregnancy?

1. Inhaled corticosteroid > Budesonide is the preferred inhaled corticosteroid for use during pregnancy and it comes in Respules fore infants 2. Rescue inhaler (inhaled albuterol)

List 2 types of C-R packaging?

1. Screw caps 2. Unit dose packaging

List 2 drugs used to treat metabolic acidosis of CKD

1. sodium bicarbonate 2. Sodium citrate > Metabolized to bicarbonate by liver, may not be effective in concomitant liver failure

Why we try to attack the immune system via multiple MOA?

1. to lower toxicity risk 2. To reduce the risk of graft rejection

Genetic factors contribute between ....... of the differences in drug metabolism and response between patients: A. 10-15% B. 20-40% C. 50-80% D. 75%

20%-40%

Engage in aerobic physical activity 3-4 session per week, lasting 40 minutes/session and involving moderate-to-vigorous intensity can reduce LDL-C by?

3 to 6 mg/dL

How long it takes for iron deficiency anemia to be restored after supplements with ferrous sulfate?

3-6 months to adequately replete iron stores

T or F. If live vaccines (MMR, MMRV, Varicella, Zoster and yellow fever) and live intranasal influenza vaccine (LAIV) are not administered at the same visit, they should be separated by at least ........ weeks

4

Noticeable improvement may be see after .... or more weeks of treatment for anxiety.

4 weeks

Define pernicious anemia?

A type of macrocystic anemia that results in low vitamin B12 levels due to lack of intrinsic factor which is required for adequate vitamin b12 absorption in the small intestine.

How to diagnosis UTI?

A urinalysis is considered positive when there is evidence of pyuria (positive leukocyte esterase or >=10 WBC/ml and bacterurima >= 10^5 bacteria/ml in uncomplicated patient and >= 10^3 bacteria/ml in complicated patients including in men

Sometimes heat is used to mix things easier, but it lead to over-heating which produce undesired chemical-reactions. What is the best way to prevent over-heating?

A water bath will help prevent over-heating

Whenever a young woman enters the pharmacy, the pharmacist can ask if she is consuming adequate folic acid A........... , calcium B........... and vitaminD C..... * because many pregnancy are not planned *

A. 400-800 mcg 0.4-0.8mg daily B. 1000mg calcium daily C. 600IU daily

Which of the following has higher potency? A. Elocon (mometasone) ointment B. Elocon (mometasone) cream

A. Elocon ointment is high potency and B is low-medium potency and reason behind this is ointment is 80% oil

Which of the following agents has BBW for risk for progressive multifocal leukoencephalopahty (PML)? A. Natalizumab (Tysabri) B. Vedolizumab (Entyvio)

A. Natalizumab (Tysabri)

Which Migraine medications comes as Injection formulation? A. Sumatripitan B. Eletriptan C. Zolmitriptan D. Naratriptan

A. Sumatriptan

Smoking is protective in A. UC B. CD

A. UC

B4 starting patient on 2nd generation anti-psychotic; they should be screen for all of the following EXCEPT? A. overweight B. obesity C. dyslipidemia D. hyperglycemia E. Hypertension

ALL OF THEM

Refer to URGENT care with any of the following signs and symptoms

Age < 3 months old with a temperature of 100.4 F Age 3-6 months with a temperature of 101 F Age > 6 months with a temperature of 103 F

Anemia treatment in prgnancy

Anemia due to iron deficiency can occur during pregnancy and will be treated with supplemental iron in addition to prenatal vitamin which contain some iron

Tyrosine Kinase Inhibitors (TKIs) Targeting VEGF

Axitinib (Inlyta) > Inhibits VEGF - SE > Hypertension, hepatotoxicity, hand-food syndrome, dysphonia

Which ARB must be kept in original container?

Azilsartan (Edarbi)

Which of the following is a pro-drug? . Phenytoin B. Fosphenytoin

B. Fosphenytoin

Which of the following is indicated for males? A. Cervarix B. Gardasil

B. Gardasil

Ambrisentan (Letairis)

BBW _ Use in pregnancy is CI - Require REMS SE - Peripheral edema, headache, decrease in HgB * NOTE * > Monitoring of LFTs was removed from package insert on March 4, 2001 (FDA)

LamiVUDine 3TC (Epivir) + abacavir (Epzicom)

BBW (2) 1. Do not use Epivir-HBV for treatment of HIV (contains lower dose of lamivudine) 2. May exacerbate HBV once drug is discontinued or when HBV resistance develops

Foscarne (Foscavir) - Resistant HSV, CMV

BBW 2 1. Renal impairment occur to some degree 2. Seizure due to electrolyte imbalances SE - Electrolyte abnormalities (decrease in K, Ca, Mg, Phos) increase in SCr, increase in BUN Notes - Vesicant , handle as chemotherapeutic agent

What is the drug of choice if urgent cardiac surgery or PCI is required ?

Bivalirudine is the preferred anticoagulant

which patch comes with a patch-disposal unit to be placed in and sealed then disposed in the trash?

Butrans patch

How does an investigator will want to ship the drug to clinical investigators in many states?

By obtaining IND. The IND is the means through which the sponsor technically obtains this exemption from the FDA in order to conduct clinical trials.

Who is required to report SE , AE ? A. Pharmacist B. Doctors C. MFT

C.MFG

What are the treatment for PAH if Acute Vasoreactivity testing is +?

CCB

What is the DOC for Gonorrhea in pregnancy?

Cephalosporin or if contraindicated azithormycin 2gram PO x 1

Important about Cefotetan?

Contains N-methylthiotetrazole (NMTT or 1-MTT) side chain, which can increase risk of hypoprothrombinemia (bleeding) and a disulfiram-like reaction with alcohol ingestion

Pregnancy Category A

Controlled studies in animal and women have shown no risk in the 1st trimester, and possible fetal harm is remote

List some methods use to determine the impact of the pharmaceutical product?

Cost-effectivness analysis (CEA) Cost-minimization analysis (CMA) Cost-utility analysis (CUA) Cost-benefit analysis (CBA)

What are the diagnosis criteria for RA?

Criteria 1-4 must be present for >= 6 weeks and 4 or more criteria must be present Diagnostic Criteria 1. Morning stiffness around joints lasting > 1 hour 2. Soft tissue swelling (arthritis) in 3 or more joints 3. Swelling (arthritis) of hand, foot, or wrist joints 4. Symmetric involvement 5. Rheumatoid nodules 6. Positive serum rheumatoid factor (present in about 70% of patients) 7. Radiographic erosions or periarticular osteopenia in hand or wrist joints

What is the most significant side effect of Nelfinavir (Viracept)?

Diarrhea (up to 20% in adults)

Which patches you apply TWICE DAILY?

Diclofenac (Flector) twice daily

Fill in the blank Oil in water , or water in Oil is example of .......

Emulsions

Tamoxifen (Soltamox)

Estrogen receptor 1 If positive , can use drug -> Available evidence indicates that patients whose tumors are estrogen receptor positive are more likely to benefit from tamoxifen therapy

What is the drug of choice for Absence seizure?

Ethosuximide - Valproate can be used too

Drugs that cause DRESS

Ethosuximide Carbamazepine Phenytoin/Fosphenytoin Minocycline DOxycyline

What is important about Etravirine?

Etra- Turan (FC-barcelona) > Increase cholesterol > Increase LDL > Hyperglycemia > Peripheral neuropathy

mTOR inhibitors

Everolimus (Afinitor) > Zortress is indicated for transplantation SE > Dyslipidemia , hyperglycemia, rash, stomatitis, interstitial lung disease, etc. Temsirolimus (Torisel) > Use non-PVC bag and tubing due to leaching of DEHP SE > Dyslipidemia, hyperglycemia, myelosuppression, intersitital lung disease.

Traveler's Diarrhea Treatment

FQ for 3 days or Azithromyin for 3 days (drug of choice for pregnancy and children) Bismuth subsalicylate to reduce incidence of TD PLUS Loperamide 4mg x 1, then 2mg after each loose stool maximum of 8mg/day (loperamide is not recommended if patient has signs of dysentery, high fever, or blood in stool)

T or F. drugs should be separated from the activated charcoal by at least one hour?

False. At least 2 hours

how to gauge the severity of kidney damage in patients with kidney disease or nephropathy?

From measuring the amount of albumin in the urine because albumin is not suppose to pass the filtrate

Afatinib

Gilotrif

Pseudomonas aeruginosa, E.Coli, Haemophilus Influenzae, Vibrio cholerae, H. Pylori, Klebsiella pneumonaie, Legionella pneumophillia, Proteus Mirabilis

Gram - Rods

What is the difference between gram- and gram + and what color it will stain ?

Gram-positive organisms have a thick cell wall and stain purple, or bluish in color from a crystal violet stain, whereas gram-negative organism have a thin cell wall and take up the safranin counterstain, resulting in a pink or reddish color.

Important drug interaction w/ Hydroxyurea

Hydroxyurea may enhance the adverse/toxic effect of Didanosine. An increased risk of pancreatitis, hepatotoxicity and/or neuropathy may exist. Didanosine may enhance the adverse/toxic effect of Hydroxyurea. An increased risk of pancreatitis, hepatotoxicity and/or neuropathy may exist. Severity Major Reliability Rating Good

Agents used to treat Acinetobacter baumanni?

Imipenem Meropenem Doripenem

How to calculate LDL-C?

LDL-C = TC - HDL-C - (TG/5) > Invalid if TG > 400 mg/dL

Which patches are applied daily?

Methylphenidate (Daytrana): Apply QAM 2 hours prior to school, alternate *hips* daily Nicotine (NicoDerm CQ): apply daily Rivastigmine (Exelon): daily Rotigotine (Neupro): Daily Selegiline (Emsam): Daily

What is unique about Concerta?

Methylphenidate IR-ER - they use OROS system > Somewhat harder to abuse (harder to crush) - Take QAM with or without food - swallow whole

What is the treatment for HHS?

NS and insulin

What is the treatment for DKA?

NS, followed by 1/2 NS, potassium replacement, insulin and occasionally sodium bicarbonate.

How to calculate non-HDL-C?

Non-HDL-C = TC - HDL-C

Rivaroxaban (Xarelto) [dose]

Non-valvular Afib - CrCl > 50 ml//min: 20mg PO daily with evening meal - CrCl 15-50 ml/min: 15mg PO daily with evening meal -CrCl < 15: Contraindicated Treatmend of DVT/PE - 15mg po BID with food for 21 days then 20mg po QD with food -CrCl < 30 ml/min: Avoid use Prophylaxis for DVT (after knee/hip replacement) -10mg po daily - without regard to meal - CrCl < 30ml/min: Do not use - * first dose given 6-10 hours after surgery * - * Take for 35 days after hip replacement surgery; take for 12 days after knee replacement surgery * Reduction in the Risk of Recurrence of DVT and PE - 20mg PO daily with food - CrCl < 30ml/min: Avoid use

Apixaban (Eliquis)

Non-valvular Afib 5 mg BID unless: >= 80 years , body weight <=60 kg, or Scr >= 1.5 mg/dL, give 2.5 mg BID Treatment of DVT/PE 10 mg PO BID x 7 days, then 5mg po BID Prophylaxis for DVT (after knee/hip replacement) 2.5 mg po BID First dose given 12-24 hours after surgery Take for 35 days after hip and 12 days afer knee - Reduction in the risk of recurrence of DVT and PE 2.5 mg PO BID after at least 6 months of treatment for DVT or PE

Norepinephrine and Epinephrine - Light protection

Norepinephrine - Oxidation turns the liquid brown. If discolored do not use Epinephrine - Oxidation turns the liquid pink, then brown. If discolored, do not use.

Fill in the blank. Example of drugs used for local effects include dermal preparations. Ex. Coal tar preparations is used topically to treat.....

Psoriasis Coal Tar 2%

Which medication of the NNRTI require a meal that contain high calorie (500+) to optimize absorption?

Rilpivirine, RPV (Edurant)

Pathophysiology of stable chest pain

Stable angina is due to plague build up causing narrowing of the arteries and reduced blood flow to the heart. lack of oxygen cause the chest pain

What is SD?

Standard Deviation shows how much variation, or dispersion, there is from the mean.

Drugs that cause SJS/TEN

Sulfamethoxazole Allopurinol Carbamazepine Clindamycin Oxcarbazepine Phenobarbital Ethosuximide Lamotrigine Phenytoin/Fosphenytoin Clopidogrel Ticlopidine Quinine Abacavir Nevirapine Letrozole Hydroxycholoroquine Piroxicam Minocycline, Doxycycline Tiagabine Zonisamide

What is the treatment for TCA (Tricyclic Antidepressants) medications?

Supportive care IV hypertonic sodium bicarbonate can be administered to decrease the widened QRS complex. Benzodiazepines may be used for symptoms relief. Vasopressors may be used for hypotension

What is the ANTIDOTE for Stimulant overdose from amphetamines, including ADHD and weight loss drugs, cocaine, ephedrine, caffeine, theophylline, MDMA (ecstasy), alcohol withdrawal?

Supportive care, possibly benozodiazepines

Primary Signs/Symptoms of Parkinson Disease

TRAP: Tremor: seen during resting, usually worsened by anxiety Rigidity: arms, legs, turnk and face Akinesia/brdykinesia: lack of movemnt or slow initiation of movement Postural instability: poor balance, which may lead to frequent falls

What you know about TSH, T4 and FT4

TSH and FT4 are used for diagnosis which is indicated by LOW FT4 and HIGH TSH - Normal TSH = 0.3 - 3.0 mIU/L - Normal FT4 = 0.9 - 2.3 ng/dL Normal T4 = 4.5 - 10.9 mcg/dL * Low iodine intake is a cause of hypothyroidism; this is rare in the U.s because iodine is added to table salt (NACL) *

Why all forms of carvedilol should be taken with food?

Take all forms of carvedilol with food to decrease rate of absorption, thus minimizing risk of orthostatic hypotension

What is the difference between Vilazodone (Viibryd) (SSRI and 5HT1a)

Take with food Less sexual SE compared to SSRI and SNRI Notes - Still at increase risk for bleeding with concurrent use of anticoauglant ,etc.

Tipranavir (Aptivus)

Take with food Must be given with Ritonavir BBW 2 1. In combination with Ritonavir can cause hepatitis (sometimes fatal) 2. Intracranial hemorrhage Warning - Use caution in patients with a sulfa allergy SE - Diarrhea, rash , increase in CPK

Indinavir (Crixivan)

Take without regard to meal Side Effects > Nausea, Nephrolithiasis [Taking with 48 oz (1.5 L) of water may reduce risk], indirect hyperbilirubinemia, alopecia etc. Notes > Must dispense in the original container with the desiccant to protect from moisture. Avoid high fat/high calories meal as indinavir absorption is decrease

Denosumab = So

Target is bone

Bevacizumab = ci

Target is circulatory system

Ipilimumab = Li

Target is immune system

Tumor = tu

Target is tumor

Which tricyclic cause more weight gain and sedation? Primary amines or Tertiary amine?

Teritary amine cause more sedation and weight gain Ex. Doxepin

What is type 1 error?

The Greek letter, alpha (a) is the probability of type 1 error. The alpha value is chosen by the researcher, before the study starts. A type 1 error occur when the null hypothesis is true, yet it was rejected in error. When a researcher chooses the p-value of < 0.05 for statistical significance, the researcher accepts the fact that this error will occur < 5% of the time. A type 1 erro is also known as false positive. To remember it easier " false positive study"

What is type 2 error?

The Greek letter, beta (B) is the probability of a type II error. Beta is generally set at 0.1 or 0.2, indicating a willingness to accept a type II error 10 or 20 times in 100 comparisons. A type II error occur when the null hypothesis is false, yet it was accepted in error. A type2 error is also known as false negative. To remember it easier false negative study"

What is the difference between Macrocystic and Microcystic anemia?

The MCV is small < 80 mm3, in microcystic anemia due to a small cell size and can be due to iron deficiency. The MCV is large > 100 mm3 in macrocystic (megaloblastic) anemia and can be due to folate or vitamin B12 deficiency. * Normocytic anemia is 80-100 *

What is Alternative Hypothesis (Ha)

The alternative hypothesis state that there *is* a treatment difference (or a relationship) between groups in the trial (i.e Drug A not equal to Drug B). If you fail to accept or Reject the No then you are accepting Ha.

what is Pneumbra?

The area surrounding the ischemic

What is the drug of choice for Hypervolemic hyponatremia and Isovelmic (euvolemic) hyponatreami?

The arginine vasopressin (AVP) receptor antagonists (conviaptan or tolvaptan) may be used to treat SIADH and hypervolemic hyponatremia. They increase water excretion while maintaining Na. They are more expensive than 3% saline.

Important about niacin

The best clinical choice is Niaspan with less flushing and less heptatoxocity - but it is the most expensive one - Formulations are not interchangeable - Take with food, avoid hot beverages and spicy food which can worsen flushing

What is the drug of choice for primary peritonitis?

The drug of choice is ceftriaxone for 5-7 days

What is Median?

The median is the value in the middle of a ranked list - to calculate the Median, arrange the observations in numerical order (lowest to highest) and pick the middle value.

What is Mode?

The mode is the value that occurs most frequently in a set of data.

List some date rape drugs

The most common date rape drugs - also called "club drugs" - are flunitrazepam (Rohypnol), also called roofies; gamma hydroxybutyric acid (GHB), also called liquid ecstasy; and ketamine, also called Special K. These drugs may come as pills, liquids, or powders

What is the normal Albumin levels and why having low albumin levels put patient at risk for toxic effects when dealing with drugs that are highly protein-bound (>90%)?

The normal albumin levels are 3.5 - 5 g/dL Low albumin levels are < 3.5 If drug is high protein bound and albumin is <3.5 g/dl it means drug will not have enough albumin to bind too so it will interact with receptors and cause therapeutic or toxic effects (depends on the binding site)

what is Null Hypothesis (No)

The null hypothesis states that there is *no difference (or relationship) * between groups (example Drug A = Drug B ). -> A study is designed to reject Null hypothesis and show there is a difference between Drug A and Drug B and this is called Alternative Hypothesis Ha

How to create a good statistical power?

The power of a study is determined by several factors including the sample size, the number of events (MIs, strokes, deaths, etc), the effect size and the statistical significance criterion used.

What does variations in 1B1 (SLCO1B1) cause ? HIN! statin

The presence of variant alleles have an increased risk of myalgia when taking statins.

Opioids

The primary mechanism for pain relief occurs when the opioid binds to the mu receptor in the CNS

What is Range?

The range is the difference between the highest and the lowest values.

Important about the use of ACE inhibitors or ARB

The use of these drugs will cause a 30% rise in serum creatinine during the initiation of treatment. This rise is generally acceptable and is not a reason to stop treatment. If the rise is greater than 30% then therapy should be discontinued and the patient should be evaluated for hemodynamic factors that may need addressing. - * It is recommended that the serum creatinine and potassium be monitored 1-2 weeks after initiating ACEIs or ARBs in patient with CKD *

Raynaud's Phenomenon

The vasospasm causes the skin to turn while and then blue, which is followed by painful swelling when the affected area warms.

which of the following is giving with Cisplatin to reduce risk of nephrotoxicity ? A. Mesnex B. Zinecard C. Procrit D. Totect E. Thyol

Thyol

T or F. Live vaccines cannot be given post-transplant.

True

True or False. All sexual partners of patient who infected with STI should be treated concurrently to prevent re-infection.

True

T or F. Bradycardia is one of the symptoms of Organophosphate poisoning, which occurs most commonly with farm workers due to pesticide exposure.

True. Pesticides are the only toxic substances released intentionally into our environment to kill living things. This includes substances that kill weeds (herbicides), insects (insecticides), fungus (fungicides), rodents (rodenticides), and others

What is the 1st line therapy for RA?

Usually, methotrexate is used 1st line and these agents are add-on therapy. However, if the initial presentation is severe, these can be started as initial therapy.

Agents used to treat nosocomial-associated methicillin-resistant Staphylococcus aureus (as long MIC is not >=2 and not Vancomycin-Intermediate Staphylococcus aureus (VISA) )

Vancomycin

What is the first line treatment for patient with sepsis shock?

Vasopressor Antibiotic administration within an hour of presentation +/- steroids

What is the first line treatment for patient with cardiogenic shock?

Vasopressors and/or inotropes

Vasodilators [Nitroglycerin]

Venous vasodilator, particularly at low doses, but is an effective arterial vasodilator at higher doses. * Preferred in ADHF with active myocardial ischemia or uncontrolled hypertension, but the effectiveness may be limited after 2-3 days * CI - SBP < 90 mmHg prior to therapy, cardiogenic shock (when used as primary therapy) Side Effects - Hypotension, headache, lightheadedness, tachycardia, tachyphylaxis Monitoring BP , HR

T or F. Prisiq has ghost tablet?

Yes.

Can you cut Lidoderm?

Yes. It is designed to be cut and applied over the painful regions.

Patient received first therapy of antibiotics for AOM but treatment after 48-72 hours failed , what is the recommended first line therapy?

amox-clav for 5-10 daysif patient 1st line was amoxicillin Ceftriaxone 50mg/kg IM/IV daily for 3 days if first option was augmentin

Important about ACE-Inhibitors

angiotensin II constricts the efferent arterioles to a greater extent than the afferent arterioles within the kidney causing increased perfusion pressure in the glomeruli resulting in kidney damage over time. > Blocking this conversion will cause vasodlation and reduce BP > ACE-I have been shown to slow progression of renal disease in patients with diabetes and/or hypertension who have albuminuria; they also slow progression of heart failure. * Captopril (Capoten) is taking 1 hour before meals (empty stomach) also Moexipril (Univasc) *

What is Tonic-clonic seizures?

characterized by five phases > flexion > extension > Tremor > clinic > loss of consciousness

Cetuximab = xi

chimeric

Tositumomab = o

mouse

What is so unique about MOA of ability?

second-generation antisychotics (SGAs) block D2 and 5HT2a receptors. Ability is unique because it also act as a D2 and 5HT1a agonist

What antibiotic to give for acute bronchitis- expect cough to last 2 weeks?

this is mild-to-moderate disease > Usually viral > Antibiotic is not indicated > Recommend anti-tussive +/- inhaled bronchodilators

T or F. Seizure is more common with phenothiazines and buyrophenoones?

true

What is the difference between glass and Wedge-wood/Porcelain(ceramic) Mortar and Pestle?

wedge-wood or porcelain is used commonly and is best for reducing particle size of dry powders and crystals Glass is used for liquids or chemical that are oily or that will stain to the porcelain

Important about Fibrates

- Gemfibrozil is the only drug that is used BID > 600mg BID 30 minutes before breakfast and dinner

Important about dosing in reduced kidney function

1. Extend the interval if u are dealing with drug that exhibit concentration dependent bacterial kill 2. Decrease the dose if you deal with drug that exhibit time-dependent killing properties

What are the 3 classic symptoms of meningitis?

1. Headache 2. Stiff neck 3. Altered mental status

When to treat HTN?

1. If no other conditions and age 60 or up > Target < 150/90 2. If no other conditions and age 18-59 > Target < 140/90 3. If Diabetes or CKD regardless of age > Target < 140/90

List the vaccines that are used during pregnancy

1. Influenza vaccine (shot, inactivated) each fall 2. Pregnancy woman should receive Tdap between weeks 27-36, *each pregnancy*. If the woman has not been vaccinated or if the history is unclear, a 3-doses series is needed (one with Tdap, the other two with Td only) and if woman deliver a babe she still need to receive it post delivery. Vaccination protects the babe from pertusis and the mother.

What are some important information about using Midazolam in ICU ?

1. Injection only 2. Highly lipophilic and can accumulate in obese patients 3. Active metabolite that accumulates in renal dysfunction (caution with continuous infusion) 4. CI: concurrent use of potent cyp450 3A4 inhibitors 5. Has 3 BBW > May cause severe respiratory depression, respiratory arres, and apnea

What important things to know about phosphate binder: Aluminum based Aluminum hydroxide (AlternaGEL, Amphojel, others)

1. It is limited to 4 weeks due to risk of accumulation of aluminum 2. It cause constipation , poor taste, aluminum intoxication, dialysis dementail and osteomalacia 3. Must monitor Ca2+ , Phosphours, Serum aluminum concentration and PTH

How can DEHP lead to systemic absorption and what is the other concern?

1. Leaching (DEHP leaches from the PCV container into the solution) 2. Sorption (The drug moves into the PCV container) * If drug is incompatible with PCV, then it can be placed in polyolefin container or glass container *

List some safety concerns with Natural Products and Vitamins?

1. Manufacturing may not follow good manufacturing practices (GMP). As a pharmacist we should be able to counsel patient about the right product. We can access consumerlab.com for help patient choose a reputable product 2. A dietary supplement manufacturer does not have to prove product's safety and effectiveness before it is market 3. * Dietary supplement Health and Education act (DSHEA), the manufacturer is responsible for ensuring their dietary supplement products are safe. [In contrast, drugs must be proven safe and effective for the intended use before they can be sold* 4. Company selling or distributing the product is required to record, investigate and forward to the FDA repots they receive of serious adverse events 5. Dietary supplement can't make claims that the product treats or cures a condition

List 2 strategy to minimize catheter-associated bloodstream infections?

1. Minimize the use of intravascular catheters 2. Peripheral catheters should be removed/replaced every 2-3 days to minimize risk of infection 3. Use of skin antiseptics (2% chlorhexidine) , antibiotic impregnated central venous catheters, and antibiotics/ethanol lock therapy.

List 2 agents that are used as Cytoprotective and what is their MOA

1. Misoprostol: A prostaglandin E1 analgo that replaces the gastro-protective prostaglandins remvoed by NSAIDs > SE: Diarrhea, abdominal pain > NOTE: Preg X 2. Sucralfate a sucrose-sulfate aluminum complex that ineract with albumin and fibrinogen to form a physical barrier over an open ulcer. This protects the ulcer from further insul by HCL acid, pepsin and bile and allows it to heal > SE: Constipation

What is the DOC for candida?

1. Oropharyngeal and vaginal: Fluconazole or topical (clotrimazole or nystatin) 2. Esophageal and invasive: Fluconazole or echinocandin or amphotericin B

The Recommended regimens for initial treatment for antiretroviral therapy-naïve patients

1. PI-Based Regimens > Darunavir + Ritonavir + Tenofovir/Emtricitabine 2. INST-BASED REGIMEN > Dolutegravir + abacavir/lamivudine (Triumeq) > Dolutegravir + tenofovir/emtricitabine > Elvitegravir/cobicistat/tenofovir/emtricitabine (Stribid) > Raltegravir + tenofovir/emtricitabine

Which antibiotics should not be used during pregnancy?

1. Quinolones (due to cartilage damage) 2. Tetracycline (due to teeth discolorations)

Patient called and believe their son consumed tablets, what to do?

1. Remove anything in the mouth and collect any suspect containers to bring to the ER 2. If unconscious, place the patient on the left side to more easily clear vomit if emesis occur > * Ipecac syrupe was used previously to induce emesis for certain exposure, but is no longer commercially available, and is no longer recommended. Instruct others not to give ipecac syrupe if still have them * 3. If the patient is unconscious, having difficulty breathing, appears agitated or is seizing emergency help should be contacted (911) 4. If patient is not breathing/ or has no pulse CPR should be initiated.

What is unique about Ritalin LA?

1/2 is IR and other half is SR = in one capsule

What is the adequate trial of an antipyschitic medication?

4-6 weeks -> Making sure patient was taking medication as prescribed. -> A previous positive or negative history with antipsychotics should be used to guide therapy

How many hours are required for elimination of greater than 95% of the drug from the body?

5 half-lives are required to eliminate more than 95% of the drug

What is the half life of theophylline if it takes 5 hours for this drug to fall from 16 mg/L to 8 mg/L ?

5 hours - half life is the time it required or the drug to decrease by 50% t1/2 = 0.693 / ke

Which group qualify for moderate-to-high intensity?

Age 40-75 years with LDL between 70-189 mg/dL with estimated 10-year ASCVD risk >= 7.5%

Which Ascites patient require transplantation?

All Ascites patients with cirrhosis should be considered for liver transplantation.

What is the drug of choice for dental work to prevent Infective Endocarditis (IE)

Amoxicillin 2gram for 1 dose 30-60 minutes prior to dental work. If allergy Clindamycin 600mg or Clarithromycin or azithormycin 500mg

Anthracyclines

BBW - Irreversible myocardial toxicity may occur as total dosage approaches cumulative max dose - AML MDS - Severe myelosuppression with impaired hepatic function - Potent vesicant CI ANC < 1500

Macitentan (Opsumit)

BBW - Pregnancy is CI SE - Decrease in HgB, and headache

Why Aspirin is more effective anti-platelet agent than other non-selective NSAIDS?

Because aspirin inhibit COX1 irreversibly

Why men with ED should be preferred to cardiac evaluation?

Because patient might have cardiovascular disease. ED can appear first since the arteries supplying blood to the penis are smaller than those supplying blood to the heart; they can become restricted sooner than the larger vessels.

What is pink sheet?

Biopharma regulatory, legislative, legal and business developments

What is the antidote for iron overdose?

Deferoxamine

Ranolzine DDI

Limit simvastatin to 20mg/day if used concurrently with Ranolzine - Do not use with strong 3A4 inhibitor - 500mg po BID in patient taking moderate CYP 3A4

List one drug that has VERY LOW Volume of distribution Vd and one drug that has VERY HIGH volume of distribution?

Low Vd = Warfarin (8 L ) High Vd = Chloroquine (15,000L)

What should be given if patient receiving Ifosfamide?

Mesna (Mesnex) is always given with ifosfamide to prevent hemorrhagic cystitis and with high doses of cyclophosphamide

Which vaccines are recommended for smokers?

Pneumovax 23 Influenza

Soluble Guanylate Cyclase (sGC) Stimulator

Riociguat (Adempas) - Approved for PAH and CTEPH BBW - Use in pregnancy is contraindicated CI - Pregnancy, concimtant use of PDE-5 inhibitors SE - HA, dyspepsia, dizziness, hypotension, N/V and diarrhea - Bleeding appears to be more common with riociugat than placebo

What is the difference between Rowasa and Canasa?

Rowasa is brand name for mesalamine enema Canasa is brand name for mesalamine suppository > Used for distal ulcerative colitis

What you must know about the fiber - Chronic/Maintenance therapy ( Bulk-producing laxatives) - Psyllium (metamucil), Methylcellulose (Citrucel), and Calcium polycarbophil (fibercon, others)

SE > increased gas, bloating, bowel obstruction etc. > Choking: make sure to drink a lot of fluid Notes > Onset of action - 12 to 24 hours > Drug of choice in pregnancy > First-line treatment for constipation -/+ an osmotic agent - * take 2 hours before or after drugs

* Must know about Ototoxicity *

Salicylates, vancomycin, aminoglycoside, cisplatin and loop diuretics

Which neurotransmitter is considered the most important biologic involved with feelings of well being?

Serotonin (5HT)

What are the symptoms of low blood sugar?

Shakiness, Iritability, hunger, headache, confusion, somnolence, weakness, dizziness, sweating, and fast heartbeat. Very low sugar can cause seizures (Convulsions), fainting or coma.

PDE_5 inhibitors for Pulmonary Arterial Hypertension

Sildenafil (Revatio) Tadalafil (Adcirca) (avoid if Crcl < 30ml/min) CI > Concurrent use of nitrates > Avoid use sildenafil for PAH in patient taking PI-based (ritonavir) HAART regimen SE - Dizziness , drop in BP , headache, etc

How to confirm thrombocytopenia?

To confirm diagnosis of heparin-induced thrombocytopenia (HIT). If the ELISA test is positive, a positive SRA (Sertotonin Release assay) is confirmatory

Where should providers report clinically significant adverse events?

To the FDA's Vaccine Adverse Event Reporting System (VARES)

Foscarnet dcrease Ca , Mg and Po4 ?

True

T or F. For capsules, the label must include mcg or mg/capsule?

True - Does not require for tablet

Most drugs reactions are characterized as type A and type B. what is the difference?

Type A: dose-dependent and predictable. Ex. we know if patient start doxazosin at 1mg QHS will have less orthostatic hypotension from starting at 4mg thats why we titrate Type B: We don't know unpredictable and it is patient specific.

How to diagnose HIT?

Unexplained drop in platelet count (defined as > 50% drop from baseline) and lab confirmation of antibodies or platelet activation by heparin

What are the sign/symptoms of Anaphylaxis?

Urticaria (HIVES) rash Swelling of the mouth and throat Difficulty breathing or wheezing sounds abdominal cramping hypotention (which can cause dizziness/light-headedness) they can become unconscious or go into shock

Where do you see mirtazepine most commonly used?

Used commonly in oncology and skilled nursing since it helps with sleep at night (dosed QHS) & increases appetite (good for weight gain in frail elderly)

What is the purpose of Coombs Test , Direct?

Used to determine the cause of hemolytic anemia (autoimmune vs drug-induced) and in assessment of transfusion compatibility - Positive in drug-induced hemolysis caused by * penicillins, cephalosporins, Isoniazid, Methyldopa, Nitrofurantoin, Quinidine, Quinine, Rifampin, and sulfonamides , DAPSONE * - If positive, discontinue the drug

Tyrosine Kinase Inhibitors (TKIs) Targeting BRAF: BRAF protein Kinase mutaiton

Vemurafenib (Zelboraf) > WARNING NEW MALIGNANCIES - V600E mutation SE > Dermatolotic disorders (rash, photosensitivity, alopecia, , QT prolongation, etc. Dabrafenib (Tafinlar) - V600E or V600K mutation - Less skin toxicities SE - Pyrexia, etc.

Which patches can be covered with *permitted * adhesive cover which go over the patch to hold it in place?

Warning: Do not cover patches with the exception of fentanyl (Duragesic) or buprenorphine (Butrans), which can be covered only with the permitted adhesive film dressings Bioclusive or Tegaderm

What is so special about Levetiracetam?

Weight Natural - maybe weight loss No significant interactions

Where do you also see the use of Buoproion beside in smoking (Zyban)?

Wellbutrin XL is approved for seasonal affective disorder (SAD) > Start in early fall, titrate to 300mg/day if desired can d/c in late spring by cutting to 150 mg daily for 2 weeks

Do we have to stop NSAID prior to elective surgery?

Yes. stop all NSAIDs at least one week

What would you expect if Lithium level > 1.5 mEg/L ?

coarse hand tremor, vomiting, persistent diarrhea, confusion, ataxia

Apolipoprotein B (ApoB) ?

is an important component of many lipoproteins that are involved in atherosclerosis and cardiovascular disease. Atherosclerosis may be described as a chronic inflammation in the arterial wall.

What important things to know about calcium based binders : Calcium Acetate (PhosLo, Phoslyra, others) Calcium carbonate (Tums, store brands)

1. First line for hyperphosphatemia of CKD 2. SE: Constipation, hypercalcemia / nausea 3. Monitor: Ca2 , phosphorus , serum calcium - phosphorous product, PTH

What are the 3 most common causes of Peptic Ulcer Disease?

1. Helicobacter Pylori (H.pylori)-positive ulcers 2. NSAID-induced ulcers 3. Stress ulcers > common in critical illness and in mechanically ventilated patients.

List drugs that raise potassium levels?

1. ACE inhibitors 2. ARBs 3. Aldosterone receptor antagonists (ARAs) 3. Aliskiren, NSAID, cyclosporine, tacrolimus, everolimus, mycophenolate, potassium supplements, glycopyrrolate, drospirenone-containing oral contraceptives, sulfamethoxazole/trimethoprim, chronic heparin use, canagliflozin, pentamidine, and potassium present in IV fluid, including parenteral nutrition.

Valproate BBW

1. Hepatic Failure > Occurs rarely in adults (1:50,000) usually during first 6 months of therapy > Children (1:600) under the age of two years and patients with mitochondrial disorders are at higher risk. Monitor LFTs frequently during the first 6 months. 2. Teratogenicity > Neural tube defects (spina bifida) and decrease in IQ scores following in utero exposure > Malformations of the face and heart, and nervous system have been reported. 3. Pancreatitis 4. Mitochondrial disease > Increase risk of valproate induced acute liver failure and death in patients with hereditary neurometabolic syndromes.

List 2 top selling Non-Depolarizing NMBAS?

1. Cisatracurium (Nimbex): Short t1/2; intermediate acting 2. Rocuronium (zemuoron) Intermediate acting

Identify the 4 statin benefit Groups

1. Clinical atherosclerotic cardiovascular disease (ASCVD), including coronary heart disease (ACS, S/P MI, stable or unstable angina, coronary or other arterial revascularization), stroke, TIA, or peripheral arterial disease thought to be of atherosclerotic origin. 2. Primary elevations of LDL-C >= 190mg/dL 3. Diabetes and 40-75 years of age with LDL-C between 70-189 mg/dL 4. 40-75 years of age with LDL between 70-189 mg/dL and estimated 10-year ASCVD risk of >= 7.5% (using Global Risk Assessment Tool)

List some important safety consideration with PCA

1. Device can be complex and require set-up and programming. This is significant cause of preventable medications errors. It should be used only with well-coordinated health care teams. 2. Patient might not be a candidate for PCA. they should have a cognitive assessment prior to using the PCA 3. Friends and Family members should not administer PCA doses. This is joint commission requirement 4. PCA do not frequently cause respiratory depression. But risk is there. Especially in * advanced age, obesity, concurrent use of CNS depressant (in addition to higher opioid doses) *

What is HIV?

HIV is RNA retrovirus that attacks the immune system, mainly the CD4+ T-helper cells causing a progressive decrease in the CD4+ T cell count.

What is the equation for Elimination Rate Constant?

Ke = Cl / Vd Ex. If Vd is 50 liters and a Cl of 5 liters/hours then Ke is 0.1hr-1 and 10% of the drug is cleared per hour

Probenecid

MOA > Probenecid (a uricosuric) inhibits reabsorption of uric acid in the kidney, which increase uric acid excretion. Competes with penicillins and cephalosporins in the kidneys, which decrease the beta lactam excretion and increase plasma levels CI > with aspirin > Blood dyscraisis > Uric acid kidney stones > children < 2 y.d > Initiation during an acute gout attack (important) Warning > Increased risk of hemolytic anemia in patient with G6PD deficiency SE > Hypersensitivity reactions > Hemolytic anemia NOTES > Require adequate renal function CI if < 30 ml/min Not recommended if CrCl < 50ml/min

What are the CCB recommended?

Long acting nifedipine , Diltiazem, and amlodipine * No verapamil because its more pronounced negative inotropic effects relative to diltiazem*

What is the function of loop diuretics?

Loop diuretics inhibit the Na+-K+ pump in the ascending limb of the loop of Henle (the part that goes back up). About 25% of the Na is reabsorbed here and inhibiting these pumps leads to a significant increase in the tubular concentration of sodium and less water reabsorption. By blocking the pump, the electrical gradient is altered and reabsorption of calcium decrease. *loop diuretics used long-term deplete calcium and have a harmful effect on bone *

Can you apply 1.62% AndroGel to abdomen?

No. Only upper arms and shoulder but not the abdomen; with the 1% the abdomen can be used. * Avoid showering, swimming, or bathing for at least 2 hours after you apply AndroGel 1.62% *

If patient has a fever of > 38C > 100.4 F? can he apply fentanyl patch?

No. This will cause the drug to be pour out of the patch, resulting in toxicity. With fentanyl and buprenorphine this can be quickly toxic (fatal)

Ranolazine (Ranexa)

Selectively inhibit the late phase Na current; decrease intracellular Ca+2; may decrease myocardial oxygen deman > 500mg po BID (maximum 1000mg BID)

Important about Tacrolimus (Prograf, Astagraf XL, Hecoria) - Protopic - Topical for Eczema

> Initial 0.1 - 0.2 mg/kg/day (depending on transplant type) in 2 divided doses given Q 12 H > Trough levels varies depending on 1. Transplant type 2. Number of months since transplant 3. Concurrent medications

Pseudomonas aeruginosa is an uncommon cause of community-acquired pneumonia (CAP), but a common cause of hospital-acquired pneumonia. True or False.

True

T or F : Like codeine, tramadol is a prodrug. Tramadol is converted by both CYP3A4 and CYP2D6 to O-desmethyltramadol, which is also metabolized by CYP2D6

True

T or F. 100 mEq of potassium (oral or IV) raises K by 1 mEq/mL

True

T or F. A cool mist humidifier helps nasal passages shrink and allow easier breathing

True

T or F. Antibiotics and Probiotics are not taken together and must be seperated by at least 2 hours.

True

For patient with a supratherapeutic INR of 4.5 - 10 without bleeding

Hold 1-2 doses. Monitor INR. resume warfarin at lower dose when INR therapeutic. Vitamin K can be used if urgent surgery needed (<= 5mg, with additional 1-2 mg in 24 hours if needed) or bleeding risk is high (1-2.5mg)

For patients with INR > 10 without bleeding

Hold warfarin. Give oral vitamin K 2.5-5 mg even if no bleeding. Monitor INR. Resume warfarin at a lower dose when INR is therapeutic

Give 1 important example of SV?

Human leukocyte antigen (HLA) have structural variations which are clinically significant. HLAs alert the immune system to target a pathogen for destruction, and certain HLA variants put patients at higher risk for hypersensitivity reactions.

T or F. Brisdelle is Pregnancy Category X , but other forms of paroxetine are pregnancy category D, due to CV risk to the new born.

True

T or F. COC is used as first line treatment to induce regular menses.

True

T or F. Cancer is considered stable if < 25% decrease or increase in tumor size?

True

T or F. No bowel movement after more than 3 days require a digital evacuation.

True

T or F. Non-selective beta blockers agents can decrease insulin secretion in type 2 diabetes

True

T or F. Prior to withdrawing any liquid from a vial, first inject an equal volume of air to the fluid removed (*exception: do not inject air prior to removing cytotoxic drugs from vials)

True

T or F. Rhythm control antiarrhythmics should not be used when AF becomes permanent.

True

T or F. Some communities have locked boxes for controlled drugs - if putting the drug into this container, the drug name must be visible (but not patient info)

True

T or F. Sometimes, surgery is not an option for initial treatment and the treatment regimen begins with chemotherapy. This is called primary induction chemotherapy. I am talking about Cancer.

True

T or F. Stable Angina is considered ASCVD so patient should be put on either high intensity statin if <= 75 or moderate intensity for > 75 years.

True

T or F. The non-benzodiazepines work by acting at the benzodiazepines receptor and acting as GABA agonists.

True

List some Herbals that increase blood pressure?

- Bitter orange - Ephedra (ma-huang) - Ginseng - Guarana - St. John's wort

While giving these drugs for CINV what if patient has anxiety or upper GI symptoms similar to GERD?

Give lorazepam and H2RA for GI

Prinzmetal's Angina

It is presented in patients with normal coronary arteries, where symptoms are caused by vasospasm in the arteries.

List 1 drug that is Non-Selective Beta blocker + Alpha-1 blocker?

Labetalol (Trandate) Dose 100-1200 mg BID

What is the ANTIDOTE for Acetaminophen ?

N-acetylcysteine - Oral - IV (Aetadote)

What could be one of the reasons to switch from Venalfaxine IR to ER?

Nausea and stomach upset

What is the preferred reigmen with Olanzapine?

Olanazepine + Dexamethasone + Palonosteron

What is personalized medicine?

Personalized medicine is the ability to offer: 1. The right drug 2. The right person 3. At the right time 4. with the right dosage

What is HCG used for?

Tested in blood or urine to determine pregnancy

What if patient has adherence issue?

They make take the once monthly dose of oral risedronate or oral ibandronate

What would you give for Anthacycline extravasation ?

Totet

Relative antiinflammatory potency for steroids

betamethasone (0.6mg)/dexamethasone (0.75mg) > methylprednisolone (4mg) / triamcinolone (4mg) > prednisone (5mg)/prednisolone (5mg) > hydrocortisone (20mg)/cortisone (25mg)

5th generation cephalosporin

- Best gram positive activity including MRSA, some gram - same as ceftriaxone (no pseudomonas) - Ceftaroline fosamil (Telfaro)

What are the ANTIDOTE for cyanide?

1- Hydroxycobalamin (cyanokit) > Do not use if not *DARKED RED* 2. Sodium thiosulfate + Sodium Nitrite (Nithiodote) > Poison is caused due to: A. Long treatment or renal impairment with nitroprusside B. ingestion of amygdalin, synthetic form of laetrile

List the 4 types of phosphate binders

1. Aluminum based agents (AlternaGEL) > Aluminum can accumulate in CKD and is toxic to nervous system and maybe cause dialysis dementia. > Use short term if at all > Not commonly used 2. Calcium based agents (primarily calcium acetate and carbonate) > Effective first line agents for hyper phophatemia in CKD > The dose limiting effect is hypercalcemia which is problem especially if patient taking vitmain D (it will increase calcium in blood) 3. Iron-based agents > New agents utilize iron as the cation to bind with phosphate in the gut 4. Aluminum free- calcium free agents > Expensive > because they do not contain aluminum or calcium

*EXAM* Inferential use looks at the value as a way of comparing groups and determining level of significance. A few rules apply when looking at CI for inferential use:

1. When the 95% CI for the estimated *differences* between groups (or within the same group over time) does not include *zero* the results are significant at the 0.05 level 2. When the 95% CI for an * odds ratio, risk ratio , hazard ratio * that compare 2 groups does not include *1* the results are significant at the 0.05 level

Can you list 2 products that are used topically for systemic conditions?

1. Nitro-Bid: Nitroglycerin ointment 2. AndroGel and other hormones

What is the drug therapy consist of?

1. Non-absorbent disaccharides (such as Lactulose) 2. Antibiotics (Rifaximin, neomycin, others) for acute and chronic therapy 3. zinc > Zinc administration has the potential to improve hyperammonemia by increasing the activity of ornithine transcarbamylase, an enzyme in the urea cycle. The subsequent increase in ureagenesis results in the loss of ammonia ions

List some methods to reduce phosphorous in the blood?

1. Restricting dietary phosphorus 2. Phosphate binders bind meal-time phosphate in the gut that is coming from the diet > if a dose is missed and the food is absorbed, that dose should be skipped, and the patient should resume normal dosing at the next meal or snack

What important things to know about Iron based phophate binders? Sucroferric oxyhydroxide and Ferric citrate

1. SE (diarrhea, discolored feces) 2. Monitor phosphorous: iron studies should be monitored with ferric citrate not with sucroferric oxyhdroxide, ferritin and TSAT

BBW 2 for Testosterone C-III

1. Secondary exposure to testosterone in children and women can occur with use of testosterone. Cases of secondary exposure resulting in virilization of children have been reported. Women and Children should avoid contact with any unwashed or unclothed application sites in men using testosterone gel. 2. Injection only: Pulmonary oil microembolism (POME) reactions. CI > Breast cancer > Prostate Cancer SE > Increase in appetite > Increase risk of hepatotoxicity > Sensitive nipples > Acne > Gynecomastia > Dyslipidemia > Increase in Serum Creatine > Reduced sperm count > Sleep apnea > Increase in PSA

If unsure if drug has a new formulation, these questions might be helpful to determine if there is a new formulation..

1. Who typically uses the drug? 2. Would there be a reason to have this type of formulation for this patient population? * These can be useful on exam if unsure if the formulation exists*

Important information about treatment of anemia in CKD

1. Treatment of anemia generally involves combination of erythropoiesis-stimulating agent (ESAs) and Iron supplementation 2. Intravenous iron if preferred over PO for patient on hemodialysis due to poo iron absorption in GI 3. ESA, including epoetin alfa (Epogen, Procrit) and Darbepoetin alfa (Aranesp) are generally necessarily to treat anemia of CKD to prevent the need for blood transfusions. 4. The serum hemoglobin concentration should not be correct to the "normal" concentration for patients without CKD (with CKD they are lower) due to an * increased risk of cardiovascular events, stroke and death *

List 2 agents used to treat secondary hyperparathyrodisim

1. Vitamin D analog which increase absorption of Ca2 and provide negative feedback to the parathyroid gland > Calcitriol (Rocaltrol, Calcijex) > CI: Hypercalcemia, Low vitamin D toxicity 2. Calcimimetic: Increase sensitivity of calcium-sensing receptor on the parathyroid gland, thereby decrease PTH, CA , Phos and preventing progressive bone disease > Cinacalcet (Sensipar) > CI: Hypocalcemia > Caution in patient with history of seizure > * Take tablet whole, do not crush or chew *

Which of the following are the primary insect that cause infection to travelers and they are mosquitos which carry parasite? A. Japanese encephalitis B. Yellow Fever C. Dengue D. Malarai

A B C D

what does Center for Drug Evaluation and Research (CDER) regulate? A. OTC B. RX C. Biological therapeutics D. Generic drug E. Also publish orange book

A B C D E all correct

List medicaitions used for maintenance therapy?

A CNI (Tacrolimus is the 1st line CNI) or cyclosporine PLUS An antiproliferative agent (mycophenolate is 1st line in most protocols), or everolimus, sirolimus, belatacept or azathioproine PLUS/MINUS Steroids (typically prednisone) * If patient is low immunological risk, the steroids will be discontinued; otherwise, they are not and the long-term adverse effects will need to be considered*

Which of the following is safer to give for pregnant woman? A. Bupropion B. Nicotine (patch) C. Nicotine (nasal spray) D. Nicotine (lozenges) E. Nicotine (gum)

A D E (all pregnancy category C) Rest are D

What T-score indicate a normal bone density?

A T-score from -1 and higher indicates normal bone density

The TakeAway mailers are for: A. OTC B. Rx (regular) C. Rx (controlled). Select all that apply

A and B correct

What is Multiple Sclerosis (MS)?

A chronic, progressive autoimmune disease in which the patient's immune system attacks the myelin sheaths that surround the axons in the brain and spinal cord (CNS). As demyelination progresses, the symptoms worsen because the nerves can no longer properly conduct electrical transmission.

What is the difference between Dependent and Independent?

A dependent variable is the outcome of interest, which should change in response to some intervention. An independent variable is the intervention, or what is being manipulated. For example, aspirin is compared against placebo to see if it leads to a reduction in coronary events. The dependent variable (our outcome of interest ) is the number of coronary events while the independent variable (the intervention) is aspirin

Who is qualified to receive Methylnaltrexone (Relistor)

A laxative for constipation due to opioids (It blocks gut opioid-receptors) -> The patient must have failed DSS + Laxative (senna, bisacodyl) > Administer SC QOD

Which of the following is classified as yeast fungi? A. Candida B. Cryptococcus C. Aspgergillus D. Mucor E. Histoplasma F. Blastomyces

A. Candida B. Cryptococcus

Which of the following takes longer time for its effect. A. Inducer B. Inhibitor

A. Inducers > Inhibition of an enzyme is fast and at most takes a few days to take effect and will end quickly when the inhibitor is discontinued. Induction most often requires additional enzyme production, which takes time. The full effect may not be present for up to two weeks. When the inducer is stopped it could take 2-4 weeks for the induction to disappear completely; the enzyme have been produced and will die off based on their half-lives.

Which of the following statements regarding the median is correct? A. It is the value in the middle of ranked list B. It is not appropriate to use the skewed data C. It is not sensitive to outliers D. It is a measure of dispersion E. In a normal distribution, it is the same as the mean

A. It is the value in the middle of ranked list C. It is not sensitive to outliers E. In a normal distribution, it is the same as the mean

What is ARR Absolute Risk Reduction?

Absolute risk reduction, or attributable risk, is the difference between the control group's event rate and the treatment group's event rate ARR = % risk in control group - % risk in treatment group Ex. 16 - 15 = 12. the ARR is 12%. the risk that can be contributed to the intervention (DRUG). This means for every 100 patients treated with drug 12 fewer patients experience heart failure progression.

Why there is a contraindication between Simvastatin, Lovastatin and Pitavastatin?

According to studies > The clearance of simvastatin was reduced in one study of 20 heart transplant patients receiving concurrent cyclosporine.

What is the difference between Active and Passive immunity?

Active immunity is protection that is produced by the person's own immune system. One way is to acquire active immunity is to survive an infection. Another way to produce active immunity is by vaccination. Passive immunity is protection by antibody containing products produced by an animal or human and transferred to a human, usually injection. This protection wanes with time, usually within a few weeks, or months. The most common form of passive immunity is the antibodies an infant receives from the mother. Many types of blood products contain antibody, including IVIG and plasma products.

Why adjuvant agents are given with CNI ?

Adjuvant agents such as steroids , azathioprine or mycophenolate mofetil (CellCept) are given with CNI to reduce the dose of the CNI * to reduce nephrotoxicity risk)

List some agents used for migraine prophylaxis ?

Anti-depressants Anti-convulsants > Do not recommend valproic acid due to safety concerns + its preg category X Vitamins Natural products * A full trial, at a reasonable dose, should be 2-6 months *

What is the significant side effect of Disopyramide?

Anticholinergic effects > 10% (dry mouth, constipation, urinary retention), hypotension, QT prolongation, HF

These medications have high known risk to increase uric acid

Aspirin > Hight dose > 81mg giving daily is okay Diuretics (thiazides, loops) Niacin (big one) Pyrazinamide Ribavirin Tacrolimus and Cyclosporine

What is the duration of steroids used for asthma exacerbations?

Asthma exacerbations are generally treated with steroids for 3-10 days

When to step down if possible after putting patient on asthma medications?

Asthma is well controlled at least 3 months

Non-Drug treatment for Allergic Rhiniits

Avoiding exposure to known or suspected allergens HEPA vacuum cleaner weekly or more often will reduce household allergens - Allergen block > Nasal gels with petrolatum can be applied around the nostrils to physically block pollens and allergens from entering the nose

What is the DOC for chlamydia in pregnancy?

Azithromycin 1 g x 1 or amoxicillin 500mg po TID for 7 days

Which of the following antibiotics used for UTI can cause hyperbilirubinemia and kernicterus in 3rd trimester and is pregnancy D? A. Macrobid B. Bactrim DS C. Cephalosporin D. Monurol

B - Kernicterus is a bilirubin-induced brain dysfunction. Bilirubin is a highly neurotoxic substance that may become elevated in the serum, a condition known as hyperbilirubinemia

How severe iron deficiency can cause Tachycardia and Heart Failure?

The heart will try to compensate for low oxygen levels by pumping faster (tachycardia) and by increasing the mass of the ventricular wall, which can eventually lead of heart failure.

What is the duration of treatment for a provoked VTE? A. 2 months B. 3 months C. 6 months D. 12 months

B. 3 months - If the cause of the VTE is unprovoked, extending therapy longer than 3 months is recommended, as long as the bleeding risk is low-to-moderate. - If patient has 2 episodes of unprovoked VTE, long-term treatment may be warranted.

Which of the following agencies focus on pharmacy ? A. The joint commission B. The pharmacy Quality Alliance (PQA) C. The Agency for Health care Research and Quality (AHRQ)

B. PQA specific goals that involve medications include increasing adherence, avoiding unnecessary or unsafe medication (such as high risk medication in the elderly) and increasing the use of medications indicated for certain conditions

Bupropion

BBW - - Antidepressant increase risk of suicidal thinking and behavior in children, adolescents and young adults (18-24 years of age) with major depression disorder and other psyo disorder CI - Seizure disorder; history of anorexia/bulumia, abrupt d/c of ethanol or sedatives; concurrent use with MAO Is, linezolid, IV methyelen blue, or other forms of bupropion SE - Dry mouth, Insomnia, headache/migraine, N/V, constipation, and tremors/seizures (dose-related), weight loss , etc. - No effects on 5HT and therefore no sexual dysfunction; may be used if issues with other antidepressants

SSRI - Safety/Side effect/etc

BBW - Antidepressants increase the risk of suicidal thinking and behavior in children, adolescents and young adults (18-24 years of age) with major depressive disorder (MDD) and other psychiatric disorders; consider risk prior to prescribing. Contraindications - Concurrent use with MAO Is, Linezoid , IV methylene blue, etc. Side Effects - Sexual side effects - Smnolence, insomnia, nausea, xerostomia, diaphoresis (dose-related), weakness, tremor, dizziness, * headache (but may help with headache of taking continuously) * - Fluoextine can be activation; take dose in AM - SIADH, hyponatremia (elderly at high risk) - Increase bleeding with concurrent use of anticoagulatns, etc. - Increase risk falls

Which of the followings is 80% oil and 20% water? A. Cream B. Lotion C. Ointment D. Paste

C. Ointment Cream is 50% oil and 50% water Lotion is mostly water Paste are the thickest ointments and are also used as protective barriers Ex. Triple paste medicated diaper rash ointment

Which of the following promote a better medication absorption? A. Lotions B. Creams C. Ointments D. Pastes

C. Ointments They are not applied easily to hairy skin

Platinum Based compounds

CARBOplatin (Paraplatin-AQ) Oxaliplatin (Eloxatin) > Not used commonly > In FOLFOX colorectal cancer > Neuropathy exacerbated by exposure to cold

What are the preferred agents for Prinzmetal's angina?

CCB * Make sure to use LONG ACTING *

What is the initial therapy in black hypertensive patients?

CCB and/or thiazide-type diuretic unless they have CKD and proteinuria.

What is CUA?

Cost-utility analysis is a specialized form of CEA that includes a quality of life component associated with morbidity using common health indices such as quality adjusted life year (QALYs) and disability-adjusted life year (DALYs). An advantage of CUA is that different types of outcomes and diseases with multiple outcomes of interest can be compared (unlike CEA) using one common unit, like QALY

Fingolimod (Gilenya)

Contraindication > Recent (within the last 6 months) MI, unstable angina, stroke , TIA, HF requiring hospitalization, or NYHA class III/IV HF; history of 2nd or 3rd degree heart block or sick sinus syndrome (without a functional pacemaker), QT interval > 500 msec, concurrent use of Class Ia or III anti-arrhythmics. Warning - Decrease in heart rate (must monitor), BP, macular edema, infections, Decrease in pulmonary function tests, increase in LFTs Side Effects > Headache, diarrhea, flu-like symptoms, back pain, increase in LFT, cough hypertension, posterior reversible encephalopathy syndrome (PRES- rate but can cause stroke/hemorrhage) Monitoring CBC (baseline and periodically thereafter); ECG (baseline; repeat after initial dose observation period), etc. *Eye Exam * at baseline and 3-4 months after initiation of treatment Note > * Avoid live vaccines until 2 months after stopping treatment * > Blister pack: protect from moisture. > Caution when used with drugs that slow heart rate, monitor continuous ECG overnight after first dose if concomitant use is necessary

Correction factor and Correction dose.

Correction dose; is the amount of additional insulin needed to keep their blood glucose in range. For example if patient going to wedding and wants to enjoy a piece of cake. Knowing how to calculate a correction dose will allow the patient to accurately dose the additional insulin needed for the extra carb load. - Correction factor - 1800 Rule (rapid-acting) 1800 / TDD = correction factor for 1 unit of rapid-acting insulin - Correction factor - 1500 Rule (regular insulin) 1500/TDD = correction factor for 1 unit of regular acting insulin - Correction dose [Blood glucose now - target glucose] / correction factor = correction dose.

Define Correlation?

Correlation describes the relationship between two or more variables which is then plotted on a linear scale. Example how is disease progression and exercise correlated. -> If positive correlation then 2 variables tend to increase or decrease together -> If negative correlation then 2 variables are inversely related -> If 0 correlation then the 2 variables have no relationship or correlation

Agents used to treat VRE. Faecium?

Daptomycin

Define Sentinel Event?

Death or serious injury

What affect Bile Acid have on LDL HDL TG?

Decrease LDL Increase HDL *increse TG* < BAD!! thats why ACC/AHA guidelines do not recommend using these agents when TGs are greater or equal to 300 mg/dL

What decrease WBC?

Decrease due to clozapine Decrease due to chemotherapy that target bone marrow Decrease due to carbamazepine Decrease due to cephalosporins, procainamide, vancomycin

What is the ANTIDOTE for Iron?

Deferoxamine (desferal) Deferiprone (Ferriprox) and Deferasirox (Exjade) for iron overload from blood tranfusion

Important about Denosumab (Prolia)

Denosumab - Monoclonal antibody that binds to nuclear factor kappa ligand (RANKL) and prevents interaction betwen RANKL and RANK (a receptor on osteoclasts), preventing osteoclast formation; leads to decrease bone resorption and increase bone mass. CI > Hypocalcemia (must be corrcted prior to using) > Pregnancy SE > Back Pain > Limb Pain > Dermatitis > Eczema > Rash, Hypocalcemia, increased serum cholesterol NOTE must be given in MD office

What is the drug of choice for bed wetting?

Desmopressin (DDAVP): 0.2 mg PO QHS, can titrate to 0.6mg (maximum) given QHS > Limit fluid intake 1 hour before dose and until next morning

Which non-benzodiazepines is the only sedative approved for use in non-intubated patients?

Dexmetetomidine (Precedex)

What is the ANTIDOTE for Insulin or other hypoglycemic, severe low blood glucose?

Dextrose > Glucagon if unconscious and outpatient > For Hypoglycemia due to Sulfonylurea: Octreotide (SandoSTATIN) may be given with dextrose

Which group qualify for moderate intensity?

Diabetes 40-75 years with LDL between 70-189 mg/dl with estimated 10-year ASCVD risk < 7.5% Clinical atherosclerotic cardiovascular disease (ASCVD) > 75 years

Digoxin

Digoxin (Digox, Lanoxin) > Tablet, solution, injection Antidote: DigiFab CI > Ventricular fibrillation Warnings > Vesicant > Avoid in 2nd or 3rd degree heart block with a functional pacemaker > Avoid in Wolff-Parkinson-White syndrome (WPW) with AFib

Which CCB cause peripheral vasodilation?

Dihydropyridine CCBS > The peripheral vasodilation leads to reflex tachycardia, headache, flushing, and peripheral edema > Always pick amlodipine or Nifedipine ER (less side effects)

What is the initial DOC for Parkinson and is the most effective agent and better tolerate as initial treatment in the elderly ?

Levodopa

Pregnancy and Hypothyrodisim

Levothyroxine is FDA pregnancy category A - TSH elevation in pregnant woman will put children at risk of impairment in their intellectual function and motor skills. - Pregnant women being treated with thyroid hormone replacement will * require 30-50% increase* in levothyroxine dose throughout the course of their pregnancy.

Which patches you apply 12 hours ON and 12 hours OFF?

Lidocaine (Lidoderm) 1-3 patches on for 12 hours, then off 12 hours Nitroglycerin (Minitran, Nitro-Dur): On 12-14 hours/day, off 10-12 hours per day

List 2 drugs that have equal PO and IV dose and this makes it simple to convert from IV (hospital) to PO(outpatient).

Linezolid and Levofloxacin 100% bioavailability

Fish Oils

Lovaza and Epanova contain EPA + DHA SE > Eructation (burping), dyspespia, taste pervesrions (lovaza, Epanova) > Arthalgia (Vascepa) Lipid Effects - Decresae in TG up to 45%, Increase in HDL around 9% - Can increase LDL (up to 44% with lovaza; 25% with Omtryg, and 15% with Epanova), No increase seen with Vascepa.

What is the difference in MOA for Low, Medium and high dose dopamine?

Low dose: D1 agonist (D1 receptor in the kidney cause vasodilation) Med dose: Beta-1 agonist High dose: Alpha 1 agonist

How to diagnose Hypothyrodism ?

Low free thyroxine (Low FT4) (normal 0.9 - 2.3 ng/dL) High thyroid stimulating hormone (High TSH) (normal range 0.3 - 3 mIU/L

Why we use TOBI?

Lung infections occur intermittently at first, but eventually become chronic. * In particular, chronic lung infections with P.aeruginosa are associated with more rapid decline in pulmonary function. * - Inhaled antibiotics like TOBI are used to eradicate P.aeruginosa from the lungs.

What would be a good option to use as mood stabilizer in pregnancy?

Lurasidone

what would you give a patient that comes to ur community pharmacy (Walgreens) and has 1. anxiety 2. neuropathic pain. Looking for one medication that treat both?

Lyrica - * is not fda * approved for anxiety but is useful if a patient has anxiety with neuropathic pain. - Pregabalin has immediate anxiolytic effect similar to BZDs. - Schedule C-V

Compare Prophylactic medications for Malaria?

Mefloquine is started 1-2 weeks prior to travel and taken for 4 weeks post-travel. High resistance in many areas. Side effects including psychiatric symptoms (anxiety, paranoia, depression, hallucination, and psychosis) and neurologic symptoms (seizures, dizziness or vertigo, tinnitus, and loss of balance), can't be used in cardiac conditions however it is an option in *pregnancy* - Atovaquone/Proguanil (Malarone): quick acting 1-2 days before travel and 7 days post travel > SE: Increase in LFT , etc. > Good option in area with Chloroquine-resistan Palsmodium Falciparum - Primaquine: 1-2 days before travel and taken 7 days after travel. watch out for G6DP deficiency prior to initiating treatment with primaquine SE: Hemolytic anemia (in aptient with G6DP defieicney) Aranulocytosis, anemia. > CDC require screening for G6DP defciency prior to initating treatment -Chloroquine: 1-2 weeks before and 4 weeks post. cause *visual problems , must be avoided in pregnancy and should not be used in area with high chloroquine or mefloquine resistance * WARNING > Retinopathy > QT prolongaton SE > GI upset > Visual distrubances > Photosensitivity > Etc. CI > Hypersensitivity to 4-aminoquinoline compounds (e.g, hydroxycholoquine); retinal or visual field changes

What is the indication of Megestrol (Megace) in HIV?

Megestrol tablets treat breast and uterine cancer. Megace® ES liquid is used to increase appetite and prevent weight loss in patients with AIDS.

What are the common drugs associated with PF?

Methotrexate Nitrofurantoin Sulfasalazine Amiodarone

Which stimulant is tried first?

Methylphenidate, which is available in various formulations is tried first or lisdexamfetamine (Vyvanse), the prodrug of dexroamphetamine.

WHAT IS THE DOC for Bacterial Vaginosis caused by many

Metronidazole 500mg po TID for 7 days Metronidazole 0.75% gel 5g vaginally daily for 5 days or Clindamycin 2%

Why we need the drug to be hydrophilic during absorption and Metabolism?

More of the drug will stay in the blood, the blood pass through the kidneys, and the drug can renally excreted

Who should be informed about a medication error in hospital settings?

The hospital's Pharmacy and Therapeutics (P&T) committee should be informed of the error as well as the Medication Safety Committee.

Missed COC pills

Single Pill Missed > If a single pill is missed anywhere in the packet, the forgotten pill needs to be taken when noticed and the next pill is taken when it is due which may mean taking two pills on the same day. No additional contraception is required 2+ Pills Missed > Back-up generally needed if two or more are missed > Woman should take one of the missed active (hormonal) pills as soon as possible and then continue taking one pill each day as prescribed. Depending on when she remembers her missed pill, she may take two pills on the same day > If the two or more pills are missed in the first week of the cycle and unprotected intercourse occurs during this week, use of emergency contraception could decrease the risk of pregnancy > If pills were missed in the last week of hormone pills, days 15 to 21 of a 28 day pack, omit the hormone-free week by finishing the hormone pills in the current pack and start a new pack the next day. If unable to start a new pack, use back-up until hormonal pills from a new pack are taken for seven consecutive days.

Important about Teriparatide injection (Forteo)

Teriparatide Injection > For patients who are at very high risk of fracture, or who have already had a fracture due to osteoporosis, or for GIO or if cannot take other medications > Doe: 20mcg SC daily for max of 2 years BBW > Osteosarcoma (bone cancer) WARNING > Orthostatic hypotension with initial doses, use cautiously if history or current urolithiasis (urinary stones) SE * Hypercalcemia (transient, post-dose), arthralgias, pain, nausea, orthostasis/dizziness and increase in HR * Notes > You may feel dizzy or have a fast heart beat after the first few doses. This usually happens within 4 hours of taking the medication and goes away within a few hours. For the first few doses, inject this medication where you can sit or lien down right away if these symptoms occur > The pen should be kept in the refrigerator. After 28 days, the pen should be discarded even if some medicine left inside. > Inject in thigh or abdomen > * Do not exceed 2 years of use * > PPI may increase fracture risk. You may need to use a calcium citrate tablets.

Which patch is applied daily AT QHS?

Testosterone (Androderm): Nighlty, not on scrotum

What is the difference between genotype and phenotype?

The 2 alleles of a specific gene comprise a genotype. If both alleles are the same, the person is homozygous for that gene or trait. If the alleles are different, the person is heterozygous for that gene or trait. Phenotype is the set of observable traits of an individual that result from his/her genotype.

Who is the provision of the patient safety act and the patient safety rule dealing with PSO operations?

The Agency for Healthcare Research and Quality (AHRQ) administers

Who develops written recommendations for the routine administration of vaccine to children and adults in the civilian population?

The CDC Advisory Committee on Immunization Practices (ACIP)

Which patches dose the DEA permits flushing?

The DEA permits flushing of highly potent narcotic patches (Duragsic, Daytrana, Butrans), since these are dangerous and a child or animal could ingest a fatal amount.

Metreleptin (Myalept)

This is a recombinant human leptin analog used as adjunct to diet to treat the complications of leptin deficiency in patient with congenital or acquired generalized lipodystrophy

What is the advantage of Nascobal?

This is cyanocobalamin (vitamin B 12) Nasal spray * A compound that requires a gut factor for absorption (Intrinsic factor) can be given nasally (vitamin B12)*

When doing bowel prep.. make sure to tell the patient now to consume these?

Do not consume anything with red or blue/purple food coloring (including gelatin and popsicles), milk, cream, tomato, orange, or grapefruit juice, etc.

Can we use 2 biologics concurrently?

Do not use more than one biologic concurrently Do not use live vaccines if using these drugs.

Can you use chantix + nicotine product?

Do not use varenicline with nicotine products due to increasaed side effects

What are the HR goal in patient with A-FIB?

< 80 BPM in patient with symptomatic AFib < 100 BPM may be reasonable in patients who are asymptomatic and have preserved left ventricular function.

T or F. The majority of AE re due to damaging effects on non-cancerous, rapidly dividing cells in the GI tract, hair follicles and bone marrow (blood cells). Thus, N/V, alopecia (turkey 1700$ including hotels) and myelosuppression are common side effects of most chemotherapy

True

T or F. There is a drug absorption with topical administration but the rate of absorption is affected by some factors (ex. open wound that would increase absorption)

True

T or F. Treating opioid dependence with methadone lowers testosterone levels significantly.

True

T or F. You may see a ghost tablet in the feces if you take this medication. Asacol HD

True

T or F. dopamine agonist are often given as the initial option in younger patients ( < 65 years)

True

T or F. it is important to put patient with RA on treatment within 3 months.

True

True or False. Do not block three inches from the back of the hood and leave six inches from the front edge of the hood clear

True

Which testing are required for carbamazepine (Tegretol)?

HLA-B*1502 for Asian patients -> If positive, do not use unless benefit clearly outweighs risk -> Serious dermatologic reactions with positive HLA-B*1502 (TEN, SJS)

T or F. Ziprasidone the SGA has the highest QT prolongation.

True

T or F. mood stabilizers is a medication that can treat either mania or depression without inducing either.

True

Tricyclics

- 3rd amines > Doxepin >> Zonalon cream is used for pruritus >> Silenor is for insomnia > Amitriptyline - 2nd amines > Nortirptyline (Pamelor)

What do people with schizophrenia suffer from?

- Hallucinations - Delusions (false beliefs) - Disorganized thinking and behavior * It has to do with dopamine and glutamine *

Crizotinib (Xalkori) Certinib (Zykadia) which test required/recommended?

ALK (Anaplastic lymphoma kinase) -> If positive use it.

What test must be + to be able to use Imatinib (Gleevec)?

CD117+ (KIT, C-KIT) Patients will response to therapy if GIST is KIT and KIT means CD117+

Zetia

Do not exceed simvastatin 20mg per day when using in combination with Zetia if CrCl < 60 ml/min

Excretion can occur via the A. kidney B. Liver C. Gut D. Lungs E. Skin (sweat) F. All of the above

F

What is the goal of hemoglobin level in SCD when providing blood transfusion?

No higher than 10 g/dL post infusion

Which of the following is immature RBC? A. Reticulocytes B. Erythrocyte

Reticulocytes are immature RBCs which circulate in the blood for 1-2 days before maturing into an erythrocyte.

What is recommended for pregnant woman who may stop ART durign post-partum period?

*- Lopinavir + ritonavir - Kalerta * + 2-NRTI backbone - Atazanavir + ritonavir + 2-NRTI backbone > Once daily lopinavir + ritonavir is not recommended in pregnant women. Always use twice daily dosing regimen.

What would you recommend for a patient who want an item that needs to defecate?

- A suppository (bisacodyl or glycerol) will provide fast relief > Take 30 minutes after a meal because this is the best time; GI peristalsis has increased - Oral stimulatns such as senna, bisacodyl can be added on at QHS as chronic agents.

Important about treatment goals for non-pregnant adult with diabetes - THE ABC's of diabetes

- A- A1C > At least test every 3 months in patients not well controlled (at goal) > Twice per year if at goal and have stable control.

Which chemotherapy agents put patient at high emetic risk (>90% frequency of emesis) IV

- AC (Doxorubicin or Epirubicin) with cyclophosphamide - Cisplatin - Cyclophosphamide > 1500 mg/m2 - Doxorubicin >= 60 mg/m2 - Decarbazine - Epirubicin > 90 mg/m3 - Ifosfamide >= 2g/m2 per dose

List some AEDs that are strong inducers?

- Carbamazepines - Oxcarbazepines - Fosphenytoin - Phenytoin - Phenobarbital - Primidone - Topiramate >= f200mg/d

* Measles *

- Kapolik spots small white spots on the inside of the cheeks and appear 2-5 days prior to rash - Maculopapular rash - Fever -malaise - cough, rhinitis, conjunctivitis (URS)

Important information to know about Activated Charcoal?

- Patient who are unconscious; we should protect their airway to prevent vomiting and aspiration - Activated charcoal is contraindicated when the airway is unprotected or if the ingested compound increase risk of aspiration such as *hydrocarbons*.

Important about another type of Aminosalicylate - Olsalazine (Dipentum)

- SE > Diarrhea, abdominal pain

List the 4 reasons people use ODT and Films?

1. Dysphagia from stroke or other causes 2. Children: Too young to swallow tablets or capsules 3. Nausea: make it difficult to tolerate anything PO 4. Non-adherence: ODT prevent cheeking

Must know about Diphenoxylate/atropine (Lomotil)

CI - Children < 2 years of age, etc. - SE > Sedation, constipation, urinary retention, etc.

Which patches you apply q 72 hours?

Fentanyl (Duragesic) Q72H, if it wears off after 48 hours, change to Q48 hours Scopolamine (Transderm-Scop) Q72H, if needed

Is there a positive correlation or negative correlation between exercise and diabetes disease progression?

Negative correlation

What is a solution for patients who complain of GI problems with oral bisphosphonates?

They may prefer an injectable bisphosphonate

This drug MOA is increase in GABA activity, NMDA receptor blocker and has BBW for hepatic failure and Aplastic anemia. This drug also require an informed consent to be signed by patient and prescriber prior to dispensing.

Felbamate (Felbatol) > Refracotry seizure

What is the SE associated with Rx Linzess and what is the generic name?

- Linaclotide > Diarrhea, etc. BBW * Do not use in patients less than 18 years of age *

Cetuximab (Erbitux) Panitumumab (Nectibix)

- Pharmacogenomics > Must test for EGFR and K-ras mutations before treatment SE - Acne-like rash - Mg and Ca wasting - etc.

What is the first line treatment for patient with Hypovolemic shock?

The first line treatment is fluid resuscitation with crystalloids or colloids

Hypothyroid Treatmen

- 25mcg 50mcg 75mcg 88mcg 100mcg 112mcg 125mcg 137mcg 150mcg 175mcg 200mcg 300mcg Orangutans Will Vomit On You Right Before They Become Large, Proud Giants - Usual Dose > 0.5 mcg/kg/day for elderly > 1.6 mcg/kg/ day in younger patients - If formulations changed, check signs and symptoms and levels in 4-6 weeks - If known CAD, start low 12.5 - 25 mcg daily - BBW > Thyroid supplements are ineffective and potentially toxic when used for the treatment of obesity or for weight reduction, especially in euthyroid patients. High doses may produce serious or even life-threatening toxic effects particularly when used with some anorectic drugs as sympathomimetic amines. CI > Acute MI, Thyrotoxicosis, uncorrected adrenal insufficiency Monitoring > Check TSH levels and clinical symptoms every 4-6 weeks until levels are normal, then 4-6 months later, then yearly. Overdosing put patient at risk of A-fib. Notes Highly protein bound > 99% Pregnancy category A

Rilpirivine (Edurant)

- 25mg daily with meal - Keep in original container CI > ALL PPIs (require acidic) >>> Antacids should be given at least 2 hours before or 4 hours after rilpivirine. > Concurrent use of rifabutin, rifampin, all inducers, etc. Side Effects > CNS (Depression, mood changes, suicidal ideation, insomnia) Notes > Higher rates of failure have been seen in patient with HIV viral load > 100,000 at treatment initiation

Important about Atazanavir ATV (Reyataz)

- 300mg + 100mg ritonavir daily - Take with food and water Side effects - PR interval prolongation - Indirect hyperbilirubinemia (leading to jaundice or scleral icterus - think "bananavir) - Rash - Nephrolithiasis - Cholelithiasis - Increase in CPK

Must know about Loperamide

- 4 mg PO after first loose stool initially; then 2 mg after each subsequent stool; not to exceed 16mg per day - Contraindications > Abdominal pain w/o diarrhea, children < 2 years of age, etc. NOTE > Do not self-treat for > 2 days

Raltegravir , RAL (Isentress)

- 400mg po BID Side effects > N/D , HA , Insomnia, Pyrexia, Increase in CPK, Myopathy and Rhabdomyolysis, Rash (including SJS) Monitoring > CPK (if symptomatic)

Dolutegravir (Tiviicay)

- 50mg po daily (not indicated for weight <40kg) - CI > Co-administration with dofetilide due to risk for significantly increase dofetilide concentration resulting in increased toxicity - Side Effects > Insomnia > Headache > Diarrhea, Rash , Increase in CPK , Increase in LFT among hepB/C patients , Increase in SCr without affecting GFR

Efavirenz EFV (Sustiva) + emtricitabine and tenofovir (Atrilpa)

- 600mg daily - Take on an empty stomach, preferably at bedtime - CI > Concurrent use of ergot alkaloids, midazolam, pimozide, triazolam, voriconazole, and st.john's wort - Side effects > CNS (impaired concentration, somnolence, vivid dreams- usually resolve within 2-4 weeks) and psychiatric symptoms (depression, paranoia, mania, suicide), rash, hyperlipidemia and increase in LFTs. Note > Pregnancy category D >>> Use other ART agents in women of childbearing potential who are planning to become pregnant or who are sexually active and not using effective contraception. For women who present in the first trimester already on an efavirenz containing reigemen and who have adequate viral suppression, efavirenz may be continued as changing regimens may lead to loss of viral control and increase risk of perinatal transmission. Note > Methadone levels can be decreased by Efavirenz and nevirapine. Monitor for signs and symptoms of possible methadone withdrawal. > > Hormonal contraceptive levels may be decreased by efavirenz and nevirapine and result in unintentional pregnancy.

What is pseudo-addiction?

- A drug-seeking behaviour that simulates true addiction, which occurs in patients with pain who are receiving inadequate pain medication - Usually due to poorly controlled pain - Must have patient return to their pain management provider for reassessment and additional help in developing a tailored pain management provider - Most likely the reason is over-reliance on a short acting agent such as a NORCO combination.

What are the recommendations to reduce risk of gout attacks?

- A healthy diet (including low fat dairy products and vegetables), hydration, weight control, smoking cessation and exercise are recommended in the guidelines to reduce risk of gout attacks)

DA-AGONIST (Pramipexol - Mirapex, Mirapex ER) and Ropinirol - Requip and Requip XL

- A slow titration is required (at least 1 week) due to orthostasis, dizziness, sleepiness - SE > Drowsiness, including sudden daytime sleep attacks > Nausea, dizziness, orthostasis > Hallucination, dyskinesias NOTE Renal _ Decrease pramipexol dose if CrCl < 50ml/min

Nicotine Replacement Therapy (NRT)

- ALL OTC EXCEPT Nicotrol inhaler and Nicotrol NS - if < 25: use 2mg gum if >= 25 use 4mg gum - * 1 gum Q1-2 hours for 6 weeks, then 1 gum q 2-4 hours for 3 weeks then 1 gum q 4-8 hours for 3 weeks max 24 pieces/day * - Patches > if more than 10 cig/day, use 21 mg x 6 week, 14mg x 2 week, then 7mg x 2 week > if less or equal to 10 cig/day, use 14 mg x 6 wk, then 7mg x 2 weeks - Lozenge > 1st cig > 30 mins after waking up: use 2 mg lozenge > 1st cig <= 30 mins after waking up: use 4 mg lozenge ( do not exceed 20 lozenges per day or 24 pieces of gum per day * >> same direction as the gum

ARBs

- ARBs block angiotensin II from binding to angiotensin II type-1 (AT1) receptor on vascular smooth muslce, preventing vasoconstriction. Boxed warning - Can cause injury and death to developing fetus; discontinue as soon as pregnancy is detected Contraindication - Angioedema - Do not use in bilateral renal artery stenosis since renal function will worsen. - Do not use concurrently with aliskiren in patients with diabetes. WARNING - Olmesartan only > Sprue-like enteropathy > Severe, chronic diarrhea with substanital weight loss has been reported in patients taking olmesartan months to years after drug initiation SE > Hyperkalemia > Hypotension > Dizziness and HA Monitoring > BP, K, SCr, BUN Notes Preg D

Importanta bout ART

- ART has dramatically reduced HIV-associated morbidity and mortality - The primary GOALs of ART are to: restore and preserve the immune system, suppress HIV viral load to undetectable levels, reduce HIV-associated morbidity, prolong survival and prevent HIV transmission - ART is recommended in ALL HIV-infected individual - ART is also recommended for HIV-infected individuals for the prevent of HIV transmission to non-HIV infected sexual partners. - An adherence of 95% or higher is required in order for ART regimen to be effective.

List some Natural treatment of cancer (can be used for prevention)?

- Beta-carotene (It is the precursor of Vitamin A) - Fish oil - Black or Green Tea - Garlic - Soy - Vitamin A and D - Colon Cancer (calcium) - Prostate Cancer (Lycopene: In cooked tomatoes)

Biologic Agents (Tumor Necrosis Factor (TNFa) Inhibitors (Anti-TNF biologics)

- BBW (2) > Risk for serious infections - some fatal (including active TB, fungal, viral, bacterial, or opportunistic infections); lymphomas and other malignancies; discontinue therapy if a serious infection develops; screen for latent TB and treat before starting therapy - Warning > TNF inhibitors can cause demyelinating disease, hepatitis B reactivaiton, heart failure, hepatotoxicity, lupus-like syndrome, and severe infections. Do not use with other TNF inhibitors or immunosuppressive biologics, or live faccine SE > Infections and injection site reactions (redness, rash, swelling, itching, bruising), positive anti-nuclear antibodies, headache, nausea, Increase iN CPK (adalimumab) Monitoring > TB test (prior to administration and annually), signs and symptoms of infection, CBC, LFTs, HBV (prior to initiation), HF, malignancies. NOTE > Do not shake. Requires refrigeration (biologics will denature if hot). Do not freeze. Etanercept may be stored at room temperature for a maximum of 14 days. Allow to reach room temperature before injecting (15-30 minutes)

zidovudine ZDV or AZT (Retrovir)

- BBW 2 1. Hematologic toxicities (neutropenia and anemia) especially in advanced HIV 2. Prolonged use has been associated with symptomatic myopathy and myositis SE - N/V, skin/nail hyperpigmentation (blue) - Increase in LFT - Macrocytic anemia Note > Do not combine with stavudine (antagonist effect on HIV as both are thymidine analogs: d4T and Azt); erythropoietin is indicated to manage ZDV-induced anemia. > IV zidovudine should be administered in the setting of labor for HIV-infected pregnant women, unless viral load is <1,000 copies/mL

Nitrofurantoin (Macrodantin, Macrobid, Furadantin)

- Bacterial cell wall inhibitor - Bactericidal - Used for uncomplicated UTI (cystitis only) due to E.coli, etc - MacroBID > 100mg po BID x 7 days - Macrodantin 50-100 mg po QID with food for 3-7 days - Macrodantin 50-100 mg po QHS with food for prophylaxis CI CrCL < 60 due to concerns of inadequate urinary concentration and risk of accumulation of neurotoxins

Nevirapine , NVP (Viramune, Viramune XR)

- Bad drug - BBW (2) > Severe Hepatotoxic reactions may occur (liver failure, death) > Severe, life-threatening skin reactions (SJS/TEN) - SE > GI (N/D) > Increasae LFT > Rash (more common with this agent) Notes - Do not initiate therapy in *women* with CD4+ counts > 250 cells/mm3 and in men with CD4+ counts >400 cells/mm3 due to increase risk of hepatotoxicity. Notes > Methadone levels can be decreased by Efavirenz and nevirapine. Monitor for signs and symptoms of possible methadone withdrawal. > Hormonal contraceptive levels may be decreased by efavirenz and nevirapine and result in unintentional pregnancy.

How to treat Acute Variceal bleeding?

- Band ligation or sclerotherapy are recommended first line treatment for bleeding varices - In addition, vasoactive therapy is used to stop or minimize the bleeding by decreasing portal blood flow > Octreotide (Sandostatin) is selective > Vasopression is non-selective - Surgical interventions may be considered if the patient is not responding to treatment or to prevent future rebleeding episodes > Balloon tamponade > TIPS - The addition of antimicrobial therapy ceftriaxone for 5 days to prevent subseuent infection and albumin 1.5mg/kg IV on day 1 and 1mg/kg IV on day 3 are recommended as they are both associatd with mortality benefit - * non-selective beta blockers should be added after resolution of variceal bleeding for 2nd prevention of variceal bleeding recurrence *

Why Progestin is okay to use if Patient has Migraine with Aura but Estrogen is not Okay to use for Migraine with Aura?

- Because estrogen put patient at risk for clots etc.. and giving for Migraine with AURA is danger because Migraine w/ Aura are more risk of stroke and other cardiovascular disease.

What is the generic name for Benlysta?

- Belimumab > IgG1- lambda monoclonal antibody that prevents the survival of B lymphocytes by blocking the binding of soluble human B lymphocyte stimulator protein (BLyS) to receptors on B lymphocytes. This reduces the activity of B-cell mediated immunity and the autoimmune response. WARNING Risk of serious infection including progressive multifocal leukoencephalopathy (PML)

Important notes about Mesalamine

- Best to avoid concomitant use of oral formulations with antacids, H2RAs, or PPIs (interfere with absorption) - * Mesalamine is better tolerated than other aminosalicylates * - Topical mesalamine is more effective than oral mesalamine and seroids for distal disease/proctitis in UC; can use oral and topical together. Topical agents should not be used in proximal disease - Swallow caps/tabs whole; do not crush, chew or break due to delayed release coating

* Penicillins *

- Beta lactams; inhibit bacterial cell wall synthesis by binding to one or more penicillin-binding proteins (PBPs) which in turn prevents the final transpeptidation step of peptidoglycan synthesis in bacterial cell walls. - Exhibit time-dependent killing and bactericidal, except against Entercocci species where aminoglycoside are needed for bactericidal activity - Nafcillin and Oxacillin have enhaned activity against methicillin susceptible staphylococcus aureus (MSSA) - Pipercillin and Ticarcillin have extended gram - coverage including pseudomonas aeruginosa - BBW > Pencillin G benzathine: Not for IV use, can cause cardiorespiratory arrest and death - CI > Augmentin XR: history of jaundice, or hepatic dysfunction; severe renal impairment (crcl < 30 mL/minute) - Can cause positive direct coombs test - Augmentin oral suspention must be refrigerated, amoxil oral suspension is refrigerated to improve taste, but stable for 14 days at room temperature - take ER amox/clauv w/ food - *take ampicillin PO on empty stomach 1 hour before or 2 hours after meals* - Ampicillin IV is compatible with NS only; stable for 8 hours at room temperature - Piperacillin and Ticarcillin have activity against Pseudomonas - Nafcillin is a vesicant - if extravasation occurs, use cold packs and hyaluronidase infections

What is the first line therapies for migraine prophylaxis in adults?

- Beta-Blockers > Propranolol > Metoprolol > Timolol - Valproic acid (many side effects and bbw) - Topiramate (good choice if patient wants to lose weight) - Extendend-cycle contraceptives if pre-menstrual migraine > Or start NSAID or triptan 2 days before menses and continue for 5-7 days

Beta Blockers

- Beta-adrenergic receptor antagonists, or simply beta blockers, antagonize the effects of catecholamiens (especially norepinephrine) at the beta-1 and beta-2 adrenergic receptors - They decrease morbidity and mortality in HF - A beta blocker is recommended for all heart failure patients, especially those in NYHA FC II-IV. - Only carvediolol (IR an ER), metoprolol succinate ER and bisoprolol are recommended in the guidelines. - * Beta blockers with instrinsic sympathomimetic activity should be avoided*

Polymyxins - Concentration dependent killing - Bactericida

- Colistimethate sodium (Coly-Mycin M) > Colistimethate is the inactive prodrug that is hydrolyzed to colistin which act as a cationic detergent and damages the bacterial cytoplasmic membrane causing leak of intracellular substance Warning: Dose dependent nephrotoxicity neurotoxicity > Solution for inhalation must be mixed immediately prior to administration - P olymyxin B sulfate > BBW 6 1. Nephrotoxicity (dose-dependent) 2. Neurotoxicity 3. Safety in pregnancy not established 4. Intramuscular/Intratheical administered only to hospitalized patients 5. Neurotoxicity can cause respiratory paralysis from neuromuscular blockage 6. Should be avoided w/ other drugs that cause neurotoxicity / nephrotoxicity SE > Nephrotoxicity > Neurologic disturbances

Testosterone Formulations

- Comes in IM injections which are painful and require medical visits - The injectable pellets in Testopel require medical visits. The pellets are a little smaller than Tic-Tacs and have the unfortunate tendency to popping out - Striant is a buccal form that is held inside cheek BID and has high tendency of buccal irritation - Androderm patch has a high incidence of skin irritation. - AndroGel is the most popular formulation and are relatively well-tolerated - The risk of early virilization is a boxed warning. Pharmacist needs to counsel men to wash their hands after each application and to be carefull that no one touches the areas of application when wet. - Several topical formulations are made to reduce accidental exposure risk: Axiron is applied to the underarms using an applicator that looks like deodorant. - Fortesta is applied to the thighs with one finger - Naetesto is applied to the nostrils.

Clinical Presentation and Assessment of ADHF

- Common non-cardiac causes include non-adherence with medications or dietary restrictions. - Patients with ADHF present with either worsening congestion and or/hypoperfusion. - Evidence of hypoperfusion include cool extremities, hypotension, decreased renal function, and impaired mental function. - Majority present with Congestion - In all ADHF cases, counsel on the important of medication adherence and optimize drug doses. Beta blockers should only be stopped if hypotenson or hypoperfusion is present.

Important about SulfaSALAine (Azulfidine, Azulfidine EN-tabs, Sulfazine, Sulfazine EC)

- Contraindication > Patients with sulfa or salicylate allergy, GI or GU obstruction. - Side Effects > Headache, Rash, Anorexia, Dyspepsia , GI upset (N/V/D), Oligospermia (Reversible) (all > 10%); folate deficiency Warning > G6DP deficiency NOTE > Can cause yellow-orange coloration of skin/urine > impairs folate absorption may be given 1mg/day folate supplement.

What is the mechanism of action of Altepase?

- Converts entrapped plasminogen to plasmin

List of drugs that can cause Hyperglycemia

- Corticosteroids - Protease inhibitors - Atypical antipsychotics (olanzapine, clozapine, quetiapine) - Niacin - Thiazide and loop diuretics - Statins - Octreotide (in type 2) - Fluoroquinolones - Azole antifungals - Carvediolol, propranolol and possibly other beta-blockers - Cyclosporine, tacrolimus - Interferons - Diazoxide - Cough syrups (sugar content usually modest)

Tyrosine Kinase Inhibitors (TKIs) Targeting Analastic Lymphoma Kinase (ALK)

- Crizotinib (Xalkori) - Ceritinib (Zykadia) > Must test for ALK SE Crizotinib > Edema, N/V/D, constipation, vision disturbances etc. Ceritinib > Interstitial pneumonitits, hepatotoxicity, GI toxicity, QT prolongation, hyperglycemia, bradycardia

Important about Humulin R-500

- Currently, all insulins have a concentraiton of 100 units/mL, except Humulin R U-500 which has a concentraiton of 500 units/mL. This concentration of insulin is for patients requiring a large volume of insulin (>200 units/day). Humulin R U-500 has an onset of action of 30 minutes, has a peak similar to U-100 reqular insulin, and has a relatively long duration of action (up to 24 hours after a single dose)compared to U-100 regular insulin. The prescribed dose of Humulin R U-500 should be expressed in actual units of Humulin R-500 along with corresponding markings on the syrgine the patient is using [e.g., 200 units (0.4 ml]

List a cytotoxic drug that is used for flares as induction therapy (used in severe disease) to treat SLE?

- Cyclophosphamide - Side Effects > Myelosuppression, infections, hemorrhagic cystitis, malignancy, sterility, and teratogenesis > Give MESNA therapy and keep hydrated to prevent hemorrhagic cystitis (bladder)

QUEtiapine

- DOC for psychosis in parkinson - SE - Sedating, orthosasis - Weight gain, increase lipids, and glucose - little risk EPS - Lowest QT prolongation

What you must know about Direct Observed Therapy (DOT)?

- DOT regiment are preferred for regimens dosed 2 or 3 times per week instead of daily - daily dosing is strongly encouraged unless DOT is possible - Patients with TB should be isolated in a single negative pressure room. Healthcare workers should wear N95 respirator mask when entering the room of a patient. - *Isoniazid carriers a risk of neuropathy which can be reduced by taking pyridoxine (vitamin B6) 25-50mg PO daily

Dalbavancin (Dalvance)

- Dalbavancin is a lipoglycopeptide which binds to the D-alanyl-D-alanine terminus of cell wall peptidoglycan prevening corss-linking and interfering with cel wall synthesis - Concentration dependent bactericidal activity - WARNING > Red name syndrome if rapid IV administration - SE > N/D rash and headache - Compatible with D5W only *dalbavancin is given as two doses (one week apart).*

List chelation used to reduce Iron overload from transufions

- Deferoxamine (Desferal) > SE: Ototoxicity, visual impairment, arthralgia, acute respiratory distress syndrome, growth failure, hypersensitivity reaction, injection site reaction, etc. - Deferasirox (Exjade) *BBW: HF, HEPATIC FAILURE, GI HEMOORHAGE/ULCERATION* > Advantage: taken once daily by mouth mixed w/ liquid > SE: Headache, rash, abdominal pain, nausea, arthralgia, visual impairment , hepatic dysfunction, kidney impairment > CI: GI hemorrhage , hepatic injury, acute renal failure

important about Daytrana patch

- Hold patch on skin for 30 seconds and smooth down edges. It should stay on during swimming or bathing. Remove the patch after 9 hours so your child can sleep well at night.

ESAs for chemotherapy

- Epoetin alfa (Epogen, Procrit) - Darbepoetin (Aranesp) - Initiate ESA when Hgb < 10 g/dL and when at least 2 additional months of chemo planned. BBW > ESA increase risk of death, MI, stroke , VTE, thrombosis of vascular access, and tumor progression and recurrence Cancer > ESA decreased overall survival and increased risk of tumor progression or recurrence in clinical studies in patient with some cancers. > Require APPRISE > * ESAs are not indicated when the anticipated outcome is cure * - D/C after completion of chemotherapy course. SE - Hypertension, Thrombosis, etc. Monitoring - Hgb, Hct, transferrin saturaiton, serum ferritin , BP , etc. NOTE - If Hgb increases > 1 g/dL in any 2 weeks interval , decrese dose by 25% for epoetin alfa and by 40% for darbeopetin alfa - MEDGUIDE required

Mammalian target of rapamycin (mTOR) kinase inhibitor which inhibits T-lymphocyte activation and proliferation; may be synergistic with CNIs

- Everolimus (Zortress, Afinitor) BBW > Increase risk of infection; increase risk of lymphoma and skin cancer; reduced doses of cyclosporine are recommended when used concomitantly; increased risk of renal thrombosis may resul tin graft loss; not recommended in heart transplant SE > Peripheral edema, constipation, hypertension, hyperglycemia, hyperlipidemia, delayed wound dhealing, pneumonitits, etc. - Sirolimus (Rapamune) BBW > Increase risk of infection; increase risk of lymphoma; not recommended for use in liver transplantation; not recommended for use in lung transplantation SE > Delayed wound healing, pneumonitis/bronchitis, cough, hyperglycemia, hyperlipidemia, etc. - Belatacept (Nulojix) BBW > Inreased risk post-transplant lymphoproliferative disorder (PTLD), recipients without immunity to Epstein-Barr virus (EBV) are at highest risk, use in EBV seropositive patients only. Increased susceptibility to infection and malignancies. Avoid use in liver transplant patients due to risk of transplant rejection and death. Should be administered under the supervision of a physician experienced in immunosuppressive therapy.

Naloxone

- Evzio auto-injector - Dose > 0.4-4 mg IV/IM/SC; repeat q2-3min PRN; not to exceed 10 mg (0.01 mg/kg) - Will cause an acute withdrawal syndrome (pain, anxiety, tachypnea) in patients physically dependent on opioids) - It has voice directions that can be given by patient in thight through clothing, hold for 5 seconds, if any doubt inject then call 911

What is the test used to look for signs/symptoms of stroke

- F.A.S.T Face (Ask person to smile. dose one side droop or numb? Arms: Ask person to raise both arms. Does one arm drift downward? Speech: Ask the person to repeat a simple sentence. Are the words slurred? Is the sentence repeated Time: If the person shows any of these symptoms, even if the symptoms go away call 9-11 immediately.

What is the MOA of Phenytoin/Fosphenytoin

- Fast sodium channel blockers that stabilize neuronal membrane and reduce seizures by increasing efflux or decreasing influx of Na ions.

What are the diagnosis criteria of prediabetes?

- Fasting plasma glucose (FPG) 100-125 mg/dL OR - 2-hr plasma glucose in the 75-g oral glucose tolerance test (OGTT) of 140-199 mg/dL OR - A1C 5.7%-6.4%

Clinical Presentation of Systemic Lupus Erythematosus (SLE)

- Fatigue, Fever, Anorexia, Weight loss, Muscle aches, arthritis, rash (buttery-fly rash), photosensitivity, and joint pain and stiffness. - Arthritis and cutaneous manifestations are most common, but renal, hematologic and neurologic manifestations contribute largely to morbidity and mortality. Lupus nephritis (kidney disease) develops in over 50% of patients with SLE.

List some natural products used in migraine?

- Feverfew - Willow Bark - Butterbur - Magnesium - Riboflavin (Vitamin B2)

What are the toxicity associated with Digoxin?

- First sign of toxicity are n/v, loss of appetite and bradycardia. Other signs of toxicity include blurred/double vision, altered color perception, greenish-yellow halos around lights or objects, abdominal pain, confusion, delirium, arrhythmia (prolonged PR interval, accelerated junctional rhythm, bidirectional ventricular tachycardia) - * Hypokalemia (K < 3.5 mEq/L), hypomagnesemia, hypercalcemia increase the risk of digoxin toxicity * - Hypothyrodisim can increase digoxin levels.

Importanta bout Quillivant XR suspention

- First, shake the bottle for at least 10 seconds - Measure to the white end of the plunger - The medicine can be stored at room temperature for up to 4 months

Natural Products used in Schizophrena

- Fish Oil > Also used for ADHD and Depression - Do not recommend cod liver oil due to the risk of vitamin A toxicity

Lactulose (Constulose, Enulose, Generlac, Kristalose) this drug is used to treat HE (hepatic Encephalophathy) What is the side effects of this drug? Give me the big once.

- Flatulence - Diarrhea - Dyspepsia - Abdominal discomfort - Dehydration - Hypernatremia (Elevated Na levels in the blood) - Hypokalemia

Which of the following has the fastest onset of action? A. Bisacodyl rectal B. Magnesium salts (MOM) C. Fleet enema (sodium phosphate) D. Miralax

- Fleet enema > Onset of action: 1 to 5 minutes

Gemcitabine (Gemzar)

- Flu-like symptoms during first 24 hours > use tylenol - Prolonged infusion time may increase toxicities SE - Myelosuppression, hepatotoxicity , etc.

Important drug-drug interactions?

- Fluoextine (CYP2D6 inhibitors-strong) > Tamoxifen is widely prescribed to patients with estrogen receptor-positive breast cancer, and it is a prodrug that requires bioactivation by cytochrome P450 enzymes CYP2D6 and 3A4 to generate the active metabolite, endoxifen. - Paroextine = D-D interaction X with Tamoxifen

List of drugs that can cause hypoglycemia

- Fluoroquinolones - Linezolid - Octreotide (in type 1) - Propranolol - Lorcaserin (belviq) - Quinine

WARNING about fluoroquinolones

- Fluoroquinolones can prolong QT interval - Peripheral neuropathy - CNS effect, seiures - Hypo/hyperglycemia - Hepatotoxicity - Photosensitivity/phototoxicity - Use in children should be avoided due to concerns of musculoskeletal toxicity - Cipro is not the first choice medicine in patient under 18 years of age due to a risk of bone and joint problems. however. it is used occasionally on a short term basis for certain conditions

Direct Vasodilators

- Hydralazine , Minoxidil - Minoxidil used for hair growth (OTC) - topical Warning - Drug-induced lupus erythematosus (DILE - dose and duraiton related) SE > Headache, reflex tachycardia, palpitations SE > hair growth (nice)

NRTI for Hepatitis B only!!!! what must know about these drugs?

- For the entire class decrease dose if CrCL < 50 mlmin - BBW > Lactic acidosis and severe hepatomegaly with steatosis which may be fatal > Exacerbations of HBV may occur upon discontinuation, monitor closely > [LamiVUDine (Epivir HBV)]: Do not use Epivir HBV for treatment of HIV (contains lower doses of lamivudine) > [Adefovir (Hepsera) and Entecavir (Barcaclude)]: May cause HIV resistence in patients with unrecognized or untreated HIV infection. - Adefovir (Hepsera): * Use caution in patients with renal impairment or those at risk of renal toxicity * (including concurrent nephrotoxic agents or NSAIDS) - Tenofovir (Viread): Fanconi syndrome, renal insufficiency, osteomalacia and decrease bone density - *Telbivudine (Tyzeka): Increase in CPK* - Note that food reduce AUC of Entecavir (Baraclude) b 18-20%; take on an empty stomach (2 hours before or after a meal) HINT: Barcelona

What can happen as a result of GERD?

- GERD can lead to esophageal erosion , strictures, bleeding and Barrett's esophagus, abnormal cell growth of the esophageal lining due to years of acid reflux which can lead to esophageal cancer

List some risk factors for low bone density

- Genetic factors most important > Caucasian and Asian American women at highest risk - Advanced age - Low bone mineral density evidenced by the T-score - Previous fracture as an adult after age 50, not due to traumatic injury - More than 2 alcoholic per day - Oral or IM glucocorticoid use for >= 3 months at a daily dose of 5mg prednisone equivalent or greater - Excessive thinness - A decline in adult estrogen levels, from menopause, anorexia nervosa, lactation, hypogonadism - Rheumatoid Arthritis and Lupus - Low level of physical activity and adequate nutrition > Low over the life span - Calcium and Vitamin D - Low intake - Smoking

List some medications/diseases that can cause PAH?

- Genetics - connective tissue - Advanced liver disease - HIV - Cocaine - Methamphetamine - Dasatinib (Sprycel) >SPRYCEL may cause high blood pressure in the vessels of your lungs. PAH may happen at any time during your treatment with SPRYCEL. Your healthcare provider should check your heart and lungs before and during your treatment with SPRYCEL. Call your healthcare provider right away if you have shortness of breath, tiredness, or swelling all over your body (fluid retention) - SSRI during pregnancy cause PPHN (new-born)

Important about this Aminosalicylates - Balsalazide

- Giazo is only approved in males (failed to show a benefit in females) - Colazal capsule contents may be sprinkled on applesauce. Beads are not coated, so mixture can be chewed if needed. This may cause staining of teeth/tongue.

Counseling with Nicotine Gum

- Gum should be chewed slowly until a "peppery" or "falvored" taste emerges, then "parked" between cheek and gum to facilitate nicotine absorption thorugh the oral mucosa - It must be parked for 30 minutes or until the taste or tingle goes away - Acidic beverages (coffee, juices, soft drinks) interfere with the buccal absorption of nicotine, so eating and drinking anything except water should be avoided for 15 mins before or during chewing - Patient often do not use enough gum to obtain optimal effects. instruct to use at least 1 piece Q1-2 hours

Important about HAP

- HAP that occurs < 5 days (early onset) after admission is often caused by pathogens similar to that of CAP except for the fact the incidence of *enteric gram-negative bacteria is more prevalent and atypical pathogens (e.g., Legionella, Mycoplasma) are less prevalent. Nosocomial pathogens (MRSA, Pseudomonas aeruginosa) is more prevalent with hospitalization >=5 days (late onset)

What are the most common cause of Hypothyroidism?

- Hashimoto's disease - most common - Iodine deficeincy - Etc - Drugs > Amiodarone > Interferons > Lithium > Carbamazepine > Oxcarbazepine > Nitroprusside > Tyrosine kinase inhibitors - most notably sunitinib - * Myxedema coma is a life-threatening emergency characterized by poor circuation, hypothermia, and require IV thyroid hormone *

Where do you see Iron given IV ?

- Hemodialysis (most common use of IV iron) > The national kidney foundation guidelines state that to achieve and maintain a hemoglobin level of 11-12 g/dL, most hemodialysis patients will require IV iron on a regular basis.

Important information about Hepatitis A

- Hep A is one of the most common vaccine preventable infections acquired during international travel - Person from developed countries who travel to developing countries are at highest risk - * transmitted with a person ingests fecal matter * - Vaccination is recommended for susceptible travelers to countries with high or intermediate risk of hepatitis A. there are two monovalent (hep A only) vaccine (Havrix, vaqta) and a combination Hepatitis A/B vaccine (Twinrix). * immunoglobulin is given with hepatitis A vaccine in some high risk groups *

Important information about Hepatitis B

- Hep B is a DNA virus transmitted with blood or other blody fluid - The risk for travelers who do not participate in high risk behaviors is low - Vaccination for Hepatitis B is extremely important to consider for travelers who plan to have sexual encounters or who want to provide medical work - Vaccine is 3 dose series on a 0,1,6 month schedule - Two vaccines available (Engerix-B, Recombivax HB) and the HepA/B twinrix combination

Notes about Hyperthyrodism

- High FT4 and low TSH - Goiter and Exopthalmos can occur - Causes of Hyperthyrodism > Grave's disease - most common cause > Toxic multinodular goiter > Toxic adenoma > Thyroiditis > Drugs (Iodine, amiodarone, interferons, too much thyroid hormone) Clinical Signs/Symptoms > Heat intolerance or increased sweating > Weight loss (or gain) > Agitation, nervousness, irritability, anxiety > Palpitations and tachycardia > Fatigue and muscle weakness > Frequent bowel movement or diarrhea > insomnia > Light or absent menstrual periods > Goiter > Thinning hair > Tremor > Exopthalmos > Dipolpia

The difference between Hihg potency FGA and Low potency FGA?

- High potency FGAs such as haloperidol are associatd with a high risk of EPS effects , a moderate risk of sedation an low risk of orthostatic hypotension, tachycarida, and anticholenergic compared to low potency FGAs. - In contrast, low potency FGAs, are associated with a lower risk of EPS, high degree of sedation, a high risk of orthostatic hypotension , tachyardia and high risk of anticholineric effect.

There are 3 categories of transmission-based precautions defined by CDC. What you must know about the third one (Airborne Precautions)?

- Intended to prevent transmission of infectious agents that remain infectious over long distances when suspended in the air. - Patient should be placed in an airborne infection isolation room (AIIR). An AIIR is a single-patient room that is equipped with special air and ventilation handling pressure rooms. The air is exhausted directly to the outside or re-circulated through HEPA filtration before return - Healthcare personnel wear a mask or respirator (N95 levels or higher), depending on the disease, which is donned prior to room entry - * Airborne precautions are recommended for patient with rubella virus (measles), varicella virus (chickenpox) or M.tuberculosis. *

There are 3 categories of transmission-based precautions defined by CDC. What you must know about the second one (Droplet precaution?

- Intended to prevent transmission of pathogens spread through close respiratory contact with respiratory secretions - Single patient rooms are preferred. If not available, keep >3 feet spatial separation and drawing a curtain between beds is especially important for diseases transmitted via droplet - * Healthcare personal wear a mask (a respiratory is not necessary) for close contact with the patient. The mask is donned upon room entry * - Droplet precautions are recommended for patient with Active B. pertussis, influenza virus, adenovirus, rhinovirus, N. meningitides , and group A streptococcus (for the first 24 hours of antimicrobial therapy)

Bleomycin

- Intercalating agent blocking topoisomerase II - Not myelosuppressive - Maximum lifetime dose of 400 units due to pulmonary toxicity risk BBW - Pulmonary fibrosis, anaphylaxis SE - Hypersensitivity reaction, pulmonary reactions (10%) - such as pneumonitis, which may progress to pulmonary firbosis, mucositis, hyperpigmentation, fever, chills

Itravenous (Parenteral) Iron Supplementation

- Iron Dextran (INFeD, Dexferrum) - Sodium Ferric GLuconate (Ferrlecit) - Iron Sucrose (Venofer) BBW (Iron Dextran only) . Risk of anaphylactic reactions. A test dose should be given to all patients prior to first therapeutic dose. Fatal reactions have occurred even in patients who tolerated the test dose. History of drug allergy and/or concomitant use of ACE inhibitor may increase risk SE - Hypotension, chest tightness, peripheral edema, risk of anaphylaxis with all agents (especially with dextran which require test)

RIBAVIRIN for hepatitis C only!! Things you must know abour RIBAVIRIN Brand names: Copegus, Rebetol, Ribasphere, Ribasphere RibaPak, Virazole

- It is indicated for HCV in combination with interferon alfa, simeprevir, and/or sofosbuvir. Aerosolized ribavirin has been used for respiratory syncytial virus - *medguide required* - BBW (4) 1. Significant teratogenic effects 2. Monotherapy not effective and should not be used 3. Hemolytic anemia (most likely within first 4 weeks) 4. Caution with inhalation formulation because precipitation of drug may interfere with ventilation - CI in CrCl < 50 mL/min - SE > Hemolytic anemia - may worsen cardiac disease - Pregnancy category X - Avoid pregnancy in female patients and female partners of male patient during therapy and for 6 months after completing therapy. - At least 2 reliable forms of effective contraception must be utilized during treatment and during the 6 months post-treatment follow-up period - always take with food

Xtandi (Enzalutamide)

- It used with CYP 2C8 inhibitor, reduce dose to 80mg daily CI - pregnancy SE - Fatigue, insomnia, weakness, diarrhea, edema, pulmonary infections, *seizure in 1% * MOA > Block the downstream transfer of information from the activated receptor. Use din patients with castration-resistant metastatic prostate cancer who failed docetaxel chemotherapy.

What are the 3 BBW for all NRTIs?

- Lactic acidosis - Severe Hepatomegaly with Steatosis - Steatosis: infiltration of liver cell with fat.

Carbidopa/Levodopa

- Levodopa is a precursor of dopamine. Carbidopa inhibit dopa decarboxylase, preventingperipheral metabolism of levodopa - If switching from levodopa IR to levodopa-carbidopa CR, dosage should be substituted at an amount that provided around 10% more of levodopa/day - It comes in ODT formulation > Dissolve on the tongue w/o water - SR tab can be cut in half - 70-100 mg of carbidopa is required to inhibit the peripheral conversion (dopa decarboxylase) and to decrease nausea CI - MAOIs within last 14 days, narrow angle glaucoma SE - Dizziness, Nausea, orthostasis, vomiting, dry mouth - Dyskinesias (abnormal movement), dystonias (painfull) - can cause brown, black, or dark urine, * positive coombs test* Note - Possibility of unusual sexual urges - Separate from iron, possibly separate from protein - may increase uric acid

Thyroid Storm

- Life-threatening medical emergency characterized by deocompensated hyperthyrodisim that can be precipitated by infection, trauma etc. - Treatment PTU + SSKI or Lugol's + Propranolol + Dexamethasone + Aggressive cooling with Tylenol and cooling blankets and other supportive treatment (e.g, antiarrhythmics, insulin, fluids, electrolytes, etc.)

* Telavancin (Vibativ) *

- Lipoglycopeptide and derivative of vancomycin - Same MOA of vancomycin - inhibit bacterial cell wall synthesis by blocking polymerization and cross-linking of peptidoglycan by binding to the D-Ala-D-Ala portion of cell. - Addtional MOA since it is lipophilic , it disrupt membrane potential and change cell permeability - BBW (3) > Fetal risk; must obtain pregnancy test prior to starting therapy > Nephrototixicy > Study showed that mortality was higher VS Ae in patient with moderate-to-severe renal impairment (CrCl <= 50ml/min trated for HAP/VAP - Prolong QT interval - Rapid IV administration cause red man syndrome - SE > Metallic taste N/V

List some Long acting formulations used in Schizophrenia?

- Long acting injections > Haldol decanoate used every 4 weeks - Risperdal Consta > Used every * 2 weeks * - Invega Sustenna > Used every 4 weeks - Ability maintena > Used every 4 weeks

Why non-benzodiazepins are preferred over benzodiazepines?

- Long term non-benzodiazepines are preferred over benzodiazepines due to a decreased risk of physical dependence and less daytime cognitive effect

Pharmacology of COPD

- Long-term bronchodilators are more effective and more convenient than treatment with short acting inhaled bronchodilators. - Combining bronchodilators of different pharmacologic classes may improve efficacy and decrease the risk of side effects compared to increasing the dose of a single agent - Long-term mono-therapy with oral or inhaled corticosteroids is not recommended in COPD

Important information about Aminoglycoside (AMGs)

- MOA: Bind to 30S and 50S ribosomal subunits resulting in defective bacterial cell membrane - Exhibit concentration-dependent killing - Have post-antibiotic effect (PAE) - The above 2 points are the reasons Aminoglycosides are dosed once daily - Extended Interval dosing has also been shown to decrease nephrotoxicity relative to traditional dosing - Cover gram - (Psedumonas) - Gentamicin and Streptomycin used synergy in treating gram + (Staph and Enterococcus) in setting of Endocarditits in combination with beta-lactam or vancomycin - BBW 1. Neurotoxicity 2. Nephrotoxicity 3. Fetal harm if given in pregnancy 4. Nephrotoxicity if used with other nephrotoxic agents > Nephrotoxic agents (Amphotericin B, Cisplatin, colistimethate, cyclosporine, loop diuretics, NSAID, radiocontrast dye, tacrolimus and vancomyin - Monitor Peak and Trough > Peak 1/2 hour after end of drug infusion of 3rd dose > Trough level right before 3rd dose - Pregnancy D - Amikacin (IV,IM) - NO PO - Streptomyci: ONLY IM - Dose on TBW unless total body weight > 130% of IBW then use adjusted body weight - Peak levels > Gentamicin for gram - 5-10 mcg/mL trough < 2mcg/ml > Gentamicin for gram + 3-4 mcg/ml trough < 1mcg/ml >Tobramycin 5-10- mcg/ml trough < 2 mcg/ml > Amikacin 20-30 mcg/ml trough < 5mcg/ml

What is significant about Quinupristin/Dalfopristin (Synercid)?

- MOA: Binds to different sites on the 50S bacterial ribosomal subunit inhibiting protein synthesis - Bactericidal - Side effects > Arthralgias/myalgias (up to 47%) > Infusion reactions, including edema and pain at infusion site (up to 44%) > Phlebitis (40%) > Hyperbilirubinemia (up to 35%) > GI upset > CKP elevation and increase in LFT NOTES > Compatible in D5W only!

What can be used as second line therapy for CINV

- Marijuana - Dronabinol (Marinol) and Nabilone (Cesamet) can be used as second line agents. - These are synthetic analogs of delta-9 tetrahydrocannabinol a natruallyoccuring compoenet of canabis sativa (marijuana)

What is the treatment for Intracerebral Hemorrhage?

- Measures should be taken to lower the ICP - Elevate head of the bed by 30 degrees and use Mannitol - If thrombocytopenia: give appropriate factor or platelet respectively - * prophylaxis of anticonvulsants medications should not be used *

List some factors that cause Stress Ulcer?

- Mechanical ventilation - Coagulopathy - Sepsis - Traumatic brain injury - Burn patients - Acute renal failure - High dose corticosteroids

What type of drugs do we give for PAH?

- Medications include prostacyclin analogues which cause vasodilation. These drugs may be given by continuous IV infusion, infusion under the skin, inhalation or as oral therapy. Endothelin receptor antagonist block endothelin, a vasoconstrictor. Phosphodiesterase-5 inhibitors w/ low doses and a soluble sGC stimulators relax the blood vessels in the lungs.

Hormone Therapy (HT)

- Menopause means that menses has ceased for 12 months - Many women experience vasomotor symptoms as their ovaries produce less estrogen. A decrease in estrogen causes an increase in LH which can result in hot flashes and night sweats.

Maintenance therapy for UC and CD - Aminosalicylates - not recommended for CD

- Mesalamine > Apriso, Asacol HD, Delizcol, Pentasa, Lialda are all long-acting - Suppository > Rowasa - enema CI > Hypersensitivity to salicylates or aminosalicylates or any component of the formulation Warning > Apriso contains pheylalanine- avoid in PKU SE > Abdominal pain, nausea, headache, etc.

Methotrexate Drug Interactions

- Methotrexate should not be taken with alcohol; this combination increase risk of liver toxicity - Active transport renal elimination is decreased by aspirin, beta-lactams, probenecid and NSAIDs, resulting in toxicity. Avoid concurrent use - Sulfanomides and topical tacrolimus increase AE of methotrexate. Avoid concurrent use - Methotrexate can decrease effectiveness of loop diuretic; loop diuretic can increase the concentration of methotrexate concentration. Use caution if using these agents concomitantly. - Methotrexate and cyclosporine concentration will both increase when used together AVOID!

Potassium chloride

- Micro-K: capsules may be opened and content sprinkled on a spoonful of applesauce or pudding and imeediately swallowed without chewing - K-Tab, Kaon-Cl, Klor-Con > Swallow whole, do not crush, cut, chew or suck on ablet - Klor-Con M > Swallow whole, do not crush, chew or suck on tablet. > Tablet may also be cut in half and swallowed separately or cna dissolve the whole tablet in 4 oz of water - stir for 2 mins and drink immediately

What cause migraines and what is the rule of Triptan drugs in migraine?

- Migraines can be caused by changes in the Trigeminal nerve and imbalances in neurotransmitters, including serotonin, which decrease during a migraine causing a chemical release of neuropeptides that trigger vasodilation in cranial blood vessels. - * Triptan drugs are serotenin-receptor agonists that cause vasoconstriction of cranial blood vessels *

What are the common drugs used for Narcolepsy?

- Modafinil (Provigil) > This one contain the R and S - Armodafanil (Nuvigil) > the R-isomer of modafinil SE > dizziness, headache, anxiety, agittation, *rash* , etc > Because of rash they require MEd-GUIDE

Non-Drug (Lifestyle) Treatment What do you instruct patients with heart failure?

- Monitor and document body weight daily > AM and after voiding - Notify provider if heart failure symptoms worsen or when weight increase > 2-4 lb gain in 1 day 3-5 in 1 week and other symtpoms of HF - Maintain Na restriction of less than 1500mg/d for most patients with stage A and B (not enough evidence in stage C and D) - Maintain fluid restriction to 1.5 - 2 L/day in * stage D *especially in patients with hyponatremia - Stop smoking and limit alcohol and avoid illicit drug use - Obtain pneumococcal polysaccharid and annual influenza vaccinations. > Consider Tdap and zoster ( for 60 and up) - Consider wt reduction to < 30 BMI - Exercise

What are the 1st line agents used to treat mania and maintaining a stable mood?

- Mood stabilizers (lithium, valproate, lamotrigine, carbamazepine), and the SGAs are first line for treating and maintaining a stable mood (pre-venting the patient's mood from swinging into mania or depression.

Trazodone

- Mostly used for sedation - off label for sleep - BBW - Antidepressant increase risk of suicidal thinking and behavior in children, adolescents and young adults (18-24 years of age) with major depression disorder and other psyo disorder - SE - sedation - Priapism (medical emergency - require immediate medical attention if painful erection longer than 4 hours ) - * Do not exceed 450mg daily in anyone* > Seizure risk

Platinum-Based Compounds

- Non-cell specific: Cross-links DNA, leading to apoptosis > Programmed cell death - CISplatin > Nephrotoxicity is managed with vigorous hydration and sometimes mannitol, electrolyte wasting requires magnesium and potssium supplementaiton. Amifostine (Ethyol) may also be used propylactially for renal protection. > Recommend audiogram prior to each cycle to screen for ototxicty. Manage acute and delayed N/V with 3 drug combination antiemetic regiemn BW (CISplatin) > Anaphylactic like reaction cumulative renal toxicity, ototoxcitiy, caution against cisplatin overdose (doses < 100 Q3-4 weeks are rarely used and should be verified) CI Cisplatin > Pre-excisiting renal impariement, myelosuppression, hearing impairemnet SE - Neuropathy, myelosuppression, etc

Non-Nucleoside Reverse Transcriptase Inhibitor (NNRTIs)

- Non-competitive binding to reverse transcriptase and blocking the RNA-dependen and DNA-dependent DNA polymerase activities including HIV-1 replication. - All NNRTI can cause rash including SJS/TEN (monitor for erythema, facial edema, skin necrosis, blisters, tongue swelling) and hepatotoxicity. - No renal dose adjustments - Delavirdine is CYP-inhibitors - Efavirenz is an inducer and an inhibitor of the cytochrome P450 system - Rest are inducers except Rilpirivine which is only substrate of cyp3A4.

List drugs used to treat DRY MOUTH ?

- Non-drug treatment for dry mouth includes salivary stimulation, using sugar-free chewing gum (with xylitol) and lozenges, and daily rinses with antimicrobial mouthwash. - Glycopyrrolate is used to decrease excessive salivation; this may be used in a few conditions such as MG (maybe dry mouth from this) - Pilocarpine (Salagen) - Pilocarpine opthalamic (Isoptocarpine, Pilopine HS) is used for glaucoma - Cevimeline (Evoxac) CI > Uncontrolled asthma, narrow-angel glaucoma, severe hepatic impairment (for Pilocarpine) and Acute iritis (Evoxac) SE > Diaphoresis

Buspirone

- Not controlled, no abuse orphysiological dependence potential - Safety/side effect/monitoring > Nausea, dizziness,headache, lightheadedness, excitement. - Avoid use if severe kidney or liver dysfunction - pregnancy category B

What is occasional anxiety?

- Occasional anxiety can occur in the general population when faced with challenging issues at work, home, or school - The symptoms of occasional anxiety dissipate once the issue is gone.

SSRI - Selective Serotonin Reuptake Inhibitor

- Olanzapine + Fluoextine = Symbac > Used for resistant depression - Flouextine > Long half life - Sarafem (Fluoextine) > used for PMDD 20mg every day of menstrual cycle or 20 mg daily starting 14 days prior to menstruation through 1st full day of bleeding - Fluvoxamine has more drug interactions > That's why only approved for seasonal effective disorder - Setraline also used for PMDD - Citalopram > * 2011 FDA warning not to use > 40mg/day due to QT risk * > Max 20mg/day in poor CYP 2C19 (elderly) metabolizers or concurrent use of CYP2C19 inhibitors -Escitalopram (Lexapro) > Do not exceed > 20mg/day due to QT risk > Maximum 10mg/day in CYP 2C19 poor metabolizers.

OTC and Alternative Medications

- Omega-3 polyunsaturated fatty acid (PUFA) > Good to use in II-IV symptoms to decrease mortality an cardiovascular hospitalizations. - Avoid using products containing ephedra (ma huang) or ephedrine - Avoid NSAIDs (including COX-2 inhibitors) - Hawthorn and coenzyme Q10 may improve heart failure symptoms based on small studies.

5-HT3 receptor antagonists

- Ondansetron (Zofran, Zuplenz film) - Granisetron (Sancuso transderamal patch) - Dolasetron (Anzemet) - Palonosetron (Aloxi) - Single max IV dose for zofran is 16 - Single max PO dose for zofran is 24mg - Sanucos patch > Apply to upper arm 24-48 hours before day 1 of chemo, leave on at least 24 hours after last chemo session > Can be worsen up to 7 days Contraindication - Concomitant use of apomorphine (Apokyn) - Do not use dolasetron IV for acute CINV due to QT prolongation SE - HA, fatigue, dizziness, constipation Note - Give 30 minutes before chemo (Except patch)

CloZAPine (Clozaril)

- Only use if failed to respond to treatment with 2 standard AP or had significant ADRs. One of the AP must be 2nd generation - BBW > Arganulocytosis > Seziures - dose correlated > Tachycarida, orthostatic etc. > Myocarditits and cardiomyopathy, d/c if present - SE - Orthostatis , sialorrhea (hypersalivation), etc. Monitoring - REMS - Patient must register with Clozaril Registery. Only pharmacies using Registry can fill this drug - To start: WBC must be > 3,500 and ANC > 1500/mm3 - check WBC and ANC weekly for 6 months then every 2 weeks for 6 months then monthly Note - *smoking reduce drug levels *

H2RAs Notes

- Onset 30-45 minutes and last 4-10 hours - Decrease dose if CrCl < 50 ml/min (famotidine, ranitidine, nizatidine); CrCl < 30 ml/min (cimetidine) - * Acid suppressive therapy (H2AR and PPI) increase risk of GI infections (including C.difficile associated diarrhea and may increase risk of pneumonia in hospitalized patients * - Avoid all H2RAs in elderly with delirium, dementia, cognitive impairment due to risk of adverse CNS effects per Beers criteria. - Avoid cimetidine entirely due to drug interactions and side effects - Cimetidine is CYP450 inhibitor

Why is Dabigatran (Pradaxa) better than warfarin?

- Oral direct thrombin inhibitor - Does not require blood test to monitor for effectiveness - Is not subject to food interactions - Has few drug interactions * Disadvantages * > Significant dyspepsia/gastritis > Increased risk of GI bleeding compared to other oral anticoagulants.

Treatment of Iron Deficiency Anemia > Oral Iron Therapy

- Oral iron therapy (ferrous sulfate) can adequately treat patients with iron-deficiency anemia, except for patients on hemodialysis - Ferrous iron (Fe2) is absorbed more readily than the ferric (Fe3) form - Treatment should be continued for 3-6 months - SR formulations or EC formulation of iron are not recommended as initial therapy because they reduce the amount of iron that is present for absorption in the duodenum - Absorption of iron is enhanced in an acidic gastric environment. Administration with vitamin C 200mg may enhance absorption to a minimal extent. - Food will decrease the absorption of iron. Best if taken 1 hour before meals. > People still take iron with food because they experience GI upset (NAUSEA) which is ok.

Important about Biologic Agents

- People taking this medicine should not get live vaccines. Make sure your vaccines are up to date before starting this medicine. You can continue to take your annual influenza shot (but not the nasal mist vaccine, since it is a live vaccine) - These drugs will lower the ability to fight infections. Patient should be tested for tuberculosis before and during treatment. - This medicine has the possibility of causing liver damage. call healthcare provider right away if you have any of these symptoms: feel very tired, skin or eyes look yellow, poor appetite or vomiting, pain on the right side of y our stomach (abdomen) - Store the medication (single-use syringe or multiple-use vial) in the refrigerator (etanercept may be stored at room temeprature for a maximum of 14 days). Allow the medicine to warm to room temperature before injecting (takes 15-30 minutes). Do not shake the med. before using, check for particles or discoloration. If either is present do not use the medicine. Injectors require protection from light prior to administration. - For entanercept (Enbrel) virals for reconstitution: when reconstituting Enbrel powder from the multidose vial, some foaming is normal. The final solution should be clear and colorless, with no particulate matter.

Drug Treatment of Hypothyroidism

- Per guidelines, levothyroxine (T4) is the drug of choice and current recommendations encourage the use of a consistent preparation for the patient to minimize variability from refill to refill - Other treatment options include liothyronine (T3, Cytomel and Triostat) or desiccated thyroid (T3 and T4, Armour Thyroid). - Levothyroxine should be taken with water 60 minutes before breakfast or at bedtime 3 hours after last meal.

Lifestyle recommendation for patients with low bone density

- Perform weight-bearing exercise (such as walking, jogging, Tai-Chi) - Muscle-strengthening exercise (weight training, yoga) - Taking adequate vitamin D and calcium - Stopping smoking and avoiding secondhand smoke - Reducing alcohol intake and adopting fall prevention strategies listed in the previous card.

List some drugs that worsen DEMENTIA

- Peripheral anticholinergics (including incontinence and IBS drugs) - Central anticholinergic (benztropine) - Antihistamines and antiemetics - Antipsychotics - Barbiturates - Benzodiazepines - Skeletal muscle relaxants - Other CNS depressants

List some drugs that cause Parkinson disesae

- Phenothiazines - Haloperidol -2nd generations Antipsychotics Risperidone at highter dose and Invega (Paliperidone) - Dopamine blocking agent that is used for gastroparesis - Reglan (metoclopramide) > Most likely will cause parkinson when it is overdosed, especially in elderly patient who require a dose reduction with renal dysfunction

* Must know about Hyperkalemia Risk *

- Potassium is renally cleared; severe renal disease cause hyperkalemia by itself - The largest increase is expected with aldosterone blockers (spironolactone and elepernone) because aldosterone regulate potassium excretion - the AHA has issued recommendations to minimize the risk of hyperkalemia in patients treated with these agents which includes avoiding use if the potassium is high at baseline > 5 mEg/L and monitoring renal function - Additive potassium accumulation: ACEIs, ARBs, aliskiren, amiloride, triamterene, epeleronone, spirolonactone, salt substate (KCL) and the drospirenone- containing oral contraceptive. - Additional drugs that can cause or worsen hyperkalemia include calcineurin inhibitors (Tacrolimus and cyclosporine), canaglifozin, etc.

What is the MOA of benzodiazepines?

- Potentiation of GABA - They are differentiated by onset of action and duration of activity

1. Binds the KCNQ voltage-gated K channels, enhancing the M-current and suppressing seizure activity 2. BBW (Retinal abnormalities that can progress to vision loss in 33% of patients after 4 years of treatment 3. WARNING: skin discoloration, urinary retention, and QT prlongation 4. NOTE: Require eye exam and monitoring of QT interval and it will cause urine discoloration * Used in refractory seizure *

- Potiga > Generic is Ezogabine

Alpha blockers

- Prazosin (minipress), Terazosin (Hytrin), Doxazosin (Cardura, Cardura XL) - Warning > Can cause significant orthostatic hypotension, and syncope, especially with the first dose, restarting the dose, rapidly increase the dose, or initiation with another anti-hypertensive agent or PDE-5 inhibitor. > Intraoperative floppy iris syndrome has occured in cataract surgery patients who were on or were previously treated with an alpha-1 blocker

List some steroids that are used in UC and CD ?

- Prednisone > Dexamethasone 0.75 mg is equal to 5 mg of prednisone. Therefore, setting up a ratio, the correct dose of prednisone is 40 mg daily. - Side Effects > Short term: Increase appetite/weight gain, fluid retention, emotional instability, etc. > Long term: Adrenal suppression/Cushing's syndrome, immunosuppression/impaired wound healing etc. NOTE (all steroids) > For acute flare management: steroids should not be used long term - however some patients are on it for long term > If used long term: access bone density (consider use of bisphosphonates, optimize calcium and vitamin D intake) > If used longer than 2 weeks, must taper (over 3-4 weeks) to avoid withdrawal symptoms > May use alternative day therapy ADT to decrease adrenal suppression and adverse effects - Budesonie > Undergoes extensive first pass metabolism; lower systemic exposure than other oral steroids > Entocort indicated for mild-moderate Crohn's invovling the ileum or ascending colon; Uceris indicated in UC only > Swallow whole - Do not crush, chew or break

Immunotherapy - Ipilimumab (Yervoy)

- Primarily autoimmune system unchecked BBW - Fatal immune mediate reactions SE - Dermatologic (rash, pruritus)

When to take these enzymes?

- Prior to meals and snacks - Full dose given before meal and 50% of the mealtime dose is given with snacks - High fat content require higher doses - Do not chew > Can be sprinkled on soft food with a low ph that does not require chewing - There is a powder formulation that can be taken entirely at the beginning of each meal or snack along with a generous amount of liquid - * Do not substitute pancreatic enzyme products. This is an FDA recommendations. They do not require refrigerations. * SIDE EFFECTS - mucosal irritation, abdominal pain, nausea, headache, neck pain

What are the drugs that most commonly associated with drug-induced lupus erythematosus (DILE)?

- Procainamide - Hydralazine (alone, and in BiDil) - Isoniazid - Quinidine - Methyldopa - Propylthiouracil - Methimazole - Minocycline - Terbinafine - Anti-TNF agents

ADR to Progestin

- Progestin can cause breast tenderness, headache, fatigue or changes in mood. If late cycle breakthrough bleeding occurs a higher progestin dose may be required.

Sodium Glucose Co-Transporter-2 (SGLT2) Inhibitors

- SGLT2 is expressed in the proximal renal tubules which is responsible for the majority of the reabsorption of filtered glucose from the tubular lumen. By inhibiting SGLT2, these agents reduce reabsorption of filtered glucose and lowers the renal threshold of glucose which increase urinary glucose excretion. - Canagliflozin (Invokana) > 100mg prior to first meal of the day, can increase to 300mg daily CrCL 45-60 ml/min: 100mg max dose CrCL < 45 ml/min: Do not use - Dapagliflozin (Farxiga) > 5mg daily in the am; can increase to 10mg daily CrCL < 60 ml/min: do not use -Empagliflozin (Jardinace) CrCl< 45 do not use CI > Severe renal impairement (CrCL < 30ml/min) ESRD, or on dialysis WARNING - Increase in LDL - Dapagliflozin can increase risk of bladder cancer SE Decrease weight by 4-7 pounds Increase in K with canagliflozin Etc. Drug interactions > Monitor digoxin levels if taking digoxin concurrently due to Increase in AUC of digoxin.

What are the reasons to use Sublingual (SL)?

- SL and buccal absorption has a faster onset than a tablet or capsule that is swallowed; the drug is readily absorbed in the venous circulation right under the absorption site (under the tongue). Less of the drug is lost to gut degradation and first pass metabolism. Example: - Saphris - Asenapine SL tablet (Antipsychotic) - Fentanyl patches > Onsolis: Fentanyl buccal film > Subsys: Fentanyl SL spray > Actiq : Fentanyl buccal lozenge > Abstral : Fentanyl SL tablet

What are the classic symptoms of CF?

- Salty testing skin - poor growth and weight gain - thick and sticky mucus production - frequent lung infections - coughing and SOB - digestive symptoms > msteatorrhea, malnutrition due to poor absorption of nutrients, including fat soluble vitamins.

Neutropenia Drugs

- Sargramostim (Leukine) > Limited to use in stem cell transplanation - Filgrastim (Neupogen) - Pegfilgrastim (Neulasta) - Tbo-filgrastim (Granix) SE Filgrastim/pegfilgrastim/tbo-filgraastim > Bone pain, fever, generalized rash, injection site reaction Sargramostim > Fever, bone pain, arthralgias, myalgias, rash, dyspnea, peripheral edema, pericardial effusion, cardiovascular edema, HTN, chest pain Important notes > administer 24-72 hours after chemo > Must document when pegfilgrastim was given; it should not be given within 14 days before or 24 hours after chemo. All other should not be given ebfore or within 24 hours after chemo

Side effects of First-Generation Antipsychotics

- Sedating > Low potency 1st generation cause less EPA and high somonolene (chlorpromazine) > High potency 1st generation cause more EPA and less sedating (but still sedating) haloperidol - Dystonias > Painfull muscle contraction > High risk in young male > consider centrally acting anticholenergic (Benadryl, benztropine) for prophylaxis during therapy initiating - Akathisia > restlessnes with anxiety and inability to stay still > Treatment with anticholinergics, benzodiazepines or propranolol - Parkinosim > treat with anticholinergic or propranolol - TD (Tardive dyskinesais) > High risk in elderly females > Stop the drug ASAP and replae with SGA with low EPS risk (quetiapine, clozapine). TD can be irreversible - Dyskinesia

Important information about Tolvaptan (Samsca)?

- Selective AVP antagonist V2 - Use for up to 30 days due to hepatotoxicity - BBW (2) > Should be initiated and re-initiated in a hospital under close monitoring of serum Na > Overly rapid correction of hyponatremia (> 12 mEq/L/24 hours) associated with osmotic demyelination syndrome (life-threatening) - CI > Patients who are unable to sense or respond appropriately to thirst, urgent need to raise Na, hypo hypo, concurrent use with strong 3A4 inhibitors, anuria - Warning > Hepatotoxicty ( avoid use > 30 days and in liver disease/cirrhosis)

Antiestrogens/SERMs

- Selective Estrogen Receptor Modulators (SERMs) are estrogen antagonists in breast tissue, but act as estrogen agonist in some other tissue including bone. - These are used for breast cancer in hormone receptor tumor (estrogen/progesterone). Most SERMs are used in post-menopausal women, except tamoxifen which is indicated in pre- and post- menopausal women and in men.

MAO Is

- Selegiline transdermal patch > EMSAM > No dietary issues with 6mg patch BBW - Antidepressant increase risk of suicidal thinking and behavior in children, adolescents and young adults (18-24 years of age) with major depression disorder and other psyo disorder Warning - Not commonly used but watch for D-D and D-Food interactions - if missed could be fatal - Hypertensive crisis can occur when taken with TCAs, SSRI, SNRIs, many other drugs and tyramine rich foods

Why Meperidine (Demerol) is bad?

- Short duration of action (pain control for max 3 hrs) - Avoid as agent for chronic pain management and even short-term in elderly. Warning > Renal impairment/elderly at risk for CNS toxicity Notes Serotonergic

Important notes about Tacrolimus

- Should be taken on an empty stomach to avoid variability in absorption. If taking with food, take consistently (higher fat food decreases absorption the most) - Do not interchange XL to immediate release. Conversion IV to PO IR is 1:4. Start oral dosing 8-12 hours after last IV dose.

Class III anti-arrhythmic agents

- Significantly prolong the refractory period - Most drugs in this class act thorugh blockage of potassium channels - Ibutilide is the exception and words by activating late inward of Sodium current which result in a significant increase in refractory perioid - * Amiodarone and Dronedarone also block alpha- and beta-adrenergic receptors and calcium and sodium channels. - Sotalol also has significant beta-adrenergic receptor blocking activity

Some drugs that cause or worsen heart failur

- Some chemotherapeutic drugs such as anthracyclines > Doxorubicin (Adriamycin, Doxil), daunorubicin (Cerubidine, DaunoXome) etc. - Tyrosine kinases inhibitors > lapatinin and sunitin etc - Aphemtamines and other sympathomimetics - Non-dihyropyridines calcium channel blockers - Antiarrythmic drugs > Do not use class I antiarrhythmic - Avoid itraconazole for non-life threatening infecitons - Immunomodulators, including inerferons, TNF inhibitors, ritximab - NSAIDs, including celecoxib - Steroids - Triptans - TZD due to edema - Excessive alcohol > mild use might have cardiovascular benefit

Important about POP start Day options

- Start at any time. Use another method of birth control for the first 48 hours of progestin-pill use - protection begins after two days. All come in 28 days packs and all pills are active - * POPs need to take exactly around the same time of day everyday, if 3 hours have elapsed from the regular scheduled time, back up is needed for 48 hours after taking the late pill. If a dose is missed, patient could be pregnant and EC may be suitable. *

* Sulfanomides *

- Sulfamethoxazole (SMX) interferes with bacterial folic acid synthesis via inhibition of dihydrofolic acid formation from para-aminobenzoic acid and trimethoprim (TMP) inhibits dihydrofolic acid reduction to tetrahydrofolate resulting in inhibition of enzymes of the folic acid pathway. - Individually they are bacteriostatic, however collectively they are bactericidal - Active against some opportunistic pathogens (Nocardia, Pneumocystis, Toxoplasmosis) - Single Strength (SS) 400mg smx/80mg TMP - Double Strength (DS) 800mg smx/160mg TMP CI > Sulfa allergy > Pregnancy > breastfeeding, anemia due to folate deficiency > Market renal or hepatic disease, infants < 2 months of age - SE > N/V/D > Anorexia, skin reaction (rash) > Crystalluria (take with 8oz of h2o) > Photosensitivity > QT prolongation > Decrease folic acid, positive coombs test

Important abot Dabigatran (Pradaxa)

- Swallow capsule whole - Do not break , chew, crush or open - Do not put in NG tube - Take the missed dose ASAP unless it is within 6 hours of next scheduled dose; do not double up - Keep in original container. Discard 4 months after opening the original container. Keep bottle tightly closed to protect form moisture - * blisters packs are good until the date on the pack (usually 6-12 months)

Conversion (Dabigatran)

- Switching from warfarin to dabigatran > Discontinue warfarin and start dabigatran when INR < 2 - Switching from UFC/LMWHs to dabigatran > Start dabigatran <=2 hours before the next scheduled dose of LMWH or at the time of discontinuation of UFH infusion - Switching from dabigatran to parental anticoagulants: > Wait 12 horus (CrCl>=30 ml/min) or 24 hours (CrCl < 30 ml/min) ater the last dose of dabigatran before starting therapy - Switching from dabigatran to warfarin > start warfarin 3 days before stopping dabigatran when CrCl >= 50ml/min - Start warfarin 2 days before stopping dabigatran when SrCl 30-50 ml/min - Starting warfarin 1 day before stopping dabigatran when CrCl 15-30 ml/min

Carbamazepine

- TEGretol, TEGretol XR, Carbatrol, Epitol - Equetro - for bipolar - Therapeutic range 4-12 mcg/mL BBW - Serious skin reactions including SJS and TEN > All Asians MUST be tested for HLA-B*1502 - Aplastic anemia and arganulocytosis > Monitor CBC, Platelets , and differential prior to and during therapy; discontinue if significant bone marrow suppression occurs. CI - Bone marrow suppression, hypersensitivity to TCA - MAO inhibitor within past 14 days (Since same as TCA) - etc.

Lithium (Lithobid)

- TR: 0.6 - 1.2 mEq/L (trough level) - Cannot use with renal impairment: Lithium is 100% renal cleared - and if not eliminated, toxicity will result - Side Effects > GI upset (take with food in the stomach can change to ER) > Cognitive effects, cogwheel rigidity, fine hand tremor, weigh gain > polyuria, polydipsia, Hypothyroid: must monitor serotonergic, avoid co-admin with other sertonergic agents > Cardiac abnormalities (inverted T waves) > Edema, worsening of psoriasis

Thiazolidinediones (TZDs)

- TZDs are peroxisome proliferator-activated receptor gamma (PPARy) agonists causing increase peripheral insulin sensitivity (Increase uptake and utilization of glucose by the peripheral tissues; insulin sensitizers) - Pioglitazone (Actos) > +metformin (Actoplus Met, Actoplus Met XR) > + glimepiride (Duetact) > + alogliptin ) (Oseni) - Rosiglitazone (Avandia) > + metformin (Avandamet) > + glimepridie (Avandaryl) BBW > May cause or exacerbate heart failure in some patients CI > NYHA class III/IV heart failure Warning > Avoid pioglitazone in patients with active bladder cancer SE > Peripheal edema, weight gain, URTIs, > Macular edema, HF, Increase fracture risk, Increase LFT, pioglitazone has increased risk of bladder cancer when used for more than 1 year > Desirable side effects of pioglitazone include Increase in HD, Decrease in TG and total cholesterol

Saquinavir (Invirase)

- Take with food (or within 2 hours of a full meal) - Must be given with Ritonavir CI > Severe hepatic impairment, prolonged QT interval, and refractory hypokalemia or hypomagnesemia SE > N/V/D, HA, PR, and QT interval prolongation (Avoid use if QT > 450 msec) Monitoring ECG (baseling and ongoing), electrolyte (K and Mg)

QuiNIDine - Tablet, Injection

- Take with food or milk to decrease GI upset - BBW > Quinidine may increase mortality in treatment of AFib or flutter; control AV conduction before initiating. Anti-arrythmic drugs have not been shown to enhance survival in non-life threatening ventricular arrhythmias and may increase mortality; the risk is greatest in patients with structural heart disease. - Side Effects > Diarrhea (35%), stomach cramping (22%), lightheadedness, QT prlongation, nausea, vomiting, anorexia, cinchonism (tinnitus, hearing loss, blurred vision, headache, delieiurm) rash, positive coombs test (risk for hemolytic anemia), hemolysis risk in G6DP positive patients, drug-induced lupus erythematosus (DILE) NOTES - Different salt forms are not interchangeable - Avoid changes in Na intake. Decrease Na intake can increase quinidine levels - Alkaline foods may increase qunidine levels

Tamoxifen (Soltamox) , Fulvestrant (Faslodex), Raloxifene (Evista) , Toremifene (Fareston)

- Tamoxifen increase risk of endometrial cancer - Tamoxifen cause hot flashes as side effect and in this cause do not use Fluoxetine and paroxetine, instead use Venalfaxine for hot flashes > Tamoxifen is cyp 3A4 substrate 2C9 and 2D6 BBW - Uterine malignancy (tamoxifene); increase risk of thromboembolic events such as dVT , PE, stroke (tamoxifen , raloxifene), QT prolongation (Torefemene) CI - Taxmosifene: DVT?PE, concomitant warfarin therapy - Raloxifene: DVT/PE, pregnancy , breastfeeding - Toremifene: QT prolongation, hypokalemia, hypomagnesemia SE - DVT/PE , menopausal symptoms, hot flashes, flushing, N/V, edema , weigh gain, ypertension, mood changes, amenorrhea, vaginal bleeding/discharge, skin change, cataracts (tamoxifene)

Important about ART therapy

- Tenofovir should be used with caution in patients with renal insufficiency - Atazanavir should not be used in patient who require > 20mg omeprazole equivalent per day - Elvitegravir/combicistat/tenofovir/emtricitabine should be only initiated in patient with CrCL >70 ml/min and should not be used with other ART or with other nephrotoxic drugs. - Abacavir should not be used in patient who test positive for HLA-B 5701. When combined with dolutegravir-lamivudiine, abacavir can be used in patient with pre-treatment HIV viral load > 100,000 copies/mL - Rilpirivine is not recommended if CD4 < 200 cells/mm3. Use of proton pump inhibitor is contraindicated with rilprivine.

What is the MOA of SubstanceP/Neurokin-1 receptor antagonist in preventing N/V?

- They inhibit the substance P/neurokinin 1 receptor, therefore augmenting the antiemetic activity of 5HT3 receptor antagonists and corticosteroids to inhibit acute and delayed phases of chemotherapy-induced emesis SE - Dizziness, fatigue, constipation, weakness, hiccups Notes - Aprepitant increase the concentration of corticosteroids

List some methods that improve automated dispensing cabinet (ADC)?

- The joint commission requires that the pharmacist review the order before the mediation can be removed from the ADC for a patient, except in special circumstances such as emergency which can override but these override must be investigated. - Bar-code improve ADC safety. This means that if nurse wants to remove a medication from the cabinet, she/he should match the drug by the patient's MARs via bar-coding prior to removal of drug from cabinet - LASA should be stored in different locations within the ADC - * Certain medications should not be put in the ADC such as Insulin, Warfarin, and high-dose narcotics (such as hydromorphone 10mg/ml and morphine 20mg/ml) - Returned medications should be placed in a seperate drawer. never to put back in ADC - ADC must be in area that doesn't have a busy, noisy environment, or in one with poor lighting.

Define Breakpoint?

- The level of MIC at which a bacterium is deemed either susceptible or resistant to an antibiotic. Breakpoint vary for different antimicrobial classes. Breakpoints are established by the FDA and clinical and Laboratory Standards Institute (CLSI) and can change based on clinical data in order to optimize antimicrobial therapy and patient outcomes

Myelosuppression Overview

- The lowest point that WBCs and platelets reach occur about 7-14 days after chemotherapy - RBC nadir is much later due to their long life span (- 120 days) - WBC and platelets generally recover 3-4 weeks post treatment - All treatment given Sub-Q

Immunizations and Antibiotics in SCD

- The major cause of death for children with SCD under 5 years of age is infeciton. - Bacterial proliferates and causes increased risk of infection, including septicemia with encapsulated bacteria (S. pneumonia, H. influenza, Salmonella spp) - Prophylactic antibiotics (Penicillin) has been shown to dramatically reduce the mortality associated with invasive pneumococcal infection in young children and should be initiated at age 2 months and continued minimally until age 5 years.

Warfarin - Pharmacodynamic Drug Interactions

- The most common pharmacodynamic interactions are with NSAIDs, Antiplatelet agents, other anticoagulatns, SSRI SNRI > These interactions increase bleeding risk but the INR may be in the usual range or slightly elevated - Drugs that increase clotting risk (including Estrogen and SERMs) should be discontinued.

Estrogen-Progestin Use: Health Risks/Considerations for Use

- The most effective therapy for vasomotor symptoms (hot flashes, night sweats) is estrogen, which causes a decrease in LH and consequently more stable temperature control. - In women with a uterus, estrogen should not be given alone because this will put patient at elevated risk for endometrial cancer. Risk is 5X higher.

Non-Dihydropyridine CCBs

- The non-dihydropyridines consists of verapamil and dilitiazem and are used primarily for arrhythmias to control/slow HR, and sometimes for HTN and angina. > They are negative inotropes (decrease contraction force) and negative chronotropes (decrease HR) > The difference from Dihydropyridine is that these agents inhibit Ca2+ ions from entering the "slow" channels or voltage-sensitive areas of vascular smooth muscle and myocardium during depolarization, resulting in coronary vasodilation.

PT/INR to monitor what?

- The normal PT 10-13 seconds - The normal INR <1.2 for healthy person not on warfarin and 2-3 for person on warfarin - It is used to monitor warfarin

How the body compensate when left ventricular systolic function continue to decline which results in decrease CO?

- The structural changes in shape and composition of the myocardium and activation of neurohormonal system. - Activation of the sympathetic nervous system and the renin-angiotensin-aldosterone system > Endothelin and Vasopressin (anti-diuretic hormone) are also activated results in vasoconstriction or increased systemic vascular resistance (SVR) > > this vasoconstriction helps maintain blood pressure and perfusion to vital organs. - Sympathetic (adrenergic) activation also increases heart rate and contractility which augments cardiac output. - Aldosterone increase sodium and water retention which increase pre-load in an attempt to increase stroke volume and cardiac output.

Voriconazole (VFEND)

- Therapeutic range: 1-5 mcg/mL - Take 1 hour before or 1 hour after meals (empty stom) - Suspension: shake for 10 seconds before use - Do not refrigerate WARNING > Qt prolongation: Correct K/Ca, MG before start Side Effects > Visual changes (around 20%) > Increase Serum Creatine > CNS toxicity (hallucination) Monitoring > LFT, renal function, electorlytes, visual function, CBC, trough concentraiton NOTES > Caution driving at night due to vision changes > Avoid direct sunlight More information > More active against Aspergillus species, C. glabrata, and C. krusei, compared to itraconazole/fluconazole

GLP- AGONIST

- These agents are analogs of glucagon-like peptide-1 (GLP-1) which increase insulin secretion, decrease glucagon secretion, slow gastric emptying, improve satiety, and may result in weight loss. these are incretin mimetics. - - All can be stored for 28-30 days except Trulicity which can be stored at room temperature for up to 14 days - - All can be taken without regard to meals except Byetta which should be given within 60 minutes before the morning and evening meal BBW > For all except byetta: Thyroid C-cell carinomas seen in rats - unknown if this could happen in humans. Contraindicated in patients with a personal or family history of medullary thyroid carcinoma (MTC) or patients with multiple endocrine neoplasia syndrome type 2 (MEN2) WARNIN > Pancreatitis > Use caution with moderate renal impairmen, not recommended in severe impairment (CrCl < 30ml/min) (Byetta-Bydureon) SE > Nausea , weight loss (2-6 pounds) etc. NOtes > Exenatide is a synthetic version of exendin, a substance found in Gila monster saliva. BYETA > May enhance the anticoagulatn effect of warfarin, monitor INR

List some Direct Thrombin INhibitors

- These are agents directly inhibit thrombin (Factor IIa); they bind to the active thrombin site of free and clot-associated thrombin - Argatroban - Bivalirudin (Angiomax) >*Indicated for patients with ACS undergoing PCI and are at risk for HIT * - Desirudin

What is the rule of Guanfacine and Clonidine in ADHD?

- They are used as add-on to the stimulant after the stimulant was tried and additional benefit was needed. - They can also help w/ sleeping since they are sedating.

Topiramate

- Topamax sprinkle > May be swallowed whole or opened to sprinkle the contents on a small amount (around 1 teaspoon) of soft food (drug/food mixture should not be chewed; swallow immediately) - CI > Trokendi XR only - recent alcohol use (within 6 hours prior to and 6 hours after dose) Warnings - Metabolic acidosis - Oligohydrosis - Nephrolithiasis - Acute myopia and secondary angel closure glaucoma - Hyperammonemia - Visual problems (reversible) Side Effects - Somnolence, dizziness, difficulty with memory, difficulty with concentration/attention, cognitive problems, psychomotor slowing, paresthesias, weight loss, anorexia, mood changes, *decrease sodium bicarbonate concentration*, vitamin D and calcium deficiency (bone loss) Note - Pregnancy category D - > cause cleft lip and palate in newborn - Supplementation with calcium and vitamin D recommended.

Prostate Cancer

- Typically patients are started on Luteinizing Hormone-Releasing Hormone (LHRH) agonist for chornic suppression of testosterone - Antiandrogen are started 1-4 weeks prior (to help mitigate the tumor flare) to the LHRH agonist. - Many technicians will discontinue Anti-androgen a week or more after starting the LHRH agonist. - PS level are checked routinely - Docetaxel/prednisone are used in metastatic disease

Heparin [MOA and Safety concerns]

- UFH binds to antithrombin (AT) and inactivates thrombin (Factor IIa) and Factor Xa (as well as factors IXa, XIa, XIIa, and plasmin) and prevents the conversion of fibrinogen to fibrin Safety Note - Heparin lock-flushes (HepFlush) are used to keep IV lines open (patent), not used for anticoagulation. - There have been fatal errors, especially in neonates, made by choosing the incorrect heparin strength. - Heparin injection 10,000 units/mL and heparin flushes 10 or 100 units/ml LASA. > Using higher dose to flush a line could cause fatal hemorrhage.

What is the issue with codeine and Ultra-Rapid metabolizers?

- Ultra-rapid metabolizers may have exaggerated response due to extensive conversion to morphine metabolite. Over-production of morphine can result in increased in CNS effects, including an increase risk of respiratory depression.

What are the 2 BBW with Itraconazole?

- Use is CI for treatment of onychomycosis in patients with ventricular dysfunction or a history of HF - Co-administration with itraconazole can cause increase plasma concentrations of certain drugs and lead to QT prolongation and V-tachyarrhythmias

Azithormycin role in CF

- Used to decrease inflammation and reduce exacerbations; not an FDA-indicated use * azithromycin may have indirect actions, including anti-inflammatory, in addition to the standard antibacterial properties* - clinical trial > CONCLUSION: Azithromycin treatment was associated with improvement in clinically relevant end points and should be considered for patients with CF who are 6 years or older and chronically infected with P aeruginosa.

List drugs that work on the M-phase?

- Vinca alkaloids - Taxanes

What cause increase in Lymphocytes?

- Viral infection - Lymphoma * Decrease in bone marrow suppression, HIV, or due to corticosteroids *

List some natural products used in Dementia

- Vitamin E > Doses > 150 IU carry risk - Ginkgo Biloba > 8 year old study did not find benefit > Earlier studies the use of ginkgo provided modest benefit - * Ginkgo can increase bleeding risk * - Huperzine A is being used for dementia with promising efficacy - Low serum vitamin D levels have an increased risk of developing Alzheimer's disease.

Important about non-benzodiazepines

- Warning > Increase risk of mortality due to interference with brathing at night, causes accidents/falls, confusion, and may increase risk of infection and cancer > Potential for abuse and dependence - Side Effects > Somnolence > Dizziness, ataxia > Lightheadedness > Pins and needles feeling on skin > * May cause parasomnias *

* Infusion Inhibitor *

- We currently have one - MOA: > Fusion inhibitors block the fusion of the HIV-1 virus with the CD4+ cells by blocking the conformational change in gp41 required for membrane fusion and entry into CD4+ cells. - Enfuvirtide (Fuzeon) > Not metabolized via CYP (no drug interactions) > 90mg SC BID - SIDE EFFECTS > Local infection site reactions in almost 100% of patients (pain, erythema, induration, nodules, and cysts, pruritus, ecchymosis) Increase risk of Bacterial pneumonia, hypersensitivity reactions (rare) NOTE > Reconstituted solution should be refrigerated and used within 24 hours.

When do you stop the treatment of PCP for patient with HIV+?

- When CD4+ count is > 200 cells/mm3 for > 3 months on ART

MEGLITINIDES

- Work by stimulating insulin secretion from the pancreatic beta cells * Do not use with Sulfonylurea due to similar MOA * - Repaglinide (Prandin) > Take 15-30 minutes before meals - Nateglinide (Starlix) > Take 1-30 minutes before meals SE > Hypoglycemia > Mild weight gain > URTI

Warfarin Use - Key points from Chest 2012

1. In a healthy outpatients, the initial starting dose of warfarin should be 10mg daily for the first 2 days, then adjust per INR values. 2. For patients with stable therapeutic INRs presenting with a single subtherapeutic INR value, routinly briding with heparin is not recommended. 3. Routine pharmacogenomic testing is not recommended 4. For patient with consistently stable INRs on warfarin therapy, INR testing can be done up to every 12 weeks rather than every 4 weeks 5. For patient with single out of range INR <= 0.5 below or above , continue with same regimen and obtain INR within 1-2 weeks 6. Routine use of Vitamin K supplements is not recommended 7. * Start warfarin therapy at the same day as the parenteral anticoagulant and continue both anticoagulants for a minimum of 5 days and until INR is 2 or above for at least 24 hours. Once the INR is therapeutic for 24 hours, the parenteral anticoagulant can be discontinued.

What are the risk associated with smoking during pregnancy?

1. Spontaneous abortion 2. Low birth weight 3. sudden infant death -> If smoke 5 or less: behavioral support -> if smoke more than 5: Bupropion (CategoryC) and other sources recommend nicotine replacement in pregnancy however the efficacy is not as high in non-pregnant patient. Nicotine gum and lozenges are category C

What is the recommended folic acid supplement for any woman planning to conceive and all women of child-bearing age?

400-800 mcg/day to help prevent birth defects of the brain and spinal cord (neural tube defects. Folic acid should be taken at least one month before pregnancy, since it takes time to build up adequate body stores.

Vasodilators (Nesiritide (Natrecor)

A recombinant B-type natriuretic peptide that binds to vascular smooth muscle, increase in cGMP resulting in smooth muscle cell relaxation which cause vasodilation. - Nesiritide provides both arterial and venous vasodialtion with increase in CO CI - Persistent SBP < 100mmHg prior to therapy, cardiogenic shock SE Hypotension, SCr Monitoring BP, SCr, BUN, urine output Notes - Limited experience with infusions lasting longer than 96 hours.

What is root cause analysis (RCA)?

A root cause analysis is a retrospective investigation of an event that has already occurred which includes reviewing the sequence of events that led to the error. The information obtained in the analysis is used to design changes that will hopefully prevent future errors.

Pegloticase (Krystexxa)

A pegylated form of uricase, an enzyme which converts uric acid to allantoin (an inactive and water soluble metabolite of uric acid); it does not block uric acid formation - This medication is used for refractory cases only - IV BBW > Anaphylactic reactions require pre-medicate > Discontinue if UA is > 6mg/dL CI > G6PD deficiency SE > Antibody formation, gout flare, infusion reactions, nausea, bruising, urticaria, erythema, pruritus >> use NsAID or colchicine as prophylaxis (1 week prior to infusion and continue for at least 6 months: prophylaxis of acute gout flares

What si the primary goal of therapy when it comes to MS?

A primary goal of therapy must be prevention of disease progression; what is lost in neuronal function cannot be regained.

What to do if you miss a dose of Rivaroxaban?

Administer the dose as soon as possible on the same day as follows: > For patient receiving 15mg BID: Take rivaroxaban immediately to ensure intake of 30mg rivaroxaban per day. In this particular instance, two 15mg tablets may be taken at once. Then continue with the regular 15mg twice daily intake as recommended on the following day. > For patient receiving 20,15,10 once daily: Take the missed rivaroxaban dose as soon as possible on the same day; otherwise skip

What is the MOA of Leukotriene Modifying Agents?

All agents help decrease airway edema, constriction and inflammation - Zileuton is a 5-lipoxygenase inhibitor which inhibit leukotriene formation

Which of the following is NOT MAO-Inhibitor? A. Isocarboxazid (Marplan) B. Phenelzine (Nardil) C. Tranylcypromine (Parnate) D. EMSAM E. Selegiline ?

All are MAO-I

Which of the following natural products can be hepatotoxic? A. Chaparral C. Comfrey D. Kava

All of them FDA is against the use of Chaparral

Which of the following must be removed prior to MRI? A. Testesterone (Androderm) B. Clonidine (Catapre-TTS) C. Fentanyl (Duragesic) D. Rotigotine (Neupro) E. Scopolamine (Transderm-Scop) F. Salonpas Power Plus (OTC) G. Nicotine (NicoDerm CQ)

All of them must be removed

Which one would you give if patient has Hepatic impairment or Kidney ? Argatroban and Bivalirudin

Argatorban is good if patient has kidney problems Bivalirudin is good if patient has liver problems

Tofacitinib (Xelijanz)

BBW 2 1. Increased risk of serious infections (including active tuberculosis, fungal, viral, bacterial, or OI). Screen for latent TB and treat before starting therapy 2. Increased risk of lymphomas and other malignancies NOTES 1. Can be used as monotherapy or with non-biologic DMARDs 2. DO NOT use with biologic DMARDs or potent immunosupprsants.

SNRI - Safety/Side effects/monitoring

BBW - Antidepressants increase the risk of suicidal thinking and behavior in children, adolescents, and you adults (18-24 years of age) with MDD and other psychiatric disorders CI - Potentialy lethal DI: SNRI and MAO Is SE - All have warning for increase in BP, but risk is greatest with venalfaxine when dosed > 150mg/day - Bleeding risk with concurrent use of anticoagulatns, antiplatelets, etc. NOTE - Do not use Fetzima or Cymbalta if CrCl < 30 ml/min

Alkylators (BBW/CI/SE)

BBW - Hemorrhagic cystitis (ifosfamide, hcyclophosphamide), severe bone marrow suppression, secondary malignancy CI - Severe bone marrow suppression, bladder or urinary obstruction SE - Myelosuppression, N/V , alopecia - Pulmonary toxicity (busulfan, carmustine, lomustine) - skin pigmentation changes (busulfan, carmustine) - Decarbazine is highly emetogenic and can cause flu-like sytmpms - * impairs fetrility *

Endothelin Receptor Antagonist (ERAs) Bosentan (Tracleer)

BBW - Hepatotoxicity - Use in pregnancy is contraindicated (Pregnancy Category X) * Due to above 2 BBW; this drug only available through the Tracleer Access Program (T.A.P.) * Side Effects - Headache - Decrese in HgB (usually in the first 6 weeks of therapy) > Monitor at baseline and at 1 month and 3 months then every 3 months - Increase in LFT (dose related) > Monitor at baselne every month thereafter - Edema (>10%) - etc. * Avoid combination with Cyclosporine and Glyburide* > Glyburid will put patient at increase risk of hepatotoxicity > Cyclosporine will increase Conc. of Bosentan

Tenofovir (Viread)

BBW - May exacerbate HBV once drug is discontinued or when HBV resistance develops SE - Renal insufficiency, Fanocni's syndrome, osteomalacia, and decrease in bone density - Increase in LFT and CPK - etc - Take Atripla on an empty stomach, preferably at bedtime. Take complera with a meal. Take stribid with food - Dispense in original container. * Consider vitamin D and calcium supplementation * Notes > Tenofovir powder: This medication comes with a dosing scoop; use only the dosing scoop to measure the oral powder. Mix the oral powder with soft foods that can be swallowed without chewing (applesauce, baby food or yoqurt). Do not mix with liquid as the powder may float to the top even after stirring. Give the entire dose right away after mixing to avoid a bad taste.

Rivaroxaban (Xarelto)

BBW - Patients receiving neuraxial anesthesia (epidural, spinal) or undergoing spinal puncture are at risk of hematomas and subsequent paralysis - Premature discontinuation of rivaroxaban increase the risk of thrombotic events Contraindication - Active major bleeding Warning - Avoid use in severe renal impairment (DVT treatment and prophylaxis (CrCl < 30ml/min) and AFib (CrCl < 15 ml/min) - * use is not recommended with prosthetic heart valves SE - Bleeding anemia Notes - No antidote - No monitoring of efficacy required - Discontinue 24 hours prior to elective surgery

OxyCODONE

BBW - Report abuse , misuse and diversion - Avoid use with 3A4 inhibitor > Will increase oxycodone levels

Isoniazid (INH)

BBW - Severe (and fatal) hepatitis may occur; usually within first 3 months of treatment SE - Headache, GI upset, Increase in LFTs, peripheral neuropathy, drug-induced lupus erythematosus (DILE), positive coombs test, etc. NOTE - Add pyridoxine 25-50 mg daily to reduce risk of peripheral neuropathy

Spironolactone (Aldactone) Eplerenone (Inspra)

BBW - Tumor risk with spironolactone; tumorigenic in chronic rat toxicity studies. Avoid unnecessary use CI - Anuria, significant renal impairment (CrCl <= 30ml/min) Hyperkalemia (addision disease or other conditions that increase K); concomitant use of strong 3A4 inhibitor with eplerene. SE > Hyperkalemia, Increase in SCr, Dizziness - For spiro > Gynecomastia, brast tenderness, impotenece, irregular menses Monitoring - Check K before starting and frequently thereafter, BP, SCr/BUN; fluid status (input and output, weight) signs and symptoms of HF

Morphine

BBW > Fatal respiratory depression has occured > Crushing, dissolving, or chewing of the long acting products can cause the delivery of a potentially fatal dose SE > Constipation, Nausea, Vomiting CNS effects > Somnolence, dizziness, changes in mood, confusing, delerium SKIN REACTION > Flushing, pruritus, diaphoresis - may need antihistamine, etc. > * Impotence might occur with chronic use*

Important about ABC - Abacavir (Ziagen) +lamivudine (Epzicom) + lamivudine and dolutegravir (Triumeq)

BBW > Serious, sometimes fatal, hypersensitivity reaction - look for fever, skin rash, respiratory symptoms (dyspnea, cough), fatigue, malaise, and/orGI symptoms *discontinue drug and do not re-challenge* > Must screen for the HLA 5701 allele prior to starting abacavir therapy- if positive this will increase risk of hypersensitivity so do not use. SE > Increase in LFT, Hyperlipidemia, etc Note > Require MEDGUIDE due to CVD risk > Take w/o regard to meal except if you are taking it in combo.

Important points about Induction Agent Antithymocyte Globulin (Atgam-Equnine) (Thymocyte-Rabbit)

BBW > Should be administered under the supervision of a physician experienced in immunosuppressive therapy. Adequate laboratory and supportive medical resources must be readily available (e.g., epinephrine) SE > Anaphylaxis (intra-dermal sin testing recommended prior to 1st dose), fever, chills pruritus, rash, etc. Monitoring > Lymphocyte profile (T-cell count) etc Note > May need to pre-medicate (Benadryl, Tylenol, Steroids) Epinepherine should always be read MOA > Antibodies - reverse rejection by binding to antigen on T-lymphocyte (killer cells) and interfere with their function

Azathioprine (Azasan, Imuran)

BBW (2) - Chronic immunosuppression can increase risk of neolasia (esp. lymphomas) - Hematologic toxicities (leukopenia, htromboctopenia) and mutagenic potential WARNINGS - GI (Severe N/V/D), hematologic (leukopenia, thrombocytopenia, anemia) and hepatotoxicity; patients with genetic deficiency of thiopurine methyltransferase (TPMT) are at increase risk for myelosuppression and may require lower dose. - PREG D

AzaTHIOprine (Azasan, Imuran)

BBW (2) 1. Chronic immunosuppression can increase risk of neoplasia (especially lymphomas) 2. Hematologic toxicities (Leukopenia, Thrombocytopenia, and mutagenic potential) WARNING GI (Severe N/V/D), hematologic (leukopenia, thrombocytopenia, anemia) abnormalities and hepatotoxicity; patients with genetic deficiency of thiopurine methyltransferase (TPMT) are at increased risk for myelosuppression and may require lower dose SE N/V , rash, increase in LFT, myelosuppression Notes > Pregnancy category D

Procainamide > injection ONLY > Active metabolite N-acetyl procainamie (NAPA), is renally cleared, decrease dose when CrCl < 50 mL/min

BBW (3) - Potentially fatal blood dyscrasias (e.g, arganulocytosis) > Monitor patient closely in the first 3 months of therapy and periodically thereafter. - Long-term use leads to positive antibody (ANA) test in 50% of patients which may result in drug-induced lupus erythematosus (DILE) in 20-30% of patients. - CAST trial > Negative study CI > 2nd/3rd degree heart block (unless patient has a functional artificial pacemaker), SLE, torsade de pointes, procaine or other ester-type local anesthetics SE > Hypotension, rash, lupus-like syndrome, QT prlongation

Maintenance medication - Mycophenolate Mofetil (CellCept) and Mycophenolic acid (Myfortic)

BBW (4) - Increase risk of infections, Increase in development of lymphoma and skin malignancies; Increase risk of congenital malformations and spontaneous abortions when used during pregnancy, should only be prescribed by health care providers experienced in immunosuppresive therapy Side Effects - Diarrhea, GI upset, Vomiting, hyper-and hypotension, edema, tachycardia, pain, hyperglycemia, hypo/hyperkalemia, hypomagnesemia, hypocalcemia, hypercholesterolemia , tremor, acne, infections NOTE > CellCept and Myfortic should not be used interchangeably due to difference in absorption. Myfortic is enteric coated which helps to decrease diarrhea - Do not use IV if allergy to polysorbate 80 - Shoul dbe taken on an empty stomach to avoid variability in absorption. - Preg D

Amiodarone (Cordarone, Pacerone, Nexterone) > Tablet > Injection

BBW (4) 1. Only for life-threatening arrhythmias due to toxicity: patients should be hospitalized when therapy is initiated 2. Pulmonary toxicity may occur without symptoms 3. Liver toxicity 4. Proarrhythmic: Exacerbation of arrhtmias making them more difficult to tolerate or reverse.

Ergotamine drugs (DHE 45, Migranal) > IM/SUB-Q > Intranasal

BOXED WARNING - Serious and life-threatening peripheral ischemia have been associated with the co-administration of DHE with potent CYP3A4 inhibitor including protease inhibitors and macrolide antibiotics. - Inhibition increase conc. and put patient at risk of vasoconstriction complications Notes > Contraindicated with many drugs including ergot/5-HT derivatives within last 24 hours, pressor/vasoconstrictive medications. Avoid during or within 2 weeks of discontinuing MAO inhibitors.

Recomendations of BP and Cholesterol in patient with Diabetes per ADA

BP > Goal blood pressure for patient with diabetes is <140/80. A lower <130 may be appropriate for certain individuals such as younger patients. > A regimen consisting of an ACE inhibitor or ARB shoudl be chosen first-line due to reducing of CVD outcomes and progression of diabetic nephropathy in patients with albumunira. Cholesterol > Life style modification should focus on reducing saturated fat, trans fat and cholesterol. Stating therapy should be added in patient A. with overt CVD B. With out CVD and >40 and have 1 more risk factor Cholesterol GOALS - LDL < 100 mg/dl in patient without CVD - In patient with CVD goal < 70mg/dL is an option - Triglyceride <150 - HDL > 40 men - HDL > 50 women

LABA

BW - LABA increase the risk of asthma-related deaths and should only be used in asthma patients who are currenlty receiving but are not adequately controlled on long-term asthma control medications CI - Status asthmaticus, acute episodes of asthma or COPD, monotherapy in treatment of asthma SE - Tachycardia, tremor, shakiness, lightheadedness, cough, palpitations, hypokalemia and hyperglycemia Notes - Bronchodilators are used on a PRN schedules basis reduce symptoms

Enoxaparin [BW, CI, SE]

BW - Patient receiving neuraxial anesthesia (epidural, spinal) or undergoing spinal puncture at risk of hematoma and subsequent paralysis CI - History of HIT, active major bleed, hypesensitivity to pork SE - Bleeding, anemia, Increase in LFT, thrombocytopenia, hyperkalemia, injection site reactions (bruising)

Why variations in CYP2D6 expression is significant?

Because based on the presence or absence of CYP2D6 polymorphism, patients can be CYP2D6 ultra-rapid metabolizer (UMs), extensive metabolizer (EMs), intermediate metabolizer (IMs) or poor metabolizer (PMs). Codeine is metabolized to morphine by CYP2D6; morphine provides the analgesic effect. UMs will have higher morphine levels while PMs will have low, or absent morphine levels and thus minimal analgesic effect from the use of codeine.

If most of the common covered DME items require the pharmacy to become CMS-approved DME provider, then why some items are billed under Medicare part B such as Albuterol nebulizer solution with out being accredited?

Because these drugs that are billed under Part B (e.g. drugs used in a nebulizer) do not require DMEPOS accreditation for Medicare to pay the bill.

IF HIT!! then patient is at risk of thrombosis. then why is it advised to stop warfarin if patient develop HIT?

Because warfarin use with a low platelet count has a high correlation with warfarin-induced limb gangrene and necrosis - Then when to restart Warfarin? > When platelet are at least 150,000 /mm3 > Warfarin should be initiated at lower doses (5mg maximum) > overlap warfarin with a nonheparin anticoagulant for a minimum of 5 days and until INR is within target range for 24 hours.

The first line treatment of bed wetting in kids age 6 or up is behavioral therapy. What is Behavioral therapy?

Behavioral therapy that can be effective include *positive reinforcement*, establishing a normal daytime voiding patter and a normal bowel patter, and establishing a normal hydration patter. Fluid intake should be limited prior to bedtime. Behavioral approaches are effective in many children and should be tried for up to *3 months*. If failed, then we can move on to Alarm therapy alone or w/ desmopressin.

What is the drug of choice in ICU - Agitated patient with seizures or alcohol/benzodiazepines withdrawal?

Benzodiazepines

Proteasome inhibitors - Bortezomib (Velcade) - Carfilzomib (Kyprolis)

Bortezomib > Give acyclovir prevent of zoster reactivation SE - Peripheral neuropathy Carfilzomib SE > Peripheral neuropathy (but less than bortezomib)

What is the advantage of Fosaprepitant IV over Aprepitant PO?

Both are Neuokinin 1 receptor antagonist - Fosaprepitant 150 mg IV once on Day 1 lasts up to 72 hours post chemotherapy, thus neuokinin 1 receptor antagonist regimen is not needed on days 2-4

Hydroxyurea

Brand names (Droxia, Hydrea) - Hydroxyurea reduces the severity of sickle cell disease by stimulating production of HbF levels, by blocking the enzyme ribonucleotide reductase. - It is indicated for adults with >= 3 moderate-severe pain crisis in 1 year or patient with severe ACS, anemia, or disability. - May be used in children > 9 months regardless of disease severity

What are the WARNING SIGNS of cancer?

C A U T I O N Change in bowel or bladder habits A sore that does not heal Unusual bleeding or discharge Thickening or lump in breast or elsewhere Indigestion or difficulty swallowing Obvious change in wart or mole Nagging cough or hoarseness

What is the DOC for Gonorrhea- caused by Neisseria gonorrhea, a gram negative diplococcus?

Ceftriaxone 250 IM x 1 PLUS Azithromcyin 1gram PO x 1 or Doxycycline 100mg po BID for 7 days If severe allergies to cephalosporin Azithormycin 2 g Po x 1 will cover both gonorrhea and chlamydia

What is the difference between Class Ia Ib Ic Vaughan Williams Classification of Antiarrhythmics

Class I antiarrhythmics are sodium channel blockers. - Class Ia are intermediate sodium channel blockers and they are also * block the potassium channels * > Decrease conduction velocity and automaticity and increase refractory period - Class Ib are fast sodium channel blockers > decrease conduction and automaticity and * little effect on decreasing refractory period * > CNS adverse effects. - Class Ic are long sodium channel blockers > * Absolutely contraindicated in patients with heart failure or with a recent myocardial infraction (due to negative inotropic properties * > Decrease conduction velocity, and decrease automaticity, have little or no effect on refractory period

Oral medication used commonly for Fertility

Clomiphene (Clomid, Serophene) GnRH -> Increase in FSH and LH to Increase Ovulation > Selective Estrogen Receptor Modulator (SERM) CI > Liver disease, pregnancy, uncontrolled adrenal or thyroid disorders SE > Hot flashes, ovarian enlargement, abdominal bloating/ discomfort , thrombosis risk etc.

Drugs that cause TTP - Thrombotic thrombocytopenic purpura (TTP

Clopidogrel Ticlopidine Sulfamethoxazole Quinine Acyclovir, Valacyclovir, famicilovir

What is Continuous data and what are the 2 types of Continuous data?

Continuous Data: Can take a infinite number of possible values (such as height, weight, A1C, blood pressure) within a defined range. The 2 types 1. Interval: Interval data is used to measure continuous data that have legitimate mathematical values. Ex. Celsius temperature scale 2. Ratio: Ratio data has equal intervals between values and a meaningful zero point; meaning there is none of the variable (e.g. height, weight, time, length)

Depression diagnosis

DSM-5 - Must have 1 or 2 during same two weeks period 1. Depressed mood 2. Marked diminished interest/pleasure 3. Significant weight loss or weight gain 4. Insomnia or hypersomnia 5. Psychomotor agitation or retardation 6. Fatigue or loss of energy 7. Feelings of worthlessness 8. Diminished ability to concentrate 9. Recurrent suicidal ideation

What is the function of Distal Convoluted tubule?

Distal means farthest away and the distal convoluted tubule is the farthest away from the entry point in the nephron. The distal tubule is also invovled in regulation K+, Na+, Ca2+ and pH. * Thiazide diuretics inhibit the Na-Cl pump in the distal tubule *. Only about 5% of the sodium is reabsorbed at this point which makes thiazides weaker than loops. Thiazides *increase* calcium level by affecting the calcium pump in the distal convoluted tubule. Consequently, long term use of thiazide diuretics have a protective effect on bone

What is the most commonly used formula in adult oncology practice?

DuBois and DuBois formula > For calculating based on surface area

What is the gold standard to diagnose osteoporosis?

Dual energy x-ray absorptiometry (DEXA or DXA)

how much elemntal iron in Ferrous sulfate?

Ferrous gluconate - 12% Ferrous sulfate - 20% Ferrous sulfate - exsiccated - 30% Ferrous fumarate 33% Carbonyl iron - 100% Polysaccharid iron complex 100% > No advantage of using last two because body can only absorb certain amount

Vasodilators [Nitroprusside]

Equal arterial and venous vasodilator at all doses. - Do not use if active myocardial infraction as it can cause " coronary steal" or shunting of blood away from areas with diseased coronary arteries. - Nitroprusside metabolism results in the formation of thiocynate and cyanide both of which can cause toxicity - Hydroxycobalamin can be administered to reduce risk of thiocynate toxicity, whereas, sodium thiosulfate may be used to treat cyanide toxicity - CI > SBP < 90 mmHG, CI with PDE-5 inhibitor, increase in intracranial prssure sE > Hypotension, headache, tachycardia, thiocyanate,cyanide toxicity BBW - Except when used briefly or at low ( < 2mcg/kg/minute) infusion rates. Nitroprusside gives rise to large cyanide quanitites - Can cause excessive hypotension; continuous BP monitoring required - Solution must be further diluted with D5W; do not give undiluted

1. cause weight loss 2. hiccups 3. Drug of choice for Absence (big hint)

Ethosuximide (Zarontin) MOA > T-type calcium channel blocker that increase seizure threshold and suppreses paroxysmal spike and -wave pattern in absense seizures SE - GI upset (weight loss, abdominal pain, nausea and vomiting), hiccups, dizziness, somnolence. WARNNING - Locosamide prolongs PR interval and increase risk of arrhythmias. Obtain an ECG prior to use and after titrated to steady state. Use with caution in patients with cardaic conduction problems and severe cardiac disease (MI, HF)

How to calculate Odds Ratio?

Ex. In a group of 100 smokers, 40 people developed lung CA while 60 people did not. In a similar group of 100 non-smokers, lung CA developed in 10 people. The odds of a smoker developing lung CA would be 40/60=0.67, whereas the odds of a non-smoker developing lung CA would be 10/90 = 0.11. The odds ratio is then 0.67/0.11, or 6, meaning that smokers are 6 times as likely to develop lung cancer than non-smokers. An odds ratio of 1 indicates no difference between groups. The smaller the event rate, the closer the odds ratio is to the relative risk.

What equation is used to determine the bioavailability? You will be given AUC for PO and IV and Dose for IV and PO

F(%) = 100 x AUC (extra) x Dose (intra) AUC (intra) x dose (extra)

Where do you report SE, AE and allergies? A. VAERS B. FAERS C. NONE

FAERS These repots should be reported to the FDA's MedWatch program, which is called the FDA's adverse Event Reporting System (FAERS) ** VAERS is for immunizations **

T or F. TST (Tuberculin skin test) is used to diagnose active tuberculosis.

False. - Active tuberculosis is not diagnosed with a TST but rather through a sputum sample. - The acid fast result is not specific to MTB. Final diagnosis must be made through PCR testing or culture results. Final results can take several weeks.

T or F. There is no drug absorption when drug is administered via *Extravascular * route.

False. Drug absorption occurs as the drug moves from the site of administration to the circulatory system.

T or F. The necessary interval between an antibody-containing blood product and MMR or varicella-containing vaccine (except zoster-vaccine - this is not affected by circulating antibody) is a minimum of 4 months and may be up to 12 months.

False. Minimum of 3 months and may be up to 11 months

T or F. Liquids in a container curve up, therefore measure at the top of the meniscus.

False. Must measure at the bottom of meniscus

T or F. Anticoagulation break existing clots

False. Only thrombolytics such as tPA break existing clot.

T or F. The larger the size of capsule, the higher the number.

False. The smaller the size, the higher the number and the smallest capsule size is 5

Dobutamine may turn slightly pink oxidation and potency will be lost. T or F

False. potency will not be lost

What is required to be given to patient BEFORE EACH vaccine is administered?

Federal law requires that patients receive the most-up-date version of the Vaccine Information Statement (VIS) BEFORE EACH vaccine is administered.

Fibrates

Fenofibrate (Tricor) Fenofibric acid (Trilipix) Gemfibrozil (Lopid) Contraindication - Severe liver disease - Severe renal disease (CrCl < 30 ml/min) - Gallblader disease - Nursing mothers - Concurrent use with repaglinide SE > Increase in LFT, abdominal pain, Increase in CPK, dyspesia , URTIs - Fenofibrate,Fenofibric acid: * Fenoglide, Lofibra, Lipofen with FOOD *

What are the signs of Toxicity?

First sign of toxicity are nausea/vomiting, loss of appetite and bradycardia. Other signs of toxicity include blurred/double vision, altered color perception, greenish-yellow halos around lights or objects, abdominal pain, confusion, delirium, arrhythmias (prolonged PR interval, accelerated junctional rhythm, bidirectional ventricular tachycardia)

What is the drug of choice for oropharyngeal candidiasis (thrush) in HIV patients or moderate-severe disesae in non-hiv infected patients?

Fluconazole

which steroid has the highest mineralcorticoid potency causing Na+ and H2O retention and is indicated for Addison's disease; not for inflammation.

Fludrocortisone

GELs

Gels are oil in water with alcohol base Example: Benzoyl peroxide and erythromycin topical acne gel (benzamycin) Also used as Thickeners > example of thickeners are alginates (including Na, K, Ca alginate), agar, carrageenan, gelatin, carbomer, tergacanth, bentonite)

What is included in the Global Risk Assessment Tool?

Gender Age Total Cholesterol HDL-C Systolic blood pressure Wether antihypertensive treatment is used Smoker > Presence of diabetes > Race

Signs and Symptoms of Heart Failure

General - Dyspnea at rest or on exertion - Weakness/fatigue - SOB - Reduction in exercise capacity - LVH - Increase in BNP (B-type Natriuretic Peptide): normal < 100 pg/mL - Increase in NT-proBNP (N-terminal pro B-type Natriuretic Pepdite): normal < 300 pg/ml Left-side HF - Orthopnea - Nocturnal cough - Bibasilar rales - S3 gallop (abnormal heart sound) - EF < 40% Right-Sided HF - Edema - Ascites - JVD - Hepatojugular reflux - Hepatomegaly

Herbal/Natrual Product Drug Interactions

Ginkgo biloba increase bleeding risk with no effect on INR Danshen can increase INR Dong Quai can increase INR Glucosamine can increase INR Grapefruit can increase INR Wintergreen oil can increase INR etc. - Any addition of vitamin K will decrease INR > Stay consistent with the amount of vitamin K consumed through the diet

GAP-BA (acronym)

Glycoprotein (GP) IIb/IIIa receptor antagonist > Given for patients going PCI +/- stend > Abciximab only given to patient who PCI is planned Anticoagulants > Used to prevent further clotting P2Y12 inhibitors > give loading dose followed by maintenance dose > * do not give if urgent CABG surgery required* > Ticagrelor (preferred)- 180mg x 1 > Clopidegrol 600mg x 1 > * Prasugrel should only be given if going PCI Beta blockers > beta-1 selective should be given unless patient has: a. signs of HF 2. evidence of a low-output state c. high risk for cardiogenic shock d. others > if patient has HF then give: metoprolol succ or bisoprolol or carvediolol ACE-I Should be given within the first 24 hours and continued indefinitely in all patients with LVEF < 40%, those iwth HTN, DM, or stable CKD unless CI (use ARB if CI to ACE-I) * Do not use IV ACE-I due to risk of hypotension *

Where does gout occur?

Gout typically occurs in one joint, which is most often the MetaTarsoPhalangeal joint (MTP, the big toe) > Uric acid is produced as an end-product of purine metabolism. - Symptoms > Severe, painful gout attack > Gout typically strikes after many years of persistent hyperuricemia

Clostridium difficile, Listeria monocytoegnes?

Gram + Rods * is a spore-forming, Gram-positive anaerobic bacillus that produces two exotoxins: toxin A and toxin B. It is a common cause of antibiotic-associated diarrhea (AAD*

Patient without risk factors for stress ulcer should not receive stress ulcer prophylaxis. What are the disadvantage of H2RA and PPI

H2RA > Cause thrombocytopenia > Mental status changes in the elderly or those with renal/hepatic impairment > Tachyphylaxis (Tolerance) have been reported PPI > Risk of GI infections (C.difficile) > Fractures > Nosocomial pneumonia

What is the leading cause of death in ICUs?

HAP is the leading infectious cause of death in ICUs

Pertuzumab (Perjerta)

HER2 gene over-expression BBW - Embryo-fetal death and birth defects, cardiac failure SE - Cardiomyopathy (HF, decresae in LVEF) etc.

What is the difference between hallucination and delusions?

Hallucination - Hearing voices (auditory hallucinations are common), visual hallucinations Delusions - Beliefs the patient has, but are without a basis in reality

Where to find information about pharmaceutics?

Handbook of pharmaceutical Excipients Merck Index Remington: The science and practice of pharmacy

What is Hazard Ratio (HR)?

Hazard ratios are often used when clinical trials present data related to the time survived to an event (mortality, cure, specified level of symptoms reduction) * HR are used in clinical trials with time-to-event (or survival) analysis* HR assume that the ratio is constant over tim. HR = HR in treatment group divided by HR in the control group ** Interpreting HR** HR = 1: Event rates are the same in both arms over time HR < 1: At any given time relatively fewer patient in the treatment group have had an event compared to control group HR > 1: At any given time, relatively more patients in the treatment group have had an event compared to control group Example: In a clinical trial assessing the cure rate provided by drug A vs. placebo the HR is reported to be 4. This means that a treated patient who has not been cured by a certain time point has four times the chance of being cured by the next time point compared to someone on the placebo group.

Do we have vaccine for Hepatitis A? What is the treatment of hepatitis A? How is hepatitis A transmitted? When is the dose of Hepatitis A vaccine should be started? What specific age?

Hepatitis A virus (HAV) is a vaccine preventable disease that cause an acute, self-limiting illness in most patients. - Transmission is primarily via the fecal-oral route through improper hand washing after exposure with an infected person or via contaminated food/water - 2 shots of vaccine is required at one year of age. - Vaccine is also recommended for older persons who is at risk of catching the disease - Treatment of HepA is supportive and no antiviral agents is needed - * Immunoglobulin (IgG) can be given for post-exposure prophylaxis in select cases *

Hyponatremia Hypertonic? Isotonic? Hypotonic: (hypovolemic hyponatremia, hypervolemic hyponatremia, and Isovolemic hyponatremia)

Hypertonic: Is a state in which serum osmolality is increase and caused by hyperglycemia or use of hypertonic solutions that do not contain sodium Isotonic: Has normal osmolality and can be associated with hyperlipidemia Hypotonic: may occur with changes in volume status: Hypovolemic hyponatreamia: diuretic use, salt-wasting syndromes, adrenal insufficiency blood loss, v/d) treatment is correcting the underlying cause and administer saline solutions Hypervolemic hyponatremia: is caused by fluid overload, usually with cirrhosis, heart failure, renal failure. Diuresis with fluid restriction is the preferred treatment Isovolemic hyponatreamia is usually caused by the syndrome of inappropriate antidiuretic hormone (SIADH). It is treated with fluid restriction or diuresis.

What is the route of administration of Palivizumab?

IM injection in the anterolateral thigh muscle Can use deltoid in children > 1 year if the muscle mass is adequate, but this is unlikely until the child is older

What is commonly used for treatment of alcohol withdrawal?

INPATIENT > Benzodiazepines OUTPAITENT > Anticonvulsants OTHERS USED TO PREVENT RELAPSES > Naltrexone (ReVia) > Acamprosate (Campral) > Disulfiram (Antabuse)

Important on monitoring of Warfarin

INR target is 2.5, range 2-3 for most indications (DVT, AFib, bioprosthetic mitral valve, mechanical aortic valve, antiphospholipid syndrome) and should be 2.5-3.5 for some high risk indications such as a mechanical mitral valves or 2 mechanical heart valve - INR monitoring begin after the initial 2 or 3 doses, or if on a chronic stable doses of warfarin, monitor of intervals up to 12 weeks

When do you start treating HIV+ patient for primary prophylaxis of Toxoplasma gondii?

Indication > Toxoplasma IgG + patients with CD4+ count < 100 cells/mm3

List the 3 immunizations that we must screen if patient has allergies to eggs prior to administrations?

Influenza > Flublock does not use eggs at all in its production > Expensive Yellow fever Rabies vaccine (precaution with egg allergy)

Digoxin

Inhibit the Na/K ATPase pump which results in positive inotropic effect (Increase in CO). It also exerts a parasympathetic effect which provides a negative chronotropic effect (decrease in HR). - It is added in patients who remain sympotmatic despite receiving standard therapy of ACE inhibitors (or ARB) with a beta blocker. - It improve symptoms, exercise tolerance, and QOL. - Overall, digoxin does not improve survival of heart failure patients, but it does reduce hospitalizations of heart failure. - Lower dose for renal insufficiency, smaller, older, female - HF should be kept < 1 ng/mL (range 0.5 - 0.9 ng/mL)

Why is it important to educate patient not to chew on ER formulation?

It is danger if long-acting formulations are crushed: a fatal dose could be released. This includes ER opioids. Be sure to look for the suffix and counsel * There are a few long-acting formulations that can be cut at the score line- but still NOT crushed, such as Toprol XL and Sinemet CR *

List one drug that you can cut the patch into pieces?

Lidoderm which is designed to be cut and applied over the painful regions - Do not cut fentanyl patch because it will change the dose - Heat will release the drug faster from the patch (Do not heat the patch)

Dipeptidyl Peptidase 4 inhibitors

MOA > Prevent the enzyme DPP-4 from breaking down incretin hormones, glucagon-like peptide-1 (GLP-1) and glucose-dependent insulinotropic polypeptide (GIP) - SitaGLIPtin (januvia) - Saxagliptin (Onglyza) Linagliptin (Tradjenta) - Alogliptin (Nesina) - No renal dose adjustment for Trajenta SE > Nasopharyngitits > Upper respiratory tract infections > Peripheral edema > Rash and hypoglycemia > Rarely acute pancreatitis Notes > Natural weight > Saxagliptin is major 3A4 and P-glycoprotein substrate. use the lower 2.5 mg dose with strong CYP3A4 inhibitors. > Linagliptin is a major 3A4 and P-glycoprotein substrate. Linagliptin levels are decreased by strong inducers

Summary of Drugs Used Acutely for ACS

Morphine > Pain relief > May be used in patients with ongoing chest discomfort despite nitroglycerin (NTG) therapy > SE: Hypotension, bradycardia, N/V, sedation etc. > Antidote: Narcan - naloxone Oxygen > Give if SaO2 < 90 or respiratory distress Nitrates > Sublingual NTG 0.3 to 0.4 mg every 5 minutes for up to 3 doses should be taken for immediate relief of ischemic pain. call 911 if pain not improved 5 minutes after first dose > Seperate from PDE5 Aspirin > * non-enteric-coated, chewable aspirin (162-325) should be given to all patients immediately if no contraindication are present* > Maintenance dose of aspirin 81mg should be continued indefinitely > IF CI: give P2Y12 inhibitors

DMS 5 for Schizophrenia

Must have 1 of the following - Delusion , hallucination, disorganized speech PLUS one or more of the following A. Negative signs and symptoms > Loss of interest in everyday activity > Lack of emotions > Inability to plan or carry out activities > Poor hygiene > Social withdrawal > Loss of motivation > Poverty of speech B. + signs and symptoms > Hallucination > Delusion > Disorganized thinking/behavior > Difficulty paying attention

Mixing Insulins and Insulin Mixes

NPH can be mixed with regular and rapid acting insulins. This is done by first drawing up the clear insulin to a syringe (regular or rapid-acting insulin) and then drawing up the cloudy insulin (NPH). In a mix, the first number is the % of the long-acting intermediate insulin (N also written as NPH, or aspart protamine or lipsor protamine), followed by the % of the shorter-acting insulin (R, for regular or rapid acitng aspart for lispor) Ex. Humulin 70/30 contains 70% NPH and 30% regular insulin. They are named after the regular or rapid-acting insulin (ex. Novlog 70/30 contain 70% insulin aspart protamine and 30% insulin aspart.

Medications to Avoid in Acute Setting

NSAID (except for aspirin) should be avoided IR nifedipine should be avoided due to risk of mortality IV fibrinolytic therapy should not be administered unless ST-segment elevation MI or new left bundle branch block (which is equivalent to STEMI )

Why we should not use Racepinephrine (Asthmanefrin EZ breath Atomizer) OTC

Not beta-2 selective

DNA , Chromosome, Nucleotides, Gene , Allele

Nucleotides: These consists of a Nitrogen base [ adenine (A), thymine (T), guanine (G) and cytosine (C)], a five-carbon sugar, and a phosphate group. They are the building block of DNA. DNA is the genetic information or characteristics of each individual that has been inherited from both parents. DNA is wrapped around proteins and packaged into chromosomes, which appear as threadlike structures. there are 46 chromosomes. 23 from father and 23 from mother. 22 of these are called autosomes, look the same in both males and females. the 23rd pair, the sex chromosome different between males and females. Gene: The DNA can be divided into specific sequences of nucleotides that code for a single protein. The sequence of the nucleotides are called genes. Allele: Specific form of a gene ( 7 to 10 nucleotide looking at that sequence).

What is NNT

Number needed to treat (Always round up) Is the number needed to treat represent the number of people who would need to be treated with the intervention (drug) for a certain period of time (one year) in order to achieve the desired outcome 1/ARR ex. ARR = 0.12 1/0.12 = 8.3 (*always round up*) = 9 Therefore, for ever 9 patients who received metoprolol for 1 year, heart failure progression is prevented in one patient

Iburprofen

OTC > 200-400 mg Q 4 to 6 H Maximum of 1.2g/day > Limit to 10 days or less RX > 400-800 mg Q 6 to 8 H Maximum of 3.2g/day

What is ODD?

Odds are not the same as risk. Risk is the probability that a person who has not developed the event will develop the event whereas odds represent the probability of the event occurring compared with the probability that it will not occur. Ex. 100 smokers if 40 smokers develop lung cancer and 60 smokers did not develop then risk of developing is 40% and odds would be 40:60 40/60 67%

Important notes about Amiodarone

Pregnancy category D Infusion longer than 2 hours must be administered in *non-PVC container such as polyolefin or glass. Premixed Nexterone comes in GALAXY containers which are non-PCV and non-DEHP and can be stored up to 24 months at room temperature. - Flush with saline - Premixed IV bag advantages: longer stability, PCV bag not an issue, available in most commonly used concentrations. If hypotension or bradycardia occurs, slow infusion rate or discontinue. - * t1/2 40-60 days * - Recommended as drug of choice in patient with concomitant heart failure *

The difference between Prolia and Xgeva?

Prolia 60mg SC every 6 months used for osteoporsois Xegva 120mg SC monthly used used early in metastatic disease to prevent skeletal-related evens.

Important about Vasopressin

SE: Arrythmias, necrosis/gangrene > Doses > 0.04 units/min cause more cardiovascular side effects (asystole, MI); hyponatremia > Must monitor BP, HR, CO, ECG, fluid balance, sodium concentration

What are the reasons to use Films ?

Same as ODTs; they dissolve in the mouth easily and do not have issues associated with swallowing Examples - Zuplenz - Ondansteron

What is the difference between Tricycles and SNRI when it comes to MOA?

Same thing - NE and 5HT reuptake inhibitors but Ticyclic has more MOA such as block of ACH and Histamine receptors and thats the reason we have the SE.

What would be a good option for a patient that always get Nausea from chemotherapy that last for a while?

Sancuso (Granisetron) is one of the 5HT3-antagonist that comes in long-acting patch that reduces nausea for up to *seven* days * It should be applied prior to chemo because it takes time for it to work, not help if you apply it at the same time patient is having nausea *

What are the benefits of Sancuso? What is it?

Sancuso is granisetron patch It is 5-HT3-receptor antagonists comes in a long acting patch (Sancuso) that reduces nausea for up to *Seven days*; this would be useful for a patient with nausea from chemotherapy that lasts a while.

What is the different between Boceprevir (Victrelis) and Simeprevir (Olysio)

Simeprevir just a new drug - same indication as Boceprevir (can't use alone) - New thing about this drug is that it can be combind with Sofosbuvir (Sovaldi) and this will eliminate the need of Peg + Ribavirin - Must screen patients with HCV for the presence of NS3 Q80K polymorphism because patient with this will have reduced response - Another nice feature about this drug compared to Boceprevir does not affect hepatic 3A4 acitivty - It is a substracte to 3A4 so do not administer with moderate or strong inhibitors * if Simeprevir combined with PEG-INF-alfa and RBV and RNA >= 25 units/mL at treatment week 4 or 12 then stop therapy but no treatment stopping rules apply to the simeprevir + sofosubvir combination *

What is the only management strategy proven to slow progression of disease?

Smoking cessation is the only management strategy proven to slow progression of disease. Other important management strategies include vaccinations, pulmonary rehabilitation programs, and drug therapy (often using inhalers). Some patients go on to require long term oxygen therapy either given in the hospital for acute exacerbations or used chronically outpatient with the use of portable oxygen systems

What is the dose of aspirin for primary prevention for CVD in type 1 and type 2 diabetes? What if patient has allergies ?

The dose of aspirin should be 75-162 mg daily (usually 81mg EC) if there is no contraindication such as aspirin allergy or bleeding. If the patient has an aspirin allergy, other antiplatelet agents may be considered (plavix)

* Important * About initiation of Hypertension medication in AA

Thiazides are more protective against stroke and CCBs also have better outcomes, including blood pressure reduction, in black patients. However, when CKD and proteinuria are present, an ACE inhibitor or ARB is recommended as initial therapy instead because of the higher likelihood of progression to ESRD (these drugs classes help slow progression to kidney disease). Emphasize medication adherence and sodium reduction.

Why DEHP is an issue in general?

The DEHP compound is considered environmentally unsafe, is a potential human carcinogen and may affect human fertility

What are the most sensitive and specific biomarkers for ACS?

The cardiac troponin T and I (TnT and TnI) - The ischemic leads to myocyte necrosis and subsequent release of biochemical markers into the bloodstream.

What works in the collecting ducts?

The collecting duct is a network of tubules and ducts that connect the nephrons to the ureter. Urine passes from the ureter into the bladder, and from there out of the body via the urethra. Aldosterone also works in the distal tubule. The primary function of aldosterone is to increase Na and water retention and decrease K. By blocking aldosterone (with antagonist like spironolactone or eplerenone), serum potassium increases.

What is Confidence Level?

The confidence interval (CI) Confidence Zone : is a range of value derived from the sample data that has a given probability of encompassing the "true" value. CI = 1 - Type 1 error and type 1 error is the 5% out of the CI because CI is 95%

What's the difference between DT and Td vaccines?

The difference is the amount of diphtheria toxoid contained in each dose. Pediatric DT ("big D") contains 3 to 5 times more diphtheria toxoid than the adult Td ("little d"). DT is used for the few children who cannot receive the pertussis component of the DTaP vaccine, and Td is used for adults and children seven years of age and older who need booster doses of diphtheria and tetanus toxoid.

How to calculate the Relative Risk/Risk Ratio (RR)?

The relative risk (or risk ratio) is the probability of an unfavorable event occurring in the treatment group versus he control group. First, the risk of developing the event must be calculated for both the treatment group and the control group. Then the relative risk is calculated by comparing the risk calculated for the treatment group (numerator) to the risk calculated to the control group (denominator)

Important about P-value and CI

The use of P-value and confidence intervals is complementary. P-value allow a quick decision about wether a result is statistically significant or not. Confidence intervals provide information on statistical significance plus information about the direction and strength of effect

When is the therapy for Herpex Zoster most effective?

Therapy should be initiated at the earliest sign or symptom of shingles and is most effective when started within 72 hours of the onset of zoster rash

Explain the two primary processes of Absorption in the GI?

There is passive diffusion and Active diffusion. Passive diffusion occurs when a high concentration of drug in the gut lumen moves to equalize drug concentration across the gut wall, thus reaching equilibrium. Active diffusion require transporter proteins (require ATP)

What is the MOA of ACE-Inhibitors?

These agents inhibit the angiotensin converting enzyme (ACE), preventing the convesion of angiotensin I (AngI) to angiotensin II (AngII) , a potent vasoconstrictor.

Why the majority of Polyvinly Chloride (PCV) Containers used DEHP Diethylhexyl phthalate?

They use DEHP as a "plasticizer" to make the plastic flexible

What is the MOA of Bisphosphonate?

They work by inhibiting osteoclast activity.

What are the drug therapy recommendations for Black patients w/ or wihout diabetes?

Thiazide or CCB

What is red book?

Useful resource to determine product presentations (i.e., dosage form, strength, package size) and availability and $$

What is the drug of choice for Aspergilus infections?

Voriconazole

Which of the following follow *Michaelis-Menten* or saturable kinetics which means it start as first order kinetics then once it reaches certain dose i changes to zero order kinetics.

Voriconazole Theophylline Phenytoin

Vyvanse is pro-drug for?

Vyvanse lisdexamfetamine is the pro-drug for dextroamphetamine.

Important information about Ketamine (Ketalar) (protect from light

WARNING > Emergence reactions (Vivid dreams, hallucinations, delirium), cerebrospinal fluid (CSF) pressure elevation, respiratory depression/apnea, may cause dependence/tolerance

Dimethyl fumarate (Tecfidera)

WARNINGS > Risk of fatal progressive multifocal leukoencephalopathy (PML) > GI events (n/v/diarrha, etc) SE Flushing , etc Notes > Can give aspirin 30 minutes prior to prevent flushing > Do not crush, chew, sprinkle capsule contents on food.

What are the target for the following Bevacizumab (Avastin) Cetuximab (Erbitux) Trastuzumab (Herceptin) RItuximab (Rituxan) Ipilimumab (Yervoy)

binds to VEGF-A Binds to EGFR Binds to HER2/neu Binds to CD20 Binds to Cytoxocit T-lymphocyte antigen-4 > CTL4 receptor


Related study sets

RE Exam Prep Practice of Real Estate

View Set

MTA 98-364: Database Administration Fundamentals Exam Exercise

View Set

Micro Midterm 2 Consumers and Incentives

View Set

Publications Semester 1 finals study guide

View Set

30 Questions to Test a Data Scientist on Tree Based Models

View Set

Ch. 27: WHMIS Part 2 - Labeling of Controlled Products

View Set

Фармакологія тести

View Set

Chapter 4 Loops and Files (reading)

View Set

Tversky and Kahneman- Availability heuristic

View Set